Sunteți pe pagina 1din 224

1.

SOLIVEN VS MAKASIAR (presidential privilege)


Facts:
Luis Beltran is among the petitioners in this case. He, together with others, was charged with
libel by the then president Corzaon Aquino. Cory herself filed a complaint-affidavit against
him and others. Makasiar averred that Cory cannot file a complaint affidavit because this
would defeat her immunity from suit. He grounded his contention on the principle that a
president cannot be sued. However, if a president would sue then the president would allow
herself to be placed under the court’s jurisdiction and conversely she would be consenting
to be sued back. Also, considering the functions of a president, the president may not be able
to appear in court to be a witness for herself thus she may be liable for contempt.
Issue:
Whether or not such immunity can be invoked by Beltran, a person other than the president.
Ruling:
No. The rationale for the grant to the President of the privilege of immunity from suit is to
assure the exercise of Presidential duties and functions free from any hindrance or
distraction, considering that being the Chief Executive of the Government is a job that, aside
from requiring all of the office-holder’s time, also demands undivided attention.
But this privilege of immunity from suit, pertains to the President by virtue of the office and
may be invoked only by the holder of the office; not by any other person in the President’s
behalf. Thus, an accused like Beltran et al, in a criminal case in which the President is the
complainant cannot raise the presidential privilege as a defense to prevent the case from
proceeding against such accused.
Moreover, there is nothing in our laws that would prevent the President from waiving the
privilege. Thus, if so minded the President may shed the protection afforded by the privilege
and submit to the court’s jurisdiction. The choice of whether to exercise the privilege or to
waive it is solely the President’s prerogative. It is a decision that cannot be assumed and
imposed by any other person.

1
EXECUTIVE DEPARTMENT
2. ESTRADA VS DESIERTO (presidential privilege)
(March 2, 2001)

FACTS:
It began in October 2000 when allegations of wrong doings involving bribe-taking, illegal
gambling, and other forms of corruption were made against Estrada before the Senate Blue
Ribbon Committee. On November 13, 2000, Estrada was impeached by the House of
Representatives and, on December 7, impeachment proceedings were begun in the Senate
during which more serious allegations of graft and corruption against Estrada were made
and were only stopped on January 16, 2001 when 11 senators, sympathetic to the President,
succeeded in suppressing damaging evidence against Estrada. As a result, the impeachment
trial was thrown into an uproar as the entire prosecution panel walked out and Senate
President Pimentel resigned after casting his vote against Estrada.
On January 19, PNP and the AFP also withdrew their support for Estrada and joined the crowd
at EDSA Shrine. Estrada called for a snap presidential election to be held concurrently with
congressional and local elections on May 14, 2001. He added that he will not run in this
election. On January 20, SC declared that the seat of presidency was vacant, saying that
Estrada “constructively resigned his post”. At noon, Arroyo took her oath of office in the
presence of the crowd at EDSA as the 14th President. Estrada and his family later left
Malacañang Palace. Erap, after his fall, filed petition for prohibition with prayer for WPI. It
sought to enjoin the respondent Ombudsman from “conducting any further proceedings in
cases filed against him not until his term as president ends. He also prayed for judgment
“confirming Estrada to be the lawful and incumbent President of the Republic of the
Philippines temporarily unable to discharge the duties of his office.

ISSUE(S):
1. WoN the petition presents a justiciable controversy.
2. WoN Estrada resigned as President.
3. WoN Arroyo is only an acting President.
4. WoN the President enjoys immunity from suit.
5. WoN the prosecution of Estrada should be enjoined due to prejudicial publicity.

RULING:
1. Political questions- "to those questions which, under the Constitution, are to be decided
by the people in their sovereign capacity, or in regard to which full discretionary authority

2
EXECUTIVE DEPARTMENT
has been delegated to the legislative or executive branch of the government. It is concerned
with issues dependent upon the wisdom, not legality of a particular measure."
Legal distinction between EDSA People Power I EDSA People Power II:
EDSA I EDSA II
exercise of the people power of revolution which overthrew the whole government.
exercise of people power of freedom of speech and freedom of assembly to petition
the government for redress of grievances which only affected the office of the President.
extra constitutional and the legitimacy of the new government that resulted from it cannot
be the subject of judicial review
intra constitutional and the resignation of the sitting President that it caused and
the succession of the Vice President as President are subject to judicial review.
presented a political question; involves legal questions.

The cases at bar pose legal and not political questions. The principal issues for resolution
require the proper interpretation of certain provisions in the 1987 Constitution: Sec 1 of Art
II, and Sec 8 of Art VII, and the allocation of governmental powers under Sec 11 of Art VII.
The issues likewise call for a ruling on the scope of presidential immunity from suit. They also
involve the correct calibration of the right of petitioner against prejudicial publicity.
2. Elements of valid resignation: (a)an intent to resign and (b) acts of relinquishment.

Both were present when President Estrada left the Palace.


Totality of prior contemporaneous posterior facts and circumstantial evidence— bearing
material relevant issues—President Estrada is deemed to have resigned— constructive
resignation.
SC declared that the resignation of President Estrada could not be doubted as confirmed by
his leaving Malacañan Palace. In the press release containing his final statement:
1. He acknowledged the oath-taking of the respondent as President;
2. He emphasized he was leaving the Palace for the sake of peace and in order to begin the
healing process (he did not say that he was leaving due to any kind of disability and that he
was going to reassume the Presidency as soon as the disability disappears);
3. He expressed his gratitude to the people for the opportunity to serve them as President
(without doubt referring to the past opportunity);
4. He assured that he will not shirk from any future challenge that may come in the same
service of the country;

3
EXECUTIVE DEPARTMENT
5. He called on his supporters to join him in promotion of a constructive national spirit of
reconciliation and solidarity.
Intent to resign—must be accompanied by act of relinquishment—act or omission before,
during and after January 20, 2001.
3. The Congress passed House Resolution No. 176 expressly stating its support to Gloria
Macapagal-Arroyo as President of the Republic of the Philippines and subsequently passed
H.R. 178 confirms the nomination of Teofisto T. Guingona Jr. As Vice President. Senate passed
HR No. 83 declaring the Impeachment Courts as Functius Officio and has been terminated. It
is clear is that both houses of Congress recognized Arroyo as the President. Implicitly clear in
that recognition is the premise that the inability of Estrada is no longer temporary as the
Congress has clearly rejected his claim of inability.
The Court therefore cannot exercise its judicial power for this is political in nature and
addressed solely to Congress by constitutional fiat. In fine, even if Estrada can prove that he
did not resign, still, he cannot successfully claim that he is a President on leave on the ground
that he is merely unable to govern temporarily. That claim has been laid to rest by Congress
and the decision that Arroyo is the de jure, president made by a co-equal branch of
government cannot be reviewed by this Court.
4. The cases filed against Estrada are criminal in character. They involve plunder, bribery and
graft and corruption. By no stretch of the imagination can these crimes, especially plunder
which carries the death penalty, be covered by the alleged mantle of immunity of a non-
sitting president. He cannot cite any decision of this Court licensing the President to commit
criminal acts and wrapping him with post-tenure immunity from liability. The rule is that
unlawful acts of public officials are not acts of the State and the officer who acts illegally is
not acting as such but stands in the same footing as any trespasser.
5. No. Case law will tell us that a right to a fair trial and the free press are incompatible. Also,
since our justice system does not use the jury system, the judge, who is a learned and legally
enlightened individual, cannot be easily manipulated by mere publicity. The Court also said
that Estrada did not present enough evidence to show that the publicity given the trial has
influenced the judge so as to render the judge unable to perform. Finally, the Court said that
the cases against Estrada were still undergoing preliminary investigation, so the publicity of
the case would really have no permanent effect on the judge and that the prosecutor should
be more concerned with justice and less with prosecution.

PRESIDENTIAL IMMUNITY:
Presidential Immunity; Impeachment; Since the Impeachment Court is now functus officio, it
is untenable for former President Estrada to demand that he should first be impeached and
then convicted before he can be prosecuted.—We shall now rule on the contentions of
petitioner in the light of this history. We reject his argument that he cannot be prosecuted
for the reason that he must first be convicted in the impeachment proceedings. The
4
EXECUTIVE DEPARTMENT
impeachment trial of petitioner Estrada was aborted by the walkout of the prosecutors and
by the events that led to his loss of the presidency. Indeed, on February 7, 2001, the Senate
passed Senate Resolution No. 83 “Recognizing that the Impeachment Court is Functus
Officio.” Since the Impeachment Court is now functus officio, it is untenable for petitioner to
demand that he should first be impeached and then convicted before he can be prosecuted.
The plea if granted, would put a perpetual bar against his prosecution. Such a submission has
nothing to commend itself for it will place him in a better situation than a non-sitting
President who has not been subjected to impeachment proceedings and yet can be the
object of a criminal prosecution. To be sure, the debates in the Constitutional Commission
make it clear that when impeachment proceedings have become moot due to the resignation
of the President, the proper criminal and civil cases may already be filed against him.

Incumbent Presidents are immune from suit or from being brought to court during the period
of their incumbency and tenure but not beyond.—This is in accord with our ruling in In Re:
Saturnino Bermudez that “incumbent Presidents are immune from suit or from being brought
to court during the period of their incumbency and tenure” but not beyond. Considering the
peculiar circumstance that the impeachment process against the petitioner has been aborted
and thereafter he lost the presidency, petitioner Estrada cannot demand as a condition sine
qua non to his criminal prosecution before the Ombudsman that he be convicted in the
impeachment proceedings. His reliance on the case of Lecaroz vs. Sandiganbayan and related
cases are inapropos for they have a different factual milieu.

By no stretch of the imagination can the crimes of plunder, bribery and graft and corruption,
especially plunder which carries the death penalty, be covered by the alleged mantle of
immunity of a non-sitting president.—We now come to the scope of immunity that can be
claimed by petitioner as a non-sitting President. The cases filed against petitioner Estrada are
criminal in character. They involve plunder, bribery and graft and corruption. By no stretch
of the imagination can these crimes, especially plunder which carries the death penalty, be
covered by the alleged mantle of immunity of a non-sitting president. Petitioner cannot cite
any decision of this Court licensing the President to commit criminal acts and wrapping him
with post-tenure immunity from liability. It will be anomalous to hold that immunity is an
inoculation from liability for unlawful acts and omissions. The rule is that unlawful acts of
public officials are not acts of the State and the officer who acts illegally is not acting as such
but stands in the same footing as any other trespasser.

A critical reading of current literature on executive immunity will reveal a judicial


disinclination to expand the privilege, especially when it impedes the search for truth or
impairs the vindication of a right.—Indeed, a critical reading of current literature on executive
immunity will reveal a judicial disinclination to expand the privilege, especially when it
impedes the search for truth or impairs the vindication of a right. In the 1974 case of US v.
5
EXECUTIVE DEPARTMENT
Nixon, US President Richard Nixon, a sitting President, was subpoenaed to produce certain
recordings and documents relating to his conversations with aids and advisers. Seven
advisers of President Nixon’s associates were facing charges of conspiracy to obstruct justice
and other offenses which were committed in a burglary of the Democratic National
Headquarters in Washington’s Watergate Hotel during the 1972 presidential campaign.
President Nixon himself was named an unindicted co-conspirator. President Nixon moved to
quash the subpoena on the ground, among others, that the President was not subject to
judicial process and that he should first be impeached and removed from office before he
could be made amenable to judicial proceedings. The claim was rejected by the US Supreme
Court. It concluded that “when the ground for asserting privilege as to subpoenaed materials
sought for use in a criminal trial is based only on the generalized interest in confidentiality, it
cannot prevail over the fundamental demands of due process of law in the fair administration
of criminal justice.” In the 1982 case of Nixon v. Fitzgerald, the US Supreme Court further
held that the immunity of the President from civil damages covers only “official acts.”
Recently, the US Supreme Court had the occasion to reiterate this doctrine in the case of
Clinton v. Jones where it held that the US President’s immunity from suits for money damages
arising out of their official acts is inapplicable to unofficial conduct.

Public Officers; The constitutional policies on accountability of public officers—of public


office being of public trust—will be devalued if the Court sustains the claim that a non-sitting
president enjoys immunity from suit for criminal acts committed during his incumbency.—
There are more reasons not to be sympathetic to appeals to stretch the scope of executive
immunity in our jurisdiction. One of the great themes of the 1987 Constitution is that a public
office is a public trust. It declared as a state policy that “(t)he State shall maintain honesty
and integrity in the public service and take positive and effective measures against graft and
corruption.” It ordained that “(p)ublic officers and employees must at all times be
accountable to the people, serve them with utmost responsibility, integrity, loyalty, and
efficiency, act with patriotism and justice, and lead modest lives.” It set the rule that “(t)he
right of the State to recover properties unlawfully acquired by public officials or employees,
from them or from their nominees or transferees, shall not be barred by prescription, laches
or estoppel.” It maintained the Sandiganbayan as an anti-graft court. It created the office of
the Ombudsman and endowed it with enormous powers, among which is to “(investigate on
its own, or on complaint by any person, any act or omission of any public official, employee,
office or agency, when such act or omission appears to be illegal, unjust, improper, or
inefficient.” The Office of the Ombudsman was also given fiscal autonomy. These
constitutional policies will be devalued if we sustain petitioner’s claim that a non-sitting
president enjoys immunity from suit for criminal acts committed during his incumbency.

(APRIL 2001)

6
EXECUTIVE DEPARTMENT
Impeachment; Presidential Immunity; Section 3(7) of Article XI of the Constitution conveys
two uncomplicated ideas—first, it tells us that judgment in impeachment cases has a limited
reach, i.e., it cannot extend further than removal from office and disqualification to hold any
office under the Republic of the Philippines, and second, it tells us the consequence of the
limited reach of a judgment in impeachment proceedings considering its nature, i.e., that the
party convicted shall still be liable and subject to prosecution, trial and punishment according
to law.—Petitioner reiterates the argument that he must be first convicted in the
impeachment proceedings before he could be criminally prosecuted. A plain reading of the
provision will not yield this conclusion. The provision conveys two uncomplicated ideas: first,
it tells us that judgment in impeachment cases has a limited reach . . . i.e., it cannot extend
further than removal from office and disqualification to hold any office under the Republic
of the Philippines, and second, it tells us the consequence of the limited reach of a judgment
in impeachment proceedings considering its nature, i.e., that the party convicted shall still be
liable and subject to prosecution, trial and punishment according to law. No amount of
manipulation will justify petitioner’s non sequitur submission that the provision requires that
his conviction in the impeachment proceedings is a condition sine qua non to his prosecution,
trial and punishment for the offenses he is now facing before the respondent Ombudsman.

Presidential Immunity; Administrative Law; Words and Phrases; “Term” and “Tenure,”
Distinguished; The intent of the framers is clear that the immunity of the president from suit
is concurrent only with his tenure and not his term.—Petitioner, however, fails to distinguish
between term and tenure. The term means the time during which the officer may claim to
hold the office as of right, and fixes the interval after which the several incumbents shall
succeed one another. The tenure represents the term during which the incumbent actually
holds office. The tenure may be shorter than the term for reasons within or beyond the
power of the incumbent. From the deliberations, the intent of the framers is clear that the
immunity of the president from suit is concurrent only with his tenure and not his term.

7
EXECUTIVE DEPARTMENT
3. CIVIL LIBERTIES UNION (presidential privilege)
FACTS:
Petitioners: Ignacio P. Lacsina, Luis R. Mauricio, Antonio R. Quintos and Juan T. David for
petitioners in 83896 and Juan T. David for petitioners in 83815. Both petitions were
consolidated and are being resolved jointly as both seek a declaration of the
unconstitutionality of Executive Order No. 284 issued by President Corazon C. Aquino on July
25, 1987.
Executive Order No. 284, according to the petitioners allows members of the Cabinet, their
undersecretaries and assistant secretaries to hold other than government offices or positions
in addition to their primary positions. The pertinent provisions of EO 284 is as follows:
Section 1: A cabinet member, undersecretary or assistant secretary or other appointive
officials of the Executive Department may in addition to his primary position, hold not more
than two positions in the government and government corporations and receive the
corresponding compensation therefor.
Section 2: If they hold more positions more than what is required in section 1, they must
relinquish the excess position in favor of the subordinate official who is next in rank, but in
no case shall any official hold more than two positions other than his primary position.
Section 3: AT least 1/3 of the members of the boards of such corporation should either be a
secretary, or undersecretary, or assistant secretary.
The petitioners are challenging EO 284’s constitutionality because it adds exceptions to
Section 13 of Article VII other than those provided in the constitution. According to the
petitioners, the only exceptions against holding any other office or employment in
government are those provided in the Constitution namely: 1. The Vice President may be
appointed as a Member of the Cabinet under Section 3 par.2 of Article VII. 2. The secretary
of justice is an ex-officio member of the Judicial and Bar Council by virtue of Sec. 8 of article
VIII.
Issue:
Whether or not Executive Order No. 284 is constitutional.
Decision:
No. It is unconstitutional. Petition granted. Executive Order No. 284 was declared null and
void.
Ratio:
In the light of the construction given to Section 13 of Article VII, Executive Order No. 284 is
unconstitutional. By restricting the number of positions that Cabinet members,
undersecretaries or assistant secretaries may hold in addition their primary position to not
more than two positions in the government and government corporations, EO 284 actually
allows them to hold multiple offices or employment in direct contravention of the express
8
EXECUTIVE DEPARTMENT
mandate of Sec. 13 of Article VII of the 1987 Constitution prohibiting them from doing so,
unless otherwise provided in the 1987 Constitution itself.
The phrase “unless otherwise provided in this constitution” must be given a literal
interpretation to refer only to those particular instances cited in the constitution itself: Sec.
3 Art VII and Sec. 8 Art. VIII.

The intent of the framers of the Constitution was to impose a stricter prohibition on the
President and his official family in so far as holding other offices or employment in the
government or elsewhere is concerned.—Evidently, from this move as well as in the different
phraseologies of the constitutional provisions in question, the intent of the framers of the
Constitution was to impose a stricter prohibition on the President and his official family in so
far as holding other offices or employment in the government or elsewhere is concerned.

Such intent is underscored by a comparison of Section 13, Article VII when other provisions
of the Constitution on the disqualifications of certain public officials or employees from
holding other offices or employment.—Moreover, such intent is underscored by a
comparison of Section 13, Article VII with other provisions of the Constitution on the
disqualifications of certain public officials or employees from holding other offices or
employment. Under Section 13, Article VI, “(N)o Senator or Member of the House of
Representatives may hold any other office or employment in the Government . . .”. Under
Section 5(4), Article XVI, “(N)o member of the armed forces in the active service shall, at any
time, be appointed in any capacity to a civilian position in the Government, including
government-owned or controlled corporations or any of their subsidiaries.” Even Section 7
(2), Article IX-B, relied upon by respondents provides “(U)nless otherwise allowed by law or
by the primary functions of his position, no appointive official shall hold any other office or
employment in the Government.”

The prohibition imposed on the President and his official family is therefore all-embracing
and covers both public and private office or employment.—It is quite notable that in all these
provisions on disqualifications to hold other office or employment, the prohibition pertains
to an office or employment in the government and government-owned or controlled
corporations or their subsidiaries. In striking contrast is the wording of Section 13, Article VII
which states that “(T)he President, Vice-President, the Members of the Cabinet, and their
deputies or assistants shall not, unless otherwise provided in this Constitution, hold any other
office or employment during their tenure.” In the latter provision, the disqualification is
absolute, not being qualified by the phrase “in the Government.” The prohibition imposed
on the President and his official family is therefore all-embracing and covers both public and
private office or employment.

9
EXECUTIVE DEPARTMENT
The all-embracing prohibition imposed on the President and his official family are proof of
the intent of the 1987 Constitution to treat them as a class by itself and to impose upon said
class stricter prohibitions.—Going further into Section 13, Article VII, the second sentence
provides: “They shall not, during said tenure, directly or indirectly, practice any other
profession, participate in any business, or be financially interested in any contract with, or in
any franchise, or special privilege granted by the Government or any subdivision, agency or
instrumentality thereof, including government-owned or controlled corporations or their
subsidiaries.” These sweeping, all-embracing prohibitions imposed on the President and his
official family, which prohibitions are not similarly imposed on other public officials or
employees such as the Members of Congress, members of the civil service in general and
members of the armed forces, are proof of the intent of the 1987 Constitution to treat the
President and his official family as a class by itself and to impose upon said class stricter
prohibitions.

While all other appointive officials in the civil service are allowed to hold other office or
employment in the government during their tenure when such is allowed by law or by the
primary functions of their positions, members of the Cabinet, their deputies and assistants
may do so only when expressly authorized by the Constitution itself.—Thus, while all other
appointive officials in the civil service are allowed to hold other office or employment in the
government during their tenure when such is allowed by law or by the primary functions of
their positions, members of the Cabinet, their deputies and assistants may do so only when
expressly authorized by the Constitution itself. In other words, Section 7, Article IX-B is meant
to lay down the general rule applicable to all elective and appointive public officials and
employees, while Section 13, Article VII is meant to be the exception applicable only to the
President, the Vice-President, Members of the Cabinet, their deputies and assistants.
Executive Order No. 284 is unconstitutional as it allows Cabinet members, undersecretaries
or assistant secretaries to hold multiple offices or employment in direct contravention of the
express mandate of Section 13, Article VII of the 1987 Constitution prohibiting them from
doing so.—In the light of the construction given to Section 13, Article VII in relation to Section
7, par. (2), Article IX-B of the 1987 Constitution, Executive Order No. 284 dated July 23, 1987
is unconstitutional. Ostensibly restricting the number of positions that Cabinet members,
undersecretaries or assistant secretaries may hold in addition to their primary position to not
more than two (2) positions in the government and government corporations, Executive
Order No. 284 actually allows them to hold multiple offices or employment in direct
contravention of the express mandate of Section 13, Article VII of the 1987 Constitution
prohibiting them from doing so, unless otherwise provided in the 1987 Constitution itself.

10
EXECUTIVE DEPARTMENT
4. TECSON VS COMELEC (presidential privilege)
FACTS:
Petitioners sought for respondent Poe’s disqualification in the presidential elections for
having allegedly misrepresented material facts in his (Poe’s) certificate of candidacy by
claiming that he is a natural Filipino citizen despite his parents both being foreigners. Comelec
dismissed the petition, holding that Poe was a Filipino Citizen. Petitioners assail the
jurisdiction of the Comelec, contending that only the Supreme Court may resolve the basic
issue on the case under Article VII, Section 4, paragraph 7, of the 1987 Constitution.
ISSUES:
1. Whether or not it is the Supreme Court which had jurisdiction.
2. Whether or not Comelec committed grave abuse of discretion in holding that Poe
was a Filipino citizen.

RULING:
1.) The Supreme Court had no jurisdiction on questions regarding “qualification of a
candidate” for the presidency or vice-presidency before the elections are held.
"Rules of the Presidential Electoral Tribunal" in connection with Section 4, paragraph 7, of
the 1987 Constitution, refers to “contests” relating to the election, returns and qualifications
of the "President" or "Vice-President", of the Philippines which the Supreme Court may take
cognizance, and not of "candidates" for President or Vice-President before the elections.
2.) Comelec committed no grave abuse of discretion in holding Poe as a Filipino Citizen.
The 1935 Constitution on Citizenship, the prevailing fundamental law on respondent’s birth,
provided that among the citizens of the Philippines are "those whose fathers are citizens of
the Philippines."
Tracing respondent’s paternal lineage, his grandfather Lorenzo, as evidenced by the latter’s
death certificate was identified as a Filipino Citizen. His citizenship was also drawn from the
presumption that having died in 1954 at the age of 84, Lorenzo would have been born in
1870. In the absence of any other evidence, Lorenzo’s place of residence upon his death in
1954 was presumed to be the place of residence prior his death, such that Lorenzo would
have benefited from the "en masse Filipinization" that the Philippine Bill had effected in
1902. Being so, Lorenzo’s citizenship would have extended to his son, Allan---respondent’s
father.
Respondent, having been acknowledged as Allan’s son to Bessie, though an American citizen,
was a Filipino citizen by virtue of paternal filiation as evidenced by the respondent’s birth
certificate. The 1935 Constitution on citizenship did not make a distinction on the legitimacy

11
EXECUTIVE DEPARTMENT
or illegitimacy of the child, thus, the allegation of bigamous marriage and the allegation that
respondent was born only before the assailed marriage had no bearing on respondent’s
citizenship in view of the established paternal filiation evidenced by the public documents
presented.
But while the totality of the evidence may not establish conclusively that respondent FPJ is a
natural-born citizen of the Philippines, the evidence on hand still would preponderate in his
favor enough to hold that he cannot be held guilty of having made a material
misrepresentation in his certificate of candidacy in violation of Section 78, in relation to
Section 74 of the Omnibus Election Code.
The jurisdiction of the Supreme Court defined by Sec. 4, par. 7, of the 1987 Constitution, does
not include cases directly brought before it questioning the qualifications of a candidate for
the presidency or vice-presidency before the elections are held; A quo warranto proceeding
is generally defined as being an action against a person who usurps, intrudes into, or
unlawfully holds or exercises a public office.—The rules categorically speak of the jurisdiction
of the tribunal over contests relating to the election, returns and qualifications of the
“President” or “Vice-President”, of the Philippines, and not of “candidates” for President or
Vice-President. A quo warranto proceeding is generally defined as being an action against a
person who usurps, intrudes into, or unlawfully holds or exercises a public office. In such
context, the election contest can only contemplate a post-election scenario. In Rule 14, only
a registered candidate who would have received either the second or third highest number
of votes could file an election protest. This rule again presupposes a post-electionscenario. It
is fair to conclude that the jurisdiction of the Supreme Court, defined by Section 4, paragraph
7, of the 1987 Constitution, would not include cases directly brought before it questioning
the qualifications of a candidate for the presidency or vice-presidency before the elections
are held.
Election Law; Cancellation of Certificates of Candidacy; While the totality of the evidence may
not establish conclusively that Fernando Poe, Jr. is a natural-born citizen of the Philippines,
the evidence on hand still would preponderate in his favor enough to hold that he cannot be
held guilty of having made a material misrepresentation in his certificate of candidacy in
violation of Section 78, in relation to Section 74, of the Omnibus Election Code.—But while
the totality of the evidence may not establish conclusively that respondent FPJ is a natural-
born citizen of the Philippines, the evidence on hand still would preponderate in his favor
enough to hold that he cannot be held guilty of having made a material misrepresentation in
his certificate of candidacy in violation of Section 78, in relation to Section 74, of the Omnibus
Election Code. Petitioner has utterly failed to substantiate his case before the Court,
notwithstanding the ample opportunity given to the parties to present their position and
evidence, and to prove whether or not there has been material misrepresentation, which, as
so ruled in Romualdez-Marcos vs. COMELEC, must not only be material, but also deliberate
and willful.

12
EXECUTIVE DEPARTMENT
5. BRILLANTES VS COMELEC (presidential privilege)
FACTS:
Comelec issued resolutions adopting an Automated Elections System including the assailed
resolution, Resolution 6712, which provides for the electronic transmission of advanced
result of “unofficial” count. Petitioners claimed that the resolution would allow the
preemption and usurpation of the exclusive power of Congress to canvass the votes for
President and Vice-President and would likewise encroach upon the authority of NAMFREL,
as the citizens’ accredited arm, to conduct the "unofficial" quick count as provided under
pertinent election laws. Comelec contended that the resolution was promulgated in the
exercise of its executive and administrative power "to ensure free, orderly, honest, peaceful
and credible elections” Comelec added that the issue is beyond judicial determination.

ISSUE:
Whether or not Comelec's promulgation of Resolution 6712 was justified.

RULING:
The Comelec committed grave abuse of discretion amounting to lack or excess of jurisdiction
in issuing Resolution 6712. The issue squarely fell within the ambit of the expanded
jurisdiction of the court.
Article VII, Section 4 of the Constitution, further bolstered by RA 8436, vest upon Congress
the sole and exclusive authority to officially canvass the votes for the elections of President
and Vice-President. Section 27 of Rep. Act No. 7166, as amended by Rep. Act No. 8173, and
reiterated in Section 18 of Rep. Act No. 8436, solely authorize NAMFREL, the duly-accredited
citizen’s arm to conduct the “unofficial counting of votes for the national or local elections.
The quick count under the guise of an “unofficial” tabulation would not only be preemptive
of the authority of congress and NAMFREL, but would also be lacking constitutional and/or
statutory basis. Moreover, the assailed COMELEC resolution likewise contravened the
constitutional provision that "no money shall be paid out of the treasury except in pursuance
of an appropriation made by law." It being “unofficial”, any disbursement of public fund
would be contrary to the provisions of the Constitution and Rep. Act No. 9206, which is the
2003 General Appropriations Act.
The Omnibus Election Code in providing the powers and functions of the Commission
subjects the same to certain conditions with respect to the adoption of the latest
technological and electronic devices, to wit: (1) consideration of the area and available funds
(2) notification to all political parties and candidates. The aforementioned conditions were
found to have not been substantially met.

13
EXECUTIVE DEPARTMENT
Resolution 6712 was null and void.
Political questions are concerned with “issues dependent upon the wisdom, not legality of a
particular measure.”—The Court does not agree with the posture of the respondent
COMELEC that the issue involved in the present petition is a political question beyond the
jurisdiction of this Court to review. As the leading case of Tañada vs. Cuenco put it, political
questions are concerned with “issues dependent upon the wisdom, not legality of a particular
measure.”
The assailed resolution usurps x x x the sole and exclusive authority of Congress to canvass
the votes for the election of President and Vice-President.—The assailed resolution usurps,
under the guise of an “unofficial” tabulation of election results based on a copy of the
election returns, the sole and exclusive authority of Congress to canvass the votes for the
election of President and Vice-President.
Contention of the COMELEC that its tabulation of votes is not prohibited by the Constitution
and Rep. Act No. 8436 as such tabulation is “unofficial,” is puerile and totally unacceptable.—
The contention of the COMELEC that its tabulation of votes is not prohibited by the
Constitution and Rep. Act No. 8436 as such tabulation is “unofficial,” is puerile and totally
unacceptable. If the COMELEC is proscribed from conducting an official canvass of the votes
cast for the President and Vice-President, the COMELEC is, with more reason, prohibited from
making an “unofficial” canvass of said votes

14
EXECUTIVE DEPARTMENT
6. SANTIAGO VS RAMOS (presidential privilege)
FACTS:
The protestant, Miriam Defensor-Santiago ran for presidency and lost in the May 1992
election. In her Motion on the 16th day of August in the year 1995, reiterated in her comment
of the 29th of August of the same year, protestant Defensor-Santiago prayed that the revision
in the remaining precincts of the pilot areas be dispensed with and the revision process in
the pilot areas be deemed computed.

The Court deferred action on the motion and required, instead, the protestant and protestee
to submit their respective memoranda. Hence, this petition.

ISSUE:
Whether or not the election protest filed by Defensor-Santiago is moot and academic by her
election as a Senator in the May 1995 election and her assumption of office as such on the
30th of June in the year 1995.

RULING:
YES. The Court held that the election protest filed by Santiago has been abandoned or
considered withdrawn as a consequence of her election and assumption of office as Senator
and her discharge of the duties and functions thereof.
The term of office of the Senators elected in the 8 May 1995 election is six years, the first
three of which coincides with the last three years of the term of the President elected in the
11 May 1992 synchronized elections. The latter would be Protestant Santiago’s term if she
would succeed in proving in the instant protest that she was the true winner in the 1992
elections. In assuming the office of Senator then, the Protestant has effectively abandoned
or withdrawn this protest, or at the very least, in the language of Moraleja, abandoned her
“determination to protect and pursue the public interest involved in the matter of who is the
real choice of the electorate.” Such abandonment or withdrawal operates to render moot
the instant protest. Moreover, the dismissal of this protest would serve public interest as it
would dissipate the aura of uncertainty as to the results of the 1992 presidential election,
thereby enhancing the all—too crucial political stability of the nation during this period of
national recovery.
The protestant abandoned her “determination to protest and pursue the public interest
involved in the matter of who is the real choice of the electorate.
Moreover, the dismissal of this protest would serve public interest as it would dissipate the
aura of uncertainty as to the results of the 1992 presidential elections, thereby enhancing
the all too crucial political stability of the nation during this period of national recovery.
15
EXECUTIVE DEPARTMENT
Also, the PET issued a resolution ordering the protestant to inform the PET within 10 days if
after the completion of the revision of the ballots from her pilot areas, she still wishes to
present evidence. Since DS has not informed the Tribunal of any such intention, such is a
manifest indication that she no longer intends to do so.
Election Law; Statutes; B.P. Blg. 881; Section 67 of B.P. Blg. 881 applies exclusively to an
incumbent elective official who files a certificate of candidacy for any office “other than the
one he is holding in a permanent capacity.”—We cannot subscribe to the view of the
Protestee that by filing her certificate of candidacy for Senator Protestant Santiago ipso facto
forfeited her claim to the office of President pursuant to Section 67 of B.P. Blg. 881. Plainly,
the said section applies exclusively to an incumbent elective official who files a certificate of
candidacy for any office “other than the one he is holding in a permanent capacity.” Even
more plain is that the Protestant was not the incumbent President at the time she filed her
certificate of candidacy for Senator nor at any time before that. Thus, the holding in
Dimaporo does not apply to the Protestant.
Election Contests; Protestant, in filing her certificate of candidacy for the Senate without any
qualification, condition, or reservation, entered into a political contract with the electorate
that if elected, she would assume the office of Senator, discharge its functions and serve her
constituency as such for the term for which she was elected.—Indeed, it would be entirely
different where the protestant pursued the new position through a popular election, as in
the case of Protestant Santiago who filed a certificate of candidacy for Senator in the 8 May
1995 election, campaigned for such office, and submitted herself to be voted upon. She knew
that the term of office of the Senators who would then be elected would be six years, to
commence at noon on the thirtieth day of June next following their election and to end at
noon of 30 June 2001. Knowing her high sense of integrity and candor, it is most unlikely that
during her campaign, she promised to serve the electorate as Senator, subject to the
outcome of this protest. In short, she filed her certificate of candidacy for the Senate without
any qualification, condition, or reservation. In so doing, she entered into a political contract
with the electorate that if elected, she would assume the office of Senator, discharge its
functions and serve her constituency as such for the term for which she was elected. These
are givens which are in full accord with the principle enshrined in the Constitution that public
office is a public trust, and public officers and employees must at all times be accountable to
the people and serve them with utmost responsibility, integrity, loyalty and efficiency.

Abandonment; In assuming the office of Senator, the Protestant has effectively abandoned
or withdrawn her protest, or at the very least, abandoned her, “determination to protect and
pursue the public interest involved in the matter of who is the real choice of the electorate,”
such abandonment or withdrawal operating to render moot the protest.—The term of office
of the Senators elected in the 8 May 1995 election is six years, the first three of which
coincides with the last three years of the term of the President elected in the 11 May 1992
synchronized elections. The latter would be Protestant Santiago’s term if she would succeed
16
EXECUTIVE DEPARTMENT
in proving in the instant protest that she was the true winner in the 1992 elections. In
assuming the office of Senator then, the Protestant has effectively abandoned or withdrawn
this protest, or at the very least, in the language of Moraleja, abandoned her “determination
to protect and pursue the public interest involved in the matter of who is the real choice of
the electorate.” Such abandonment or withdrawal operates to render moot the instant
protest. Moreover, the dismissal of this protest would serve public interest as it would
dissipate the aura of uncertainty as to the results of the 1992 presidential election, thereby
enhancing the all—too crucial political stability of the nation during this period of national
recovery.

Grounds for the Dismissal of Election Protests Before the Presidential Electoral Tribunal.—It
must also be stressed that under the Rules of the Presidential Electoral Tribunal, an election
protest may be summarily dismissed, regardless of the public policy and public interest
implications thereof, on the following grounds:

(1) The petition is insufficient in form and substance;


(2) The petition is filed beyond the periods provided in Rules 14 and 15 hereof;
(3) The filing fee is not paid within the periods provided for in these Rules;
(4) The cash deposit, or the first P100,000.00 thereof, is not paid within 10 days after the
filing of the protest; and
(5) The petition or copies thereof and the annexes thereto filed with the Tribunal are not
clearly legible.
Other grounds for a motion to dismiss, e.g., those provided in the Rules of Court which apply
in a suppletory character, may likewise be pleaded as affirmative defenses in the answer.
After which, the Tribunal may, in its discretion, hold a preliminary hearing on such grounds.
In sum, if an election protest may be dismissed on technical grounds, then it must be, for a
decidedly stronger reason, if it has become moot due to its abandonment by the Protestant.
The failure of the Protestant to inform the PET whether, after the completion of the revision
of the ballots from her pilot areas, she still intends to present evidence in connection
therewith is nothing short of a manifest indication that she no longer intends to do so.—Until
the present, however, the Protestant has not informed the Tribunal whether after the
completion of the revision of the ballots from her pilot areas, she still intends to present
evidence in connection therewith. This failure then, is nothing short of a manifest indication
that she no longer intends to do so. It is entirely irrelevant at this stage of the proceedings
that the Protestant’s revisors discovered in the course of the revisions alleged irregularities
in 13,510 out of the 17,525 contested precincts in the pilot areas and have objected to
thousands of ballots cast in favor of the Protestee.

17
EXECUTIVE DEPARTMENT
The protest “has been rendered moot and academic by its abandonment or withdrawal by
the Protestant as a consequence of her election and assumption of office as Senator and her
discharge of the duties and functions thereof.”—The dispositive portion of this resolution
leaves no room for any doubt or miscomprehension that the dismissal is based on the ground
that the protest “has been rendered moot and academic by its abandonment or withdrawal
by the Protestant as a consequence of her election and assumption of office as Senator and
her discharge of the duties and functions thereof.” There is, therefore, ONLY ONE reason or
ground why the protest has been rendered moot and academic, i.e., it has been abandoned
or withdrawn. This was the very issue upon which the parties were required, in the resolution
of 26 September 1995, to submit their respective memoranda

18
EXECUTIVE DEPARTMENT
1. PLANAS VS GIL (Executive and administrative powers in general)

FACTS:

Petitioner, a member of the municipal board of the City of Manila, criticized the acts
of certain government officials in connection with the general election for Assemblymen held
on November 8, 1938 in one of the local dailies. The statement as published in the issue of
La Vanguardia of November 17, 1938, included the following statements:

“… In Manila, the opposition should have won the November 8 elections, but lost
instead because of a disastrous
division due to people who commercialized their candidacies.
"The Constitution prohibits the reelection of the President precisely so that the Presi
dent maydevote all his time to the administration of public affairs for the welfare of the peo
ple, but thePresident was the first to play politics. Publicly expressing his preference for
candidates of his liking; and with the President all other officials of the government also
moved, taking part in electoral campaigns.
"With the government machinery feverishly functioning to flatten the opposition and
preventcandidates supported by the people from going to the National Assembly, and with
frauds and violations of all rules of the civil service to push to victory the candidates of the
Nacionalista Party and the administration, all constructive opposition in the country is useless
...
"… It is reasonable to believe
that the President is from this moment paving the way for his reelection. It is to be feared
that the new National Assembly will change this wise provision of our Constitution to permit
the reelection of President Manuel L. Quezon."

The day following the publication of the foregoing statement, the petitioner received
a letter, where she is informed that she needs to appear before the Commissioner of Civil
Service to prove her statements otherwise she may be suspended or removed from office.

At the appointed time, the petitioner, accompanied by her counsel, appeared at the
office of the respondent and delivered to him a letter, Annex B, in which she voiced objection
to the authority of the respondent to conduct the investigation. The respondent
Commissioner did not desist from proceeding with the investigation, but announced before
adjourning the hearing of November 22nd that he would decide the question raised as to his
jurisdiction on November 26, 1938.

It was at this state of the investigation that the petitioner filed in this court her original
petition for prohibition of November 25, 1938, in which she at the same time prayed for the
issuance of a writ of preliminary injunction enjoining the respondent commissioner from
continuing with the investigation.
19
EXECUTIVE DEPARTMENT
ISSUES:

WON the courts have jurisdiction over this case


WON the president has the power to order the investigation
WON the investigation is in accordance with the law

RULING:

1. Yes.

In the present case, the President is not a party to the proceeding. The Commissioner
of Civil Service is the party respondent and the theory is advanced by the Government that
because an investigation undertaken by him is directed by authority of the President of the
Philippines, this court has no jurisdiction over the present proceedings instituted by the
petitioner, Carmen Planas. The argument is farfetched. A mere plea that a subordinate officer
of the government is acting under orders from the Chief Executive may be an important
averment, but is neither decisive nor conclusive upon this court. Like the dignity of his high
office, the relative immunity of the Chief Executive from judicial interference is not in the
nature of a sovereign passport for all the subordinate officials and employees of the
Executive Department to the extent that at the mere invocation of the authority that it
purports the jurisdiction of this court to inquire into the validity or legality of an executive
order is necessarily abated or suspended.

Not infrequently, "the writ is granted, where it is necessary for the orderly
administration of justice, or to prevent the use of the strong arm of the law in an oppressive
or vindictive manner, or a multiplicity of actions." This court, therefore, has jurisdiction over
the instant proceedings and will accordingly proceed to determine the merits of the present
controversy.

2. Yes.

It is not denied that the President did authorize the issuance of the order, but it is
contended "that the said investigation with a view to petitioner's suspension or removal is
against Article VII, sec. 11 (1) of the Constitution of the Philippines and is not warranted by
any statutory provision." (Par. XV [b], amended petition.) It, therefore, becomes necessary
to inquire into the constitutional and legal authority of the President to order the
investigation which has given rise to the present controversy.

A perusal of our Constitution will show that extensive authority over the public service
is granted the President of the Philippines. Article VII of the Constitution begins in its section
1 with the declaration that "The Executive power shall be vested in a President of the
20
EXECUTIVE DEPARTMENT
Philippines." All executive authority is thus vested in him, and upon him devolves the
constitutional duty of seeing that the laws are "faithfully executed." (Art. VIII, sec. 11, subsec.
1, last clause.) In the fulfillment of this duty which he cannot evade, he is granted specific and
express powers and functions. (Art. VII, sec. 11.) In addition to these specific and express
powers and functions, he may also exercise those necessarily implied and included in them.
The National Assembly may not enact laws which either expressly or impliedly diminish the
authority conferred upon the President of the Constitution. The Constitution provides that
the President "shall have control of all the executive departments, bureaus, and offices" (Art.
VII, sec. 11 [1], first clause) and shall "exercise general supervision over all local governments
as may be provided by law" (Ibid, second clause). This power of control and supervision is an
important constitutional grant. The President in the exercise of the executive power under
the Constitution may act through the heads of the executive departments.

Independently of any statutory provision authorizing the President to conduct an


investigation of the nature involved in this proceeding, and in view of the nature and
character of the executive authority with which the President of the Philippines is invested,
the constitutional grant to him of power to exercise general supervision over all local
governments and to take care that the laws be faithfully executed must be construed to
authorize him to order an investigation of the act or conduct, of the petitioner herein.
Supervision is not a meaningless thing. It is an active power. It is certainly not without
limitation, but it at least implies authority to inquire into facts and conditions in order to
render the power real and effective. If supervision is to be conscientious and rational, and
not automatic and brutal, it must be founded upon a knowledge of actual facts and
conditions disclosed after careful study and investigation.

Viewed from the totality of powers conferred upon the Chief Executive by our
Constitution, we should be reluctant to yield to the proposition that the President of the
Philippines who is endowed with broad and extraordinary powers by our Constitution, and
who is expected to govern with a firm and steady hand without vexatious or embarrassing
interference and much less dictation from any source, is yet devoid of the power to order the
investigation of the petitioner in this case. We should avoid that result.

The deliberations of the Constitutional Convention show that the grant of the
supervisory authority to Chief Executive in this regard was in the nature of a compromise
resulting from the conflict of views in that body, mainly between the historical view which
recognizes the right of local self-government and the legal theory which sanctions the
possession by the state of absolute control over local governments. The result was the
recognition of the power of supervision and all its implications and the rejection of what
otherwise would be an imperium in imperio to the detriment of a strong national
government.

Apart from the constitutional aspect, we find that section 64 of the Administrative
Code of 1917 provides as follows:
21
EXECUTIVE DEPARTMENT
"In addition to his general supervisory authority, the Governor-General (President)
shall have such specific powers and duties as are expressly conferred or imposed on him by
law and also, in particular, the powers and duties set forth in this chapter.
"Among such special powers and duties shall be:
*******
"(c) To order, when in his opinion the good of the public service so requires, an
investigation of any action or the conduct of any person in the Government service, and in
connection therewith to designate the official, committee, or person by whom such
investigation shall be conducted."

This provision of the law, in existence before the taking effect of the Constitution, still
subsists. It is not inconsistent with the Constitution and has not been abrogated or repealed
by the National Assembly. (See sec. 2, Art. XV, Constitution.)

3. Yes.

The interest of the public service requires that these charges be investigated, so that,
if found to be true, appropriate action may be taken against the parties alleged to have been
guilty of illegal acts, and if found untrue and made without justifiable motives, the party
making them may be proceeded against in accordance with section 2440, in connection, with
section, 2078, of the Revised Administrative Code." Assuming that this is not one of the
grounds provided by law for which the petitioner may be investigated administratively (sec.
2078, Rev. Adm. Code), there is weight in the argument that the investigation would still be
in order if for no other purpose than to cause a full and honest disclosure of all the facts so
that, if found proper and justified, appropriate action may be taken against the parties
alleged to have been guilty of the illegal acts charged. The enforcement of the law and the
maintenance of peace and order are primarily an executive obligation. The declaration that
the President should "take care that the laws be faithfully executed" is more an imposition
of an obligation than a conferment of power. His oath requires him to "faithfully and
conscientiously fulfill" his duties as President, "preserve and defend" the Constitution and
"execute" the law. This duty of the Executive to see that the laws be faithfully executed is not
limited to the enforcement of legislative acts or the express terms of the Constitution but
also includes the due enforcement of rights, duties, obligations, prerogatives and immunities
growing out of the Constitution itself and of the protection implied by the nature of the
government under the Constitution.

We are vigilantly alive to the necessity of maintaining and protecting the constitutional
guaranty of freedom of speech and of the press, no less than the right of assembly and
petition which, according to Stimson (The American Constitution As It Protects Private Rights,
152), is its origin rather than its derivation. In the present case, however, the petitioner is not
denied the right, nor is she being investigated because she had exercised that right. She has
22
EXECUTIVE DEPARTMENT
a perfect right to criticize the Government, its administration, its policies and officials, but
she may not, on the plea of freedom of speech and of the press, impute violations of law and
the commission of frauds and thereafter fold her arms and decline to face an investigation
conducted to elicit the truth or falsity of the charges formulated by her. Otherwise, the
guarantee which, in the language of Wendell Phillips, is "at once the instrument, and the
guarantee, and the bright consummate flower of all liberty" would degenerate into an
unbridled license, and render the Government powerless to act.

23
EXECUTIVE DEPARTMENT
2. MYERS VS UNITED STATES (Executive and administrative powers in general)
Brief Fact Summary. Appointee to the postmaster of the first class in Oregon was forced to
resign.
Synopsis of Rule of Law. The Constitution “grants to the President the executive power of
the government- i.e., the general administrative control of those executing the laws,
including the power of appointment and removal of executive officers-a conclusion
confirmed by his obligation to take care that the laws be faithfully executed; that article 2
excludes the exercise of legislative power by Congress to provide for appointments and
removals, except only as granted therein to Congress in the matter of inferior offices; that
Congress is only given power to provide for appointments and removals of inferior officers
after it has vested, and on condition that it does vest, their appointment in other authority
than the President with the Senate’s consent; that the provisions of the second section of
article 2, which blend action by the legislative branch, or by part of it, in the work of the
executive, are limitations to be strictly construed, and not to be extended by implication; that
the President’s power of removal is further established as an incident to his specifically
enumerated function of appointment by and with the advice of the Senate, but that such
incident does not by implication extend to removals the Senate’s power of checking
appointments.”
Facts. Under an 1876 rule, the President had to get the Senate’s permission to remove the
postmaster of Portland, Oregon. That individual had been appointed with the Senate’s advice
and consent. The President asked for the individual’s resignation without consulting the
Senate first, and the Senate refused the President permission to do so.
Issue. “[W]hether under the Constitution the President has the exclusive power of removing
executive officers of the United States whom he has appointed by and with the advice and
consent of the Senate.”
Held. Yes. The Supreme Court of the United States (the Supreme Court) produced a long-
winded opinion, examining the legislative and adjudicative history of executive
appointments, including Marbury v. Madison. It concluded that Tenure of Office Act of 1867,
“in so far as it attempted to prevent the President from removing executive officers who had
been appointed by him by and with the advice and consent of the Senate, was invalid, and
that subsequent legislation of the same effect was equally so.” Dissent. Justice McReynolds
found that “it is impossible for me to accept the view that the President may dismiss, as
caprice may suggest, any inferior officer whom he has appointed with consent of the Senate,
notwithstanding a positive inhibition by Congress after his own lengthy review of precedent.
Justice Brandeis felt that the central issue was “May the President, having acted under the
statute in so far as it creates the office and authorizes the appointment, ignore, while the
Senate is in session, the provision which prescribes the condition under which a removal may
take place?” Justice Holmes emphasized the fact that the office was created by Congress.
Discussion. “To hold [an opposite rule] would make it impossible for the President, in case of
political or other difference with the Senate or Congress, to take care that the laws be
faithfully executed.”

24
EXECUTIVE DEPARTMENT
3. FLORES VS DRILON (Power of appointment; discretionary)
FACTS:
The constitutionality of Sec. 13, par. (d), of R.A. 7227, otherwise known as the "Bases
Conversion and Development Act of 1992," under which respondent Mayor Richard J.
Gordon of Olongapo City was appointed Chairman and Chief Executive Officer of the Subic
Bay Metropolitan Authority (SBMA), is challenged with prayer for prohibition, preliminary
injunction and temporary restraining order. Said provision provides the President has the
power to appoint an administrator of the SBMA provided that in the first year of its operation,
the Olongapo mayor shall be appointed as chairman and chief of executive of the Subic
Authority. Petitioners maintain that such infringes to the constitutional provision of Sec. 7,
first par., Art. IX-B, of the Constitution, which states that "no elective official shall be eligible
for appointment or designation in any capacity to any public officer or position during his
tenure," The petitioners also contend that Congress encroaches upon the discretionary
power of the President to appoint.

ISSUE:
Whether or not said provision of the RA 7227 violates the constitutional prescription
against appointment or designation of elective officials to other government posts.

RULING:
The said provision is unconstitutional.
In the case before us, the subject proviso directs the President to appoint an elective
official, i.e., the Mayor of Olongapo City, to other government posts (as Chairman of the
Board and Chief Executive Officer of SBMA). Since this is precisely what the constitutional
proscription seeks to prevent, it needs no stretching of the imagination to conclude that the
proviso contravenes Sec. 7, first par., Art. IX-B, of the Constitution. Here, the fact that the
expertise of an elective official may be most beneficial to the higher interest of the body
politic is of no moment. It is argued that Sec. 94 of the Local Government Code (LGC) permits
the appointment of a local elective official to another post if so allowed by law or by the
primary functions of his office. But, the contention is fallacious. Section 94 of the LGC is not
determinative of the constitutionality of Sec. 13, par. (d), of RA 7227, for no legislative act
can prevail over the fundamental law of the land. Moreover, since the constitutionality of
Sec. 94 of LGC is not the issue here nor is that section sought to be declared unconstitutional,
we need not rule on its validity. Neither can we invoke a practice otherwise unconstitutional
as authority for its validity.
It is further argued that the SBMA posts are merely ex officio to the position of Mayor
of Olongapo City, hence, an excepted circumstance, citing Civil Liberties Union v. Executive
Secretary, where we stated that the prohibition against the holding of any other office or
employment by the President, Vice-President, Members of the Cabinet, and their deputies
or assistants during their tenure, as provided in Sec. 13, Art. VII, of the Constitution, does not
comprehend additional duties and functions required by the primary functions of the officials
concerned, who are to perform them in an ex officio capacity as provided by law, without
25
EXECUTIVE DEPARTMENT
receiving any additional compensation therefor. This argument is apparently based on a
wrong premise. Congress did not contemplate making the subject SBMA posts as ex officio
or automatically attached to the Office of the Mayor of Olongapo City without need of
appointment. The phrase “shall be appointed” unquestionably shows the intent to make the
SBMA posts appointive and not merely adjunct to the post of Mayor of Olongapo City. Had it
been the legislative intent to make the subject positions ex officio, Congress would have, at
least, avoided the word “appointed” and, instead, “ex officio” would have been used.

Appointing Power; The appointing power has the right of choice which he may exercise
freely according to his judgment, deciding for himself who is best qualified among those who
have the necessary qualifications and eligibilities.—Considering that appointment calls for a
selection, the appointing power necessarily exercises a discretion. According to Woodbury,
J., “the choice of a person to fill an office constitutes the essence of his appointment,” and
Mr. Justice Malcolm adds that an “[a]ppointment to office is intrinsically an executive act
involving the exercise of discretion.” In Pamantasan ng Lungsod ng Maynila v. Intermediate
Appellate Court we held: The power to appoint is, in essence, discretionary. The appointing
power has the right of choice which he may exercise freely according to his judgment,
deciding for himself who is best qualified among those who have the necessary qualifications
and eligibilities. It is a prerogative of the appointing power x x x x”

26
EXECUTIVE DEPARTMENT
4. LUEGO VS CIVIL SERVICE COMMISSION (Power of appointment; discretionary)

FACTS:

Petitioner was appointed Administrative Officer II, Office of the City Mayor, Cebu City,
by Mayor Florentino Solon on 18 February 1983. The appointment was described as
“permanent” but the Civil Service Commission approved it as “temporary.” On 22 March
1984, the Civil Service Commission found the private respondent better qualified than the
petitioner for the contested position and accordingly directed herein private respondent in
place of petitioner’s position. The private respondent was so appointed on 28 June 1984, by
the new mayor, Mayor Ronald Duterte. The petitioner is now invoking his earlier permanent
appointment as well as to question the Civil Service Commission’s order and the private
respondent’s title.

ISSUE:

Whether or not the Civil Service Commission is authorized to disapprove a permanent


appointment on the ground that another person is better qualified than the appointee and,
on the basis of this finding, order his replacement by the latter?

RULING:

No, the Civil Service Commission is not empowered to determine the kind or nature of
the appointment extended by the appointing officer, its authority being limited to approving
or reviewing the appointment in the light of the requirements of the Civil Service Law. When
the appointee is qualified and all the other legal requirements are satisfied, the Commission
has no choice but to attest to the appointment in accordance with the Civil Service Laws. As
Justice Ramon C. Fernandez declared in the case of In Re: Elvira C. Arrega, 89 SCRA 318, 322,
"It is well settled that the determination of the kind of appointment to be extended lies in
the official vested by law with the appointing power and not the Civil Service Commission.
The Commissioner of Civil Service is not empowered to determine the kind or nature of the
appointment extended by the appointing officer. When the appointee is qualified, as in this
case, the Commissioner of Civil Service has no choice but to attest to the appointment. Under
the Civil Service Law, Presidential Decree No. 807, the Commissioner is not authorized to
curtail the discretion of the appointing official on the nature or kind of the appointment to
be extended."

Administrative Law; Civil Service; Appointments; A permanent appointment is protected


by the Constitution.—While the principle is correct, and we have applied it many times, it is
not correctly applied in this case. The argument begs the question. The appointment of the
27
EXECUTIVE DEPARTMENT
petitioner was not temporary but permanent and was therefore protected by Constitution.
The appointing authority indicated that it was permanent, as he had the right to do so, and
it was not for the respondent Civil Service Commission to reverse him and call it temporary.

Words stamped '”Approved as Temporary” by the Civil Service Commission in the


appointment for Administrative Officer II, Administrative Division, Cebu City, do not change
the character of the appointment as permanent.—The stamping of the words “APPROVED as
TEMPORARY” did not change the character of the appointment, which was clearly described
as “Permanent” in the space provided for in Civil Service Form No. 33, dated February 18,
1983. What was temporary was the approval of the appointment, not the appointment itself.
And what made the approval temporary was the fact that it was made to depend on the
condition specified therein and on the verification of the qualifications of the appointee to
the position.

Civil Service Commission, not empowered to determine the kind or nature of the
appointment.—The Civil Service Commission is not empowered to determine the kind or
nature of the appointment extended by the appointing officer, its authority being limited to
approving or reviewing the appointment in the light of the requirements of the Civil Service
Law. When the appointee is qualified and all the other legal requirements are satisfied, the
Commission has no choice but to attest to the appointment in accordance with the Civil
Service Laws.

Attestation; Approval of appointment by the Civil Service Commission called an


“attestation;” Requirement of attestation, nature of.—Indeed, the approval is more
appropriately called an attestation, that is, of the fact that the appointee is qualified for the
position to which he has been named. As we have repeatedly held, such attestation is
required of the Commissioner of Civil Service merely as a check to assure compliance with
Civil Service laws.
Appointment, nature of the power of issuance of.—Appointment is an essentially
discretionary power and must be performed by the officer in which it is vested according to
his best lights, the only condition being that the appointee should possess the qualifications
required by law. If he does, then the appointment cannot be faulted on the ground that there
are others better qualified who should have been preferred. This is a political question
involving considerations of wisdom which only the appointing authority can decide.

Power of the Commission to “approve” or “disapprove” appointments, limited only to


determining whether or not the appointee possesses the appropriate civil service eligibility or
the required qualifications.—However, a full reading of the provision, especially of the
underscored parts, will make it clear that all the Commission is actually allowed to do is check
whether or not the appointee possesses the appropriate civil service eligibility or the
required qualifications. If he does, his appointment is approved; if not, it is disapproved. No
other criterion is permitted by law to be employed by the Commission when it acts on—or

28
EXECUTIVE DEPARTMENT
as the Decree says, “approves” or “disapproves”—an appointment made by the proper
authorities.

Civil Service Commission, without authority to revoke an appointment because of its


belief that another person was better qualified, which is an encroachment on the discretion
vested solely in the city mayor.—Significantly, the Commission on Civil Service acknowledged
that both the petitioner and the private respondent was qualified for the position in
controversy. That recognition alone rendered it functus officio in the case and prevented it
from acting further thereon except to affirm the validity of the petitioner’s appointment. To
be sure, it had no authority to revoke the said appointment simply because it believed that
the private respondent was better qualified for that would have constituted an
encroachment on the discretion vested solely in the city mayor.

Next-in-rank rule; Application of the next-in-rank rule, not absolute.—In preferring the
private respondent to the petitioner, the Commission was probably applying its own Rule V,
Section 9, of Civil Service Rules on Personnel Actions and Policies, which provides that
“whenever there are two or more employees who are next-in-rank, preference shall be given
to the employee who is most competent and qualified and who has the appropriate civil
service eligibility.” This rule is inapplicable, however, because neither of the claimants is next
in rank. Moreover, the next-in-rank rule is not absolute as the Civil Service Decree allows
vacancies to be filled by transfer of present employees, reinstatement, reemployment, or
appointment of outsiders who have the appropriate eligibility.

29
EXECUTIVE DEPARTMENT
PONENTE/ OTHER GUTIERREZ, JR.
OPINION
DATE June 9, 1992
PETITIONERS Melanio S. Torio (Case 1); Jaime Espanola (Case
2)
RESPONDENTS Civil Service Commission, National Printing Office,
Office of the Press Secretary, Efren Camacho (vs
Torio), Letty Cangayda (vs. Espanola)
DISPOSITION The Court hereby GRANTS the petitions.
Resolutions of the Civil Service Commission set
aside. The permanent appointments of Torio and
Espanola are declared valid.
DOCTRINE ACCDG TO Power of appointment- discretionary
THE SYLLABUS

The appointing authority should appoint persons


to the positions who possess the minimum
qualifications so as to be within ambits of the law
otherwise, there would still be abuse of discretion
by the appointing authority if the other
qualifications (not limited to eligibility) are not
satisfied. For even if the appointing authority is
given a wide latitude in the exercise of its
discretion in personnel actions, the appointee
must first possess the minimum qualifications
prescribed by law.

The appointing authority’s exercise of discretion in


the choice of appointees, if valid, must be
respected even if there are other persons who are
likewise qualified for the position. In fact, the CSC
does not have the power to overrule such
discretion even if its finds that there are other
persons more qualified to the contested position.
QUICK FACTS CSC revokes appointment of petitioners
(consolidated petitions)
TORIO vs. CSC
30
EXECUTIVE DEPARTMENT
FACTS: Executive Order No. 285 abolished the General Services Administration
(GSA) including all offices and agencies under it. The General Printing Office (GPO)
which was under the GSA was merged with the relevant printing units of the Philippine
Information Agency (PIA) and out of the merger arose the National Printing Office
(NPO) which was placed under the control and supervision of the Office of the Press
Secretary (OPS). A new plantilla of personnel for the NPO was prepared and
approved and the affected officers and employees continued to perform their
respective duties and responsibilities in a hold-over capacity pending the
implementation of the reorganization.
Melanio Torio was the Chief of the Production Staff of the Printing Division, PIA, while
Jaime Espanola was a Bindery Foreman at the PIA. They continued discharging their
functions in a hold-over capacity after the PIA was merged with the GSA. In
accordance with the new staffing pattern of the NPO, Torio was temporarily appointed
as Assistant Operations Superintendent of Printing while Espanola was appointed as
Temporary Supervising Book-binder. When these appointments lapsed, Torio was
extended a renewal appointment which was likewise in a temporary capacity while
Espanola was issued another appointment as Supervising Bookbinder with a
permanent status. The positions of both petitioners were upgraded—the Assistant
Operations Superintendent of Printing was changed to Assistant Superintendent of
Printing and the Supervising Bookbinder to Bookbinder IV. This time, another
appointment was issued to Torio for the upgraded position together with his change
of status from temporary to permanent. Espanola, on the other hand, was given only
a notice of the upgrading of his position inasmuch as he was already holding it in a
permanent capacity.
Prior to the appointments of the petitioners to the permanent items, protests were
lodged with the CSC. The CSC referred the first protest to the NPO while the other
protest was referred to the Reorganization Appeals Board of the OPS. The offices
concerned did not take any action on the referrals by the CSC so the latter was
constrained to resolve the protests based on the available documents or papers
before it. The CSC issued a resolution revoking the appointment of Torio and ordering
those qualified, including Camacho, to be evaluated for the position. Subsequently,
the CSC rendered another resolution cancelling Espanola’s appointment and ordering
the reappointment of Cangayda to the position.

ARGUMENTS:

TORIO and ESPANOLA: possessed the minimum educational qualifications.


Inasmuch as the appointment has already been approved by the CSC, it cannot be
withdrawn, recalled or cancelled thus petition is moot and academic. Furthermore,
cannot be dismissed because of security of tenure under the Constitution.

31
EXECUTIVE DEPARTMENT
CAMACHO and CANGAYDA: At the time of the appointment, there were other
qualified eligibles who were not given the chance to be considered for the contested
position through no fault of their own. The Commision has the power to review
appointments for the correction of mistakes in the approval or disapproval of
appointments.

SOLICITOR GENERAL: Appointment is essentially within the discretionary power of


the appointing authority, subject to the only condition that the appointee should
possess the qualifications required by law. The subsequent acquisition of eligibility is
of no moment inasmuch as the reckoning point should be the time of appointment and
not any time before or after.

CSC: their subsequent acquisition of eligibility will not validate the otherwise invalid
appointment inasmuch as the material date is the date of appointment.

HELD:
Petitions already moot and academic, however, the court still decided on the validity
of the appointments and declared that Espanola and Torio are indeed qualified for the
positions to which they were appointed. (Torio: possesses necessary eligibility,
experience1 and education requirement; Espanola: Eligible2 but did not complete
required secondary course but may be offset by his 10yr experience)

1. Whether or not the CSC committed grave abuse of discretion

YES. The questioned resolutions of the CSC should be declared inapplicable to the
petitioners because they refer to the temporary appointments which had already
lapsed when they were issued. It must be emphasized that if a protest filed against a
temporary appointment is carried over to the subsequent permanent appointment to
the same position of the same person, an anomalous situation will arise wherein the
permanent appointee’s security to his position would be jeopardized by
considerations outside of his permanent appointment.

After giving the department or agency to which the protest is referred as reasonable
deadline to act, its inaction may be a basis for the CSC to give positive relief. The
situation in the present petitions could have been prevented if the CSC did not wait
for two years before taking the appropriate action on the protests filed.

2. Whether or not the appointments were valid

YES. Civil service positions do not only prescribe the eligibility but also the minimum
education and experience required of the position. When necessary, education,
experience or training may be used interchangeably to offset deficiencies3.The

1 Not enough experience in the department but the appointing power found Torio’s previous work experiences sufficient to tack to
the number of years of experience in actual printing operations coupled with the numerous seminars and trainings he had attended.
2 Court did not rule on the validity of the grant of testimonial eligibility. The issue is not before the SC
3 Memorandum Circular No. 23 series of 1991 expressly allowing the offsetting of deficiencies except the required eligibility

32
EXECUTIVE DEPARTMENT
necessity exists if the appointee’s training or experience is of such a level that the
same would more than supplement the deficiency in education considering the
demands of the position in question. The converse holds true if the appointee’s
deficiency is in the required training or experience. The decision as to when the
conditions give rise to a necessity to interchange education with experience and vice-
versa is upon the sound discretion of the appointing authority. This is not to be viewed
as an unbridled license given to the appointing authority to appoint whomsoever he
desires. This is rather a recognition of the fact that the appointing authority is in
the best position to determine the needs of his department or agency and how
to satisfy those needs.

A Qualification Standard provides the gauge by which the appointing authority shall
exercise his discretion. It is the minimum requirements for a class of position in terms
of education, training and experience, civil service eligibility, physical fitness and other
qualities required for successful performance.

3. Whether or not private respondent Cangayda's security of tenure was


violated by the appointment of Espanola

NO, employees or officers holding permanent appointments do not automatically get


appointed to the new positions. The appointing authority is still given latitude in making
his choice considering the duty resting on his discretion to see to it that the best
interest of the public is served with each appointment he makes. More so in cases of
reorganization of offices, where in making the new appointments, the appointing
authority has also to take into consideration the purposes and objectives of the
reorganization. As a general rule, a reorganization is carried out in ‘good faith’ if it is
for the purpose of economy or to make bureaucracy more efficient. In that event, no
dismissal (in case of a dismissal) or separation actually occurs because the position
itself ceases to exist. And in that case, security of tenure would not be a Chinese wall.
A valid abolition of office is neither removal nor separation of the incumbents. The
appointing authority should be given sufficient discretion to be able to ensure that the
purposes and objectives are met. In the present case, the reorganization was made
in good faith and was undertaken to promote economy, efficiency and effectiveness
in the delivery of public services.

"Every particular job in an office calls for both formal and informal qualifications.
Formal qualifications such as age, number of academic units in a certain course,
seminars attended and so forth, may be valuable but so are such intangibles as
resourcefulness, team spirit, courtesy, initiative, loyalty, ambition, prospects for the
future and best interest of the service. Given the demands of a certain job, who can
do it best should be left to the head of the Office concerned provided the legal
requirements for the office are satisfied." "The preference given to permanent
employees assumes that employees working in a Department for longer periods have
gained not only superior skills but also greater dedication to the public service.’ “But
the presumption is not always true and the law does not preclude the infusion of new
blood, younger dynamism or necessary talents into the government service."
33
EXECUTIVE DEPARTMENT
There is nothing in this decision which precludes the more appropriate recourse of
private respondent Cangayda to appeal to the better judgment of the Department
Head to consider her for other vacant positions more commensurate to her
qualifications.

NOTES:

Under the as the Civil Service Decree of the Philippines, an appointee with a
temporary status need not possess the civil service eligibility required by the position
provided he meets the following qualifications:
(1) it is necessary in the public interest to fill a vacancy;
(2) there are no appropriate eligibles;
(3) the temporary appointment shall not exceed twelve months; and
(4) he may be replaced sooner if a qualified civil service eligible becomes available.
Still, at the time of the temporary appointment of the petitioner, a civil service eligible
who was willing to accept the position was available. Apparently, there was disregard
of the mandate of the law when the appointment was issued. Nevertheless, the
petitioner has correctly pointed out that the protest lodged had become moot and
academic inasmuch as the appointment had already lapsed.

A permanent appointment is not a continuation of the temporary appointment—these


are two distinct acts of the appointing authority. The fact that the appointees in the
two appointments are one and the same person is purely incidental. Any irregularities
in the former appointment are not to be automatically carried over to the latter. If the
protest is directed against the temporary appointment, it would be illogical to carry-
over the merits of the protest to the subsequent permanent appointment. The
preceding ruling should not be construed to mean, however, that by the mere
expedient of appointing the temporary appointee to a permanent status, the
appointing authority can deprive the protestant of an opportunity to question the
appointment. First, the protestant is not precluded from filing another protest directed
against the permanent appointment. Second, if it can be shown that the appointment
was purposely done to moot the protest or is characterized by malice, then corrective
action can be taken and, moreover, the erring officials can be proceeded against
administratively.

34
EXECUTIVE DEPARTMENT
BINAMIRA V GARRUCHO JR.
PONENTE/ OTHER CRUZ
OPINION
DATE July 30, 1990
PETITIONERS Ramon P. Binamira
RESPONDENTS Peter D. Garrucho, Jr.
DISPOSITION Petition is DISMISSED
DOCTRINE ACCDG TO Appointment v Designation
THE SYLLABUS
Appointment and designation are distinct
from each other. The former is defined as the
selection, by the authority vested with the
power, of an individual who is to exercise the
functions of a given office. When completed,
the appointment results in security of tenure.
Designation, on the other hand, connotes
merely the imposition by law of additional
duties on an incumbent official and is
legislative in nature. The implication is that he
shall hold office only in a temporary capacity
and may be replaced at will by the appointing
authority.
QUICK FACTS in-appoint ng secretary kaso hindi feel ng
bagong pumalit na secretary. President oo
lang ng oo. Circumstances falls short of the
categorical appointment daw.

FACTS:
Binamira seeks reinstatement to the office of General Manager of the Philippine
Tourism Authority. He was designated as Gen. Manager by Jose Antonio Gonzales,
(former) Minister of Tourism and Chairman of the PTA Board which was

35
EXECUTIVE DEPARTMENT
subsequently approved by Pres. Corazon Aquino.4 However, Garrucho, the new
Secretary of Tourism demanded the immediate resignation of Binamira. In 1990,
Pres. Aquino sent Garrucho, a memorandum stating that his designation is invalid
because it was not her, the President, who appointed him as what is required by PD
No. 564. Thus, Binamira filed a quo warranto case5.

ISSUE: Whether or not Binamira was illegally removed


HELD: NO. The reason is that the decree clearly provides that the appointment of the
General Manager of the PTA shall be made by the President of the Philippines, not
by any other officer. Appointment involves the exercise of discretion, which because
of its nature cannot be delegated. Legally speaking, it was not possible for Minister
Gonzales to assume the exercise of that discretion as an alter ego of the President.
The appointment (or designation) of Binamira was not a merely mechanical or
ministerial act that could be validly performed by a subordinate even if he happened
as in this case to be a member of the Cabinet.
Indeed, even on the assumption that the power conferred on the President could be
validly exercised by the Secretary, we still cannot accept that the act of the latter, as
an extension or "projection" of the personality of the President, made irreversible the
petitioner's title to the position in question. The petitioner's conclusion that Minister
Gonzales's act was in effect the act of President Aquino is based only on half the
doctrine he vigorously invokes.
From the words of Justice Laurel in the landmark case of Villena v. Secretary of the
Interior6, The doctrine (of qualified political agency) presumes the acts of the
Department Head to be the acts of the President of the Philippines when "performed
and promulgated in the regular course of business," which was true of the designation
made by Minister Gonzales in favor of the petitioner. But it also adds that such acts
shall be considered valid only if not 'disapproved or reprobated by the Chief
Executive," as also happened in the case at bar.
The argument that the designation made by Minister Gonzales was approved by
President Aquino through her approval of the composition of the Board of Directors of
the PTA is not persuasive. It must be remembered that Binamira was included therein
as Vice- Chairman only because of his designation as PTA General Manager by
Minister Gonzales. Such designation being merely provisional, it could be recalled at
will, as in fact it was recalled by the President herself, through the memorandum she
addressed to Secretary Garrucho on January 4, 1990.

4 Binamira's in return filed a demurrer which resulted to an unpleasant exchange that led to his filing of a complaint against the
Secretary with the Commission on Human Rights. But that is another matter that does not concern us here.
5 Subsequently, while his original petition was pending, Binamira filed a supplemental petition alleging that the President of the

Philippines appointed Jose A. Capistrano as General Manager of the Philippine Tourism Authority. Capistrano was impleaded as
additional respondent.

6
Discussed Doctrine Of Qualified Political Agency
36
EXECUTIVE DEPARTMENT
Security of Tenure cannot vest upon Binamira a permanent right over a temporary
designation.
Can an officer to whom a discretion is entrusted delegate it to another?
No, the presumption being that he was chosen because he was deemed fit and
competent to exercise that judgment and discretion, and unless the power to
substitute another in his place has been given to him, he cannot delegate his duties
to another.
What is an Appointment?
It is the selection, by the authority vested with the power, of an individual who is to
exercise the functions of a given office. When completed, usually with its confirmation,
the appointment results in security of tenure for the person chosen unless he is
replaceable at pleasure because of the nature of his office. Essentially executive in
nature.
What is Designation?
It connotes merely the imposition by law of additional duties on an incumbent official,
as where, in the case before us, the Secretary of Tourism is designated Chairman of
the Board of Directors of the Philippine Tourism Authority, or where, under the
Constitution, three Justices of the Supreme Court are designated by the Chief Justice
to sit in the Electoral Tribunal of the Senate or the House of Representatives.
Essentially designation legislative in nature.
Designation may also be loosely defined as an appointment because it likewise
involves the naming of a particular person to a specified public office. That is the
common understanding of the term. However, where the person is merely designated
and not appointed, the implication is that he shall hold the office only in a temporary
capacity and may be replaced at will by the appointing authority. In this sense, the
designation is considered only an acting or temporary appointment, which does not
confer security of tenure on the person named.
The law in controversy:
Section 23-A of P.D. 564, which created the Philippine Tourism Authority, provides as
follows:
SECTION 23-A. General Manager-Appointment and Tenure. — The General
Manager shall be appointed by the President of the Philippines and shall serve for a
term of six (6) years unless sooner removed for cause; Provided, That upon the
expiration of his term, he shall serve as such until his successor shall have been
appointed and qualified. (As amended by P.D. 1400)

37
EXECUTIVE DEPARTMENT
ONG V OFFICE OF THE PRESIDENT
PONENTE/ OTHER REYES
OPINION
DATE January 30, 2012
PETITIONERS SAMUEL B. ONG
RESPONDENTS OFFICE OF THE PRESIDENT, ET AL.
DISPOSITION Petition is DENIED. Court of Appeals
decision AFFIRMED.
DOCTRINE ACCDG TO CLASSES OF APPOINTMENT:
THE SYLLABUS PERMANENT v TEMPORARY
Section 27 of the Administrative Code of
1987, as amended, classifies the
appointment status of public officers and
employees in the career service into
permanent and temporary. A permanent
appointment shall be issued to a person
who meets all the requirements for the
position to which he is being appointed,
including appropriate eligibility prescribed, in
accordance with the provisions of law, rules
and standards promulgated in pursuance
thereof. In the absence of appropriate
eligibles and it becomes necessary in the
public interest to fill a vacancy, a temporary
appointment shall be issued to a person
who meets all the requirements for the
position to which he is being appointed
except the appropriate civil service eligibility;
provided, that such temporary appointment
shall not exceed twelve months, but the

38
EXECUTIVE DEPARTMENT
appointee may be replaced sooner if a
qualified civil service eligible becomes
available. Temporary appointment is defined
as "one made in an acting capacity, the
essence of which lies in its temporary
character and its terminability at pleasure by
the appointing power7."
QUICK FACTS Non-eligible replaced by another non-
eligible, NBI

FACTS: Ong joined the National Bureau of Investigation (NBI) as a career employee
in 1978. He held the position of NBI Director I from 1998 to 1999 and NBI Director II
from 1999 to 2001. On 2001, petitioner was appointed Director III by the President.

On 2004, the petitioner received from Reynaldo Wycoco a memorandum informing


him that his appointment, being co-terminus with the appointing authority's tenure,
would end effectively at midnight on June 30, 2004 and, unless a new appointment
would be issued in his favor by the President consistent with her new tenure effective
July 1, 2004, he would be occupying his position in a de facto/hold- over status until
his replacement would be appointed. Soon thereafter, the President appointed
respondent Victor A. Bessat as NBI Director III as replacement of the petitioner. Ong
filed before the CA a petition for quo warranto. He sought for the declaration as null
and void of (a) his removal from the position of NBI Director III; and (b) his replacement
by respondent Victor Bessat. Ong likewise prayed for reinstatement and backwages8.
The CA denied the petition.

ARGUMENTS:

CA: In the present case, it is undisputed that the petitioner is a non-CESO eligible. At
best, therefore, his appointment could be regarded only as temporary and, hence, he
has no security of tenure. Such being the case, his appointment can be withdrawn at
will by the President, who is the appointing authority in this case, and "at a moment's
notice."

 In a petition for quo warranto, where the action is filed by a private person, in
his own name, he must prove that he is entitled to the controverted position,
otherwise, respondent has a right to the undisturbed possession of the office.
 The temporary appointee accepts the position with the condition that he shall
surrender the office when called upon to do so by the appointing authority.
Although petitioner's appointment is co-terminous with the tenure of the

7 Cuadra v. Cordova
8Ong died on during the pendency of the instant petition. Admittedly, Ong's death rendered the prayer for reinstatement in the
petition for quo warranto as moot and academic. However, substitution was sought because in the event that the Court would rule
that Ong was indeed entitled to the position he claimed, backwages pertaining to him can still be paid to his legal heirs.

39
EXECUTIVE DEPARTMENT
President, he nevertheless serves at the pleasure of the President and his
appointment may be recalled anytime.
 In the career executive service, the acquisition of security of tenure presupposes
a permanent appointment. As held in General v. Roco, two requisites must
concur in order that an employee in the career executive service may attain
security of tenure, to wit: 1) CES eligibility[;] and 2) appointment to the
appropriate CES rank.
 Mita Pardo de Tavera v. Philippine Tuberculosis Society, Inc.delineated the
nature of an appointment held "at the pleasure of the appointing power" in this
wise: An appointment held at the pleasure of the appointing power is in essence
temporary in nature. It is co-extensive with the desire of the Board of Directors.
Hence, when the Board opts to replace the incumbent, technically there is no
removal but only an expiration of term and in an expiration of term, there is no
need of prior notice, due hearing or sufficient grounds before the incumbent can
be separated from office. The protection afforded by Section 7.04 of the Code
of By-Laws on Removal [o]f Officers and Employees, therefore, cannot be
claimed by petitioner.

ONG: the power of removal is lodged in the appointing authority. Wycoco, and not the
President, issued the Memorandum informing Ong that his co-terminous appointment
as Director III ended effectively. The issuance of the memo was allegedly motivated
by malice and revenge since Ong led the NBI employees in holding rallies in July 2003
to publicly denounce Wycoco. Hence, Bessat's assumption of the position was null
and void since it was technically still occupied by Ong at the time of the former's
appointment.

 It is further alleged that it was erroneous for the CA to equate "an appointment
co-terminous with the tenure of the appointing authority with one that is at the
pleasure of such appointing authority." Ong distinguished a "term" as "the time
during which the officer may claim to hold office as of right" from a "tenure"
which "represents the term during which the incumbent actually holds the
office". Ong's appointment, from which he cannot be removed without just
cause, was co-terminous with the President's tenure which ended not on June
30, 2004, but only on June 30, 2010.
 Government employees, holding both career and non-career service positions,
are entitled to protection from arbitrary removal or suspension. In the case of
Ong, who started his employment in 1978 and rose from the ranks, it is allegedly
improper for the CA to impliedly infer that the President acted in bad faith by
converting his supposed promotional appointment to one removable at the
pleasure of the appointing authority.

Office of the Solicitor General (OSG): the replacement of Ong by Bessat was fair
and just. Only a temporary appointment can be issued to a person who does not have
the appropriate civil service eligibility. Appointments which are co-terminous with the
appointing authority shall not be considered as permanent.
40
EXECUTIVE DEPARTMENT
ISSUE: Whether or not the CA erred in sustaining the validity of Ong's removal

HELD: No. CA Decision Affirmed.

Whether or not the memo removed Ong from the position of Director III.

NO. It merely informed Ong that records of the NBI showed that his co-terminous
appointment had lapsed into a de facto/hold-over status and its consequences. And
even assuming arguendo that it did, the defect was cured when the President9, who
was the appointing authority herself, in whose hands were lodged the power to
remove, appointed Bessat, effectively revoking Ong's appointment.

Whether or not Ong can be removed.

YES. Ong lacked the CES eligibility (thus temporary) required for the position of
Director III and his appointment was "co-terminus with the appointing authority." His
appointment being both temporary and co-terminous in nature, it can be revoked by
the President even without cause and at a short notice. He was removable at the
pleasure of the appointing authority.

It is established that no officer or employee in the Civil Service shall be removed or


suspended except for cause provided by law. However, this admits of exceptions for
it is likewise settled that the right to security of tenure is not available to those
employees whose appointments are contractual and co-terminous in nature. Security
of tenure in the career executive service, which presupposes a permanent
appointment, takes place upon passing the CES examinations administered by the
CES Board.

An appointment is permanent where the appointee meets all the requirements for the
position to which he is being appointed, including the appropriate eligibility prescribed,
and it is temporary where the appointee meets all the requirements for the position
except only the appropriate civil service eligibility. Verily, it is clear that the possession
of the required CES eligibility is that which will make an appointment in the career
executive service a permanent one. But, the mere fact that an employee is a CES
eligible does not automatically operate to vest security of tenure on the appointee
inasmuch as the security of tenure of employees in the career executive service,
except first and second-level employees, pertains only to rank and not to the office or
position to which they may be appointed.

Indeed, the law permits, on many occasions, the appointment of non-CES eligibles to
CES positions in the government in the absence of appropriate eligibles and when
there is necessity in the interest of public service to fill vacancies in the government.
But in all such cases, the appointment is at best merely temporary as it is said to be
conditioned on the subsequent obtention of the required CES eligibility.

9
PGMA
41
EXECUTIVE DEPARTMENT
Whether or not Ong, as an appointee holding a position "co-terminus with the
appointing authority," was entitled to remain as Director III until the end of the
President's tenure

NO. A co-terminous appointment is one which is co-existent with the tenure of the
appointing authority or at his pleasure.

In Mita Pardo de Tavera v. Philippine Tuberculosis Society, Inc. it was held that the
replacement of an incumbent, who held an appointment at the pleasure of the
appointing authority is in essence temporary in nature. We categorized the
incumbent's replacement not as removal but rather as an expiration of term and no
prior notice, due hearing or cause were necessary to effect the same. In Decano v.
Edu, the court ruled that the acceptance of a temporary appointment divests an
appointee of the right to security of tenure against removal without cause. Further, in
Carillo vs. CA, it was stated that "one who holds a temporary appointment has no fixed
tenure of office; his employment can be terminated at the pleasure of the appointing
authority, there being no need to show that the termination is for cause." In Ong's
case, his appointment was temporary and co-terminous. Where a non-eligible holds
a temporary appointment, his replacement by another non-eligible is not prohibited.

Neither law nor jurisprudence draws distinctions between appointments "co-existing


with the term of the appointing authority" on one hand, and one "co-existing with the
appointing authority's tenure" on the other. In the contrary, tenure and term are used
rather loosely and interchangeably. The distinctions between term and tenure find no
materiality in the instant petition.

*In a quo warranto proceeding, the person suing must show that he has a clear right
to the office allegedly held unlawfully by another. Absent that right, the lack of
qualification or eligibility of the supposed usurper is immaterial. Ong had no legal right
over the disputed office and his cessation from office involves no removal but an
expiration of his term of office

42
EXECUTIVE DEPARTMENT
MATIBAG v BENIPAYO
PONENTE/ OTHER CARPIO
OPINION
DATE April 2, 2002
PETITIONERS MA. J. ANGELINA G. MATIBAG
RESPONDENTS ALFREDO L. BENIPAYO, RESURRECCION
Z. BORRA, FLORENTINO A. TUASON, JR.,
VELMA J. CINCO, and GIDEON C. DE
GUZMAN in his capacity as Officer-In-
Charge, Finance Services Department of the
Commission on Elections,
DISPOSITION Petition is dismissed for lack of merit.
DOCTRINE ACCDG TO AD INTERIM v. PERMANENT
THE SYLLABUS
An ad interim appointment is a
permanent appointment because it takes
effect immediately and can no longer be
withdrawn by the President once the
appointee has qualified into office. The fact
that it is subject to confirmation by the
Commission on Appointments does not alter
its permanent character. The Constitution
itself makes an ad interim appointment
permanent in character by making it effective
until disapproved by the Commission on

43
EXECUTIVE DEPARTMENT
Appointments or until the next adjournment
of Congress.
QUICK FACTS COMELEC appointments assailed

FACTS:

The COMELEC en banc appointed petitioner as "Acting Director IV" of the EID.
In 2000, then Chairperson Harriet O. Demetriou renewed the appointment of petitioner
as Director IV of EID in a "Temporary" capacity. On March 22, 2001, President Gloria
Macapagal Arroyo appointed, ad interim, Benipayo as COMELEC Chairman together
with other commisioners in an ad interim appointment.
In his capacity as COMELEC Chairman, Benipayo issued a Memorandum
addressed to petitioner as Director IV of the EID and to Cinco as Director III also of
the EID, designating Cinco Officer-in-Charge of the EID and reassigning petitioner to
the Law Department. COMELEC EID Commissioner-in-Charge Mehol K. Sadain
objected to petitioner’s reassignment in a Memorandum addressed to the COMELEC
en banc. Specifically, Commissioner Sadain questioned Benipayo’s failure to consult
the Commissioner-in-Charge of the EID in the reassignment of petitioner.
Petitioner requested Benipayo to reconsider her relief as Director IV of the EID
and her reassignment to the Law Department. Petitioner cited Civil Service
Commission Memorandum Circular No. 7 reminding heads of government offices that
"transfer and detail of employees are prohibited during the election period beginning
January 2 until June 13, 2001." Benipayo denied her request for reconsideration citing
a COMELEC exempting the Comelec from the coverage of the said memo circular.
During the pendency of her complaint before the Law Department, petitioner
filed the instant petition questioning the appointment and the right to remain in office
of Benipayo, Borra and Tuason, as Chairman and Commissioners of the COMELEC,
respectively. Petitioner claims that the ad interim appointments of Benipayo, Borra
and Tuason violate the constitutional provisions on the independence of the
COMELEC.

ISSUES
Whether or not the instant petition satisfies all the requirements before this
Court may exercise its power of judicial review in constitutional cases (Propriety
of Judicial Review)
Yes. The petitioner has complied with all the requisite technicalities. Moreover, public
interest requires the resolution of the constitutional issue raised by petitioner.
The real issue turns on whether or not Benipayo is the lawful Chairman of the
COMELEC. Even if petitioner is only an Acting Director of the EID, her reassignment
is without legal basis if Benipayo is not the lawful COMELEC Chairman, an office
created by the Constitution. On the other hand, if Benipayo is the lawful COMELEC
44
EXECUTIVE DEPARTMENT
Chairman because he assumed office in accordance with the Constitution, then
petitioner’s reassignment is legal and she has no cause to complain provided the
reassignment is in accordance with the Civil Service Law. Clearly, petitioner has a
personal and material stake in the resolution of the constitutionality of Benipayo’s
assumption of office. Petitioner’s personal and substantial injury, if Benipayo is not
the lawful COMELEC Chairman, clothes her with the requisite locus standi to raise
the constitutional issue in this petition.
It is not the date of filing of the petition that determines whether the constitutional issue
was raised at the earliest opportunity. The earliest opportunity to raise a constitutional
issue is to raise it in the pleadings before a competent court that can resolve the same,
such that, "if it is not raised in the pleadings, it cannot be considered at the trial, and,
if not considered at the trial, it cannot be considered on appeal."
Moreover, the legality of petitioner’s reassignment hinges on the constitutionality of
Benipayo’s ad interim appointment and assumption of office. Unless the
constitutionality of Benipayo’s ad interim appointment and assumption of office is
resolved, the legality of petitioner’s reassignment from the EID to the Law Department
cannot be determined. Clearly, the lis mota of this case is the very constitutional issue
raised by petitioner
Whether or not the assumption of office by Benipayo, Borra and Tuason on the
basis of the ad interim appointments issued by the President amounts to a
temporary appointment prohibited by Section 1 (2), Article IX-C of the
Constitution (The Nature of an Ad Interim Appointment)
NO. An ad interim appointment is a permanent appointment because it takes effect
immediately and can no longer be withdrawn by the President once the appointee has
qualified into office. The fact that it is subject to confirmation by the Commission on
Appointments does not alter its permanent character. The Constitution itself makes
an ad interim appointment permanent in character by making it effective until
disapproved by the Commission on Appointments or until the next adjournment of
Congress. Section 16, Article VII of the Constitution provides as follows: "The
President shall have the power to make appointments during the recess of the
Congress, whether voluntary or compulsory, but such appointments shall be effective
only until disapproval by the Commission on Appointments or until the next
adjournment of the Congress.
Thus, the ad interim appointment remains effective until such disapproval or next
adjournment, signifying that it can no longer be withdrawn or revoked by the
President. The fear that the President can withdraw or revoke at any time and for any
reason an ad interim appointment is utterly without basis.
The Constitution imposes no condition on the effectivity of an ad interim appointment,
and thus an ad interim appointment takes effect immediately. The appointee can at
once assume office and exercise, as a de jure officer, all the powers pertaining to the
45
EXECUTIVE DEPARTMENT
office. "A distinction is thus made between the exercise of such presidential
prerogative requiring confirmation by the Commission on Appointments when
Congress is in session and when it is in recess. In the former, the President
nominates, and only upon the consent of the Commission on Appointments may the
person thus named assume office. It is not so with reference to ad interim
appointments. It takes effect at once. The individual chosen may thus qualify and
perform his function without loss of time. His title to such office is complete. In the
language of the Constitution, the appointment is effective ‘until disapproval by the
Commission on Appointments or until the next adjournment of the Congress.’"
"The term is defined by Black to mean "in the meantime" or "for the time being". Thus,
an officer ad interim is one appointed to fill a vacancy, or to discharge the duties of
the office during the absence or temporary incapacity of its regular incumbent. But
such is not the meaning nor the use intended in the context of Philippine law. In
referring to appointments, the term is not descriptive of the nature of the appointments
given to him. Rather, it is used to denote the manner in which said appointments were
made, that is, done by the President of the Pamantasan in the meantime, while the
Board of Regents, which is originally vested by the University Charter with the power
of appointment, is unable to act.10"
Thus, the term "ad interim appointment", as used in letters of appointment signed by
the President, means a permanent appointment made by the President in the
meantime that Congress is in recess. It does not mean a temporary appointment that
can be withdrawn or revoked at any time. The term, although not found in the text of
the Constitution, has acquired a definite legal meaning under Philippine
jurisprudence.--- an ad interim appointment is not descriptive of the nature of the
appointment, that is, it is not indicative of whether the appointment is temporary or in
an acting capacity, rather it denotes the manner in which the appointment was made.
Ad interim appointments are permanent but their terms are only until the Board
disapproves them."
An ad interim appointee who has qualified and assumed office becomes at that
moment a government employee and therefore part of the civil service. He enjoys the
constitutional protection that "[n]o officer or employee in the civil service shall be
removed or suspended except for cause provided by law." Thus, an ad interim
appointment becomes complete and irrevocable once the appointee has qualified into
office. The withdrawal or revocation of an ad interim appointment is possible only if it
is communicated to the appointee before the moment he qualifies, and any withdrawal
or revocation thereafter is tantamount to removal from office. Once an appointee has
qualified, he acquires a legal right to the office which is protected not only by statute
but also by the Constitution. He can only be removed for cause, after notice and
hearing, consistent with the requirements of due process.

10 PLM CASE
46
EXECUTIVE DEPARTMENT
An ad interim appointment can be terminated for two causes specified in the
Constitution. The first cause is the disapproval of his ad interim appointment by the
Commission on Appointments. The second cause is the adjournment of Congress
without the Commission on Appointments acting on his appointment. These two
causes are resolutory conditions expressly imposed by the Constitution on all ad
interim appointments. These resolutory conditions constitute, in effect, a Sword of
Damocles over the heads of ad interim appointees. No one, however, can complain
because it is the Constitution itself that places the Sword of Damocles over the heads
of the ad interim appointees.
While the Constitution mandates that the COMELEC "shall be independent", this
provision should be harmonized with the President’s power to extend ad interim
appointments. To hold that the independence of the COMELEC requires the
Commission on Appointments to first confirm ad interim appointees before the
appointees can assume office will negate the President’s power to make ad interim
appointments. This is contrary to the rule on statutory construction to give meaning
and effect to every provision of the law. It will also run counter to the clear intent of
the framers of the Constitution.
The original draft of Section 16, Article VII of the Constitution - on the nomination of
officers subject to confirmation by the Commission on Appointments - did not provide
for ad interim appointments. The original intention of the framers of the Constitution
was to do away with ad interim appointments because the plan was for Congress to
remain in session throughout the year except for a brief 30-day compulsory recess.
However, because of the need to avoid disruptions in essential government services,
the framers of the Constitution thought it wise to reinstate the provisions of the 1935
Constitution on ad interim appointments. During an election year, Congress normally
goes on voluntary recess between February and June considering that many of the
members of the House of Representatives and the Senate run for re-election.
If Benipayo, Borra and Tuason were not extended ad interim appointments to fill up
the three vacancies in the COMELEC, there would only have been one division
functioning in the COMELEC instead of two during the May 2001 elections. Evidently,
the exercise by the President in the instant case of her constitutional power to make
ad interim appointments prevented the occurrence of the very evil sought to be
avoided by the second paragraph of Section 16, Article VII of the Constitution.
This power to make ad interim appointments is lodged in the President to be exercised
by her in her sound judgment. Under the second paragraph of Section 16, Article VII
of the Constitution, the President can choose either of two modes in appointing
officials who are subject to confirmation by the Commission on Appointments. First,
while Congress is in session, the President may nominate the prospective appointee,
and pending consent of the Commission on Appointments, the nominee cannot qualify
and assume office. Second, during the recess of Congress, the President may extend
an ad interim appointment which allows the appointee to immediately qualify and
47
EXECUTIVE DEPARTMENT
assume office. Whether the President chooses to nominate the prospective appointee
or extend an ad interim appointment is a matter within the prerogative of the President
because the Constitution grants her that power. This Court cannot inquire into the
propriety of the choice made by the President in the exercise of her constitutional
power, absent grave abuse of discretion amounting to lack or excess of jurisdiction
on her part, which has not been shown in the instant case.
The President’s power to extend ad interim appointments may indeed briefly put the
appointee at the mercy of both the appointing and confirming powers. This situation,
however, is only for a short period - from the time of issuance of the ad interim
appointment until the Commission on Appointments gives or withholds its consent.
The Constitution itself sanctions this situation, as a trade-off against the evil of
disruptions in vital government services. This is also part of the check-and-balance
under the separation of powers, as a trade-off against the evil of granting the
President absolute and sole power to appoint. The Constitution has wisely subjected
the President’s appointing power to the checking power of the legislature.
This situation, however, does not compromise the independence of the COMELEC
as a constitutional body. The vacancies in the COMELEC are precisely staggered to
insure that the majority of its members hold confirmed appointments, and not one
President will appoint all the COMELEC members.
Assuming that the first ad interim appointments and the first assumption of
office by Benipayo, Borra and Tuason are legal, whether or not the renewal of
their ad interim appointments and subsequent assumption of office to the same
positions violate the prohibition on reappointment under Section 1 (2), Article
IX-C of the Constitution (The Constitutionality of Renewals of Appointments)
NO. A reappointment presupposes a previous confirmed appointment. The same ad
interim appointments and renewals of appointments will also not breach the seven-
year term limit because all the appointments and renewals of appointments of
Benipayo, Borra and Tuason are for a fixed term expiring on February 2, 2008.Any
delay in their confirmation will not extend the expiry date of their terms of office.
Consequently, there is no danger whatsoever that the renewal of the ad interim
appointments of these three respondents will result in any of the evils intended to be
exorcised by the twin prohibitions in the Constitution. The continuing renewal of the
ad interim appointment of these three respondents, for so long as their terms of office
expire on February 2, 2008, does not violate the prohibition on reappointments in
Section 1 (2), Article IX-C of the Constitution.
There is no dispute that an ad interim appointee disapproved by the Commission on
Appointments can no longer be extended a new appointment. The disapproval is a
final decision of the Commission on Appointments in the exercise of its checking
power on the appointing authority of the President. The disapproval is a decision on
the merits, being a refusal by the Commission on Appointments to give its consent
48
EXECUTIVE DEPARTMENT
after deliberating on the qualifications of the appointee. Since the Constitution does
not provide for any appeal from such decision, the disapproval is final and binding on
the appointee as well as on the appointing power. In this instance, the President can
no longer renew the appointment not because of the constitutional prohibition on
reappointment, but because of a final decision by the Commission on Appointments
to withhold its consent to the appointment.
An ad interim appointment that is by-passed because of lack of time or failure of the
Commission on Appointments to organize is another matter. A by-passed
appointment is one that has not been finally acted upon on the merits by the
Commission on Appointments at the close of the session of Congress. There is no
final decision by the Commission on Appointments to give or withhold its consent to
the appointment as required by the Constitution. Absent such decision, the President
is free to renew the ad interim appointment of a by-passed appointee.
The prohibition on reappointment in Section 1 (2), Article IX-C of the Constitution
applies neither to disapproved nor by-passed ad interim appointments. A disapproved
ad interim appointment cannot be revived by another ad interim appointment because
the disapproval is final under Section 16, Article VII of the Constitution, and not
because a reappointment is prohibited under Section 1 (2), Article IX-C of the
Constitution. A by-passed ad interim appointment can be revived by a new ad interim
appointment because there is no final disapproval under Section 16, Article VII of the
Constitution, and such new appointment will not result in the appointee serving
beyond the fixed term of seven years.
Section 1 (2), Article IX-C of the Constitution provides that "[t]he Chairman and the
Commissioners shall be appointed x x x for a term of seven years without
reappointment." (Emphasis supplied) There are four situations where this provision
will apply. The first situation is where an ad interim appointee to the COMELEC, after
confirmation by the Commission on Appointments, serves his full seven-year term.
Such person cannot be reappointed to the COMELEC, whether as a member or as a
chairman, because he will then be actually serving more than seven years. The
second situation is where the appointee, after confirmation, serves a part of his term
and then resigns before his seven-year term of office ends. Such person cannot be
reappointed, whether as a member or as a chair, to a vacancy arising from retirement
because a reappointment will result in the appointee also serving more than seven
years. The third situation is where the appointee is confirmed to serve the unexpired
term of someone who died or resigned, and the appointee completes the unexpired
term. Such person cannot be reappointed, whether as a member or chair, to a
vacancy arising from retirement because a reappointment will result in the appointee
also serving more than seven years.
The fourth situation is where the appointee has previously served a term of less than
seven years, and a vacancy arises from death or resignation. Even if it will not result
in his serving more than seven years, a reappointment of such person to serve an
49
EXECUTIVE DEPARTMENT
unexpired term is also prohibited because his situation will be similar to those
appointed under the second sentence of Section 1 (2), Article IX-C of the Constitution.
This provision refers to the first appointees under the Constitution whose terms of
office are less than seven years, but are barred from ever being reappointed under
any situation. Not one of these four situations applies to the case of Benipayo, Borra
or Tuason.
In Visarra vs. Miraflor, Justice Angelo Bautista, in his concurring opinion, quoted
Nacionalista vs. De Vera that a "[r]eappointment is not prohibited when a
Commissioner has held office only for, say, three or six years, provided his term will
not exceed nine years in all." This was the interpretation despite the express provision
in the 1935 Constitution that a COMELEC member "shall hold office for a term of nine
years and may not be reappointed."
To foreclose this interpretation, the phrase "without reappointment" appears twice in
Section 1 (2), Article IX-C of the present Constitution. The first phrase prohibits
reappointment of any person previously appointed for a term of seven years. The
second phrase prohibits reappointment of any person previously appointed for a term
of five or three years pursuant to the first set of appointees under the Constitution. In
either case, it does not matter if the person previously appointed completes his term
of office for the intention is to prohibit any reappointment of any kind.
However, an ad interim appointment that has lapsed by inaction of the Commission
on Appointments does not constitute a term of office. The period from the time the ad
interim appointment is made to the time it lapses is neither a fixed term nor an
unexpired term. To hold otherwise would mean that the President by his unilateral
action could start and complete the running of a term of office in the COMELEC
without the consent of the Commission on Appointments. This interpretation renders
inutile the confirming power of the Commission on Appointments.
The phrase "without reappointment" applies only to one who has been appointed by
the President and confirmed by the Commission on Appointments, whether or not
such person completes his term of office. There must be a confirmation by the
Commission on Appointments of the previous appointment before the prohibition on
reappointment can apply. To hold otherwise will lead to absurdities and negate the
President’s power to make ad interim appointments.
In the great majority of cases, the Commission on Appointments usually fails to act,
for lack of time, on the ad interim appointments first issued to appointees. If such ad
interim appointments can no longer be renewed, the President will certainly hesitate
to make ad interim appointments because most of her appointees will effectively be
disapproved by mere inaction of the Commission on Appointments. This will nullify
the constitutional power of the President to make ad interim appointments, a power
intended to avoid disruptions in vital government services. This Court cannot
subscribe to a proposition that will wreak havoc on vital government services.
50
EXECUTIVE DEPARTMENT
Plainly, the prohibition on reappointment is intended to insure that there will be no
reappointment of any kind. On the other hand, the prohibition on temporary or acting
appointments is intended to prevent any circumvention of the prohibition on
reappointment that may result in an appointee’s total term of office exceeding seven
years. The evils sought to be avoided by the twin prohibitions are very specific -
reappointment of any kind and exceeding one’s term in office beyond the maximum
period of seven years.
Whether or not Benipayo’s removal of petitioner from her position as Director
IV of the EID and her reassignment to the Law Department is illegal and without
authority, having been done without the approval of the COMELEC as a collegial
body;(Respondent Benipayo’s Authority to Reassign Petitioner)
Petitioner claims that Benipayo has no authority to remove her as Director IV of the
EID and reassign her to the Law Department. Petitioner further argues that only the
COMELEC, acting as a collegial body, can authorize such reassignment. Moreover,
petitioner maintains that a reassignment without her consent amounts to removal from
office without due process and therefore illegal.
Petitioner’s posturing will hold water if Benipayo does not possess any color of title to
the office of Chairman of the COMELEC. We have ruled, however, that Benipayo is
the de jure COMELEC Chairman, and consequently he has full authority to exercise
all the powers of that office for so long as his ad interim appointment remains effective.
The mere fact that a position belongs to the Career Service does not automatically
confer security of tenure on its occupant even if he does not possess the required
qualifications. Such right will have to depend on the nature of his appointment, which
in turn depends on his eligibility or lack of it. A person who does not have the requisite
qualifications for the position cannot be appointed to it in the first place, or as an
exception to the rule, may be appointed to it merely in an acting capacity in the
absence of appropriate eligibles. The appointment extended to him cannot be
regarded as permanent even if it may be so designated x x x.’"
The proviso in COMELEC Resolution No. 3300, requiring due notice and hearing
before any transfer or reassignment can be made within thirty days prior to election
day, refers only to COMELEC field personnel and not to head office personnel like the
petitioner. Under the Revised Administrative Code, the COMELEC Chairman is the
sole officer specifically vested with the power to transfer or reassign COMELEC
personnel. The COMELEC Chairman will logically exercise the authority to transfer or
reassign COMELEC personnel pursuant to COMELEC Resolution No. 3300. The
COMELEC en banc cannot arrogate unto itself this power because that will mean
amending the Revised Administrative Code, an act the COMELEC en banc cannot
legally do.
COMELEC Resolution No. 3300 does not require that every transfer or reassignment
of COMELEC personnel should carry the concurrence of the COMELEC as a collegial
51
EXECUTIVE DEPARTMENT
body. Interpreting Resolution No. 3300 to require such concurrence will render the
resolution meaningless since the COMELEC en banc will have to approve every
personnel transfer or reassignment, making the resolution utterly useless. Resolution
No. 3300 should be interpreted for what it is, an approval to effect transfers and
reassignments of personnel, without need of securing a second approval from the
COMELEC en banc to actually implement such transfer or reassignment.
The COMELEC Chairman is the official expressly authorized by law to transfer or
reassign COMELEC personnel. The person holding that office, in a de jure capacity,
is Benipayo. The COMELEC en banc, in COMELEC Resolution No. 3300, approved
the transfer or reassignment of COMELEC personnel during the election period. Thus,
Benipayo’s order reassigning petitioner from the EID to the Law Department does not
violate Section 261 (h) of the Omnibus Election Code. For the same reason,
Benipayo’s order designating Cinco Officer-in-Charge of the EID is legally
unassailable.
Whether or not the Officer-in-Charge of the COMELEC’s Finance Services
Department, in continuing to make disbursements in favor of Benipayo, Borra,
Tuason and Cinco, is acting in excess of jurisdiction. (Legality of Disbursements
to Respondents)
NO.
CASES CITED:
Summers vs. Ozaeta: "x x x an ad interim appointment is one made in pursuance of
paragraph (4), Section 10, Article VII of the Constitution, which provides that the
‘President shall have the power to make appointments during the recess of the
Congress, but such appointments shall be effective only until disapproval by the
Commission on Appointments or until the next adjournment of the Congress.’ It is an
appointment permanent in nature, and the circumstance that it is subject to
confirmation by the Commission on Appointments does not alter its permanent
character. An ad interim appointment is disapproved certainly for a reason other than
that its provisional period has expired. Said appointment is of course distinguishable
from an ‘acting’ appointment which is merely temporary, good until another permanent
appointment is issued."
Gaminde vs. Commission on Appointments: The terms of office of constitutional
officers first appointed under the Constitution would have to be counted starting
February 2, 1987, the date of ratification of the Constitution, regardless of the date of
their actual appointment. By this reckoning, the terms of office of three Commissioners
of the COMELEC, including the Chairman, would end on February 2, 2001.
Guevara vs. Inocentes (Concepcion, Concurring opinion): why by-passed ad interim
appointees could be extended new appointments, thus: "In short, an ad interim
appointment ceases to be effective upon disapproval by the Commission, because
52
EXECUTIVE DEPARTMENT
the incumbent can not continue holding office over the positive objection of the
Commission. It ceases, also, upon "the next adjournment of the Congress", simply
because the President may then issue new appointments - not because of implied
disapproval of the Commission deduced from its inaction during the session of
Congress, for, under the Constitution, the Commission may affect adversely the
interim appointments only by action, never by omission. If the adjournment of
Congress were an implied disapproval of ad interim appointments made prior thereto,
then the President could no longer appoint those so by-passed by the Commission.
But, the fact is that the President may reappoint them, thus clearly indicating that the
reason for said termination of the ad interim appointments is not the disapproval
thereof allegedly inferred from said omission of the Commission, but the circumstance
that upon said adjournment of the Congress, the President is free to make ad interim
appointments or reappointments."
Achacoso v. Macaraig:‘It is settled that a permanent appointment can be issued only
‘to a person who meets all the requirements for the position to which he is being
appointed, including the appropriate eligibility prescribed.’ Achacoso did not. At best,
therefore, his appointment could be regarded only as temporary. And being so, it
could be withdrawn at will by the appointing authority and ‘at a moment’s notice’,
conformably to established jurisprudence x x x.

53
EXECUTIVE DEPARTMENT
PIMENTEL JR v ERMITA
PONENTE/ OTHER CARPIO
OPINION
DATE October 13, 2005
PETITIONERS Aquilino Q. Pimentel, Jr., Edgardo J. Angara,
Juan Ponce Enrile, Luisa P. Ejercito-Estrada,
Jinggoy E. Estrada, Panfilo M. Lacson,
Alfredo S. Lim, Jamby A.S. Madrigal, And
Sergio R. Osmea Iii
RESPONDENTS Exec. Secretary Eduardo, R. Ermita,
Florencio B. Abad, Avelino J. Cruz, Jr.,
Michael T. Defensor, Joseph H. Durano, Raul
M. Gonzalez, Alberto G. Romulo, Rene C.
Villa, Arthur C. Yap
DISPOSITION DISMISSED the present petition for certiorari
and prohibition. We find no abuse in the
present case. The absence of abuse is
readily apparent from President Arroyo’s
issuance of ad interim appointments to
respondents immediately upon the recess of
Congress, way before the lapse of one year.
DOCTRINE ACCDG TO AD INTERIM v ACTING CAPACITY/
THE SYLLABUS TEMPORARY DESIGNATION

54
EXECUTIVE DEPARTMENT
Ad-interim appointments must be
distinguished from appointments in an acting
capacity. Both of them are effective upon
acceptance. But ad-interim appointments are
extended only during a recess of Congress,
whereas acting appointments may be
extended any time there is a vacancy.
Moreover ad-interim appointments are
submitted to the Commission on
Appointments for confirmation or rejection;
acting appointments are not submitted to the
Commission on Appointments. Acting
appointments are a way of temporarily filling
important offices but, if abused, they can also
be a way of circumventing the need for
confirmation by the Commission on
Appointments.
QUICK FACTS Appointed Secretaries v senators

Facts: President Arroyo issued appointments to respondents as acting secretaries of


their respective departments without the consent of the Commission on
Appointments, while Congress is in their regular session. Subsequently after the
Congress had adjourned, President Arroyo issued ad interim appointments to
respondents as secretaries of the departments to which they were previously
appointed in an acting capacity.

Appointee Department Date of


Appointment
Arthur C. Yap Agriculture 15 August 2004
Alberto G. Romulo Foreign Affairs 23 August 2004
Raul M. Gonzalez Justice 23 August 2004
Florencio B. Abad Education 23 August 2004
Avelino J. Cruz, National Defense 23 August 2004
Jr.
Rene C. Villa Agrarian Reform 23 August 2004
Joseph H. Durano Tourism 23 August 2004
Michael T. Environment and Natural 23 August 2004
Defensor Resources

Petitioners senators assailing the constitutionality of the appointments, assert


that “while Congress is in session, there can be no appointments, whether regular or
55
EXECUTIVE DEPARTMENT
acting, to a vacant position of an office needing confirmation by the Commission on
Appointments, without first having obtained its consent. Respondent secretaries
maintain that the President can issue appointments in an acting capacity to
department secretaries without the consent of the Commission on Appointments even
while Congress is in session.

Issue: WON the President can issue appointments in an acting capacity to


department secretaries while Congress is in session.

Held: Yes. The essence of an appointment in an acting capacity is its temporary


nature. It is a stop-gap measure intended to fill an office for a limited time until the
appointment of a permanent occupant to the office. In case of vacancy in an office
occupied by an alter ego of the President, such as the office of a department
secretary, the President must necessarily appoint an alter ego of her choice as acting
secretary before the permanent appointee of her choice could assume office.

The office of a department secretary may become vacant while Congress is in


session. Since a department secretary is the alter ego of the President, the acting
appointee to the office must necessarily have the President’s confidence. Thus, by
the very nature of the office of a department secretary, the President must appoint in
an acting capacity a person of her choice even while Congress is in session.

Ad interim appointments and acting appointments are both effective upon


acceptance. But ad-interim appointments are extended only during a recess of
Congress, whereas acting appointments may be extended any time there is a
vacancy. Moreover ad-interim appointments are submitted to the Commission on
Appointments for confirmation or rejection; acting appointments are not submitted to
the Commission on Appointments. Acting appointments are a way of temporarily
filling important offices but, if abused, they can also be a way of circumventing the
need for confirmation by the Commission on Appointments.

The absence of abuse is readily apparent from President Arroyo’s issuance of ad


interim appointments to respondents immediately upon the recess of Congress, way
before the lapse of one year.

The Constitutionality of President Arroyo’s Issuance of Appointments to


Respondents as Acting Secretaries

Petitioners: President Arroyo should not have appointed respondents as acting


secretaries because "in case of a vacancy in the Office of a Secretary, it is only an
Undersecretary who can be designated as Acting Secretary." Petitioners base their
argument on Section 10, Chapter 2, Book IV of Executive Order No. 292 ("EO 292"),
which enumerates the powers and duties of the undersecretary. Paragraph 5 of
Section 10 reads:

SEC. 10. Powers and Duties of the Undersecretary. - The


Undersecretary shall xxx (5) Temporarily discharge the duties of the
56
EXECUTIVE DEPARTMENT
Secretary in the latter’s absence or inability to discharge his duties
for any cause or in case of vacancy of the said office, unless
otherwise provided by law. Where there are more than one
Undersecretary, the Secretary shall allocate the foregoing powers
and duties among them. The President shall likewise make the
temporary designation of Acting Secretary from among them; and xxx

Respondents: maintain that the President can issue appointments in an acting


capacity to department secretaries without the consent of the Commission on
Appointments even while Congress is in session. Respondents point to Section
1611, Article VII of the 1987 Constitution.

Respondents also rely on EO 292, which devotes a chapter to the President’s power
of appointment. Sections 16 and 17, Chapter 5, Title I, Book III of EO 292 read:

SEC. 16. Power of Appointment. — The President shall exercise the


power to appoint such officials as provided for in the Constitution and
laws.

SEC. 17. Power to Issue Temporary Designation. — (1) The


President may temporarily designate an officer already in the
government service or any other competent person to perform the
functions of an office in the executive branch, appointment to which
is vested in him by law, when: (a) the officer regularly appointed to
the office is unable to perform his duties by reason of illness, absence
or any other cause; or (b) there exists a vacancy[.]

(2) The person designated shall receive the compensation attached


to the position, unless he is already in the government service in
which case he shall receive only such additional compensation as,
with his existing salary, shall not exceed the salary authorized by law
for the position filled. The compensation hereby authorized shall be
paid out of the funds appropriated for the office or agency concerned.

(3) In no case shall a temporary designation exceed one (1) year.

11 SEC. 16. The President shall nominate and, with the consent of the Commission on Appointments, appoint the heads
of the executive departments, ambassadors, other public ministers and consuls, or officers of the armed forces from the
rank of colonel or naval captain, and other officers whose appointments are vested in him in this Constitution. He shall
also appoint all other officers of the Government whose appointments are not otherwise provided for by law, and those
whom he may be authorized by law to appoint. The Congress may, by law, vest the appointment of other officers lower
in rank in the President alone, in the courts, or in the heads of departments, agencies, commissions, or boards.

The President shall have the power to make appointments during the recess of the Congress, whether voluntary or compulsory, but
such appointments shall be effective only until disapproval by the Commission on Appointments or until the next adjournment of the
Congress.

57
EXECUTIVE DEPARTMENT
*IN SHORT---Petitioners assert that the President cannot issue appointments in an
acting capacity to department secretaries while Congress is in session because the
law does not give the President such power. In contrast, respondents insist that the
President can issue such appointments because no law prohibits such
appointments.

RULING OF THE COURT: Congress, through a law, cannot impose on the


President the obligation to appoint automatically the undersecretary as her
temporary alter ego. An alter ego, whether temporary or permanent, holds a position
of great trust and confidence. Congress, in the guise of prescribing qualifications to
an office, cannot impose on the President who her alter ego should be. The law
expressly allows the President to make such acting appointment. Section 17,
Chapter 5, Title I, Book III of EO 292 states that "[t]he President may temporarily
designate an officer already in the government service or any other competent
person to perform the functions of an office in the executive branch." Thus, the
President may even appoint in an acting capacity a person not yet in the
government service, as long as the President deems that person competent.

On the Mootness of the Petition

Solicitor General: the petition is moot because President Arroyo had extended to
respondents ad interim appointments on 23 September 2004 immediately after the
recess of Congress.

As a rule, the writ of prohibition will not lie to enjoin acts already done. However, as
an exception to the rule on mootness, courts will decide a question otherwise moot if
it is capable of repetition yet evading review. In the present case, the mootness of
the petition does not bar its resolution. The question of the constitutionality of the
President’s appointment of department secretaries in an acting capacity while
Congress is in session will arise in every such appointment.

On the Nature of the Power to Appoint

The power to appoint is essentially executive in nature, and the legislature may not
interfere with the exercise of this executive power except in those instances when
the Constitution expressly allows it to interfere. Limitations on the executive power to
appoint are construed strictly against the legislature. The scope of the legislature’s
interference in the executive’s power to appoint is limited to the power to prescribe
the qualifications to an appointive office. Congress cannot appoint a person to an
office in the guise of prescribing qualifications to that office. Neither may Congress
impose on the President the duty to appoint any particular person to an office.

However, even if the Commission on Appointments is composed of members of


Congress, the exercise of its powers is executive and not legislative. The
Commission on Appointments does not legislate when it exercises its power to give
or withhold consent to presidential appointments. Thus: The Commission on
Appointments is a creature of the Constitution. Although its membership is confined
58
EXECUTIVE DEPARTMENT
to members of Congress, said Commission is independent of Congress. The powers
of the Commission do not come from Congress, but emanate directly from the
Constitution. Hence, it is not an agent of Congress. In fact, the functions of the
Commissioner are purely executive in nature.

On Petitioners’ Standing

Solicitor General: the present petition is a quo warranto proceeding because, with
the exception of Secretary Ermita, petitioners effectively seek to oust respondents
for unlawfully exercising the powers of department secretaries. Petitioners may not
claim standing as Senators because no power of the Commission on Appointments
has been "infringed upon or violated by the President. If at all, the Commission on
Appointments as a body (rather than individual members of the Congress) may
possess standing in this case.”

Petitioners (wrong): the Court can exercise its certiorari jurisdiction over
unconstitutional acts of the President. They possess standing because President
Arroyo’s appointment of department secretaries in an acting capacity while
Congress is in session impairs the powers of Congress. Petitioners cite Sanlakas v.
Executive Secretary as basis, thus: To the extent that the powers of Congress are
impaired, so is the power of each member thereof, since his office confers a right to
participate in the exercise of the powers of that institution. An act of the Executive
which injures the institution of Congress causes a derivative but nonetheless
substantial injury, which can be questioned by a member of Congress. In such a
case, any member of Congress can have a resort to the courts.

Petitioners cannot claim standing. President Arroyo’s issuance of acting


appointments while Congress is in session impairs no power of Congress. Among
the petitioners, only the following are members of the Commission on Appointments
of the 13th Congress: Senator Enrile as Minority Floor Leader, Senator Lacson as
Assistant Minority Floor Leader, and Senator Angara, Senator Ejercito-Estrada, and
Senator Osmeña as members.

59
EXECUTIVE DEPARTMENT
SARMIENTO v MISON III
PONENTE/ OTHER PADILLA/ TEEHANKEE, MELENCIO-
OPINION HERRERA, SARMIENTO, CRUZ,
GUTIERREZ
DATE December 17, 1987
PETITIONERS ULPIANO P. SARMIENTO III AND JUANITO
G. ARCILLA
RESPONDENTS SALVADOR MISON, in his capacity as
COMMISSIONER OF THE BUREAU OF
CUSTOMS, AND GUILLERMO CARAGUE,
in his capacity as SECRETARY OF THE
DEPARTMENT OF BUDGET, respondents,
COMMISSION ON APPOINTMENTS
DISPOSITION Consequently, we rule that the President of
the Philippines acted within her
constitutional authority and power in
appointing respondent Salvador Mison,
Commissioner of the Bureau of Customs,
without submitting his nomination to the
Commission on Appointments for
60
EXECUTIVE DEPARTMENT
confirmation. He is thus entitled to exercise
the full authority and functions of the office
and to receive all the salaries and
emoluments pertaining thereto.
DOCTRINE ACCDG TO SUBJECT TO CONFIRMATION OF
THE SYLLABUS COMMISSION ON APPOINTMENTS
It is evident that the position of Commissioner
of the Bureau of Customs (a bureau head) is
not one of those within the first group of
appointments where the consent of the
Commission on Appointments is required. As
a matter of fact, as already pointed out, while
the 1935 Constitution includes "heads of
bureaus" among those officers whose
appointments need the consent of the
Commission on Appointments, the 1987
Constitution on the other hand, deliberately
excluded the position of "heads of bureaus"
from appointments that need the consent
(confirmation) of the Commission on
Appointments. Moreover, the President is
expressly authorized by law12 to appoint the
Commissioner of the Bureau of Customs.
QUICK FACTS Bureau heads—to confirm or not to confirm?

FACTS:

Respondent Salvador Mison was appointed as the Commissioner of the Bureau of


Customs by then President (Corazon) Aquino. The said appointment made by the
President is being questioned13 by petitioner Ulpiano Sarmiento III and Juanito Arcilla
who are both taxpayers, members of the bar, and both Constitutional law professors,
stating that the said appointment is not valid since the appointment was not submitted
to the Commission on Appointments (COA) for approval. Under the Constitution, the
appointments made for the "Heads of Bureau" requires the confirmation from COA.
As such they also pray that respondent Guillermo Carague, as Secretary of the
Department of Budget, be prohibited from disbursing Mison's salaries and
emoluments.

12
Tariff and Customs Code of the Philippines
13Because of the demands of public interest, including the need for stability in the public service, the Court resolved to give due
course to the petition and decide, setting aside the finer procedural questions of whether prohibition is the proper remedy to test
respondent Mison's right to the Office of Commissioner of the Bureau of Customs and of whether the petitioners have a standing to
bring this suit. This case assumes added significance because, at bottom line, it involves a conflict between two (2) great departments
of government, the Executive and Legislative Departments. It also occurs early in the life of the 1987 Constitution.
61
EXECUTIVE DEPARTMENT
ISSUE: WHETHER OR NOT the appointment made by the President without the
confirmation from COA is valid.

HELD:

YES, under the 1987 Constitution, Heads of Bureau are removed from the list of
officers that needed confirmation from the COA. It enumerated the four (4) groups
whom the President shall appoint:

 Heads of the Executive Departments, Ambassadors, other public minister or


consuls, Officers of the Armed Forces from the rank of Colonel or Naval
Captain, and Other officers whose appointments are vested in him in him in
this Constitution;

The above-mentioned circumstance is the only instance where the appointment


made by the President that requires approval from the COA and the following
instances are those which does not require approval from COA:

 All other Officers of the Government whose appointments are not otherwise
provided by law;
 Those whom the President may be authorized by law to appoint; and
 Officers lower in rank whose appointments the Congress may by law vest in
the President alone.

The Court construed the applicable constitutional provisions, not in accordance


with how the executive or the legislative department may want them construed, but in
accordance with what they say and provide.

It is evident that the position of Commissioner of the Bureau of Customs (a bureau


head) is not one of those within the first group of appointments where the consent of
the Commission on Appointments is required. As a matter of fact, as already pointed
out, while the 1935 Constitution includes "heads of bureaus" among those officers
whose appointments need the consent of the Commission on Appointments, the 1987
Constitution on the other hand, deliberately excluded the position of "heads of
bureaus" from appointments that need the consent (confirmation) of the Commission
on Appointments. Moreover, the President is expressly authorized by law14 to appoint
the Commissioner of the Bureau of Customs.

The first group of officers is clearly appointed with the consent of the Commission on
Appointments. Appointments of such officers are initiated by nomination and, if the
nomination is confirmed by the Commission on Appointments, the President appoints.

The second, third and fourth groups of officers:

14Tariff and Customs Code of the Philippines


62
EXECUTIVE DEPARTMENT
In the 1935 Constitution, almost all presidential appointments required the consent
(confirmation) of the Commission on Appointments. It is now a sad part of our political
history that the power of confirmation by the Commission on Appointments, under the
1935 Constitution, transformed that commission, many times, into a venue of "horse-
trading" and similar malpractices. On the other hand, the 1973 Constitution placed the
absolute power of appointment in the President with hardly any check on the part of
the legislature.

Given the above two (2) extremes, one, in the 1935 Constitution and the other, in the
1973 Constitution, it is not difficult for the Court to state that the framers of the 1987
Constitution and the people adopting it, struck a "middle ground" by requiring the
consent (confirmation) of the Commission on Appointments for the first group of
appointments and leaving to the President, without such confirmation, the
appointment of other officers, i.e., those in the second and third groups as well as
those in the fourth group, i.e., officers of lower rank.

In the course of the debates on the text of Section 16, there were two (2) major
changes proposed and approved by the Commission. These were (1) the exclusion
of the appointments of heads of bureaus from the requirement of confirmation by the
Commission on Appointments; and (2) the exclusion of appointments made under the
second sentence of the section from the same requirement.

It is contended by amicus curiae, Senator Neptali Gonzales, that the second sentence
of Sec. 16, Article VII reading with particular reference to the word "also," implies that
the President shall "in like manner" appoint the officers mentioned in said second
sentence. In other words, the President shall appoint the officers mentioned in said
second sentence in the same manner as he appoints officers mentioned in the first
sentence, that is, by nomination and with the consent (confirmation) of the
Commission on Appointments. Amicus curiae's reliance on the word "also" in said
second sentence is not necessarily supportive of the conclusion he arrives at. For, as
the Solicitor General argues, the word "also" could mean "in addition; as well; besides,
too" (Webster's International Dictionary, p. 62, 1981 edition) which meanings could,
on the contrary, stress that the word "also" in said second sentence means that the
President, in addition to nominating and, with the consent of the Commission on
Appointments, appointing the officers enumerated in the first sentence, can appoint
(without such consent (confirmation) the officers mentioned in the second sentence.
Rather than limit the area of consideration to the possible meanings of the word "also"
as used in the context of said second sentence, the Court has chosen to derive
significance from the fact that the first sentence speaks of nomination by the President
and appointment by the President with the consent of the Commission on
Appointments, whereas, the second sentence speaks only of appointment by the
President.

As to the fourth group of officers whom the President can appoint, the intervenor
Commission on Appointments underscores the third sentence in Sec. 16, Article VII
of the 1987 Constitution, and argues that, since a law is needed to vest the
63
EXECUTIVE DEPARTMENT
appointment of lower-ranked officers in the President alone, this implies that, in the
absence of such a law, lower-ranked officers have to be appointed by the President
subject to confirmation by the Commission on Appointments; and, if this is so, as to
lower-ranked officers, it follows that higher-ranked officers should be appointed by the
President, subject also to confirmation by the Commission on Appointments. The
respondents, on the other hand, submit that the third sentence of Sec. 16, Article VII,
abovequoted, merely declares that, as to lower-ranked officers, the Congress may by
law vest their appointment in the President, in the courts, or in the heads of the various
departments, agencies, commissions, or boards in the government. No reason
however is submitted for the use of the word "alone" in said third sentence.

The Court is not impressed by both arguments. It is of the considered opinion, after a
careful study of the deliberations of the 1986 Constitutional Commission, that the use
of the word alone" after the word "President" in said third sentence of Sec. 16, Article
VII is, more than anything else, a slip or lapsus in draftmanship.

CASE/S CITED:

Gold Creek Mining Corp. vs. Rodriguez:The fundamental principle of constitutional


construction is to give effect to the intent of the framers of the organic law and of the
people adopting it. The intention to which force is to be given is that which is embodied
and expressed in the constitutional provisions themselves. (Abad Santos, J)

SEPARATE OPINIONS

TEEHANKEE, C.J., concurring:

It should be noted that the Court's decision at bar does not mention nor deal with the
Manifestation of December 1, 1987 filed by the intervenor that Senate Bill No. 137
entitled "An Act Providing For the Confirmation By the Commission on Appointments
of All Nominations and Appointments Made by the President of the Philippines" was
passed on 23 October 1987 and was "set for perusal by the House of
Representatives. " This omission has been deliberate. The Court has resolved the
case at bar on the basis of the issues joined by the parties. The contingency of
approval of the bill mentioned by intervenor clearly has no bearing on and cannot
affect retroactively the validity of the direct appointment of respondent Mison and
other appointees. Any discussion of the reported bill and its validity or invalidity is
premature and irrelevant and outside the scope of the issues resolved in the case at
bar.

MELENCIO-HERRERA, J., concurring:

Agrees with the decision but worries for the issues raised by Justice Cruz.

The interpretation given by the majority may, indeed, lead to some incongruous
situations as stressed in the dissenting opinion of Justice Cruz. The remedy therefor
addresses itself to the future. The task of constitutional construction is to ascertain the
64
EXECUTIVE DEPARTMENT
intent of the framers of the Constitution and thereafter to assure its realization (J.M.
Tuason & Co., Inc. vs. Land Tenure Administration, G.R. No. 21064, February 18,
1970, 31 SCRA 413). And the primary source from which to ascertain constitutional
intent is the language of the Constitution itself.

SARMIENTO, J., concurring:

It is clear from the Constitution itself that not all Presidential appointments are subject
to prior Congressional confirmation.

GUTIERREZ, JR., J., dissenting:

The Constitution, as the supreme law of the land, should never have any of its
provisions interpreted in a manner that results in absurd or irrational consequences.
The Commission on Appointments is an important constitutional body which helps
give fuller expression to the principles inherent in our presidential system of
government. Its functions cannot be made innocuous or unreasonably diminished to
the confirmation of a limited number of appointees. In the same manner that the
President shares in the enactment of laws which govern the nation, the legislature,
through its Commission on Appointments, gives assurance that only those who can
pass the scrutiny of both the President and Congress will help run the country as
officers holding high appointive positions.

CRUZ, J., dissenting:

Following this interpretation, the Undersecretary of Foreign Affairs, who is not the
head of his department, does not have to be confirmed by the Commission on
Appointments, but the ordinary consul, who is under his jurisdiction, must be
confirmed. The colonel is by any standard lower in rank than the Chairman of the
Commission on Human Rights, which was created by the Constitution; yet the former
is subject to confirmation but the latter is not because he does not come under the
first sentence. The Special Prosecutor, whose appointment is not vested by the
Constitution in the President, is not subject to confirmation under the first sentence,
and neither are the Governor of the Central Bank and the members of the Monetary
Board because they fall under the second sentence as interpreted by the majority
opinion. Yet in the case of the multi-sectoral members of the regional consultative
commission, whose appointment is vested by the Constitution in the President under
Article X, Section 18, their confirmation is required although their rank is decidedly
lower.

The majority opinion says that the second sentence is the exception to the first
sentence and holds that the two sets of officers specified therein may be appointed
by the President without the concurrence of the Commission on Appointments. This
interpretation is pregnant with mischievous if not also ridiculous results that
presumably were not envisioned by the framers. One may wonder why it was felt
necessary to include the second sentence at all, considering the majority opinion that
the enumeration in the first sentence of the officers subject to confirmation is exclusive
65
EXECUTIVE DEPARTMENT
on the basis of expressio unius est exclusio alterius. If that be so, the first sentence
would have been sufficient by itself to convey the Idea that all other appointees of the
President would not need confirmation. One may also ask why, if the officers
mentioned in the second sentence do not need confirmation, it was still felt necessary
to provide in the third sentence that the appointment of the other officers lower in rank
will also not need confirmation as long as their appointment is vested by law in the
President alone. The third sentence would appear to be superfluous, too, again in
view of the first sentence.

My own reading is that the second sentence is but a continuation of the Idea
expressed in the first sentence and simply mentions the other officers appointed by
the President who are also subject to confirmation. The second sentence is the later
expression of the will of the framers and so must be interpreted as complementing the
rule embodied in the first sentence or, if necessary, reversing the original intention to
exempt bureau directors from confirmation. I repeat that there were no debates on
this matter as far as I know, which simply means that my humble conjecture on the
meaning of Section 16 is as arguable, at least, as the suppositions of the majority. We
read and rely on the same records. At any rate, this view is more consistent with the
general purpose of Article VII, which, to repeat, was to reduce the powers of the
Presidency.

MANALO v SISTOZA
PONENTE/ OTHER PURISIMA
OPINION
DATE August 11, 1999
PETITIONERS JESULITO A. MANALO
RESPONDENTS Pedro G. Sistoza, Regino Aro III, Nicasio Ma.
Custodio, Guillermo Domondon, Raymundo
L. Logan, Wilfredo R. Reotutar, Felino C.
Pacheco, Jr., Ruben J. Cruz, Geronimo B.
Valderrama, Merardo G. Abaya, Everlino B.
Nartatez, Enrique T. Bulan, Pedro J. Navarro,
Dominador M. Mangubat, Rodolfo M. Garcia
And Honorable Salvador M. Enriquez II In His
Capacity as Secretary of Budget and
Management
DISPOSITION DISMISSED

66
EXECUTIVE DEPARTMENT
DOCTRINE ACCDG TO SUBJECT TO CONFIRMATION OF
THE SYLLABUS COMMISSION ON APPOINTMENTS
The first group are the only ones whose
appointments are required by the
Constitution to be affirmed by the
Commission on Appointments. All others
need not be confirmed.
QUICK FACTS PNP appointments, wala pa si Bato, si
Sistoza pa lang

FACTS: In 1990, Republic Act No. 6975 was passed. This law created the Department
of Interior and Local Government. Said law, under Sections 26 and 31 thereof, also
provided on the manner as to how officers of the Philippine National Police are to be
appointed. It was provided that the PNP Chief as well as certain police officers
including Directors and Chief Superintendents, after being appointed by the
President, must be confirmed by the Commission on Appointments before said
officers can take their office.

In 1992, then president Corazon Aquino appointed Pedro Sistoza et al as Directors


and Chief Superintendents within the PNP. Said appointments were not confirmed by
the Commission on Appointments hence, Jesulito Manalo questioned the validity of
the appointments made. He insists that without the confirmation by the Commission,
Sistoza et al are acting without jurisdiction, their appointment being contrary to the
provisions of R.A. 6975.

He then went to the Supreme Court asking the court to carry out the provisions of the
said law. Manalo also insists that the law is a valid law, as it enjoys the presumption
of constitutionality, and hence, it must be carried out by the courts.

ISSUE: Whether or not Sections 26 and 31 of R.A. No. 6975 are valid.

HELD: No. Said provisions are unconstitutional. It is true that prior to this case, as
with all other laws, R.A. 6975 enjoys the presumption of constitutionality. As such,
laws enacted by Congress must be respected by courts and as much as possible,
courts must avoid delving into the constitutionality of a law. However, it is also the
duty of the courts, as guardians of the Constitution, to see to it that every law passed
by Congress is not repugnant to the Constitution.

Under Section 16, Article VII of the Constitution, there are four groups of officers of
the government to be appointed by the President:

First, the heads of the executive departments, ambassadors, other public


ministers and consuls, officers of the armed forces from the rank of colonel or

67
EXECUTIVE DEPARTMENT
naval captain, and other officers whose appointments are vested in him in this
Constitution; xxx

The first group are the only ones whose appointments are required by the Constitution
to be affirmed by the Commission on Appointments. All others need not be confirmed.
Officers of the PNP are not included therein.Sections 26 and 31 of R.A. 6975 are void
for amending the provisions set forth in the Constitution.

Is the Philippine National Police akin to the Armed Forces of the Philippines and
therefore, the appointments of police officers whose rank is equal to that of
colonel or naval captain require confirmation by the Commission on
Appointments?

NO. The Philippine National Police is separate and distinct from the Armed Forces of
the Philippines. The Constitution, no less, sets forth the distinction. Thereunder, the
police force is different from and independent of the armed forces and the ranks in
the military are not similar to those in the Philippine National Police. Thus, directors
and chief superintendents of the PNP, such as the herein respondent police officers,
do not fall under the first category of presidential appointees requiring the confirmation
by the Commission on Appointments.

CASE/S CITED:

Tarrosa vs. Singson: Congress cannot by law expand the power of confirmation of the
Commission on Appointments and require confirmation of appointments of other
government officials not mentioned in the first sentence of Section 16 of Article VII of
the 1987 Constitution.

AYTONA v CASTILLO
PONENTE/ OTHER BENGZON, C.J./ PADILLA,
OPINION
DATE January 19, 1962
PETITIONERS DOMINADOR R. AYTONA
RESPONDENTS ANDRES V. CASTILLO, ET AL.
DISPOSITION DISMISSED
DOCTRINE ACCDG TO LIMITATIONS
THE SYLLABUS

68
EXECUTIVE DEPARTMENT
ad interim appointments made by the
President during the recess of the Congress
are effective only until disapproval by the
Commission on Appointments or until the
next adjournment of the Congress
QUICK FACTS Photofinish appointment; last-minute
appointments; manok ni Garcia v manok ni
Macapagal

FACTS: On December 29, 1961, Carlos P. Garcia, who was still President that time,
made last minute appointments while the Commission on Appointments was not in
session. Said last minute appointment included Dominador R. Aytona, who was
appointed as ad interim Governor of Central Bank. The latter took oath on the same
day.

At noon on December 30, 1961, President-elect Diosdado Macapagal assumed


office. He issued Administrative Order No. 2 on December 31, 1961 recalling,
withdrawing and canceling all ad interim appointments made by President Garcia
after December 13, 1961, which was the date when Macapagal was proclaimed
President by the Congress. He then appointed Andres V. Castillo as ad interim
Governor of the Central Bank and the latter qualified immediately.

On January 2, 1962, both exercised the powers of their office. However, Aytona was
prevented from holding office the following day and thus instituted a quo warranto
proceeding, challenging Castillo’s right to exercise the powers of the Governor of the
Central Bank. Aytona claims that he was validly appointed and had qualified for the
post, therefore making Castillo’s appointment void. Castillo then contended that
Aytona’s appointment had already been revoked by Administrative Order No. 2
issued by President Macapagal.

ISSUE:

Whether President Diosdado Macapagal had power to issue the order of


cancellation of the ad interim appointments made by President Carlos P.
Garcia even after the appointees had already qualified.

RULING: Upon the ground of separation of powers, the court resolved that it must
decline and refuse jurisdiction in disregarding the Presidential Administrative Order
No. 2, canceling such “midnight” or “last-minute” appointments.
Of course, nobody will assert that President Garcia ceased to be such earlier than at
noon of December 30, 1961. But it is common sense to believe that after the
proclamation of the election of President Macapagal, his was no more than a "care-
69
EXECUTIVE DEPARTMENT
taker" administration. He was duty bound to prepare for the orderly transfer of
authority the incoming President, and he should not do acts which he ought to know,
would embarrass or obstruct the policies of his successor. The time for debate had
passed; the electorate had spoken. It was not for him to use powers as incumbent
President to continue the political warfare that had ended or to avail himself of
presidential prerogatives to serve partisan purposes.
Normally, when the President makes appointments the consent of the Commission
on Appointments, he has benefit of their advice. When he makes ad interim
appointments, he exercises a special prerogative and is bound to be prudent to insure
approval of his selection either previous consultation with the members of the
Commission or by thereafter explaining to them the reason such selection. Where,
however, as in this case, the Commission on Appointments that will consider the
appointees is different from that existing at the time of the appointment and where the
names are to be submitted by successor, who may not wholly approve of the
selections, the President should be doubly careful in extending such appointments.
Now, it is hard to believe that in signing 350 appointments in one night, President
Garcia exercised such "double care" which was required and expected of him; and
therefore, there seems to be force to the contention that these appointments fall
beyond the intent and spirit of the constitutional provision granting to the Executive
authority to issue ad interim appointments.
Of course, the Court is aware of many precedents to the effect that once an
appointment has been issued, it cannot be reconsidered, specially where the
appointee has qualified. But none of them refer to mass ad interim appointments
(three-hundred and fifty), issued in the last hours of an outgoing Chief Executive, in a
setting similar to that outlined herein.
Incidentally, it should be stated that the underlying reason for denying the power to
revoke after the appointee has qualified is the latter's equitable rights. Yet it is doubtful
if such equity might be successfully set up in the present situation, considering the
rush conditional appointments, hurried maneuvers and other happenings detracting
from that degree of good faith, morality and propriety which form the basic foundation
of claims to equitable relief. The appointees, it might be argued, wittingly or unwittingly
cooperated with the stratagem to beat the deadline, whatever the resultant
consequences to the dignity and efficiency of the public service. Needless to say,
there are instances wherein not only strict legality, but also fairness, justice and
righteousness should be taken into account.

SEPARATE OPINIONS
PADILLA, J., concurring:

70
EXECUTIVE DEPARTMENT
If the ad interim appointments made by the President during the recess of the
Congress are effective only until disapproval by the Commission on Appointments or
until the next adjournment of the Congress — a limitation on the power of the
President — there is a cogent and strong reason for holding to be the intent of the
framers of the Constitution that such appointments made by him ceased to be valid
and effective after the term of the Congress existing at the time of the making of such
appointments had ended or expired. The end or expiration of the of the Congress
existing at the time of the making of the ad interim appointments by the President is
a stronger cause or reason for the lapse or ineffectuality of such appointments than
"the next adjournment of the Congress." Since that Congress no longer exists and
hence can no longer convene and then "adjourn." The effectivity and validity of the
appointment of the petitioner as Governor of the Central Bank ceased, lapsed and
expired on thirtieth of December 1961. He is no longer entitled hold the office to which
he had been appointed. My vote, therefore, is for the denial of the petition.
DIZON, J., Concurring:
Same as Justice Padilla’ reasoning to serve as an additional ground justifying denial
of the petition under consideration.
BAUTISTA ANGELO, J., concurring:
CONCEPCION, J., concurring in part and dissenting in part:
1. Once an appointee has qualified, he acquires a legal, not merely equitable right,
which is protected not only by statute, but, also by the Constitution, for it cannot be
taken away from him, either by revocation of the appointment or by removal, except
for cause, and with previous notice and hearing, consistently with said Section 4 of
Article XII of our fundamental law, and with the constitutional requirement of due
process.
2. The case of Tipton vs. Parker (74 S. W., 298) has been cited in support of the
theory that Congress of the Philippines was not in "recess" on December 29, 1961,
and that, accordingly, ad interim appointments could not validly be made in such date.
The question involved in said case was whether a committee of the Senate of
Arkansas could be authorized by the same to function after the adjournment sine die
of the regular session of the state General Assembly. The State Supreme Court
considered as decisive authority the view expressed by Judge Cooley, to the effect
that a legislative committee "has no authority to sit during a recess of a House which
appointed him, without its permission to that effect". The issue thus hinged on the
meaning of the term "recess" as used by Judge Cooley. Resolving this question, said
court held that the recess referred to by Judge Cooley was "only the intermission
between the sittings of the same body at its regular or adjourned session and not to
the interval between the final adjournment of one body and the convening of another
at the next regular session"..

71
EXECUTIVE DEPARTMENT
3. The case of McChesney vs. Sampson (23 S. W. 2d. 584) has, also, been invoked
in support of the proposition that "an ad interim appointment is not complete until the
appointee takes the oath of office and actually takes possession of the position or
enters upon the discharge of its duties" and that, before such actual taking of
possession, though after the oath taking, the appointee may be removed without
cause.
4. Most, if not all appointments made by the President have two (2) aspects, namely,
the legal and the political. The first refers to his authority to make the appointment.
The second deals with the wisdom in the exercise of such authority, as well as with
its propriety. Whether given vacancy or number of vacancies should be filled, or who
among several qualified persons shall be chosen, or whether a given appointment or
number of appointment will favor the political party to whom the power of appointment
belongs and will injure the interest of a rival political party and to what extent, are, to
my mind, essentially and typically political matters. Hence, I believe that the question
whether certain appointments should be sanctioned or turned down by reason of the
improper, immoral or malevolent motives with which said matters were allegedly
handled is, likewise, clearly political, and as such, its determination belongs, not to
the courts of justice but to the political organ established precisely to check possible
abuses in the exercise of the appointing power — the Commission on Appointments.
5. In the present case, we have completely reversed our stand on the principle of
separation of powers. We have inquired into the motives of the Executive department
in making the appointments in question, although it is well settled, under the
aforementioned principle, that: .
BARRERA, J., dissenting:
Be this as it may, whatever may be our personal views on this matter, I agree with Mr.
Justice Concepcion that not all wrongs or even abuse of power can be corrected by
the exercise of the high prerogatives of the Supreme Court vested in it by the
Constitution. As I take it, the higher and more delicate is the prerogative, the greater
should be the degree of self-restraint in the exercise thereof, lest the fine and tested
scale of checks and balances set up by the Constitution be jarred. In the same manner
that we expect circumspection and care, even double care, on the part of the other
two co-equal coordinate departments of the government, so must we be most
cautious and slow in judging the morality, propriety and good faith involved in the
actuations of the other departments in matters coming within their competence. The
remedy, I believe, under the circumstances is with the Commission on Appointments
to which the appointments have been submitted. The more fact that it is expected that
the Commission on Appointments would be controlled by the party of the outgoing
President is immaterial, because legal processes can not be made to depend upon
the fortunes of political parties, for there is still the ultimate remedy by the people in
all authority. At any rate, as has already been aptly said: the judiciary is not the
repository of remedies for all political or social evils, and that the judicial department
72
EXECUTIVE DEPARTMENT
has no power to revise even arbitrary or unfair action of the other departments taken
in pursuance of the power committed exclusively to those departments by the
Constitution.
May I add: all the scandalous circumstances brought to the attention of this Court did
not link the petitioner herein, save for the fact that this appointment was extended on
the same day as those issued under the unusual and irregular circumstances
attending the other appointments. If at all, there is evidence in favor of Aytona to the
effect that insofar as he is concerned, his appointment to the position of Governor of
the Central Bank has been under consideration for a long time and that he is qualified
for the position. It can not, therefore be said that with respect to him there was no
mature deliberation and due consideration of his qualifications and of the need of the
service. he charge was made that the position of Governor of the Central Bank has
been vacant for several months and that the President should have filled it earlier.
Summarizing, I would say that all the circumstances cited by the respondents that
have surrounded the issuance of the appointments in question, have to do with the
mode or manner of the exercise of the authority to make the appointment, quite apart
from the existence of the authority itself. The observance of good faith, morality and
propriety by the other two co-equal coordinate departments in the performance of their
functions must be secured by their sense of duty and official oath hand not by any
supervisory power of the courts.

IN RE: APPOINTMENTS OF VALENZUELA AND VALLARTA


A.M. No. 98-5-01-SC
PONENTE/ OTHER NARVASA, CJ
OPINION
DATE November 9, 1998
PETITIONERS

73
EXECUTIVE DEPARTMENT
RESPONDENTS
DISPOSITION Appointments VOID
DOCTRINE ACCDG TO LIMITATIONS
THE SYLLABUS
During the period stated in Sec. 15, Art. VII of
the Constitution “two months immediately
before the next presidential elections and up
to the end of his term” the President is neither
required to make appointments to the courts
nor allowed to do so; and that Secs. 4(1) and
9 of Art. VIII simply mean that the President
is required to fill vacancies in the courts within
the time frames provided therein unless
prohibited by Sec. 15 of Art. VII. This
prohibition on appointments comes into effect
once every 6 years.
QUICK FACTS Landmark case: midnight appointments

Facts:
On March 30, 1998, The President signed appointments of Hon. Mateo Valenzuela
and Hon. Placido Vallarta as Judges of RTC-Bago City and Cabanatuan City,
respectively. These appointments were deliberated, as it seemed to be expressly
prohibited by Art 7 Sec 1515 of the Constitution.
A meeting was held on March 9, 1998 by the Judicial and Bar Council to discuss
the constitutionality of appointments to the Court of Appeals (CA) in light of the
forthcoming 1998 Presidential elections. Senior Associate Justice Florenz Regalado,
Consultant of the Council and Member of the 1986 Constitutional Commission, was
in the position that “election ban had no application to the CA based on the
Commission’s records”. This hypothesis was then submitted to the President for
consideration together with the Council’s nominations for 8 vacancies in the CA.
The Chief Justice (CJ) received on April 6, 1998, an official communication
from the Executive Secretary transmitting the appointments of 8 Associate Justices
of CA duly signed on March 11, 1998 (day immediately before the commencement of
the ban on appointments), which implies that the President’s Office did not agree with
the hypothesis.

15
Two months immediately before the next presidential elections and up to the end of his term, a President or Acting President shall
not make appointments, except temporary appointments to executive positions when continued vacancies therein will prejudice public
service or endanger public safety.

74
EXECUTIVE DEPARTMENT
The President, addressed to the JBC, requested on May 4, 1998 the
transmission of the “list of final nominees” for the vacancy in view of the 90 days
imposed by the Constitution (from Feb 13, date present vacancy occurred). In behalf
of the JBC, CJ sent the reply on May 6 that no session has been scheduled after the
May elections for the reason that they apparently did not share the same view
(hypothesis) proposed by the JBC shown by the uniformly dated March 11, 1998
appointments. However, it appeared that the Justice Secretary and the other
members of the Council took action without waiting for the CJ reply. This prompted
CJ to call for a meeting on May 7. On this day, CJ received a letter from the President
in reply of the May 6 letter where the President expressed his view that Article 7 Sec
15 only applied to executive appointments, the whole article being entitled
“EXECUTIVE DEPT”. He posited that appointments in the Judiciary have special and
specific provisions16.
On May 12, CJ received from Malacañang, the appointments of the 2
Judges of the RTC mentioned. Considering the pending proceedings and
deliberations on this matter, the Court resolved by refraining the appointees from
taking their oaths. However, Judge Valenzuela took oath in May 14, 1998 claiming
he did so without knowledge on the on-going deliberations. It should be noted that
the originals of the appointments for both judges had been sent to and received by
the CJ on May 12 and is still in the latter’s office and had not been transmitted yet.
According to Judge Valenzuela, he did so because of the May 7 Malacañang copy of
his appointment.
In construing Article 7 and 8: when there are no presidential elections, Art.
8 shall apply where vacancies in SC shall be filled within 90 days otherwise prohibition
in Art. 7 must be considered where the President shall not make any appointments.
According to Fr. Bernas, the reason for prohibition is in order not to tie the hands of
the incoming Pres through midnight appointments.
ISSUE: Whether, during the period of the ban on appointments imposed by
Section 15, Article VII of the, Constitution, the President is nonetheless required
to fill vacancies in the judiciary, in view of Sections 4(1) and 9 of Article VIII;
whether he can make appointments to the judiciary during the period of the ban
in the interest of public service.
HELD:
1. The provision of the Constitution material to the inquiry at bar read as follows:

16 Article 8 Sec 4: “The Supreme Court shall be composed of a Chief Justice and fourteen Associate Justices. It may sit en banc or
in its discretion, in divisions of three, five, or seven Members. Any vacancy shall be filled within ninety days from the occurrence
thereof.” AND Article 8 Sec 9: “The Members of the Supreme Court and judges in lower courts shall be appointed by the President
from the list of at least three nominees prepared by the Judicial and Bar Council for every vacancy. Such appointments need no
confirmation.

75
EXECUTIVE DEPARTMENT
Section15, Article VII:
"Two months immediately before the next presidential elections and up to the end of
his term, a President or Acting President shall not make appointments, except
temporary appointments to execute positions when continued vacancies therein will
prejudice public service or endanger public safety."

Section 4 (1), Article VIII:


"The Supreme Court shall be composed of a Chief Justice and fourteen Associate
Justices. It may sit en banc or in its discretion, in divisions of three, five, or seven
Members. Any vacancy shall be filled within ninety days from the occurrence thereof."
Section 9, Article VIII:
"The Members of the Supreme Court and judges in lower courts shall be appointed
by the President from the list of at least three nominees prepared by the Judicial and
Bar Council for every vacancy. Such appointments need no confirmation.
For the lower courts, the President shall issue the appointments within ninety days
from the submission of the list."

2. The Court's view is that during the period stated in Section 15, Article VII of the
Constitution - "(t)wo months immediately before the next presidential elections and up
to the end of his term" - the President is neither required to make appointments to the
courts nor allowed to do so; and that Sections 4(1) and 9 of Article VIII simply mean
that the President is required to fill vacancies in the courts within the time frames
provided therein unless prohibited by Section 15 of Article VII. It is noteworthy that the
prohibition on appointments comes into effect only once every six years.
3. The journal of the Commission which drew up the present Constitution discloses
that the original proposal was to have an eleven-member Supreme Court.
Commissioner Eulogio Lerum wanted to increase the number of Justices to fifteen.
He also wished to ensure that that number would not be reduced for any appreciable
length of time (even only temporarily), and to this end proposed that any vacancy
"must be filled within two months from the date that the vacancy occurs." His proposal
to have a 15-member Court was not initially adopted. Persisting however in his desire
to make certain that the size of the Court would not be decreased for any substantial
period as a result of vacancies, Lerum proposed the insertion in the provision (anent
the Court's membership) of the same mandate that "IN CASE OF ANY VACANCY,
THE SAME SHALL BE FILLED WITHIN TWO MONTHS FROM OCCURRENCE
THEREOF." He later agreed to suggestions to make the period three, instead of two,
months. As thus amended, the proposal was approved. As it turned out, however, the
Commission ultimately agreed on a fifteen-member Court. Thus it was that the section
fixing the composition of the Supreme Court came to include a command to fill up any
vacancy therein within 90 days from its occurrence.
76
EXECUTIVE DEPARTMENT
4. In this connection, it may be pointed out that that instruction that any "vacancy shall
be filled within ninety days" (in the last sentence of Section 4 (1) of Article VIII)
contrasts with the prohibition in Section 15, Article VII, which is couched in stronger
negative language - that "a President or Acting President shall not make
appointments…"

5. The commission later approved a proposal of Commissioner Hilario G. Davide, Jr.


(now a Member of this Court) to add to what is now Section 9 of Article VIII, the
following paragraph: "WITH RESPECT TO LOWER COURTS, THE PRESIDENT
SHALL ISSUE THE APPOINTMENT WITHIN NINETY DAYS FROM THE
SUBMISSION OF THE LIST" (of nominees by the Judicial and Bar Council to the
President). Davide stated that his purpose was to provide a "uniform rule" for lower
courts. According to him, the 90-day period should be counted from submission of the
list of nominees to the President in view of the possibility that the President might
reject the list submitted to him and the JBC thus need more time to submit a new one.
6. On the other hand, Section 15, Article VII - which in effect deprives the President
of his appointing power "two months immediately before the next presidential
elections up to the end of his term" - was approved without discussion.

7. Now, it appears that Section 15, Article VII is directed against two types of
appointments: (1) those made for buying votes and (2) those made for partisan
considerations. The first refers to those appointments made within the two months
preceding a Presidential election and are similar to those which are declared election
offenses in the Omnibus Election Code, viz.:
SEC. 261. Prohibited Acts. - The following shall be guilty of an election offense:
(a) Vote buying and vote selling - (1) Any person who gives, offers or promises money
or anything of value, gives or promises any office or employment, franchise or grant,
public or private, or makes or offers to make an expenditure, directly or indirectly, or
cause an expenditure to be made to any person, association, corporation, entity, or
community in order to induce anyone or the public in general to vote for or against
any candidate or withhold his vote in the election, or to vote for or against any aspirant
for the nomination of choice of a candidate in a convention or similar selection process
of a political party.
….
(g) Appointment of new employees, creation of new position, promotion, or giving
salary increases. - During the period of forty five days before a regular election and
thirty days before a special election, (1) any head, official or appointing officer of a
government office, agency or instrumentality whether national or local, including
government-owned or controlled corporations, who appoints or hires any new
employee, whether provisional, temporary, or casual, or creates and fills any new
position, except upon prior authority of the Commission. The Commission shall not
77
EXECUTIVE DEPARTMENT
grant the authority sought unless, it is satisfied that the position to be filled is essential
to the proper functioning of the office or agency concerned, and that the position shall
not be filled in a manner that may influence the election.

8. The second type of appointments prohibited by Section 15, Article VII consists of
the so-called "midnight" appointments. In Aytona v. Castillo, it was held that after the
proclamation of Diosdado Macapagal as duly elected President, President Carlos P.
Garcia, who was defeated in his bid for reelection, became no more than a "caretaker"
administrator whose duty was to "prepare for the orderly transfer of authority to the
incoming President." Said the Court
"The filling up of vacancies in important positions, if few, and so spaced as to afford
some assurance of deliberate action and careful consideration of the need for the
appointment and appointee's qualifications may undoubtedly be permitted. But the
issuance of 350 appointments in one night and the planned induction of almost all of
them in a few hours before the inauguration of the new President may, with some
reason, be regarded by the latter as an abuse of Presidential prerogatives, the steps
taken being apparently a mere partisan effort to fill all vacant positions irrespective of
fitness and other conditions, and thereby to deprive the new administration of an
opportunity to make the corresponding appointments."
As indicated, the Court recognized that there may well be appointments to important
positions which have to be made even after the proclamation of the new President.
Such appointments, so long as they are "few and so spaced as to afford some
assurance of deliberate action and careful consideration of the need for the
appointment and the appointee's qualifications," can be made by the outgoing
President. Accordingly, several appointments made by President Garcia, which were
shown to have been well considered, were upheld.
9. Section 15, Article VII has a broader scope than the Aytona ruling. It may not
unreasonably be deemed to contemplate not only "midnight" appointments - those
made obviously for partisan reasons as shown by their number and the time of their
making - but also appointments presumed made for the purpose of influencing the
outcome of the Presidential election.
On the other hand, the exception in the same Section 15 of Article VII - allowing
appointments to be made during the period of the ban therein provided - is much
narrower than that recognized in Aytona. The exception allows only the making of
temporary appointments to executive positions when continued vacancies will
prejudice public service or endanger public safety. Obviously, the article greatly
restricts the appointing power of the President during the period of the ban.
10. Considering the respective reasons for the time frames for filling vacancies in the
courts and the restriction on the President's power of appointment, it is this Court's
view that, as a general proposition, in case of conflict, the former should yield to the
78
EXECUTIVE DEPARTMENT
latter. Surely, the prevention of vote-buying and similar evils outweighs the need for
avoiding delays in filling up of court vacancies or the disposition of some cases.
Temporary vacancies can abide the period of the ban which, incidentally and as
earlier pointed out, comes to exist only once in every six years. Moreover, those
occurring in the lower courts can be filled temporarily by designation. But prohibited
appointments are long-lasting and permanent in their effects. They may, as earlier
pointed out, in fact influence the results of elections and, for that reason, their making
is considered an election offense.

11. To the contention that may perhaps be asserted, that Sections 4 (1) and 9 of
Article VIII should prevail over Section 15 of Article VII, because they may be
considered later expressions of the people when they adopted the Constitution, it
suffices to point out that the Constitution must be construed in its entirety as one,
single, instrument.

12. To be sure, instances may be conceived of the imperative need for an


appointment, during the period of the ban, not only in the executive but also in the
Supreme Court. This may be the case should the membership of the court be so
reduced that it will have no quorum or should the voting on a particularly important
question requiring expeditious resolution be evenly divided. Such a case, however, is
covered by neither Section 15 of Article VII nor Section 4 (1) and 9 of Article VIII.
13. A final word, concerning Valenzuela's oath-taking and "reporting for duty" as
Presiding Judge of RTC Branch 62, Bago City, on May 14, 1998. Standing practice is
for the originals of all appointments to the Judiciary - from the highest to the lowest
courts - to be sent by the Office of the President to the Office of the Chief Justice, the
appointments being addressed to the appointees "Thru: the Chief Justice, Supreme
Court, Manila." It is a Clerk of Court of the Supreme Court, in the Chief Justice's
behalf, who thereafter advises the individual appointees of their appointments and
also of the date of commencement of the pre-requisite orientation seminar, to be
conducted by the Philippine Judicial Academy for new Judges. The rationale of this
procedure is salutary and readily perceived. The procedure ensures the authenticity
of the appointments, enables the Court, particularly of the Office of the Court
Administrator, to enter in the appropriate records all appointments to the Judiciary as
well as other relevant data such as the dates of qualification, the completion by the
appointees of their pre-requisite orientation seminars, their assumption of duty, etc.
The procedure also precludes the possibility, however remote, of Judges acting on
spurious or otherwise defective appointments. It is obviously not advisable, to say the
least, for a Judge to take his oath of office and enter upon the performance of his
duties on the basis alone of a document purporting to be a copy of his appointment
coming from Malacañang, the authenticity of which has not been verified from the
latter of the Office of the Court Administrator; or otherwise to begin performing his

79
EXECUTIVE DEPARTMENT
duties as Judge without the Court Administrator knowing of that fact. The
undesirability of such a situation is illustrated by the case of Judge Valenzuela who
acted, with no little impatience or rashness, on a mere copy of his supposed
appointment, without having received any formal notice from this Court, and without
verifying the authenticity of the appointment or the propriety of taking oath on the basis
thereof. Had he bothered to inquire about his appointment from the Court
Administrator's Office, he would have been informed of the question concerning it and
the Court's injunction.
14. The appointments of Messrs. Valenzuela and Vallarta on March 30, 1998
(transmitted to the Office of the Chief Justice on May 14, 1998) were unquestionably
made during the period of the ban. Consequently, they come within the operation of
the first prohibition relating to appointments which are considered to be for the
purpose of buying votes or influencing the election. While the filling of vacancies in
the judiciary is undoubtedly in the public interest, there is no showing in this case of
any compelling reason to justify the making of the appointments during the period of
the ban. On the other hand, as already discussed, there is a strong public policy for
the prohibition against appointments made within the period of the ban.

DE CASTRO v JBC

80
EXECUTIVE DEPARTMENT
PONENTE/ OTHER BERSAMIN/ CARPIO-MORALES, BRION
OPINION
DATE April 20, 2010
PETITIONERS ARTURO M. DE CASTRO, JAIME N.
SORIANO, PHILIPPINE CONSTITUTION
ASSOCIATION (PHILCONSA), et al
RESPONDENTS JUDICIAL AND BAR COUNCIL
DISPOSITION motions for reconsideration are denied with
finality
DOCTRINE ACCDG TO LIMITATIONS
THE SYLLABUS
Prohibition under section 15, Article VII does
not apply to appointments to fill a vacancy in
the Supreme Court or to other appointments
to the judiciary.
QUICK FACTS Search for the next chief justice

FACTS: The compulsory retirement of Chief Justice Reynato S. Puno by May 17,
2010 occurs just days after the coming presidential elections on May 10, 2010.
These cases trace their genesis to the controversy that has arisen from the
forthcoming compulsory retirement of Chief Justice Puno on May 17, 2010, or seven
days after the presidential election. Under Section 4(1), in relation to Section 9, Article
VIII, that “vacancy shall be filled within ninety days from the occurrence thereof” from
a “list of at least three nominees prepared by the Judicial and Bar Council for every
vacancy.” Also considering that Section 15, Article VII (Executive Department) of the
Constitution prohibits the President or Acting President from making appointments
within two months immediately before the next presidential elections and up to the
end of his term, except temporary appointments to executive positions when
continued vacancies therein will prejudice public service or endanger public safety.
The JBC, in its en banc meeting of January 18, 2010, unanimously agreed to start the
process of filling up the position of Chief Justice.
Conformably with its existing practice, the JBC “automatically considered” for the
position of Chief Justice the five most senior of the Associate Justices of the Court,
namely: Associate Justice Antonio T. Carpio; Associate Justice Renato C. Corona;
Associate Justice Conchita Carpio Morales; Associate Justice Presbitero J. Velasco,
Jr.; and Associate Justice Antonio Eduardo B. Nachura. However, the last two
declined their nomination through letters dated January 18, 2010 and January 25,
2010, respectively.
81
EXECUTIVE DEPARTMENT
The OSG contends that the incumbent President may appoint the next Chief Justice,
because the prohibition under Section 15, Article VII of the Constitution does not apply
to appointments in the Supreme Court. It argues that any vacancy in the Supreme
Court must be filled within 90 days from its occurrence, pursuant to Section 4(1),
Article VIII of the Constitution; that had the framers intended the prohibition to apply
to Supreme Court appointments, they could have easily expressly stated so in the
Constitution, which explains why the prohibition found in Article VII (Executive
Department) was not written in Article VIII (Judicial Department); and that the framers
also incorporated in Article VIII ample restrictions or limitations on the President’s
power to appoint members of the Supreme Court to ensure its independence from
“political vicissitudes” and its “insulation from political pressures,” such as stringent
qualifications for the positions, the establishment of the JBC, the specified period
within which the President shall appoint a Supreme Court Justice.
A part of the question to be reviewed by the Court is whether the JBC properly initiated
the process, there being an insistence from some of the oppositors-intervenors that
the JBC could only do so once the vacancy has occurred (that is, after May 17, 2010).
Another part is, of course, whether the JBC may resume its process until the short list
is prepared, in view of the provision of Section 4(1), Article VIII, which unqualifiedly
requires the President to appoint one from the short list to fill the vacancy in the
Supreme Court (be it the Chief Justice or an Associate Justice) within 90 days from
the occurrence of the vacancy.
ISSUES:
Whether or not the petitioners have legal standing.
YES. Petitioners have legal standing because such requirement for this case was
waived by the Court. Legal standing is a peculiar concept in constitutional law
because in some cases, suits are not brought by parties who have been personally
injured by the operation of a law or any other government act but by concerned
citizens, taxpayers or voters who actually sue in the public interest.” But even if, strictly
speaking, the petitioners “are not covered by the definition, it is still within the wide
discretion of the Court to waive the requirement and so remove the impediment to its
addressing and resolving the serious constitutional questions raised.”
Whether or not there is justiciable controversy that is ripe for judicial
determination.
YES, there is a justiciable issue. The court holds that the petitions set forth an actual
case or controversy that is ripe for judicial determination. The reality is that the JBC
already commenced the proceedings for the selection of the nominees to be included
in a short list to be submitted to the President for consideration of which of them will
succeed Chief Justice Puno as the next Chief Justice. Although the position is not yet
vacant, the fact that the JBC began the process of nomination pursuant to its rules
and practices, although it has yet to decide whether to submit the list of nominees to
82
EXECUTIVE DEPARTMENT
the incumbent outgoing President or to the next President, makes the situation ripe
for judicial determination, because the next steps are the public interview of the
candidates, the preparation of the short list of candidates, and the “interview of
constitutional experts, as may be needed.” The resolution of the controversy will
surely settle – with finality – the nagging questions that are preventing the JBC from
moving on with the process that it already began, or that are reasons persuading the
JBC to desist from the rest of the process.
Whether or not the incumbent President can appoint the next Chief Justice.
Yes. Prohibition under section 15, Article VII does not apply to appointments to fill a
vacancy in the Supreme Court or to other appointments to the judiciary. The records
of the deliberations of the Constitutional Commission reveal that the framers devoted
time to meticulously drafting, styling, and arranging the Constitution. Such
meticulousness indicates that the organization and arrangement of the provisions of
the Constitution were not arbitrarily or whimsically done by the framers, but purposely
made to reflect their intention and manifest their vision of what the Constitution should
contain. As can be seen, Article VII is devoted to the Executive Department, and,
among others, it lists the powers vested by the Constitution in the President. The
presidential power of appointment is dealt with in Sections 14, 15 and 16 of the Article.
Had the framers intended to extend the prohibition contained in Section 15, Article VII
to the appointment of Members of the Supreme Court, they could have explicitly done
so. They could not have ignored the meticulous ordering of the provisions. They would
have easily and surely written the prohibition made explicit in Section 15, Article VII
as being equally applicable to the appointment of Members of the Supreme Court in
Article VIII itself, most likely in Section 4 (1), Article VIII.
Whether or not mandamus and prohibition will lie to compel the submission of
the shortlist of nominees by the JBC.
No. Writ of mandamus does not lie against the JBC. Mandamus shall issue when any
tribunal, corporation, board, officer or person unlawfully neglects the performance of
an act that the law specifically enjoins as a duty resulting from an office, trust, or
station. It is proper when the act against which it is directed is one addressed to the
discretion of the tribunal or officer. Mandamus is not available to direct the exercise
of a judgment or discretion in a particular way. For mandamus to lie, the following
requisites must be complied with: (a) the plaintiff has a clear legal right to the act
demanded; (b) it must be the duty of the defendant to perform the act, because it is
mandated by law; (c) the defendant unlawfully neglects the performance of the duty
enjoined by law; (d) the act to be performed is ministerial, not discretionary; and (e)
there is no appeal or any other plain, speedy and adequate remedy in the ordinary
course of law.

SEPARATE OPINIONS:

83
EXECUTIVE DEPARTMENT
CARPIO MORALES, J. DISSENTING:

No compelling reason exists for the Court to deny a reconsideration of the assailed
Decision. The various motions for reconsideration raise hollering substantial
arguments and legitimately nagging questions which the Court must meet head on.

Whether the incumbent President is constitutionally proscribed from


appointing the successor of Chief Justice Reynato S. Puno upon his retirement
on May 17, 2010 until the ban ends at 12:00 noon of June 30, 2010

1. In interpreting the subject constitutional provisions, the Decision disregarded


established canons of statutory construction. Without explaining the
inapplicability of each of the relevant rules, the Decision immediately placed
premium on the arrangement and ordering of provisions, one of the weakest
tools of construction, to arrive at its conclusion.

2. In reversing Valenzuela, the Decision held that the Valenzuela dictum did not
firmly rest on ConCom deliberations, yet it did not offer to cite a material
ConCom deliberation. It instead opted to rely on the memory of Justice Florenz
Regalado which incidentally mentioned only the "Court of Appeals." The
Decision’s conclusion must rest on the strength of its own favorable Concom
deliberation, none of which to date has been cited.

3. Instead of choosing which constitutional provision carves out an exception


from the other provision, the most legally feasible interpretation (in the limited
cases of temporary physical or legal impossibility of compliance, as expounded
in my Dissenting Opinion) is to consider the appointments ban or other
substantial obstacle as a temporary impossibility which excuses or releases the
constitutional obligation of the Office of the President for the duration of the ban
or obstacle.

In view of the temporary nature of the circumstance causing the impossibility of


performance, the outgoing President is released from non-fulfillment of the obligation
to appoint, and the duty devolves upon the new President. The delay in the fulfillment
of the obligation becomes excusable, since the law cannot exact compliance with
what is impossible. The 90-day period within which to appoint a member of the Court
is thus suspended and the period could only start or resume to run when the
temporary obstacle disappears (i.e., after the period of the appointments ban; when
there is already a quorum in the JBC; or when there is already at least three
applicants).

Whether the Judicial and Bar Council is obliged to submit to the President the
shortlist of nominees for the position of Chief Justice (or Justice of this Court)
on or before the occurrence of the vacancy.

1. The ruling in the Decision that obligates the JBC to submit the shortlist to the
President on or before the occurrence of the vacancy in the Court runs counter
84
EXECUTIVE DEPARTMENT
to the Concom deliberations which explain that the 90-day period is allotted for
both the nomination by the JBC and the appointment by the President. In the
move to increase the period to 90 days, Commissioner Romulo stated that "[t]he
sense of the Committee is that 60 days is awfully short and that the [Judicial
and Bar] Council, as well as the President, may have difficulties with that."

2. To require the JBC to submit to the President a shortlist of nominees on or


before the occurrence of vacancy in the Court leads to preposterous results. It
bears reiterating that the requirement is absurd when, inter alia, the vacancy is
occasioned by the death of a member of the Court, in which case the JBC could
never anticipate the death of a Justice, and could never submit a list to the
President on or before the occurrence of vacancy.

3. The express allowance in the Constitution of a 90-day period of vacancy in


the membership of the Court rebuts any public policy argument on avoiding a
vacuum of even a single day without a duly appointed Chief Justice. Moreover,
as pointed out in my Dissenting Opinion, the practice of having an acting Chief
Justice in the interregnum is provided for by law, confirmed by tradition, and
settled by jurisprudence to be an internal matter.

BRION, J. CONCURRING AND DISSENTING:

The Motions for Reconsideration

I diverged fully from the Decision on the question of whether we should maintain or
reverse our ruling in Valenzuela. I maintained that it is still good law; no reason exists
to touch the ruling as its main focus – the application of the election ban on the
appointment of lower court judges under Article VIII, Section 9 of the Constitution – is
not even an issue in the present case and was discussed only because the petitions
incorrectly cited the ruling as authority on the issue of the Chief Justice’s appointment.
The Decision proposed to reverse Valenzuela but only secured the support of five (5)
votes, while my Separate Opinion in support of Valenzuela had four (4) votes. Thus,
on the whole, the Decision did not prevail in reversing Valenzuela, as it only had five
(5) votes in a field of 12 participating Members of the Court. Valenzuela should
therefore remain, as of the filing of this Opinion, as a valid precedent.

Acting on the present motions for reconsideration, I join the majority in denying the
motions with respect to the Chief Justice issue, although we differ in some respects
on the reasons supporting the denial. I dissent from the conclusion that the Valenzuela
ruling should be reversed. My divergence from the majority’s reasons and conclusions
compels me to write this Concurring and Dissenting Opinion.

It may be asked: why does the Court have to recognize the Mendoza petition when it
can resolve the conflict between Article VII, Section 15 and Article VIII, Section 4(1)
through the Tolentino and Soriano petitions?

85
EXECUTIVE DEPARTMENT
The answer is fairly simple and can be read between the lines of the above
explanation on the relationship between the Court and the JBC. First, administrative
is different from judicial function and providing guidance to the JBC can only be
appropriate in the discharge of the Court’s administrative function. Second, the
resolution of the Tolentino and Soriano petitions will lead to rulings directly related to
the underlying facts of these petitions, without clear guidelines to the JBC on the
proper parameters to observe vis-à-vis the constitutional dispute along the lines the
JBC needs. In fact, concrete guidelines addressed to the JBC in the resolution of the
Tolentino/Soriano petitions may even lead to accusations that the Court’s resolution
is broader than is required by the facts of the petitions. The Mendoza petition, because
it pertains directly to the performance of the JBC’s duty and the Court’s supervisory
authority, allows the issuance of precise guidelines that will enable the JBC to fully
and seasonably comply with its constitutional mandate.

In my Separate Opinion, I concluded that the appointment of a Member of the Court


even during the election period per se implies no adverse effect on the integrity of the
election; a full Court is ideal during this period in light of the Court’s unique role during
elections. I maintain this view and fully concur in this regard with the majority. During
the election period, the court is not only the interpreter of the Constitution and the
election laws; other than the Commission on Elections and the lower courts to a limited
extent, the Court is likewise the highest impartial recourse available to decisively
address any problem or dispute arising from the election. It is the leader and the
highest court in the Judiciary, the only one of the three departments of government
directly unaffected by the election. The Court is likewise the entity entrusted by the
Constitution, no less, with the gravest election-related responsibilities. In particular, it
is the sole judge of all contests in the election of the President and the Vice-President,
with leadership and participation as well in the election tribunals that directly address
Senate and House of Representatives electoral disputes. With this grant of
responsibilities, the Constitution itself has spoken on the trust it reposes on the Court
on election matters. This reposed trust, to my mind, renders academic any question
of whether an appointment during the election period will adversely affect the integrity
of the elections – it will not, as the maintenance of a full Court in fact contributes to
the enforcement of the constitutional scheme to foster a free and orderly election.

The Valenzuela Decision

The ponencia’s ruling reversing Valenzuela, in my view, is out of place in the present
case, since at issue here is the appointment of the Chief Justice during the period of
the election ban, not the appointment of lower court judges that Valenzuela resolved.
To be perfectly clear, the conflict in the constitutional provisions is not confined to
Article VII, Section 15 and Article VIII, Section 4(1) with respect to the appointment of
Members of the Supreme Court; even before the Valenzuela ruling, the conflict
already existed between Article VII, Section 15 and Article VIII, Section 9 – the
provision on the appointment of the justices and judges of courts lower than the
Supreme Court. After this Court’s ruling in Valenzuela, no amount of hairsplitting can
result in the conclusion that Article VII, Section 15 applied the election ban over the
86
EXECUTIVE DEPARTMENT
whole Judiciary, including the Supreme Court, as the facts and the fallo of Valenzuela
plainly spoke of the objectionable appointment of two Regional Trial Court judges. To
reiterate, Valenzuela only resolved the conflict between Article VII, Section 15 and
appointments to the Judiciary under Article VIII, Section 9.

If Valenzuela did prominently figure at all in the present case, the prominence can be
attributed to the petitioners’ mistaken reading that this case is primary authority for the
dictum that Article VII, Section 15 completely bans all appointments to the Judiciary,
including appointments to the Supreme Court, during the election period up to the end
of the incumbent President’s term. In reality, this mistaken reading is an obiter dictum
in Valenzuela, and hence, cannot be cited for its primary precedential value. This legal
situation still holds true as Valenzuela was not doctrinally reversed as its proposed
reversal was supported only by five (5) out of the 12 participating Members of the
Court. In other words, this ruling on how Article VII, Section 15 is to be interpreted in
relation with Article VIII, Section 9, should continue to stand unless otherwise
expressly reversed by this Court.

In any case, let me repeat what I stressed in my Separate Opinion about Valenzuela
which rests on the reasoning that the evils Section 15 seeks to remedy – vote buying,
midnight appointments and partisan reasons to influence the elections – exist, thus
justifying an election appointment ban. In particular, the "midnight appointment"
justification, while fully applicable to the more numerous vacancies at the lower
echelons of the Judiciary, should not apply to the Supreme Court which has only a
total of 15 positions that are not even vacated at the same time. The most number of
vacancies for any one year occurred only last year (2009) when seven (7) positions
were vacated by retirement, but this vacancy rate is not expected to be replicated at
any time within the next decade. Thus "midnight appointments" to the extent that they
were understood in Aytona will not occur in the vacancies of this Court as nominations
to its vacancies are all processed through the JBC under the public’s close scrutiny.
As already discussed above, the institutional integrity of the Court is hardly an issue.

87
EXECUTIVE DEPARTMENT
POBRE V MENDIETA
“The rule of construction according to the spirit of the law is especially applicable
where adherence to the letter would result in absurdity or injustice or where
adherence to the letter of the law would lead to contradictions or defeat the plain
purpose of the act, or where the provision was inserted through inadvertence. In
following this rule, words may be modified or rejected and others substituted, or
words and phrases may be transposed. So the meaning of general language may
be restrained by the spirit or reason of the statute, and may be construed to
admit implied exceptions. Apparent inaccuracies and mistakes in the mere
verbiage or phraseology will be overlooked to give effect to the spirit of the law.”
FACTS:
The controversy began when the term of office of Honorable Julio B. Francia as PRC
Commissioner/ Chairman expired. At that time, Mariano A. Mendieta was the senior
Associate Commissioner and Hermogenes P. Pobre was the second Associate
Commissioner of the PRC.
Executive Secretary Franklin M. Drilon sought the opinion of Acting Secretary of Justice
Silvestre H. Bello, III on whether the President’s power to appoint the Commissioner of
the Professional Regulation Commission is restricted by Section 2 of P.D. No. 223, as
amended, which provides:
Sec. 2. Composition.—The Commission shall be headed
by one fulltime Commissioner and two fulltime Associate
Commissioners, all to be appointed by the President for a
term of nine (9) years without reappointment to start from
the time they assume office, except the first two Associate
Commissioners who shall be appointed, one for six (6) years
and the other for three (3) years, and thereafter, any vacancy
in the Commission shall be filled for the unexpired term only
with the most senior of the Associate Commissioners
succeeding the Commissioner at the expiration of his term,
resignation or removal. No person shall be appointed
chairman or member of the Commission unless he is at least
forty (40) years of age, familiar with the principles and
methods of professional regulation and/or licensing and has
at least five (5) years of executive or managerial experience.

88
EXECUTIVE DEPARTMENT
The Executive Secretary wanted to know whether the President may appoint as
Commissioner/Chairman of the PRC another Associate Commissioner or any person
other than the Senior Associate Commissioner.
In a Memorandum Acting Secretary of Justice Silvestre H. Bello, III answered that Section
2 of P.D. No. 223 does not limit or restrict the appointing power of the President and a
contrary interpretation would taint the provision with unconstitutionality since it would
countenance a usurpation by the legislature of a power which does not belong to it but
pertains to the executive. It has been said that ‘those matters which the Constitution
specifically confides to the executive, the legislative cannot directly or indirectly take
from his control.
Thereafter, President Corazon C. Aquino appointed the petitioner, then an Associate
Commissioner, as the PRC Commissioner/Chairman.
Even before Commissioner Pobre’s appointment, the private respondent, Mariano A.
Mendieta, as the Senior Associate Commissioner, filed a petition for declaratory relief
against Commissioner Pobre, Executive Secretary Drilon, and Acting Secretary of Justice
Eduardo Montenegro, praying that they be enjoined from appointing, or recommending,
the appointment of Associate Commissioner Pobre as Chairman of the PRC because
under Section 2 of P.D. No. 223, he (Mendieta), as the senior Associate Commissioner,
was legally entitled to succeed Francia as Chairman of the PRC.
Pobre opposed the issuance of a restraining order because President Aquino had
already appointed him PRC Chairman and he had, in fact, already taken his oath of office
Judge Somera denied the prayer for a restraining order as well as the petition for
declaratory relief for being moot andacademic.
Consequently, Mendieta filed a petition for quo warranto contesting Pobre’s
appointment as chairman of the PRC because he (Mendieta) allegedly succeeded Francia
as PRC Chairman by operation of law.
In his answer to the petition for quo warranto, Pobre disputed Mendieta’s claim on
the ground that only the President of the Philippines, in whom the appointing power is
vested by law and the Constitution, may name the successor of retired PRC
Commissioner/Chairman Francia upon the expiration of the latter’s term of office.
Judge Somera rendered a decision in favor of Mendieta.

ISSUE:
w/o/n (Mendieta), as the senior Associate Commissioner, was legally entitled to succeed
Francia as Chairman of the PRC.
89
EXECUTIVE DEPARTMENT
HELD: No. The petition raises an issue regarding the proper construction of the provision
in Section 2 of P.D. No. 223 that: “x x x any vacancy in the Commission shall be filled for
the unexpired term only with the most Senior of the Associate Commissioners
succeeding the Commissioner at the expiration of his term, resignation or removal,”
whereby the legality of Pobre’s appointment as PRC Chairman may be determined.
In interpreting this section of P.D. No. 223, consideration should be accorded the
provision of the Constitution vesting the power of appointment in the President of the
Philippines.
Section 10, Article VII of the 1973 Constitution which took effect on January 17, 1973
(per Proclamation No. 1102) was the source of former President Ferdinand E. Marcos’
authority to issue P.D. No. 223 on June 22, 1973, because under that constitutional
provision, the President was empowered to “appoint the heads of bureaus and offices.”
The chairman of the PRC is the head of an office.
“Sec. 10. The President shall appoint the heads of bureaus and offices, the officers of the
armed forces of the Philippines from the rank of brigadier general or commodore, and
all other officers of the Government whose appointments are not otherwise provided
for, and those whom he may be authorized by law to appoint. However, the Batasang
Pambansa may by law vest in the Prime Minister, members of the Cabinet, the Executive
Committee, courts, heads of agencies, commis-sions, and boards the power to appoint
inferior officers.”
Section 10, Article VII of the 1973 Constitution was modified by Section 16, Article VII of
the 1987 Constitution, which provides:
“Sec. 16. The President shall nominate and, with the consent of the Commission on
Appointments, appoint the heads of the executive departments, ambassadors, other
public ministers and consuls, or officers of the armed forces from the rank of colonel or
naval captain and other officers whose appointments are vested in him in this
Constitution. He shall also appoint all other officers of the Government whose
appointments are not otherwise provided for by law, and those whom he may be
authorized by law to appoint. The Congress may, by law, vest the appointment of other
officers lower in rank in the President alone, in the courts, or in the heads of
departments, agencies, commissions, or boards.”
This provision empowers the President to appoint “those whom he may be authorized
by law to appoint.” The law that authorizes him to appoint the PRC Commissioner and
Associate Commissioners, is P.D. 223, Section 2, which provides that the Commissioner
and Associate Commissioners of the PRC are “all to be appointed by the President for a
term of nine (9) years, without reappointment, to start from the time they assume office
x x x.”
90
EXECUTIVE DEPARTMENT
In holding that Mendieta, as the senior PRC associate commissioner, has a valid claim to
the office of chairman/commissioner vacated by Francia, Judge Somera relied on what
she called the “succession clause” (p. 38, Rollo) in Section 2, P.D. No. 223 which provides
that:
“x x x any vacancy in the Commission shall be filled for the unexpired term only with the
most Senior of the Associate Commissioners succeeding the Commissioner at the
expiration of his term, resignation or removal.” (Sec. 2, P.D. 223.)
She rejected Pobre’s theory that said provision refers to a vacancy in the office of
Commissioner/Chairman caused by the latter’s retirement, resignation or removal (also
death or incapacity) before the expiration of his 9-year term, thereby leaving “an
unexpired term” which shall be served by “the most senior among the Associate
Commissioners.”
We do not agree with Judge Somera’s opinion that the filling up of the vacancy “for the
unexpired portion of the term only” refers to the unexpired portion of the term of the
successor (the “most senior Associate Commissioner”) rather than the unexpired portion
of the Chairman’s term. The Court holds that the succession clause operates only when
there is an “unexpired term” of the Chairman/Commissioner to be served. Otherwise, if
the Chairman’s term had expired or been fully served, the vacancy must be filled by
appointment of a new chairman by the President.
It may be candidly admitted that the language of Section 2, P.D. 223 leaves much to be
desired for clarity. For instance, the provision speaks of “any vacancy in the
Commission” but it obviously refers only to a vacancy in the position of Commissioner or
Chairman for it is only her (or she) who may be succeeded by the “most senior Associate
Commissioner.” Furthermore, the same section speaks of “the most senior of the
Associate Commissioners succeeding the Commissioner.” Only the Chairman of the
Commission bears the title of “Commissioner;” the others are “Associate
Commissioners.”
The Court finds unacceptable the view that every vacancy in the Commission (except the
position of “junior” Associate Commissioner) shall be filled by “succession” or by
“operation of law” for that would deprive the President of his power to appoint a new
PRC Commissioner and Associate Commissioners-all to be appointed by the President”
under P.D. No. 223. The absurd result would be that the only occasion for the President
to exercise his appointing power would be when the position of junior (or second)
Associate Commissioner becomes vacant.

91
EXECUTIVE DEPARTMENT
GAMINDE v. COA.
“Term of office”
In the law of public officers, there is a settled distinction between “term” and “tenure.”
“[T]he term of an office must be distinguished from the tenure of the incumbent. The
term means the time during which the officer may claim to hold office as of right, and
fixes the interval after which the several incumbents shall succeed one another. The
tenure represents the term during which the incumbent actually holds the office. The
term of office is not affected by the hold-over. The tenure may be shorter than the term
for reasonswithin or beyond the power of the incumbent.”

FACTS: On June 11, 1993, the President of the Philippines appointed petitioner
Thelma P. Gaminde, ad interim, Commissioner, Civil Service Commission. She assumed
office on June 22, 1993, after taking an oath of office. On September 07, 1993, the
Commission on Appointment, Congress of the Philippines confirmed
the appointment. However, on February 24, 1998, petitioner sought clarification from
the Office of the President as to the expiry date of her term of office. In reply to her
request, the Chief Presidential Legal Counsel, in a letter dated April 07, 1998. Opined that
petitioner’s term of office would expire on February 02, 2000, not on February 02,
1999.Relying on said advisory opinion, petitioner remained in Leon, wrote office after
February 02, 1999. On February 04,1999, Chairman Corazon Alma G. de the
Commission on Audit requesting opinion on whether or not Commissioner
Thelma P. Gaminde and her co-terminus staff may be paid their salaries notwithstanding
the expiration of their appointments on February 02, 1999.
On February 18, 1999, the General Counsel, Commission on Audit, issued an opinion that
“the term of Commissioner Gaminde has expired on February 02, 1999 as stated inher
appointment conformably with the constitutional intent.”Consequently, on March 24,
1999, CSC Resident Auditor Flovitas U. Felipe issued notice of disallowance No. 99-002-
101 (99), disallowing in audit the salaries and emoluments pertaining to petitioner and
her co-terminus staff, effective February 02, 1999. On April 5, 1999, petitioner appealed
the disallowance to the Commission on Audit Enbanc.
On June 15, 1999, the Commission on Audit issued Decision dismissing petitioner’s appeal.
The Commission on Audit affirmed the propriety of the disallowance, holding that the
issue of petitioner’s term of office may be properly addressed by mere reference to her
appointment paper which set the expiration date on February 02,1999, and that the
Commission is bereft of power to recognize an extension of her term, not even with the
implied acquiescence of the Office of the President.

92
EXECUTIVE DEPARTMENT
In time, petitioner moved for reconsideration; however, on August 17, 1999, the
Commission on Audit denied the motion.

ISSUE: The basic issue raised is whether the term of office of Atty. Thelma P. Gaminde, as
Commissioner, Civil Service Commission, to which she was appointed on June 11, 1993,
expired on February 02, 1999, as stated in the appointment paper, or on February 02, 2000, as
claimed by her.

RULING:

On June 11, 1993, the President appointed Atty. Thehna P. Gaminde Commissioner, Civil
Service Commission, for a term expiring February 02, 1999.18 This terminal date is specified
in her appointment paper. On September 07, 1993, the Commission on Appointments
confirmed the appointment. She accepted the appointment and assumed office on June 22,
1993. She is bound by the term of the appointment she accepted, expiring February 02,
1999. In this connection, the letter dated April 07, 1998, of Deputy Executive Secretary
Renato C. Corona19 clarifying that her term would expire on February 02, 2000, was in
error. What was submitted to the Commission on Appointments was a nomination for a
term expiring on February 02, 1999. Thus, the term of her succes-
sor20 must be deemed to start on February 02, 1999, and expire on February 02, 2006.
Thus, The term of office of Ms. Thelma P. Gaminde as Commissioner, Civil
Service Commission, under an appointment extended to her by President Fidel V.
Ramos on June 11, 1993. Expired on February 02, 1999.However, she served as de facto
Officer in good faith until February 02, 2000, and thus entitled to receive her salary and other
emoluments for actual service rendered.Consequently, the Commission on Audit erred
in disallowing in audit such salary and other emoluments, including that of her co-terminus
staff. ACCORDINGLY, The Court REVERSED the decisions of the Commission on Audit insofar
as they disallow the salaries and emoluments of Commissioner Thelma P. Gaminde and her
coterminous staff during her tenure as de facto officer from February 02, 1999, until
February 02, 2000.

93
EXECUTIVE DEPARTMENT
3. FUNA V VILLAR
On February 15, 2001, Pres Arroyo appointed Carague as Chairman of the COA for a term of
7 years. Carague’s term of office started on February 2, 2001 to end on February 2, 2008.
On February 7, 2004, Villar was appointed as the third member of the COA for a term of 7
years starting February 2, 2004 until February 2, 2011.

Following the retirement of Carague on February 2, 2008 and during the fourth year of
Villar as COA Commissioner, Villar was designated as Acting Chairman of COA from
February 4, 2008 to April 14, 2008. Subsequently, on April 18, 2008, Villar was nominated
and appointed as Chairman of the COA. Shortly thereafter, the Commission on
Appointments confirmed his appointment. He was to serve as Chairman of COA, as
expressly indicated in the appointment papers, until the expiration of the original term of
his office as COA Commissioner or on February 2, 2011.

Issue 1: W/N a promotional appointment from the position of Commissioner to Chairman is


constitutionally permissible and does NOT constitute reappointment as barred by the
Article IX (D), Sec 1 (2) of the Constitution
Yes. A commissioner who resigns after serving in the Commission for less than seven years
is eligible for an appointment to the position of Chairman for the unexpired portion of the
term of the departing chairman. Such appointment is not covered by the ban on
reappointment, provided that the aggregate period of the length of service as
commissioner and the unexpired period of the term of the predecessor will not exceed 7
years and provided further that the vacancy in the position of Chairman resulted
from death, resignation, disability or removal by impeachment.

Reappointment found in Sec. 1(2), Art. IX(D) means a movement to one and the same office
(Commissioner to Commissioner or Chairman to Chairman). On the other hand, an
appointment involving a movement to a different position or office (Commissioner to
Chairman) would constitute a new appointment and, hence, not, in the strict legal sense, a
reappointment barred under the Constitution.

Issue 2: W/N the appointment of Villar to the position of COA Chairman which is
made vacant by the expiration of term of the predecessor is valid
No. The Constitution clearly provides that if the vacancy results from the expiration of the
term of the predecessor, the appointment of a COA member shall be for a fixed 7-year
term.

Here, the vacancy in the position of COA chairman left by Carague in February 2, 2008
resulted from the expiration of his 7-year term. Under that circumstance, there can be
no unexpired portion of the term of the predecessor to speak of. Hence, in light of the 7-

94
EXECUTIVE DEPARTMENT
year aggregate rule, Villar’s appointment to a full term is not valid as he will be allowed to
serve more than seven 7 years under the constitutional ban.

Villar had already served 4 years of his 7-year term as COA Commissioner. A shorter term,
however, to comply with the 7-year aggregate rule would also be invalid as the
corresponding appointment would effectively breach the clear purpose of the Constitution
of giving to every appointee so appointed subsequent to the first set of commissioners, a
fixed term of office of 7 years.

95
EXECUTIVE DEPARTMENT
City of Iligan v Director of Lands
1. President issued Proclamation 335:
a. Withdrawingcertain parcels of public land in Iligan from sale or settlement and
b. Reserving such for the use of NPC (Nat’l Power Corporation)
2. By virtue of said proclamation,NPC constructed a fertilizer plant named “Maria Cristina”
3. Later, NPC:
a. Sold the fertilizer plant to “Marcelo Tire and Rubber Corp” with all the machineries,
right of occupancy, and use of land
b. Covenanted to collaborate with DANR in facilitating sale and right to lease for at
least 25 years, the lands where plant is erected
4. Proclamation 20 and 198 were issued:
a. Proc. 20 – excluding from operation of Proc. 335 certain areas occupied by “Ma.
Cristina” and Employees Housing and declaring such lands for OPEN DISPOSITION
b. Proc. 198 – changing the technical description of said areas (6 lots)
5. “Marcelo Steel” and “Ma. Cristina” filed a Msc. Sales Application with the Bureau of Lands
a. “Marcelo Tire” and “Ma. Cristina” are sister corporations.
b. Purchaser was “Marcelo Tire” but another sister corp. “Marcelo Steel” operated said
plant
6. In the notice of sale issued in Manila, Director of Lands advised that Bureau will sell in an
auction said lands of “Marcelo Steel”
7. President then issued Proc. 469 – excluding from the reservation made in favor to NPC
certain lands in Iligan (Lot 1, 1-a, 3, and 4) and DONATING said lands in favor of Iligan City.
8. Mayor of Iligan wrote to Director of Lands informing him that City is the owner of said
lands and foreshores in auction.
9. BUT no action was taken on said request for exclusion and so City filed a complaint for
injunction in CFI against Director. Injunction temporarily issued.
10.Pending case, President Marcos issued Proc. 94 – excluding from the donation in Proc.
469 certain lands (Lot 1-a, 2-a, and 3) and declaring same for open disposition.
11.CFI dismissed the complaint of City and dissolved injunction. Hence, this appeal.

ISSUE: WON President has the authority to grant a portion of public domain to any
government like the City of Iligan.
HELD: Yes.Since it is the Director of Lands who has direct executive control among others in
the lease, sale or any form of concession or disposition of the land of the public domain
subject to the immediate control of the Secretary of Agriculture and Natural Resources, and
considering that under the Constitution the President of the Philippines has control over all
executive departments, bureaus, and offices, etc., 15 the President of the Philippines has
therefore the same authority to dispose of portions of the public domain as his subordinates,
the Director of Lands, and his alter ego the Secretary of Agriculture and Natural Resources

96
EXECUTIVE DEPARTMENT
Thus, Having found that the President of the Philippines has the authority to donate or
grant lands of the public domain for residential, commercial or industrial purposes or other
similar purposes, Proclamation No. 469, donating the parcels of lands in question to the
plaintiff is thus valid and binding and consequently the second assigned error is well taken.
The ownership of the parcels of land in question had thereby been vested in the plaintiff City
of Iligan by the issuance of said Proclamation No. 469 on October 4, 1965. Indeed what the
records show is that the Mayor of the City of Iligan upon said proclamation immediately had
the lots surveyed and entered into negotiation with the National Investment & Development
Corporation and those interested in promoting a coco-chemical plant in Iligan City with the
end in view of accelerating the economic expansion of the City.
The Requirement that plaintiff should have filed the appropriate public land
application has no lawful basis.—The observation of the court a quo that this land grant to
the plaintiff City of Iligan is not valid and that plaintiff should have filed the appropriate public
land application by way of lease or sale as provided by the Public Land Act has no lawful basis.
Moreover, Proclamation No. 94 of President Ferdinand E. Marcos is null and void and
of no force and effect as the saidparcels of land had since been segregated from the land of
the public domain and became of the exclusive ownership and property of the plaintiff as of
October 4, 1965, Proclamation No. 94 of August 25, 1966 of President Ferdinand E. Marcos
excluding from the operation of said Proclamation No. 469 dated October 4, 1965 and
declaring the same open to dispositions Lots 1-a, 2-a and 3 is null and void and of no force
and effect. Under Section 60 aforecited it is specifically provided that "the land so granted,
donated or transferred to a province, municipality or branch or subdivision of the
government shall not be alienated, encumbered or otherwise disposed of in a manner
affecting its title, except when authorized by Congress,
Only Congress may authorize the alienation, encumbrance or disposition of such land
already donated.—From the foregoing, it is clear that it is only Congress not the President of
the Philippines that may authorize the alienation, encumbrance or disposition of such land
already donated to a province, municipality or branch or subdivision of the government.

97
EXECUTIVE DEPARTMENT
Kilusang Bayan v Dominguez

Facts:

Petitioners question the validity of the order of then Secretary of Agriculture Hon.
Carlos G. Dominguez which ordered: (1) the take-over by the Department of Agriculture of
the management of the petitioner Kilusang Bayan sa Paglilingkod Ng Mga Magtitinda ng
Bagong Pamilihang Bayan ng Muntilupa, Inc. (KBMBPM) pursuant to the Department’s
regulatory and supervisory powers under Section 8 of P.D. No. 175, as amended, and Section
4 of Executive Order No. 13, (2) the creation of a Management Committee which shall assume
the management of KBMBPM upon receipt of the order, (3) the disbandment of the Board of
Directors, and (4) the turn over of all assets, properties and records of the KBMBPM the
Management Committee.

The exordium of said Order unerringly indicates that its basis is the alleged petition
of the general membership of the KBMBPM requesting the Department for assistance in the
removal of the members of the Board of Directors who were not elected by the general
membership” of the cooperative and that the ongoing financial and management audit of
the Department of Agriculture auditors shows that the management of the KBMBPM is not
operating that cooperative in accordance with P.D. 175, LOI 23, the Circulars issued by
DA/BACOD and the provisions and by-laws of KBMBPM. It is also professed therein that the
Order was issued by the Department “in the exercise of its regulatory and supervisory powers
under Section 8 of P.D. 175, as amended, and Section 4 of Executive Order No. 113.

Issue:

whether or not the Order issued by the Secretary of Agriculture is illegal

Held:

Regulation 34 of Letter of Implementation No. 23 (implementing P.D. No. 175)


provides the procedure for the removal of directors or officers of cooperatives, thus:

An elected officer, director or committee member may be removed by a vote of majority of


the members entitled to vote at an annual or special general assembly. The person involved
shall have an opportunity to be heard.

A substantially identical provision, found in Section 17, Article III of the KBMBPM’s by-
laws, reads:

98
EXECUTIVE DEPARTMENT
Sec. 17. Removal of Directors and Committee Members. — Any elected director or committee
member may be removed from office for cause by a majority vote of the members in good
standing present at the annual or special general assembly called for the purpose after having
been given the opportunity to be heard at the assembly.

Under the same article are found the requirements for the holding of both the annual
general assembly and a special general assembly.

Indubitably then, there is an established procedure for the removal of directors and
officers of cooperatives. It is likewise manifest that the right to due process is respected by
the express provision on the opportunity to be heard. But even without said provision,
petitioners cannot be deprived of that right.

The procedure was not followed in this case. Respondent Secretary of Agriculture
arrogated unto himself the power of the members of the KBMBPM who are authorized to
vote to remove the petitioning directors and officers. He cannot take refuge under Section 8
of P.D. No. 175 which grants him authority to supervise and regulate all cooperatives. This
section does not give him that right.

An administrative officer has only such powers as are expressly granted to him and those
necessarily implied in the exercise thereof. These powers should not be extended by
implication beyond what may to necessary for their just and reasonable execution.

Supervision and control include only the authority to: (a) act directly whenever a specific
function is entrusted by law or regulation to a subordinate; (b) direct the performance of
duty; restrain the commission of acts; (c) review, approve, reverse or modify acts and
decisions of subordinate officials or units; (d) determine priorities in the execution of plans
and programs; and (e) prescribe standards, guidelines, plans and programs. Specifically,
administrative supervision is limited to the authority of the department or its equivalent
to: (1) generally oversee the operations of such agencies and insure that they are managed
effectively, efficiently and economically but without interference with day-to-day
activities; (2) require the submission of reports and cause the conduct of management
audit, performance evaluation and inspection to determine compliance with policies,
standards and guidelines of the department; (3) take such action as may be necessary for
the proper performance of official functions, including rectification of violations, abuses
and other forms of mal-administration; (4) review and pass upon budget proposals of such
agencies but may not increase or add to them.

The power to summarily disband the board of directors may not be inferred from any
of the foregoing as both P.D. No. 175 and the by-laws of the KBMBPM explicitly mandate the
manner by which directors and officers are to be removed. The Secretary should have known
better than to disregard these procedures and rely on a mere petition by the general
membership of the KBMBPM and an on-going audit by Department of Agriculture auditors in
99
EXECUTIVE DEPARTMENT
exercising a power which he does not have, expressly or impliedly. We cannot concede to
the proposition of the Office of the Solicitor General that the Secretary’s power under
paragraph (d), Section 8 of P.D. No. 175 above quoted to suspend the operation or cancel the
registration of any cooperative includes the “milder authority of suspending officers and
calling for the election of new officers.” Firstly, neither suspension nor cancellation includes
the take-over and ouster of incumbent directors and officers, otherwise the law itself would
have expressly so stated. Secondly, even granting that the law intended such as postulated,
there is the requirement of a hearing. None was conducted.

100
EXECUTIVE DEPARTMENT
ANGANGCO V CASTILLO

There is some point in the argument that the Power of control of the President may extend
to the Power to investigate, suspend or remove officers and employees who belong to the
executive department if they are presidential appointees or do not belong to the classified
service for such can be justified under the principle that the power to remove is inherent in
the power to appoint (Lacson V. Romero, supra), but not with regard to those officers or
employees who belong to the classified service for as to them that inherent power cannot
be exercised. This is in line with the provision of our Constitution which says that "the
Congress may by law vest the appointment of the inferior officers, in the President alone, in
the courts, or in heads of department" (Article VII, Section 10 [3], Constitution). With regard
to these officers whose appointments are vested on heads of departments, Congress has
provided by law for a procedure for their removal precisely in view of this constitutional
authority. One such law is the Civil Service Act of 1959. We have no doubt that when
Congress, by law, vests the appointment of inferior officers in the heads of departments it
may limit and restrict power of removal as it seem best for the public interest. The
constitutional authority in Congress to thus vest the appointment implies authority to limit,
restrict, and regulate the removal by such laws as Congress may enact in relation to the
officers so appointed. The head of a department has no constitutional prerogative of
appointment to officers independently of legislation of Congress, and by such legislation he
must be governed, not only in making appointments but in all that is incident thereto. (U.S.
v. Perkins, 116 U.S. 483)

FACTS: Collector of Customs Isidro Ang-Angco was “resigned effective on the date of notice,
with prejudice to reinstatement in the Bureau of Customs, being found guilty of conduct
prejudicial to the best interest of the service” by Executive Secretary Natalio P. Castillo, by
authority of the President.
Upon learning of the decision through the newpapers, Ang-Angco requested for
reconsideration, calling attention to the fact that the action taken by Secretary Castillo in
removing him from office had the effect of depriving him of his statutory right to have his
case originally decided by the Commissioner of Civil Service, as well as his right to appeal to
the Civil service Board of Appeals, whose decision under Republic Act No. 2260 is final.
Moreover, he assails that the decision is in violation of the guaranty by the Constitution
to officers or employees in the civil service against removal or suspension except for a cause
and in a manner provided by law.
Secretary Castillo, on authority of President Garcia denied the appeal. According to
him: (a) the President by virtue of his power of control over all executive departments,
bureaus and offices, can take direct action and dispose of the administrative case in question

101
EXECUTIVE DEPARTMENT
(b) as much as the provisions of law that would seem to vest final authority in subordinate
officers of the executive branch of the government over administrative matters falling under
their jurisdiction such cannot divest the President of his power of control nor diminish the
same.
Thus, After exhausting all administrative remedies available for Ang-Angco to secure
his reinstatement to the office from which he was removed in violation of his right to due
process of law, he filed before the Supreme Court, a petition for certiorari, prohibition and
mandamus with a petition for the issuance of a preliminary mandatory injunction.
He assails that Secretary Castillo violated:
(a) Sec. 16 (i) of the Civil Service Act o f 1959 which vests in the Commissioner of
Civil Service the original and exclusive jurisdiction to decide administrative
cases against officers and employees in the classified service and deprived
him of his right to appeal under Section 18 of the same Act of the Civil Service
Board of Appeals whose decision on the matter is final, removed him from
the service without due process in violation of Section 32 of the same Act
(b) Section 4, Article XII of the Constitution, which provides “ No officer or
employee in the civil service shall be removed except for cause as provided
for by law.” According to Angangcao, he is an officer who belongs to the
classified civil service. He is not a presidential appointee, but one appointed
by the Secretary of Finance under the Revised Administrative Code. Thus, he
cannot be removed from the service in utter disregard of the provisions of
the Civil Service Act of 1959.
Respondent, On the other hand, contended that whether the officers or
employees concerned are presidential appointees or belong to the classified
civil service, if they are officers and employees in the executive department,
they all come under the control of the President and therefore, his power of
removal may be exercised over them dir ectly without distinction.

Issue :Whether or not the President has the power to take direct action on
the case of petitioner even if he belongs to the classified services, in spite of
provisions now in force in the Civil Service Act of 1959.

Held : No. It is the Commissioner of the Civil Service who has the original and
exclusive jurisdiction to decide administrative cases of all officers and
employees in the classified service.
The only limitation to this power is that the decision of the Commissioner
may be appealed to the Civil Service Board of Appeals, the said Board will
decide within a period of 90 days and its decision in such case shall be final.
The only law that can be cited for the President to be empowered to
remove offic ers and employees in the classified civil service is Section 64 (b)
of the revised Administrative Code:
(b) To remove officials from office conformably to law and to declare
vacant the offices held by such removed officials. For disloyalty to the (United
102
EXECUTIVE DEPARTMENT
States) Republic of the Philippines, the (Governor-General) President of the
Philippines may at any time remove a person from any position of trust or
authority under the Government of the (Philippine Islands) Philippines.

but the phrase “conformably to law” is significant. It show s that the


president does not have blanket authority to remove any officer or employee
of the government but that his power is still subject to the law that may be
passed by the legislative body particularly with regard to the procedure, ca
use and finality of the removal of the persons who may be subject to the
disciplinary action.
The power of the President merely applies to the exercise of control over
the acts of the subordinate and not over the actor or agent himself of t he
act. It means that the President may set aside the judgment or action taken
b y a subordinate in the performance of his duties. It is still the Department
Head, pursuant to Section 79 (C ) who is given the direct control of all bureaus
and offices under his department by virtue of which he may “repeal” or
modify decisions of the chiefs of said bureaus or offices, and under section
74 of the same Code, the President’s control over the executive department
refers to matters of general policy. The Civil Service system has the
beneficient purpose of giving stability to the tenure of office of those who
belong to the classified service. In conclusion, the direct action taken by
Secretary Castillo with authority of t he President on the administrative case
of the petitioner, without submitting the same to the Commissioner of Civil
Service is contrary to law and should be set aside. The petitioner was
reinstated to service, without prejudice to submitting his case to the
Commissioner of Civil Service to be dealt according to law.

103
EXECUTIVE DEPARTMENT
NAMARCO v. Arca

Respondent Juan T. Arive was the Manager of the Traffic-Storage Department of the
NAMARCO.Pursuant to the General Manager's Administrative he was investigated by a
committee for violating Management Memorandum Order directing "that the allocation
and deliveries of merchandise imported under the so-called Trade Assistance Program to its
designated beneficiaries be stopped;" and causing the improper release of shipments
intended for delivery upon full payment thereof by the Federation of United NAMARCO
Distributors (FUND), which were covered by certain domestic letters of credit for the total
sum of P361,053.85.

After due hearing, the investigating committee found Arive guilty of the charges but
left the imposition of the penalty to the discretion of the General Manager and the Board of
Directors. Subsequently, the General Manager held Arive guilty of the charges and
dismissed him from the service. Thereafter, the Board of Directors adopted Resolution No.
584-60 dismissing Arive from the service. He filed a motion for reconsideration, which was
denied.

Arive appealed from the decision of the NAMARCO to the President of the
Philippines. Executive Secretary Ramon A. Diaz, presumably acting for the President,
handed down a decision setting aside Resolution No. 584-60 of the NAMARCO and
reinstating Juan T. Arive to his former position. In the decision it was pointed out that the
order of the NAMARCO stopping the further delivery of commodities imported under the
trade assistance program to the designated beneficiaries had been subsequently declared
illegal by the Supreme Court in the case of Federation of United NAMARCO Distributors vs.
NAMARCO, G. R. No. L-17819, March 31, 1962, on the ground that said order was a
violation of the contract of sale; hence, it would not be proper to hold Arive
administratively liable for his failure to comply with said order; and that the Pasig River
Bodegas being private warehouses over which Arive did not have supervision, much less
control, the release of the commodities therefrom could have been effected even had Arive
tried to block it

The NAMARCO, through its General Manager, in a letter addressed to the President,
asked for a reconsideration of the decision ordering Arive's reinstatement. In that letter it
was contended that the Office of the President had no jurisdiction to review any decision of
the NAMARCO Board of Directors removing, suspending, or otherwise disciplining any of its
subordinate employees, because Republic Act No. 1345 (the NAMARCO Charter), which
grants that power to the General Manager and to the Board of Directors, does not provide
for an appeal to any governmental body. I

n a letter to the NAMARCO, then Executive Secretary Ramon A. Diaz, this time
expressly acting "[b]y authority of the President," refused to reconsider the decision,
stating that the President had jurisdiction under his constitutional power of control over all
104
EXECUTIVE DEPARTMENT
executive departments, bureaus and offices, and directing that the decision be
implemented. The NAMARCO filed a second motion for reconsideration; the President,
through Salvador Marino, as Acting Executive Secretary, denied the motion and again
directed immediate compliance with the order of reinstatement.

But NAMARCO refused to heed to the order. Thus, he Office of the President, acting
on complaints of Arive that he had not been reinstated in spite of the denial of the
NAMARCO's two motions for reconsideration, sent a telegram to the General Manager
requesting him to act on the case and to comment within forty-eight hours; but the said
General Manager neither acted on the case nor commented.

Juan T. Arive filed a complaint with the Court of First Instance of Manila against the
NAMARCO and the members of its Board of Directors for reinstatement and damages, with
prayer for a writ of preliminary mandatory injunction.The,respondent Judge issued an order
stating that the President of the Philippines does not only exercise supervision but also
control over all government-owned and controlled corporations including the NAMARCO;
hence, he may review, revise, alter, modify or nullify the decision or action of the Board of
Directors of any government-owned and controlled corporation and substitute his
judgment for that of the latter. Plaintiff's right to reinstatement, therefore, appears to be
very clear; and considering that the effect of the issuance of the writ prayed for is rather to
re-establish and maintain a pre-existing continuing relation between the parties and
considering further that there is an invasion of plaintiff's right and the injury is a continuing
one, the Court hereby grants plaintiff's prayer and hereby orders the issuance of a writ of
preliminary mandatory injunction directing the defendants to immediately reinstate the
plaintiff to his position as Manager of the Traffic Storage Department of the National
Marketing Corporation upon filing a bond in the amount of P5,000.00.

Defendant (NAMARCO) filed a motion for reconsideration; and when the motion was
denied, they filed the present petition with this Court, which on March 15, 1966, issued a
writ of preliminary injunction.

Issue:

whether the President of the Philippines had authority to reverse the decision of the
Board of Directors of the NAMARCO and to order the reinstatement of Juan T. Arive

Held:

The President of the Philippines’ authority to review and reverse the decision of the
NAMARCO Board of Directors dismissing Juan T. Arive from his position in the NAMARCO and
to order his reinstatement falls within the constitutional power of the President over all
executive departments, bureaus and offices. Under our governmental set-up, corporations
owned or controlled by the government, such as the NAMARCO, partake of the nature of
government bureaus or offices, which are administratively supervised by the Administrator
105
EXECUTIVE DEPARTMENT
of the Office of Economic Coordination, “whose compensation and rank shall be that of a
head of an Executive Department” and who “shall be responsible to the President of the
Philippines under whose control his functions ... shall be exercised.” (Executive Order No. 386
of December 22, 1950, section 1, issued under the Reorganization Act of 1950).

The fact that section 13(d) of Republic Act No. 1345 (the NAMARCO Charter and
likewise section 11(d) of the Uniform Charter for Government Owned or Controlled
Corporations (Ex. Order No. 399 of January 5, 1951) which authorize the general manager of
such corporations, with the approval of the Board of Directors, to remove for cause any
subordinate employee of the Corporation do not provide for an appeal from the general
manager’s decision of removal to any superior officer, body or agency, does not mean that
no appeal lies from such decision to the President.

The right to appeal to the President reposes upon the President’s power of control
over the executive departments. And control simply means the power of an officer to alter
or modify or nullify or set aside what a subordinate officer had done in the performance of
his duties and to substitute the judgment of the former for the latter.

106
EXECUTIVE DEPARTMENT
DRILON V LIM

FACTS:

The Secretary of Justice had, on appeal to him of four oil companies and a taxpayer,
declared Ordinance No. 7794, otherwise known as the Manila Revenue Code, null and void
for non-compliance with the prescribed procedure in the enactment of tax ordinances and
for containing certain provisions contrary to law and public policy. 1

In a petition for certiorari filed by the City of Manila, RTC Judge Rodolfo C. Palattao
revoked the Secretary's resolution, sustained the ordinance and declared Section 187 of the
Local Government Code unconstitutional insofar as it empowered the Secretary of Justice
to review tax ordinances and, inferentially, to annul them. He cited the familiar distinction
between control and supervision, the first being "the power of an officer to alter or modify
or set aside what a subordinate officer had done in the performance of his duties and to
substitute the judgment of the former for the latter," while the second is "the power of a
superior officer to see to it that lower officers perform their functions in accordance with
law." 6 His conclusion was that the challenged section gave to the Secretary the power of
control and not of supervision only as vested by the Constitution in the President of the
Philippines. This was, in his view, a violation not only of Article X, specifically Section 4
thereof, 7 and of Section 5 on the taxing powers of local governments, 8 and the policy of
local autonomy in general.

ISSUE:. W/O/N Section 187 of the Local Government Code is unconstitutional


because it gives the Secretary a power of control

HELD:No. Section 187 of the Local Government Code reading as follows:

Procedure For Approval And Effectivity Of Tax Ordinances And Revenue


Measures; Mandatory Public Hearings. — The procedure for approval of local
tax ordinances and revenue measures shall be in accordance with the
provisions of this Code: Provided, That public hearings shall be conducted for
the purpose prior to the enactment thereof; Provided, further, That any
question on the constitutionality or legality of tax ordinances or revenue
measures may be raised on appeal within thirty (30) days from the effectivity
thereof to the Secretary of Justice who shall render a decision within sixty (60)
days from the date of receipt of the appeal: Provided, however, That such
appeal shall not have the effect of suspending the effectivity of the ordinance
and the accrual and payment of the tax, fee, or charge levied therein:
Provided, finally, That within thirty (30) days after receipt of the decision or the
lapse of the sixty-day period without the Secretary of Justice acting upon the
appeal, the aggrieved party may file appropriate proceedings with a court of
competent jurisdiction.
107
EXECUTIVE DEPARTMENT
Section 187 authorizes the Secretary of Justice to review only the constitutionality or
legality of the tax ordinance and, if warranted, to revoke it on either or both of these
grounds. When he alters or modifies or sets aside a tax ordinance, he is not also permitted
to substitute his own judgment for the judgment of the local government that enacted the
measure. Secretary Drilon did set aside the Manila Revenue Code, but he did not replace it
with his own version of what the Code should be. He did not pronounce the ordinance
unwise or unreasonable as a basis for its annulment. He did not say that in his judgment it
was a bad law. What he found only was that it was illegal. All he did in reviewing the said
measure was determine if the petitioners were performing their functions in accordance
with law, that is, with the prescribed procedure for the enactment of tax ordinances and
the grant of powers to the city government under the Local Government Code. As we see it,
that was an act not of control but of mere supervision.

An officer in control lays down the rules in the doing of an act. If they are not followed, he
may, in his discretion, order the act undone or re-done by his subordinate or he may even
decide to do it himself. Supervision does not cover such authority. The supervisor or
superintendent merely sees to it that the rules are followed, but he himself does not lay
down such rules, nor does he have the discretion to modify or replace them. If the rules are
not observed, he may order the work done or re-done but only to conform to the
prescribed rules. He may not prescribe his own manner for the doing of the act. He has no
judgment on this matter except to see to it that the rules are followed. In the opinion of the
Court, Secretary Drilon did precisely this, and no more nor less than this, and so performed
an act not of control but of mere supervision.

108
EXECUTIVE DEPARTMENT
CARPIO V. EXECUTIVE SECRETARY

FACTS:

Petitioner Antonio Carpio as citizen, taxpayer and member of the Philippine Bar, filed this
petition, questioning the constitutionality of RA 6975 with a prayer for TRO.

RA 6875, entitled “AN ACT ESTABLISHIGN THE PHILIPPINE NATIONAL POLICE UNDER A
REORGANIZED DEPARTMENT OF THE INTERIOR AND LOCAL GOVERNMENT, AND FOR
OTHER PURPOSES,” allegedly contravened Art. XVI, sec. 6 of the 1986 Constitution: “The
State shall establish and maintain one police force, which shall be national in scope and
civilian in character, to be administered and controlled by a national police commission.
The authority of local executives over the police units in their jurisdiction shall be provided
by law.”

ISSUEs:

o Whether or not RA 6975 is contrary to the Constitution


o Whether or not Sec. 12 RA 6975 constitutes an “encroachment upon, interference with,
and an abdication by the President of, executive control and commander-in-chief powers”

HELD:

Power of Administrative Control

NAPOLCOM is under the Office of the President.

SC held that the President has control of all executive departments, bureaus, and offices.
This presidential power of control over the executive branch of government extends over
all executive officers from Cabinet Secretary to the lowliest clerk. In the landmark case of
Mondano vs. Silvosa, the power of control means “the power of the President to alter or
modify or nullify or set aside what a subordinate officer had done in the performance of his
duties and to substitute the judgment of the former with that of the latter.” It is said to be
at the very “heart of the meaning of Chief Executive.”

As a corollary rule to the control powers of the President is the “Doctrine of Qualified
Political Agency.” As the President cannot be expected to exercise his control powers all at
the same time and in person, he will have to delegate some of them to his Cabinet
members.

Under this doctrine, which recognizes the establishment of a single executive, “all executive
and administrative organizations are adjuncts of the Executive Department, the heads of
109
EXECUTIVE DEPARTMENT
the various executive departments are assistants and agents of the Chief Executive, and,
except in cases where the Chief Executive is required by the Constitution or law to act in
person or the exigencies of the situation demand that he act personally, the multifarious
executive and administrative functions of the Chief Executive are performed by and
through the executive departments, and the acts of the Secretaries of such departments,
performed and promulgated in the regular course of business, unless disapproved or
reprobated by the Chief Executive, are presumptively the acts of the Chief Executive.

Thus, “the President’s power of control is directly exercised by him over the members of
the Cabinet who, in turn, and by his authority, control the bureaus and other offices under
their respective jurisdictions in the executive department.”

The placing of NAPOLCOM and PNP under the reorganized DILG is merely an administrative
realignment that would bolster a system of coordination and cooperation among the
citizenry, local executives and the integrated law enforcement agencies and public safety
agencies.

Power of Executive Control

Sec. 12 does not constitute abdication of commander-in-chief powers. It simply provides for
the transition period or process during which the national police would gradually assume
the civilian function of safeguarding the internal security of the State. Under this instance,
the President, to repeat, abdicates nothing of his war powers. It would bear to here state,
in reiteration of the preponderant view, that the President, as Commander-in-Chief, is not a
member of the Armed Forces. He remains a civilian whose duties under the Commander-in-
Chief provision “represent only a part of the organic duties imposed upon him. All his other
functions are clearly civil in nature.” His position as a civilian Commander-in-Chief is
consistent with, and a testament to, the constitutional principle that “civilian authority is, at
all times, supreme over the military.”

110
EXECUTIVE DEPARTMENT
SANLAKAS V. EXECUTIVE SECRETARY

FACTS: They came in the middle of the night. Armed with high-powered ammunitions and
explosives, some three hundred junior officers and enlisted men of the Armed Forces of the
Philippines (AFP) stormed into the Oakwood Premiere apartments in Makati City in the wee
hours of July 27, 2003. Bewailing the corruption in the AFP, the soldiers demanded, among
other things, the resignation of the President, the Secretary of Defense and the Chief of the
Philippine National Police (PNP).[1]
In the wake of the Oakwood occupation, the President issued later in the day
Proclamation No. 427 and General Order No. 4, both declaring a state of rebellion and
calling out the Armed Forces to suppress the rebellion. by evening, soldiers agreed to
return to barracks. GMA, however, did not immediately lift the declaration of a state of
rebellion, only doing so on August 1, 2003 thru Proc NO. 435.

ISSUES:
1. whether or not petitioners have standing
2. whether or not case has been rendered moot by the lifting of the proclamation
3. whether or not the proclamation calling the state of rebellion is proper

RULING:
1. NOT EVERY PETITIONER. only members of the House and Sen Pimentel have standing.
Sanlakas & PM have no standing by analogy with LDP in Lacson v Perez"… petitioner has not
demonstrated any injury to itself which would justify the resort to the Court. Petitioner is a
juridical person not subject to arrest. Thus, it cannot claim to be threatened by a
warrantless arrest. Nor is it alleged that its leaders, members, and supporters are being
threatened with warrantless arrest and detention for the crime of rebellion." At best they
seek for declaratory relief, which is not in the original jurisdiction of SC. Even assuming that
Sanlakas & PM are "people's organizations" in the language of Ss15-16, Art13 of the Consti,
they are still not endowed with standing for as inKilosbayan v Morato "These provisions
have not changed the traditional rule that only real parties in interest or those with
standing, as the case may be, may invoke the judicial power. The jurisdiction of this Court,
even in cases involving constitutional questions, is limited by the "case and controversy"
requirement of S5,Art8. This requirement lies at the very heart of the judicial function." SJS,
though alleging to be taxpayers, is not endowed with standing since "A taxpayer may bring
suit where the act complained of directly involves the illegal disbursement of public funds
derived from taxation.No such illegal disbursement is alleged." Court has ruled out the
doctrine of "transcendental importance" regarding constitutional questions in this
particular case. Only members of Congress, who's (?) powers as provided in the Consti on
giving the Pres emergency powers are allegedly being impaired, can question the legality of
the proclamation of the state of rebellion.
111
EXECUTIVE DEPARTMENT
2. YES. As a rule, courts do not adjudicate moot cases, judicial power being limited to the
determination of "actual controversies." Nevertheless, courts will decide a question,
otherwise moot, if it is "capable of repetition yet evading review."19 The case at bar is one
such case, since prior events (the May 1, 2001 incident when the Pres also declared a state
of rebellion) prove that it can be repeated. 3. YES. S18, Art 7 grants the President, as
Commander-in-Chief, a "sequence" of "graduated power[s]." From the most to the least
benign, these are: the calling out power, the power to suspend the privilege of the writ of
habeas corpus, and the power to declare martial law. In the exercise of the latter two
powers, the Constitution requires the concurrence of two conditions, namely, an actual
invasion or rebellion, and that public safety requires the exercise of such power. However,
as we observed in Integrated Bar of the Philippines v. Zamora, "[t]hese conditions are not
required in the exercise of the calling out power. The only criterion is that 'whenever it
becomes necessary,' the President may call the armed forces 'to prevent or suppress
lawless violence, invasion or rebellion.'"Nevertheless, it is equally true that S18, Art7 does
not expressly prohibit the President from declaring a state of rebellion. Note that the
Constitution vests the President not only with Commander-in-Chief powers but, first and
foremost, with Executive powers. The ponencia then traced the evolution of executive
power in the US (Jackson and the South Carolina situation, Lincoln and teh 'war powers',
Cleveland in In re: Eugene Debs) in an effort to show that "the Commander-in-Chief powers
are broad enough as it is and become more so when taken together with the provision on
executive power and the presidential oath of office. Thus, the plenitude of the powers of
the presidency equips the occupant with the means to address exigencies or threats which
undermine the very existence of government or the integrity of the State." This,
plus Marcos v Manglapus on residual powers, the Rev Admin Code S4, Ch2, Bk3 on the
executive power of the Pres to declare a certain status, argue towards the validity of the
proclamation. However, the Court maintains that the declaration is devoid of any legal
significance for being superflous. Also, the mere declaration of a state of rebellion cannot
diminish or violate constitutionally protected rights. if a state of martial law does not
suspend the operation of the Constitution or automatically suspend the privilege of the writ
of habeas corpus,61 then it is with more reason that a simple declaration of a state of
rebellion could not bring about these conditions. Apprehensions that the military and police
authorities may resort to warrantless arrests are likewise unfounded. In Lacson vs. Perez,
supra, majority of the Court held that "[i]n quelling or suppressing the rebellion, the
authorities may only resort to warrantless arrests of persons suspected of rebellion, as
provided under Section 5, Rule 113 of the Rules of Court,63 if the circumstances so warrant.
The warrantless arrest feared by petitioners is, thus, not based on the declaration of a
'state of rebellion.'"64 In other words, a person may be subjected to a warrantless arrest
for the crime of rebellion whether or not the President has declared a state of rebellion, so
long as the requisites for a valid warrantless arrest are present. The argument that the
declaration of a state of rebellion amounts to a declaration of martial law and, therefore, is
a circumvention of the report requirement, is a leap of logic. There is no illustration that the
112
EXECUTIVE DEPARTMENT
President has attempted to exercise or has exercised martial law powers. Finally, Nor by
any stretch of the imagination can the declaration constitute an indirect exercise of
emergency powers, which exercise depends upon a grant of Congress pursuant to S23 (2),
Art6 of the Constitution. The petitions do not cite a specific instance where the President
has attempted to or has exercised powers beyond her powers as Chief Executive or as
Commander-in-Chief. The President, in declaring a state of rebellion and in calling out the
armed forces, was merely exercising a wedding of her Chief Executive and Commander-in-
Chief powers. These are purely executive powers, vested on the President by S1 & 18, Art7,
as opposed to the delegated legislative powers contemplated by Section 23 (2), Article VI.

113
EXECUTIVE DEPARTMENT
COMMANDER-IN-CHIEF

31. EX PARTE MILLIGAN (1866)

114
EXECUTIVE DEPARTMENT
U.S. Supreme Court
Ex parte Milligan, 71 U.S. 4 Wall. 2 2 (1866)
Syllabus
1. Circuit Courts, as well as the judges thereof, are authorized, by the fourteenth section
of the Judiciary Act, to issue the writ of habeas corpus for the purpose of inquiring into
the cause of commitment, and they have jurisdiction, except in cases where the privilege
of the writ is suspended, to hear and determine the question whether the party is
entitled to be discharged.
2. The usual course of proceeding is for the court, on the application of the prisoner for a
writ of habeas corpus, to issue the writ, and, on its return, to hear and dispose of the
case; but where the cause of imprisonment is fully shown by the petition, the court may,
without issuing the writ, consider and determine whether, upon the facts presented in
the petition, the prisoner, if brought before the court, would be discharged.
3. When the Circuit Court renders a final judgment refusing to discharge the prisoner, he
may bring the case here by writ of error, and, if the judges of the Circuit Court, being
opposed in opinion, can render no judgment, he may have the point upon which the
disagreement happens certified to this tribunal.
4. A petition for a writ of habeas corpus, duly presented, is the institution of a cause on
behalf of the petitioner, and the allowance or refusal of the process, as well as the
subsequent disposition of the prisoner is matter of law, and not of discretion.
5. A person arrested after the passage of the act of March 3d, 1863, "relating to habeas
corpus and regulating judicial proceedings in certain cases," and under the authority of
said act, was entitled to his discharge if not indicted or presented by the grand jury
convened at the first subsequent term of the Circuit or District Court of the United States
for the District.
6. The omission to furnish a list of the persons arrested to the judges of the Circuit or
District Court as provided in the said act did not impair the right of said person, if not
indicted or presented, to his discharge.
7. Military commissions organized during the late civil war, in a State not invaded and not
engaged in rebellion, in which the Federal courts were open, and in the proper and
unobstructed exercise of their judicial functions, had no jurisdiction to try, convict, or
sentence for any criminal offence, a citizen who was neither a resident of a rebellious
State nor a prisoner of war, nor a person in the military or naval service. And Congress
could not invest them with any such power.
8. The guaranty of trial by jury contained in the Constitution was intended for a state of
war, as well as a state of peace, and is equally binding upon rulers and people at all times
and under all circumstances.
9. The Federal authority having been unopposed in the State of Indiana, and the Federal
courts open for the trial of offences and the redress of grievances, the usages of war
could not, under the Constitution, afford any sanction for the trial there of a citizen in civil

115
EXECUTIVE DEPARTMENT
life not connected with the military or naval service, by a military tribunal, for any offence
whatever.
10. Cases arising in the land or naval forces, or in the militia in time of war or public
danger, are excepted from the necessity of presentment or indictment by a grand jury,
and the right of trial by jury in such cases is subject to the same exception.
11. Neither the President nor Congress nor the Judiciary can disturb any one of the
safeguards of civil liberty incorporated into the Constitution except so far as the right is
given to suspend in certain cases the privilege of the writ of habeas corpus.
12. A citizen not connected with the military service and a resident in a State where the
courts are open and in the proper exercise or their jurisdiction cannot, even when the
privilege of the writ of habeas corpus is suspended, be tried, convicted, or sentenced
otherwise than by the ordinary courts of law.
13. Suspension of the privilege of the writ of habeas corpus does not suspend the writ
itself. The writ issues as a matter of course, and, on its return, the court decides whether
the applicant is denied the right of proceeding any further.
14. A person who is a resident of a loyal State, where he was arrested, who was never
resident in any State engaged in rebellion, nor connected with the military or naval
service, cannot be regarded as a prisoner of war.
This case came before the court upon a certificate of division from the judges of the
Circuit Court for Indiana, on a petition for discharge from unlawful imprisonment.
The case was thus:
An act of Congress -- the Judiciary Act of 1789, [Footnote 1] section 14 -- enacts that the
Circuit Courts of the United States
"Shall have power to issue writs of habeas corpus. And that either of the justices of the
Supreme Court, as well as judges of the District Court, shall have power to grant writs of
habeas corpus for the purpose of an inquiry into the cause of commitment. Provided,"
&c.
Another act -- that of March 3d, 1863, [Footnote 2] "relating to habeas corpus, and
regulating judicial proceedings in certain cases" -- an act passed in the midst of the
Rebellion -- makes various provisions in regard to the subject of it.
The first section authorizes the suspension, during the Rebellion, of the writ of habeas
corpus, throughout the United States, by the President.
Two following sections limited the authority in certain respects.
The second section required that lists of all persons, being citizens of States in which the
administration of the laws had continued unimpaired in the Federal courts, who were
then held, or might thereafter be held, as prisoners of the United States, under the
authority of the President, otherwise than as prisoners of war, should be furnished by the
Secretary of State and Secretary of War to the judges of the Circuit and District Courts.
These lists were to contain the names of all persons, residing within their respective
jurisdictions, charged with violation of national law. And it was required, in cases where
the grand jury in attendance upon any of these courts should terminate its session

116
EXECUTIVE DEPARTMENT
without proceeding by indictment or otherwise against any prisoner named in the list,
that the judge of the court should forthwith make an order that such prisoner, desiring a
discharge, should be brought before him or the court to be discharged, on entering into
recognizance, if required, to keep the peace and for good behavior, or to appear, as the
court might direct, to be further dealt with according to law. Every officer of the United
States having custody of such prisoners was required to obey and execute the
judge's order, under penalty, for refusal or delay, of fine and imprisonment.
The third section enacts, in case lists of persons other than prisoners of war then held in
confinement or thereafter arrested, should not be furnished within twenty days after the
passage of the act, or, in cases of subsequent arrest, within twenty days after the time of
arrest, that any citizen, after the termination of a session of the grand jury without
indictment or presentment, might, by petition alleging the facts and verified by oath,
obtain the judge's order of discharge in favor of any person so imprisoned, on the terms
and conditions prescribed in the second section.
This act made it the duty of the District Attorney of the United States to attend
examinations on petitions for discharge.
By proclamation, [Footnote 3] dated the 15th September following, the President,
reciting this statute, suspended the privilege of the writ in the cases where, by his
authority, military, naval, and civil officers of the United States
"hold persons in their custody either as prisoners of war, spies, or aiders and abettors of
the enemy, . . . or belonging to the land or naval force of the United States, or otherwise
amenable to military law, or the rules and articles of war, or the rules or regulations
prescribed for the military or naval services, by authority of the President, or for resisting
a draft, or for any other offence against the military or naval service."
With both these statutes and this proclamation in force, Lamdin P. Milligan, a citizen of
the United States, and a resident and citizen of the State of Indiana, was arrested on the
5th day of October, 1864, at his home in the said State, by the order of Brevet Major-
General Hovey, military commandant of the District of Indiana, and by the same authority
confined in a military prison at or near Indianapolis, the capital of the State. On the 21st
day of the same month, he was placed on trial before a "military commission," convened
at Indianapolis, by order of the said General, upon the following charges, preferred by
Major Burnett, Judge Advocate of the Northwestern Military Department, namely:
1. "Conspiracy against the Government of the United States;"
2. "Affording aid and comfort to rebels against the authority of the United States;"
3. "Inciting insurrection;"
4. "Disloyal practices;" and
5. "Violation of the laws of war."
Under each of these charges, there were various specifications. The substance of them
was joining and aiding, at different times between October, 1863, and August, 1864, a
secret society known as the Order of American Knights or Sons of Liberty, for the purpose
of overthrowing the Government and duly constituted authorities of the United States;

117
EXECUTIVE DEPARTMENT
holding communication with the enemy; conspiring to seize munitions of war stored in
the arsenals; to liberate prisoners of war, &c.; resisting the draft, &c.; . . .
"at a period of war and armed rebellion against the authority of the United States, at or
near Indianapolis [and various other places specified] in Indiana, a State within the
military lines of the army of the United States and the theatre of military operations, and
which had been and was constantly threatened to be invaded by the enemy."
These were amplified and stated with various circumstances.
An objection by him to the authority of the commission to try him being overruled,
Milligan was found guilty on all the charges, and sentenced to suffer death by hanging,
and this sentence, having been approved, he was ordered to be executed on Friday, the
19th of May, 1865.
On the 10th of that same May, 1865, Milligan filed his petition in the Circuit Court of the
United States for the District of Indiana, by which, or by the documents appended to
which as exhibits, the above facts appeared. These exhibits consisted of the order for the
commission; the charges and specifications; the findings and sentence of the court, with a
statement of the fact that the sentence was approved by the President of the United
States, who directed that it should "be carried into execution without delay;" all "by order
of the Secretary of War."
The petition set forth the additional fact that, while the petitioner was held and detained,
as already mentioned, in military custody (and more than twenty days after his arrest), a
grand jury of the Circuit Court of the United States for the District of Indiana was
convened at Indianapolis, his said place of confinement, and duly empaneled, charged,
and sworn for said district, held its sittings, and finally adjourned without having found
any bill of indictment, or made any presentment whatever against him. That at no time
had he been in the military service of the United States, or in any way connected with the
land or naval force, or the militia in actual service; nor within the limits of any State
whose citizens were engaged in rebellion against the United States, at any time during
the war, but, during all the time aforesaid, and for twenty years last past, he had been an
inhabitant, resident, and citizen of Indiana. And so that it had been
"wholly out of his power to have acquired belligerent rights or to have placed himself in
such relation to the government as to have enabled him to violate the laws of war."
The record, in stating who appeared in the Circuit Court, ran thus:
"Be it remembered, that on the 10th day of May, A.D. 1865, in the court aforesaid, before
the judges aforesaid, comes Jonathan W. Gorden, Esq., of counsel for said Milligan, and
files here, in open court, the petition of said Milligan, to be discharged.. . . . At the same
time comes John Hanna, Esquire, the attorney prosecuting the pleas of the United States
in this behalf. And thereupon, by agreement, this application is submitted to the court,
and day is given, &c."
The prayer of the petition was that, under the already mentioned act of Congress of
March 3d, 1863, the petitioner might be brought before the court and either turned over
to the proper civil tribunal to be proceeded with according to the law of the land or
discharged from custody altogether.
118
EXECUTIVE DEPARTMENT
At the hearing of the petition in the Circuit Court, the opinions of the judges were
opposed upon the following questions:
I. On the facts stated in the petition and exhibits, ought a writ of habeas corpus to be
issued according to the prayer of said petitioner?
II. On the facts stated in the petition and exhibits, ought the said Milligan to be discharged
from custody as in said petition prayed?
III. Whether, upon the facts stated in the petition and exhibits, the military commission
had jurisdiction legally to try and sentence said Milligan in manner and form, as in said
petition and exhibit is stated?
And these questions were certified to this court under the provisions of the act of
Congress of April 29th, 1802, [Footnote 4] an act which provides
"that whenever any question shall occur before a Circuit Court upon which the opinions
of the judges shall be opposed, the point upon which the disagreement shall happen
shall, during the same term, upon the request of either party or their counsel, be stated
under the direction of the judges and certified under the seal of the court to the Supreme
Court, at their next session to be held thereafter, and shall by the said court
be finally decided, and the decision of the Supreme Court and their order in the premises
shall be remitted to the Circuit Court and be there entered of record, and shall have
effect according to the nature of the said judgment and order; Provided, That nothing
herein contained shall prevent the cause from proceeding if, in the opinion of the court,
further proceedings can be had without prejudice to the merits."
The three several questions above mentioned were argued at the last term. And along
with them, an additional question raised in this court, namely:
IV. A question of jurisdiction, as -- 1. Whether the Circuit Court had jurisdiction to hear
the case there presented? -- 2. Whether the case sent up here by certificate of division
was so sent up in conformity with the intention of the act of 1802? in other words,
whether this court had jurisdiction of the questions raised by the certificate?
Primary Holding
It is unconstitutional to try civilians by military tribunals unless there is no civilian court
available.
Ex parte Milligan
Supreme Court of the United States
Argued March 5, 1866
Decided April 3, 1866
Full case name Ex parte Lambdin P.
Milligan
Citations 71 U.S. 2 (more)
4 Wall. 2; 18 L. Ed. 281;
1866 U.S. LEXIS 861
119
EXECUTIVE DEPARTMENT
Prior history This case came before the
Court upon a certificate of
division from the judges of
the Circuit Court for
Indiana, on a petition for
discharge from unlawful
imprisonment.
Holding
Trying citizens in military courts is
unconstitutional when civilian courts are still
operating. Trial by military tribunal is
constitutional only when there is no power
left but the military, and the military may
validly try criminals only as long as is
absolutely necessary.
Court membership
Chief Justice
Salmon P. Chase

Associate Justices
James M. Wayne · Samuel
Nelson
Robert C. Grier · Nathan Clifford
Noah H. Swayne · Samuel F.
Miller
David Davis · Stephen J. Field
Case opinions
Majority Davis, joined by Clifford,
Field, Grier, Nelson
Concur/dissent Chase, joined by Wayne,
Swayne, Miller
Laws applied
U.S. Const., Habeas Corpus Suspension Act
1863
Ex parte Milligan, 71 U.S. (4 Wall.) 2 (1866), was a U.S. Supreme Court case that ruled the
application of military tribunals to citizens when civilian courts are still operating is
unconstitutional. In this particular case the Court was unwilling to give PresidentAbraham
120
EXECUTIVE DEPARTMENT
Lincoln's administration the power of military commission jurisdiction, part of the
administration's controversial plan to deal with Union dissenters during the American
Civil War. Justice David Davis, who delivered the majority opinion, stated that "martial
rule can never exist when the courts are open" and confined martial law to areas of
"military operations, where war really prevails," and when it was a necessity to provide a
substitute for a civil authority that had been overthrown. Chief Justice Chase and three
associate justices filed a separate opinion concurring with the majority in the judgment,
but asserted that Congress had the power to authorize a military commission, although it
had not done so in Milligan's case.
The landmark case stemmed from a trial by a military commission of Lambdin P.
Milligan (for whom the case is named), Stephen Horsey, William A. Bowles, and Andrew
Humphreys that convened at Indianapolis on October 21, 1864. The charges against the
men included, among others, conspiracy against the U.S. government, offering aid and
comfort to the Confederates, and incitingrebellion. On December 10, 1864, Milligan,
Bowles, and Horsey were found guilty on all charges and sentenced to hang. Humphreys
was found guilty and sentenced to hard labor for the remainder of the war. (The sentence
for Humphreys was later modified, allowing his release; President Andrew
Johnson commuted the sentences for Milligan, Bowles, and Horsey to life imprisonment.)
On May 10, 1865, Milligan's legal counsel filed a petition in the Circuit Court of the United
States for the District of Indiana at Indianapolis for a writ of habeas corpus, which called
for a justification of Milligan's arrest. A similar petition was filed on behalf of Bowles and
Horsey. The two judges who reviewed Milligan's petition disagreed about the issue of
whether the U.S. Constitution prohibited civilians from being tried by a military
commission and passed the case to the U.S. Supreme Court. The case was argued before
the Court on March 5 and March 13, 1866; its decision was handed down on April 3,
1866.

Background of the case[edit]


Suppression of dissenters[edit]
During the American Civil War, the administration of President Abraham Lincoln dealt
with Union dissenters by declaring martial law, sanctioning arbitrary arrest and detention,
suspending the writ of habeas corpus that requires justification of the detention, and
initiating trials by military commission rather than conventional civil courts. The rationale
for these actions was based on Article 1, Section 9 of the U.S. Constitution, which
authorizes the suspension of the writ of habeas corpus "when in Cases of Rebellion or
Invasion the public Safety may require it."[1][2] Lincoln theorized that the civil courts in the
United States were established to try individuals and small groups "on charges of crimes
well defined in the law" and not to deal with large groups of dissenters, whose actions,
while damaging to the war effort, did not constitute a "defined crime" in states loyal to
the government.[3] Lincoln believed his administration's plan would suppress anti-
government agitators, but he was also optimistic that it would be rescinded after the war
ended.[4]
121
EXECUTIVE DEPARTMENT
The first test of Lincoln's thesis for silencing dissenters occurred in the spring of
1863. Clement Vallandigham, an Ohio politician and anti-war Democrat, was placed under
arrest on May 5, 1863, taken to Cincinnati for a trial before a military commission, and
jailed. Vallandigham was found guilty and sentenced to prison for the remainder of the
war, but Lincoln commuted the sentence. Vallandigham's petition to the U.S. Supreme
Court, known as Ex parte Vallandigham, was denied.[5]
Trial by military commission[edit]
The next test began with trials by a military commission that led to the landmark U.S.
Supreme Court case known as Ex parte Milligan. On September 17, 1864, General Alvin
Peterson Hovey, commander of the Military District of Indiana, authorized a military
commission to meet on September 19 at Indianapolis, Indiana, to begin trials of Harrison
H. Dodd, "grand commander" of the Sons of Liberty in Indiana, and others placed under
military arrest.[6] These prisoners included Democrats Lambdin P. Milligan, a lawyer living
inHuntington, Indiana, and an outspoken critic of President Lincoln and Indiana's
Republican governor Oliver P. Morton; Joseph J. Bingham, editor of the Indianapolis Daily
Sentineland chairman of Indiana's Democratic State Central Committee; William A.
Bowles of French Lick, Indiana; William M. Harrison, secretary of the Democratic Club
of Marion County, Indiana; Horace Heffren, editor of the Washington (Indiana) Democrat;
Stephen Horsey of Martin County, Indiana; and Andrew Humphreys of Bloomfield,
Indiana.[7][8]Two other men, James B. Wilson and David T. Yeakel, were also
seized.[9] Dodd, who was the first to be tried, escaped from jail before his trial was
completed, and fled toCanada. On October 10, 1864, he was found guilty, convicted in
absentia, and sentenced to hang. Charges against Bingham, Harrison, Yeakel, and Wilson
were dismissed. Heffren was released before the proceedings against Milligan began.[10]
The military commission for the trial of Milligan, Horsey, Bowles, and Humphreys
convened at Indianapolis on October 21, 1864. The commission considered five charges
against the men: conspiracy against the U.S. government, offering aid and comfort to
the Confederates, inciting insurrections, "disloyal practices," and "violation of the laws of
war."[11][12]The defendants were alleged to have established a secret organization that
planned to liberate Confederate soldiers from Union prisoner-of-war camps in Illinois,
Indiana, andOhio, and then seize an arsenal, provide the freed prisoners with arms, raise
an armed force to incite a general insurrection, and join with the Confederates to invade
Indiana, Illinois, and Kentucky and make war on the government of the United
States.[11][12][13]
The military commission's decision on December 10, 1864, found Milligan, Bowles, and
Horsey guilty. The men were sentenced to hang on May 19, 1865. Humphreys was found
guilty and sentenced to hard labor for the remainder of the war.[12][14] With President
Lincoln's support, General Hovey modified the sentence for Humphreys, allowing his
release, but Humphreys was required to remain within two specific townships in Greene
County, Indiana, and could not participate in any acts that opposed the war. Efforts were
made to secure pardons for Milligan, Bowles, and Horsey, with the decision passing to
President Johnson following Lincoln's assassination.[14][15]

122
EXECUTIVE DEPARTMENT
Circuit Court petition[edit]
On May 10, 1865, Jonathan W. Gorden, Milligan's legal council, filed a petition for a writ
of habeas corpus in the Circuit Court of the United States for the District of Indiana at
Indianapolis.[12] A similar one was filed on behalf of Bowles and Horsey. The petitions
were based on an act of the Congress titled "An Act Relating to Habeus Corpus and
Regulation Judicial Proceedings in Certain Cases" that went into effect on March 3, 1863.
The act was intended to resolve the question of whether Lincoln had the constitutional
authority to suspend the writ of habeas corpus as authorized under Article 1 of the U.S.
Constitution.[14] Milligan's petition alleged that a federal grand jury had met in
Indianapolis during January 1865, which it did, and it had not indicted him, which is also
true, making him eligible for a release from prison under the congressional act.[14][16]
On May 16, three days before their execution, Horsey's sentence was commuted to life
imprisonment and the executions of Milligan and Bowles were postponed to June 2.
President Johnson approved commutation of the sentences for Milligan and Bowles to life
imprisonment on May 30, 1865.[17] In the interim, Justice David Davis, an associate justice
of the U.S. Supreme Court and a federal circuit court judge that included Indiana, and
Judge Thomas Drummond, another federal circuit court judge, reviewed Milligan's circuit
court petition. The two judges disagreed about the issue of whether the U.S. Constitution
prohibited civilians from being tried by a military commission and passed the case to the
U.S. Supreme Court.[17][18]
Arguments[edit]
The U.S. Supreme Court was asked to consider three questions in the Ex parte
Milligan case:
 Should a writ of habeas corpus be issued based on Milligan's petition?

 Should Milligan be discharged from custody?

 Did the military commission have jurisdiction to try and sentence Milligan?

The Court case did not consider the charges or the evidence presented in the trial by the
military commission. The only issues considered were whether the military commission's
proceedings were constitutional, and whether Milligan was entitled to a discharge.[18]
The case was argued before the Court on March 5 and March 13, 1866. The team
representing the United States was U.S. Attorney General James Speed, Henry Stanbery,
andBenjamin F. Butler, a Civil War general who became a congressman and governor
of Massachusetts.[19] General Butler delivered the argument for the United States. Among
the members of Milligan's legal team were Joseph E. McDonald; David Dudley Field, who
was a New York lawyer and brother of U.S. Supreme Court justice Stephen Johnson
Field;James A. Garfield, a member of Congress in his first ever courtroom argument and a
future U.S. president;[20] and Jeremiah S. Black, President James Buchanan's U.S. Attorney
General and U.S. Secretary of State.[17][19]
The Court's decision[edit]
On April 3, 1866, Chief Justice Salmon P. Chase handed down the Court's decision, which
decreed that the writ of habeas corpus could be issued based on the congressional act of
123
EXECUTIVE DEPARTMENT
March 3, 1863; the military commission did not have the jurisdiction to try and sentence
Milligan; and he was entitled to a discharge. Milligan, Bowles, and Horsey were
discharged from prison on April 12, 1866.[21] The Court's opinion was read during the next
Court session.
On December 17, 1866, Justice Davis delivered the majority opinion explaining that
Milligan, who was a civilian not in military service and resident of a state in which civilian
courts were still functioning, had a right, when charged with a crime, to be tried and
punished according to the law.[19][21]Under the U.S. Constitution this included security
against unreasonable search and seizure, a warrant for probable cause before arrest, and
if indicted, a speedy trial by jury. Justice Davis disagreed with the federal government's
argument regarding the propriety of the military commission, stating that "martial rule
can never exist when the courts are open" and confined martial law to areas of "military
operations, where war really prevails," and when it became a necessity to provide a
substitute for a civil authority that had been overthrown. This was not the situation in
Indiana, where the civilian courts were still operating at the time of Milligan's arrest, trial,
and incarceration.[22] The majority opinion further observed that during the suspension of
the writ of habeas corpus, citizens may be only detained without charges, not "tried" or
executed under the jurisdictions of military tribunals. The writ is not the right itself, but
merely the ability to issue orders demanding the right's "enforcement."[12]
In Ex parte Milligan, a legal debate about governmental power and personal liberty, the
Court's decision stood "on the side of personal liberty."[23] In this case the Court was
unwilling to give President Lincoln's administration the power of military commission
jurisdiction. The Court's decision avoided the risk of its abuse by future administrations in
other situations.[23] It is also important to note the political environment of the decision.
Under a Republican Congress immediately after the Civil War, the Court was reluctant to
hand down any decision that questioned the legitimacy of military courts, especially in
the occupied South. The president's ability to suspend the writ of habeas corpus without
congressional approval was not addressed in this case, most likely because it was
a moot issue with respect to the case at hand. (President Lincoln suspended the writ
nationwide on September 24, 1862,[24] and Congress ratified this action on March 3, 1863,
with the Habeas Corpus Suspension Act. Milligan was detained in October 1864, more
than a year after Congress formally suspended the writ.)
Three types of military jurisdiction[edit]
This case was also important in clarifying the scope of military jurisdiction under the US
Constitution. The Supreme Court justices held:
There are under the Constitution three kinds of military jurisdiction: one to be exercised
both in peace and war; another to be exercised in time of foreign war without the
boundaries of the United States, or in time of rebellion and civil war within states or
districts occupied by rebels treated [71 U.S. 2, 142] as belligerents; and a third to be
exercised in time of invasion or insurrection within the limits of the United States, or
during rebellion within the limits of states maintaining adhesion to the National
Government, when the public danger requires its exercise. The first of these may be
124
EXECUTIVE DEPARTMENT
called jurisdiction under MILITARY LAW, and is found in acts of Congress prescribing rules
and articles of war, or otherwise providing for the government of the national forces; the
second may be distinguished as MILITARY GOVERNMENT, superseding, as far as may be
deemed expedient, the local law, and exercised by the military commander under the
direction of the President, with the express or implied sanction of Congress; while the
third may be denominated MARTIAL LAW PROPER, and is called into action by Congress,
or temporarily, when the action of Congress cannot be invited, and in the case of
justifying or excusing peril, by the President, in times of insurrection or invasion, or of civil
or foreign war, within districts or localities where ordinary law no longer adequately
secures public safety and private rights.[12]
This distinction between martial law and military government was not commonly made
before 1866. However, after the Supreme Court's clarification in this landmark case, it has
continued to be referenced. Birkhimer describes the difference on page 1 of his
opus Military Government and Martial Law (3rd edition, 1914): "Military jurisdiction is
treated in the following pages in its two branches of Military Government and Martial
Law. The former is exercised over enemy territory; the latter over loyal territory of the
State enforcing it."[25] According to the U.S. Army Field Manual FM 27-10, The Law of Land
Warfare, paragraph 362: "Military government is the form of administration by which an
occupying power exercises governmental authority over occupied territory. The necessity
for such government arises from the failure or inability of the legitimate government to
exercise its functions on account of the military occupation, or the undesirability of
allowing it to do so."[26][27]
Concurrence[edit]
Justices David Davis and four others (Nathan Clifford, Stephen Johnson Field, Robert
Cooper Grier, and Samuel Nelson) signed the majority opinion. Chief Justice Salmon P.
Chase and Justices James Moore Wayne, Noah Haynes Swayne, and Samuel Freeman
Miller filed a separate opinion concurring with the majority in the judgment, but they
disagreed with the majority's assertion that Congress did not have the power to authorize
military commissions in Indiana. Chief Justice Chase asserted that Congress had the
power to authorize a military commission, but it had not done so in Milligan's case.[28] The
separate opinion also stated that Congress could "authorize trials for crimes against the
security and safety of the national forces," and its authority to do so "may be derived
from its constitutional power to raise and support armies and to declare war;" while the
civil courts "might be open and undisturbed in their functions... yet wholly incompetent
to avert threatened danger, or to punish, with adequate promptitude and certainty, the
guilty conspirators."[29] However, as Judge Davis described the status of the federal courts
in Indiana at the time, "It needed no bayonets to protect it, and required no military aid
to execute its judgments."[23]
Aftermath[edit]
Ex parte Milligan became well known as the leading U.S. Supreme Court case that found
the president went beyond his legal powers to suppress dissenters during the American

125
EXECUTIVE DEPARTMENT
Civil War. The decision also helped to establish the tradition that presidential and military
action "based on war" had limits.[30]
After Milligan's release from prison, he returned to his home and law practice in
Huntington, Indiana.[31] In 1868 he filed a civil lawsuit in Huntington County, Indiana,
seeking damages related to the Ex parte Milligan case.[32] Milligan's civil case was referred
to the U.S. Circuit Court for Indiana at Indianapolis. Milligan v. Hovey, a two-week jury
trial held in May 1871, named as defendants several men involved in Milligan's treason
trial, including Alvin P. Hovey and Oliver P. Morton. Milligan hired Thomas A. Hendricks, a
trial lawyer, former state legislator, member of Congress, and future governor of Indiana
as his legal council in the civil suit. The defendants hired Benjamin Harrison, an
Indianapolis lawyer and future U.S. president.[33] Milligan's civil suit was "the first major
civil rights jury trial held before the federal courts."[34] At issue was what damages, if any,
Milligan had sustained relating to Ex parte Milligan. Harrison portrayed Milligan as a
traitor, while Hendricks focused on Milligan's "malicious prosecution and false
imprisonment."[34] Milligan refused to admit has affiliation or actions with a seditious
organization during the civil trial. The jury issued its verdict in Milligan's favor on May 30,
1871. Although Milligan sought thousands of dollars in damages, state and federal
statutes limited the claim to five dollars plus court costs.[35]

32. DAVID v. ARROYO

Ponente/Other Opinion: SANDOVAL-GUTIERREZ


Dispositive Portion: WHEREFORE, the Petitions are partly granted. The Court rules that
PP 1017 is CONSTITUTIONAL insofar as it constitutes a call by President Gloria Macapagal-
Arroyo on the AFP to prevent or suppress lawless violence. However, the provisions of PP
1017 commanding the AFP to enforce laws not related to lawless violence, as well as
decrees promulgated by the President, are declared UNCONSTITUTIONAL. In addition, the
provision in PP 1017 declaring national emergency under Section 17, Article VII of the
Constitution is CONSTITUTIONAL, but such declaration does not authorize the President
to take o ver privately-owned public utility or business affected with public interest
without prior legislation.
Constitutional Law; Separation of Powers; Checks and Balances; Judicial Review; One of
the greatest contributions of the American system to this country is the concept of
judicial review enunciated in Marbury v. Madison, 1 Cranch 137 (1803).—One of the
greatest contributions of the American system to this country is the concept of judicial
review enunciated in Marbury v. Madison, 1 Cranch 137 (1803). This concept rests on the
extraordinary simple foundation—The Constitution is the supreme law. It was ordained
by the people, the ultimate source of all political authority. It confers limited powers on
the national government. x x x If the government consciously or unconsciously oversteps
these limitations there must be some authority competent to hold it in control, to thwart
its unconstitutional attempt, and thus to vindicate and preserve inviolate the will of the

126
EXECUTIVE DEPARTMENT
people as expressed in the Constitution. This power the courts exercise. This is the
beginning and the end of the theory of judicial review.
Same; Same; Same; Same; Requisites; The power of judicial review does not repose upon
the courts a “self-starting capacity.”—The power of judicial review does not repose upon
the courts a “self-starting capacity.” Courts may exercise such power only when the
following requisites are present: first, there must be an actual case or controversy;
second, petitioners have to raise a question of constitutionality; third, the constitutional
question must be raised at the earliest opportunity; and fourth, the decision of the
constitutional question must be necessary to the determination of the case itself.
Same; Same; Same; Same; Same; Words and Phrases; An actual case or controversy
involves a conflict of legal right, an opposite legal claims susceptible of judicial
resolution—it is “definite and concrete, touching the legal relations of parties having
adverse legal interest,” a real and substantial controversy admitting of specific relief.—An
actual case or controversy involves a conflict of legal right, an opposite legal claims
susceptible of judicial resolution. It is “definite and concrete, touching the legal relations
of parties having adverse legal interest”; a real and substantial controversy admitting of
specific relief. The Solicitor General refutes the existence of such actual case or
controversy, contending that the present petitions were rendered “moot and academic”
by President Arroyo’s issuance of PP 1021.
Same; Same; Same; Same; Same; Moot and Academic Questions; The “moot and
academic” principle is not a magical formula that can automatically dissuade the courts in
resolving a case; Courts will decide cases, otherwise moot and academic, if: first, there is
a grave violation of the Constitution, second, the exceptional character of the situation
and the paramount public interest is involved, third, when constitutional issue raised
requires formulation of controlling principles to guide the bench, the bar, and the public,
and fourth, the case is capable of repetition yet evading review.—A moot and academic
case is one that ceases to present a justiciable controversy by virtue of supervening
events, so that a declaration thereon would be of no practical use or value. Generally,
courts decline jurisdiction over such case or dismiss it on ground of mootness. The Court
holds that President Arroyo’s issuance of PP 1021 did not render the present petitions
moot and academic. During the eight (8) days that PP 1017 was operative, the police
officers, according to petitioners, committed illegal acts in implementing it. Are PP 1017
and G.O. No. 5 constitutional or valid? Do they justify these alleged illegal acts? These are
the vital issues that must be resolved in the present petitions. It must be stressed that “an
unconstitutional act is not a law, it confers no rights, it imposes no duties, it affords no
protection; it is in legal contemplation, inoperative.” The “moot and academic” principle
is not a magical formula that can automatically dissuade the courts in resolving a case.
Courts will decide cases, otherwise moot and academic, if: first, there is a grave violation
of the Constitution; second, the exceptional character of the situation and the paramount

127
EXECUTIVE DEPARTMENT
public interest is involved; third, when constitutional issue raised requires formulation of
controlling principles to guide the bench, the bar, and the public; and fourth, the case is
capable of repetition yet evading review.
Same; Same; Same; Same; Same; Locus Standi; Words and Phrases; Locus standi is
defined as “a right of appearance in a court of justice on a given question.”—Locus standi
is defined as “a right of appearance in a court of justice on a given question.” In private
suits, standing is governed by the “real-parties-in interest” rule as contained in Section 2,
Rule 3 of the 1997 Rules of Civil Procedure, as amended. It provides that “every action
must be prosecuted or defended in the name of the real party in interest.” Accordingly,
the “real-party-in interest” is “the party who stands to be benefited or injured by the
judgment in the suit or the party entitled to the avails of the suit.” Succinctly put, the
plaintiff’s standing is based on his own right to the relief sought.
Same; Same; Same; Same; Same; Same; The difficulty of determining locus standi arises in
public suits, as here, the plaintiff who asserts a “public right” in assailing an allegedly
illegal official action, does so as a representative of the general public.—The difficulty of
determining locus standi arises in public suits. Here, the plaintiff who asserts a “public
right” in assailing an allegedly illegal official action, does so as a representative of the
general public. He may be a person who is affected no differently from any other person.
He could be suing as a “stranger,” or in the category of a “citizen,” or ‘taxpayer.” In either
case, he has to adequately show that he is entitled to seek judicial protection. In other
words, he has to make out a sufficient interest in the vindication of the public order and
the securing of relief as a “citizen” or “taxpayer.
Same; Same; Same; Same; Same; Same; Taxpayer’s Suits; Citizen’s Suits; The plaintiff in a
taxpayer’s suit is in a different category from the plaintiff in a citizen’s suit—in the former,
the plaintiff is affected by the expenditure of public funds, while in the latter, he is but
the mere instrument of the public concern.—Case law in most jurisdictions now allows
both “citizen” and “taxpayer” standing in public actions. The distinction was first laid
down in Beauchamp v. Silk, where it was held that the plaintiff in a taxpayer’s suit is in a
different category from the plaintiff in a citizen’s suit. In the former, the plaintiff is
affected by the expenditure of public funds, while in the latter, he is but the mere
instrument of the public concern. As held by the New York Supreme Court in People ex rel
Case v. Collins: “In matter of mere public right, however . . . the people are the real
parties . . . It is at least the right, if not the duty, of every citizen to interfere and see that a
public offence be properly pursued and punished, and that a public grievance be
remedied.” With respect to taxpayer’s suits, Terr v. Jordanheld that “the right of a citizen
and a taxpayer to maintain an action in courts to restrain the unlawful use of public funds
to his injury cannot be denied.”
Same; Same; Same; Same; Same; Same; Same; Same; “Direct Injury” Test; To prevent just
about any person from seeking judicial interference in any official policy or act with which

128
EXECUTIVE DEPARTMENT
he disagreed with, and thus hinders the activities of governmental agencies engaged in
public service, the United States Supreme Court laid down the more stringent “direct
injury” test, which test has been adopted in this jurisdiction.—To prevent just about any
person from seeking judicial interference in any official policy or act with which he
disagreed with, and thus hinders the activities of governmental agencies engaged in
public service, the United States Supreme Court laid down the more stringent “direct
injury” test in Ex Parte Levitt, later reaffirmed in Tileston v. Ullman. The same Court ruled
that for a private individual to invoke the judicial power to determine the validity of an
executive or legislative action, he must show that he has sustained a direct injury as a
result of that action, and it is not sufficient that he has a general interest common to all
members of the public. This Court adopted the “direct injury” test in our jurisdiction. In
People v. Vera, 65 Phil. 56 (1937), it held that the person who impugns the validity of a
statute must have “a personal and substantial interest in the case such that he has
sustained, or will sustain direct injury as a result.” The Vera doctrine was upheld in a
litany of cases, such as, Custodio v. President of the Senate, Manila Race Horse Trainers’
Association v. De la Fuente, Pascual v. Secretary of Public Works and Anti-Chinese League
of the Philippines v. Felix.
Same; Same; Same; Same; Same; Same; Being a mere procedural technicality, the
requirement of locus standi may be waived by the Court in the exercise of its discretion,
such as in cases of “transcendental importance,” or where the issues raised have “far-
reaching implications.”—Being a mere procedural technicality, the requirement of locus
standi may be waived by the Court in the exercise of its discretion. This was done in the
1949 Emergency Powers Cases, Araneta v. Dinglasan, 84 Phil. 368 (1949), where the
“transcendental importance” of the cases prompted the Court to act liberally. Such
liberality was neither a rarity nor accidental. In Aquino v. Comelec, 62 SCRA 275 (1975),
this Court resolved to pass upon the issues raised due to the “far-reaching implications”
of the petition notwithstanding its categorical statement that petitioner therein had no
personality to file the suit. Indeed, there is a chain of cases where this liberal policy has
been observed, allowing ordinary citizens, members of Congress, and civic organizations
to prosecute actions involving the constitutionality or validity of laws, regulations and
rulings.
Same; Same; Same; Same; Same; Same; Requisites in order that Taxpayers, Voters,
Concerned Citizens and Legislators may be Accorded Standing to Sue; Recent decisions
show a certain toughening in the Court’s attitude toward legal standing.—By way of
summary, the following rules may be culled from the cases decided by this Court.
Taxpayers, voters, concerned citizens, and legislators may be accorded standing to sue,
provided that the following requirements are met: (1) cases involve constitutional issues;
(2) for taxpayers, there must be a claim of illegal disbursement of public funds or that the
tax measure is unconstitutional; (3) for voters, there must be a showing of obvious
interest in the validity of the election law in question; (4) for concerned citizens, there
129
EXECUTIVE DEPARTMENT
must be a showing that the issues raised are of transcendental importance which must be
settled early; and (5) for legislators, there must be a claim that the official action
complained of infringes upon their prerogatives as legislators. Significantly, recent
decisions show a certain toughening in the Court’s attitude toward legal standing.
Same; Same; Same; Same; Same; Same; It is in the interest of justice that those affected
by Presidential Proclamation (PP) 1017 can be represented by their Congressmen in
bringing to the attention of the Court the alleged violations of their basic rights.—In G.R.
No. 171485, the opposition Congressmen alleged there was usurpation of legislative
powers. They also raised the issue of whether or not the concurrence of Congress is
necessary whenever the alarming powers incident to Martial Law are used. Moreover, it
is in the interest of justice that those affected by PP 1017 can be represented by their
Congressmen in bringing to the attention of the Court the alleged violations of their basic
rights.
Same; Same; Same; Same; Same; Same; When the issue concerns a public right, it is
sufficient that the petitioner is a citizen and has an interest in the execution of the laws.—
In G.R. No. 171400, (ALGI), this Court applied the liberality rule in Philconsa v. Enriquez,
235 SCRA 506 (1994), Kapatiran Ng Mga Naglilingkod sa Pamahalaan ng Pilipinas, Inc. v.
Tan, Association of Small Landowners in the Philippines, Inc. v. Secretary of Agrarian
Reform, Basco v. Philippine Amusement and Gaming Corporation, 197 SCRA 52 (1991),
and Tañada v. Tuvera, 136 SCRA 27 (1985), that when the issue concerns a public right, it
is sufficient that the petitioner is a citizen and has an interest in the execution of the laws.
Same; Same; Same; Same; Same; Same; Organizations may be granted standing to assert
the rights of their members.—In G.R. No. 171483, KMU’s assertion that PP 1017 and G.O.
No. 5 violated its right to peaceful assembly may be deemed sufficient to give it legal
standing. Organizations may be granted standing to assert the rights of their members.
We take judicial notice of the announcement by the Office of the President banning all
rallies and canceling all permits for public assemblies following the issuance of PP 1017
and G.O. No. 5.
Same; Same; Same; Same; Same; Same; National officers of the Integrated Bar of the
Philippines (IBP) have no legal standing where they failed to allege any direct or potential
injury which the IBP as an institution or its members may suffer as a consequence of the
issuance of PP 1017 and G.O. No. 5.—In G.R. No. 171489, petitioners, Cadiz, et al., who
are national officers of the Integrated Bar of the Philippines (IBP) have no legal standing,
having failed to allege any direct or potential injury which the IBP as an institution or its
members may suffer as a consequence of the issuance of PP No. 1017 and G.O. No. 5. In
Integrated Bar of the Philippines v. Zamora, 338 SCRA 81 (2000), the Court held that the
mere invocation by the IBP of its duty to preserve the rule of law and nothing more, while
undoubtedly true, is not sufficient to clothe it with standing in this case. This is too
general an interest which is shared by other groups and the whole citizenry. However, in

130
EXECUTIVE DEPARTMENT
view of the transcendental importance of the issue, this Court declares that petitioner
have locus standi.
Same; Same; Same; Same; Same; Same; The claim of a petitioner that she is a media
personality does not aid her where there is no showing that the enforcement of the
issuances in question prevented her from pursuing her occupation, and neither does her
submission that she has a pending electoral protest before the Presidential Electoral
Tribunal have any relevance where she has not sufficiently shown that PP 1017 will affect
the proceedings or result of her case.—In G.R. No. 171424, Loren Legarda has no
personality as a taxpayer to file the instant petition as there are no allegations of illegal
disbursement of public funds. The fact that she is a former Senator is of no consequence.
She can no longer sue as a legislator on the allegation that her prerogatives as a lawmaker
have been impaired by PP 1017 and G.O. No. 5. Her claim that she is a media personality
will not likewise aid her because there was no showing that the enforcement of these
issuances prevented her from pursuing her occupation. Her submission that she has
pending electoral protest before the Presidential Electoral Tribunal is likewise of no
relevance. She has not sufficiently shown that PP 1017 will affect the proceedings or
result of her case. But considering once more the transcendental importance of the issue
involved, this Court may relax the standing rules.
Same; Same; Same; Presidency; Parties; It is not proper to implead President Arroyo as
respondent—settled is the doctrine that the President, during his tenure of office or
actual incumbency, may not be sued in any civil or criminal case, and there is no need to
provide for it in the Constitution or law; It will degrade the dignity of the high office of the
President, the Head of State, if he can be dragged into court litigations while serving as
such.—It is not proper to implead President Arroyo as respondent. Settled is the doctrine
that the President, during his tenure of office or actual incumbency, may not be sued in
any civil or criminal case, and there is no need to provide for it in the Constitution or law.
It will degrade the dignity of the high office of the President, the Head of State, if he can
be dragged into court litigations while serving as such. Furthermore, it is important that
he be freed from any form of harassment, hindrance or distraction to enable him to fully
attend to the performance of his official duties and functions. Unlike the legislative and
judicial branch, only one constitutes the executive branch and anything which impairs his
usefulness in the discharge of the many great and important duties imposed upon him by
the Constitution necessarily impairs the operation of the Government. However, this does
not mean that the President is not accountable to anyone. Like any other official, he
remains accountable to the people but he may be removed from office only in the mode
provided by law and that is by impeachment.
Presidency; Calling-Out Power; Declaration of State of National Emergency; Petitioners
failed to show that President Arroyo’s exercise of the calling-out power, by issuing PP
1017, is totally bereft of factual basis.—As to how the Court may inquire into the

131
EXECUTIVE DEPARTMENT
President’s exercise of power, Lansang adopted the test that “judicial inquiry can go no
further than to satisfy the Court not that the President’s decision is correct,” but that “the
President did not act arbitrarily.” Thus, the standard laid down is not correctness, but
arbitrariness. In Integrated Bar of the Philippines, this Court further ruled that “it is
incumbent upon the petitioner to show that the President’s decision is totally bereft of
factual basis” and that if he fails, by way of proof, to support his assertion, then “this
Court cannot undertake an independent investigation beyond the pleadings.” Petitioners
failed to show that President Arroyo’s exercise of the calling-out power, by issuing PP
1017, is totally bereft of factual basis. A reading of the Solicitor General’s Consolidated
Comment and Memorandum shows a detailed narration of the events leading to the
issuance of PP 1017, with supporting reports forming part of the records. Mentioned are
the escape of the Magdalo Group, their audacious threat of the Magdalo D-Day, the
defections in the military, particularly in the Philippine Marines, and the reproving
statements from the communist leaders. There was also the Minutes of the Intelligence
Report and Security Group of the Philippine Army showing the growing alliance between
the NPA and the military. Petitioners presented nothing to refute such events. Thus,
absent any contrary allegations, the Court is convinced that the President was justified in
issuing PP 1017 calling for military aid.
Same; Same; Same; In times of emergency, our Constitution reasonably demands that we
repose a certain amount of faith in the basic integrity and wisdom of the Chief Executive
but, at the same time, it obliges him to operate within carefully prescribed procedural
limitations.—In the final analysis, the various approaches to emergency of the above
political theorists—from Lock’s “theory of prerogative,” to Watkins’ doctrine of
“constitutional dictatorship” and, eventually, to McIlwain’s “principle of
constitutionalism”—ultimately aim to solve one real problem in emergency governance,
i.e., that of allotting increasing areas of discretionary power to the Chief Executive, while
insuring that such powers will be exercised with a sense of political responsibility and
under effective limitations and checks. Our Constitution has fairly coped with this
problem. Fresh from the fetters of a repressive regime, the 1986 Constitutional
Commission, in drafting the 1987 Constitution, endeavored to create a government in the
concept of Justice Jackson’s “balanced power structure.” Executive, legislative, and
judicial powers are dispersed to the President, the Congress, and the Supreme Court,
respectively. Each is supreme within its own sphere. But none has the monopoly of power
in times of emergency. Each branch is given a role to serve as limitation or check upon the
other. This system does not weaken the President, it just limits his power, using the
language of McIlwain. In other words, in times of emergency, our Constitution reasonably
demands that we repose a certain amount of faith in the basic integrity and wisdom of
the Chief Executive but, at the same time, it obliges him to operate within carefully
prescribed procedural limitations.

132
EXECUTIVE DEPARTMENT
Same; Same; Same; Freedom of Expression; Facial Challenges; Overbreadth Doctrine; The
overbreadth doctrine is an analytical tool developed for testing “on their faces” statutes
in free speech cases, also known under the American Law as First Amendment cases; A
plain reading of PP 1017 shows that it is not primarily directed to speech or even speech-
related conduct—it is actually a call upon the AFP to prevent or suppress all forms of
lawless violence.—A facial review of PP 1017, using the overbreadth doctrine, is uncalled
for. First and foremost, the overbreadth doctrine is an analytical tool developed for
testing “on their faces” statutes in free speech cases, also known under the American Law
as First Amendment cases. A plain reading of PP 1017 shows that it is not primarily
directed to speech or even speech-related conduct. It is actually a call upon the AFP to
prevent or suppress all forms of lawless violence. In United States v. Salerno, the US
Supreme Court held that “we have not recognized an ‘overbreadth’ doctrine outside the
limited context of the First Amendment” (freedom of speech).
Same; Same; Same; Same; Same; Same; The overbreadth doctrine is not intended for
testing the validity of a law that “reflects legitimate state interest in maintaining
comprehensive control over harmful, constitutionally unprotected conduct”—
“overbreadth claims, if entertained at all, have been curtailed when invoked against
ordinary criminal laws that are sought to be applied to protected conduct.”—The
overbreadth doctrine is not intended for testing the validity of a law that “reflects
legitimate state interest in maintaining comprehensive control over harmful,
constitutionally unprotected conduct.” Undoubtedly, lawless violence, insurrection and
rebellion are considered “harmful” and “constitutionally unprotected conduct.” In
Broadrick v. Oklahoma, it was held: It remains a ‘matter of no little difficulty’ to
determine when a law may properly be held void on its face and when ‘such summary
action’ is inappropriate. But the plain import of our cases is, at the very least, that facial
overbreadth adjudication is an exception to our traditional rules of practice and that its
function, a limited one at the outset, attenuates as the otherwise unprotected behavior
that it forbids the State to sanction moves from ‘pure speech’ toward conduct and that
conduct—even if expressive—falls within the scope of otherwise valid criminal laws that
reflect legitimate state interests in maintaining comprehensive controls over harmful,
constitutionally unprotected conduct. Thus, claims of facial overbreadth are entertained
in cases involving statutes which, by their terms, seek to regulate only “spoken words”
and again, that “overbreadth claims, if entertained at all, have been curtailed when
invoked against ordinary criminal laws that are sought to be applied to protected
conduct.” Here, the incontrovertible fact remains that PP 1017 pertains to a spectrum of
conduct, not free speech, which is manifestly subject to state regulation.
Same; Same; Same; Same; Same; Same; Facial invalidation of laws is considered as
“manifestly strong medicine,” to be used “sparingly and only as a last resort,” and is
“generally disfavored.”—Facial invalidation of laws is considered as “manifestly strong
medicine,” to be used “sparingly and only as a last resort,” and is “generally disfavored”;
133
EXECUTIVE DEPARTMENT
The reason for this is obvious. Embedded in the traditional rules governing constitutional
adjudication is the principle that a person to whom a law may be applied will not be
heard to challenge a law on the ground that it may conceivably be applied
unconstitutionally to others, i.e., in other situations not before the Court. A writer and
scholar in Constitutional Law explains further: The most distinctive feature of the
overbreadth technique is that it marks an exception to some of the usual rules of
constitutional litigation. Ordinarily, a particular litigant claims that a statute is
unconstitutional as applied to him or her; if the litigant prevails, the courts carve away the
unconstitutional aspects of the law by invalidating its improper applications on a case to
case basis. Moreover, challengers to a law are not permitted to raise the rights of third
parties and can only assert their own interests. In overbreadth analysis, those rules give
way; challenges are permitted to raise the rights of third parties; and the court invalidates
the entire statute “on its face,” not merely “as applied for” so that the overbroad law
becomes unenforceable until a properly authorized court construes it more narrowly. The
factor that motivates courts to depart from the normal adjudicatory rules is the concern
with the “chilling;” deterrent effect of the overbroad statute on third parties not
courageous enough to bring suit. The Court assumes that an overbroad law’s “very
existence may cause others not before the court to refrain from constitutionally
protected speech or expression.” An overbreadth ruling is designed to remove that
deterrent effect on the speech of those third parties.
Same; Same; Same; Same; Same; Same; “Void for Vagueness” Doctrine; Related to the
“overbreadth” doctrine is the “void for vagueness doctrine” which holds that “a law is
facially invalid if men of common intelligence must necessarily guess at its meaning and
differ as to its application,” and like overbreadth, it is said that a litigant may challenge a
statute on its face only if it is vague in all its possible applications.—Petitioners likewise
seek a facial review of PP 1017 on the ground of vagueness. This, too, is unwarranted.
Related to the “overbreadth” doctrine is the “void for vagueness doctrine” which holds
that “a law is facially invalid if men of common intelligence must necessarily guess at its
meaning and differ as to its application.” It is subject to the same principles governing
overbreadth doctrine. For one, it is also an analytical tool for testing “on their faces”
statutes in free speech cases. And like overbreadth, it is said that a litigant may challenge
a statute on its face only if it is vague in all its possible applications. Again, petitioners did
not even attempt to show that PP 1017 is vague in all its application. They also failed to
establish that men of common intelligence cannot understand the meaning and
application of PP 1017. Same; Same; Same; Under the calling-out power, the President
may summon the armed forces to aid him in suppressing lawless violence, invasion and
rebellion.—Under the calling-out power, the President may summon the armed forces to
aid him in suppressing lawless violence, invasion and rebellion. This involves ordinary
police action. But every act that goes beyond the President’s calling-out power is
considered illegal or ultra vires. For this reason, a President must be careful in the
134
EXECUTIVE DEPARTMENT
exercise of his powers. He cannot invoke a greater power when he wishes to act under a
lesser power. There lies the wisdom of our Constitution, the greater the power, the
greater are the limitations.
Same; Same; Same; In declaring a state of national emergency, President Arroyo did not
only rely on Section 18, Article VII of the Constitution, a provision calling on the AFP to
prevent or suppress lawless violence, invasion or rebellion but also relied on Section 17,
Article XII, a provision on the State’s extraordinary power to take over privately-owned
public utility and business affected with public interest—indeed, PP 1017 calls for the
exercise of an awesome power.—President Arroyo’s declaration of a “state of rebellion”
was merely an act declaring a status or condition of public moment or interest, a
declaration allowed under Section 4 cited above. Such declaration, in the words of
Sanlakas, is harmless, without legal significance, and deemed not written. In these cases,
PP 1017 is more than that. In declaring a state of national emergency, President Arroyo
did not only rely on Section 18, Article VII of the Constitution, a provision calling on the
AFP to prevent or suppress lawless violence, invasion or rebellion. She also relied on
Section 17, Article XII, a provision on the State’s extraordinary power to take over
privately-owned public utility and business affected with public interest. Indeed, PP 1017
calls for the exercise of an awesome power. Obviously, such Proclamation cannot be
deemed harmless, without legal significance, or not written, as in the case of Sanlakas.
Same; Same; Same; Martial Law; PP 1017 is not a declaration of Martial Law—it is plain
therein that what the President invoked was her calling-out power.—Some of the
petitioners vehemently maintain that PP 1017 is actually a declaration of Martial Law. It is
no so. What defines the character of PP 1017 are its wordings. It is plain therein that what
the President invoked was her calling-out power. The declaration of Martial Law is a
“warn[ing] to citizens that the military power has been called upon by the executive to
assist in the maintenance of law and order, and that, while the emergency lasts, they
must, upon pain of arrest and punishment, not commit any acts which will in any way
render more difficult the restoration of order and the enforcement of law.”
Same; Same; Same; Same; A reading of PP 1017 operative clause shows that it was lifted
from Former President Marcos’ Proclamation No. 1081; We all know that it was PP 1081
which granted President Marcos legislative powers.—A reading of PP 1017 operative
clause shows that it was lifted from Former President Marcos’ Proclamation No. 1081,
which partly reads: NOW, THEREFORE, I, FERDINAND E. MARCOS, President of the
Philippines by virtue of the powers vested upon me by Article VII, Section 10, Paragraph
(2) of the Constitution, do hereby place the entire Philippines as defined in Article 1,
Section 1 of the Constitution under martial law and, in my capacity as their Commander-
in-Chief, do hereby command the Armed Forces of the Philippines, to maintain law and
order throughout the Philippines, prevent or suppress all forms of lawless violence as well
as any act of insurrection or rebellion and to enforce obedience to all the laws and

135
EXECUTIVE DEPARTMENT
decrees, orders and regulations promulgated by me personally or upon my direction. We
all know that it was PP 1081 which granted President Marcos legislative power. Its
enabling clause states: “to enforce obedience to all the laws and decrees, orders and
regulations promulgated by me personally or upon my direction.” Upon the other hand,
the enabling clause of PP 1017 issued by President Arroyo is: to enforce obedience to all
the laws and to all decrees, orders and regulations promulgated by me personally or upon
my direction.”
Same; Same; Same; Presidential Decrees; President Arroyo’s ordinance power is limited
to Executive Orders, Administrative Orders, Proclamations, Memorandum Orders,
Memorandum Circulars, and General or Special Orders—she cannot issue decrees similar
to those issued by Former President Marcos under PP 1081.—The President is granted an
Ordinance Power under Chapter 2, Book III of Executive Order No. 292 (Administrative
Code of 1987). She may issue any of the following: Sec. 2. Executive Orders.—Acts of the
President providing for rules of a general or permanent character in implementation or
execution of constitutional or statutory powers shall be promulgated in executive orders.
Sec. 3. Administrative Orders.—Acts of the President which relate to particular aspect of
governmental operations in pursuance of his duties as administrative head shall be
promulgated in administrative orders. Sec. 4. Proclamations.—Acts of the President fixing
a date or declaring a status or condition of public moment or interest, upon the existence
of which the operation of a specific law or regulation is made to depend, shall be
promulgated in proclamations which shall have the force of an executive order. Sec. 5.
Memorandum Orders.—Acts of the President on matters of administrative detail or of
subordinate or temporary interest which only concern a particular officer or office of the
Government shall be embodied in memorandum orders. Sec. 6. Memorandum
Circulars.—Acts of the President on matters relating to internal administration, which the
President desires to bring to the attention of all or some of the departments, agencies,
bureaus or offices of the Government, for information or compliance, shall be embodied
in memorandum circulars. Sec. 7. General or Special Orders.—Acts and commands of the
President in his capacity as Commander-in-Chief of the Armed Forces of the Philippines
shall be issued as general or special orders. President Arroyo’s ordinance power is limited
to the foregoing issuances. She cannot issue decrees similar to those issued by Former
President Marcos under PP 1081. Presidential Decrees are laws which are of the same
category and binding force as statutes because they were issued by the President in the
exercise of his legislative power during the period of Martial Law under the 1973
Constitution.
Same; Same; Same; Same; PP 1017 is unconstitutional insofar as it grants President
Arroyo the authority to promulgate “decrees.”—This Court rules that the assailed PP 1017
is unconstitutional insofar as it grants President Arroyo the authority to promulgate
“decrees.” Legislative power is peculiarly within the province of the Legislature. Section 1,
Article VI categorically states that “[t]he legislative power shall be vested in the Congress
136
EXECUTIVE DEPARTMENT
of the Philippines which shall consist of a Senate and a House of Representatives.” To be
sure, neither Martial Law nor a state of rebellion nor a state of emergency can justify
President Arroyo’s exercise of legislative power by issuing decrees.
Same; Same; Same; Same; With respect to “laws,” President Arroyo cannot call the
military to enforce or implement certain laws, such as customs laws, laws governing
family and property relations, laws on obligations and contracts and the like—she can
only order the military, under PP 1017, to enforce laws pertinent to its duty to suppress
lawless violence.—As this Court stated earlier, President Arroyo has no authority to enact
decrees. It follows that these decrees are void and, therefore, cannot be enforced. With
respect to “laws,” she cannot call the military to enforce or implement certain laws, such
as customs laws, laws governing family and property relations, laws on obligations and
contracts and the like. She can only order the military, under PP 1017, to enforce laws
pertinent to its duty to suppress lawless violence.
Same; Same; Same; President Arroyo could validly declare the existence of a state of
national emergency even in the absence of a Congressional enactment but the exercise of
emergency powers, such as the taking over of privately owned public utility or business
affected with public interest, is a different matter.—It may be pointed out that the
second paragraph of the above provision refers not only to war but also to “other
national emergency.” If the intention of the Framers of our Constitution was to withhold
from the President the authority to declare a “state of national emergency” pursuant to
Section 18, Article VII (calling-out power) and grant it to Congress (like the declaration of
the existence of a state of war), then the Framers could have provided so. Clearly, they
did not intend that Congress should first authorize the President before he can declare a
“state of national emergency.” The logical conclusion then is that President Arroyo could
validly declare the existence of a state of national emergency even in the absence of a
Congressional enactment. But the exercise of emergency powers, such as the taking over
of privately owned public utility or business affected with public interest, is a different
matter. This requires a delegation from Congress.
Same; Same; Same; Considering that Section 17 of Article XII and Section 23 of Article VI,
previously quoted, relate to national emergencies, they must be read together to
determine the limitation of the exercise of emergency powers.—Courts have often said
that constitutional provisions in pari materia are to be construed together. Otherwise
stated, different clauses, sections, and provisions of a constitution which relate to the
same subject matter will be construed together and considered in the light of each other.
Considering that Section 17 of Article XII and Section 23 of Article VI, previously quoted,
relate to national emergencies, they must be read together to determine the limitation of
the exercise of emergency powers.
Same; Same; Same; Emergency Powers; Requisites for Valid Delegation; Generally,
Congress is the repository of emergency powers.—Generally, Congress is the repository

137
EXECUTIVE DEPARTMENT
of emergency powers. This is evident in the tenor of Section 23 (2), Article VI authorizing
it to delegate such powers to the President. Certainly, a body cannot delegate a power
not reposed upon it. However, knowing that during grave emergencies, it may not be
possible or practicable for Congress to meet and exercise its powers, the Framers of our
Constitution deemed it wise to allow Congress to grant emergency powers to the
President, subject to certain conditions, thus: (1) There must be a war or other
emergency. (2) The delegation must be for a limited period only. (3) The delegation must
be subject to such restrictions as the Congress may prescribe. (4) The emergency powers
must be exercised to carry out a national policy declared by Congress.
Same; Same; Same; Same; Section 17, Article XII must be understood as an aspect of the
emergency powers clause, and the taking over of private business affected with public
interest is just another facet of the emergency powers generally reposed upon
Congress—Section 17 refers to Congress, not the President.—Section 17, Article XII must
be understood as an aspect of the emergency powers clause. The taking over of private
business affected with public interest is just another facet of the emergency powers
generally reposed upon Congress. Thus, when Section 17 states that the “the State may,
during the emergency and under reasonable terms prescribed by it, temporarily take over
or direct the operation of any privately owned public utility or business affected with
public interest,” it refers to Congress, not the President. Now, whether or not the
President may exercise such power is dependent on whether Congress may delegate it to
him pursuant to a law prescribing the reasonable terms thereof.
Same; Same; Same; Same; Words and Phrases; Emergency, as a generic term, connotes
the existence of conditions suddenly intensifying the degree of existing danger to life or
well-being beyond that which is accepted as normal—implicit in this definitions are the
elements of intensity, variety, and perception; Emergencies, as perceived by legislature or
executive in the United States since 1933, have been occasioned by a wide range of
situations, classifiable under three (3) principal heads: a) economic, b) natural disaster,
and c) national security; “Emergency,” as contemplated in our Constitution, may include
rebellion, economic crisis, pestilence or epidemic, typhoon, flood, or other similar
catastrophe of nationwide proportions or effect.—Petitioner Cacho-Olivares, et al.
contends that the term “emergency” under Section 17, Article XII refers to “tsunami,”
“typhoon,” “hurricane” and “similar occurrences.” This is a limited view of “emergency.”
Emergency, as a generic term, connotes the existence of conditions suddenly intensifying
the degree of existing danger to life or well-being beyond that which is accepted as
normal. Implicit in this definitions are the elements of intensity, variety, and perception.
Emergencies, as perceived by legislature or executive in the United States since 1933,
have been occasioned by a wide range of situations, classifiable under three (3) principal
heads: a) economic, b) natural disaster, and c) national security. “Emergency,” as
contemplated in our Constitution, is of the same breadth. It may include rebellion,

138
EXECUTIVE DEPARTMENT
economic crisis, pestilence or epidemic, typhoon, flood, or other similar catastrophe of
nationwide proportions or effect.
Same; Same; Same; Same; While the President alone can declare a state of national
emergency, however, without legislation, he has no power to take over privately-owned
public utility or business affected with public interest.—Following our interpretation of
Section 17, Article XII, invoked by President Arroyo in issuing PP 1017, this Court rules
that such Proclamation does not authorize her during the emergency to temporarily take
over or direct the operation of any privately owned public utility or business affected with
public interest without authority from Congress. Let it be emphasized that while the
President alone can declare a state of national emergency, however, without legislation,
he has no power to take over privately-owned public utility or business affected with
public interest. The President cannot decide whether exceptional circumstances exist
warranting the take over of privately-owned public utility or business affected with public
interest. Nor can he determine when such exceptional circumstances have ceased.
Likewise, without legislation, the President has no power to point out the types of
businesses affected with public interest that should be taken over. In short, the President
has no absolute authority to exercise all the powers of the State under Section 17, Article
VII in the absence of an emergency powers act passed by Congress.
Same; Same; Same; Same; One of the misfortunes of an emergency, particularly, that
which pertains to security, is that military necessity and the guaranteed rights of the
individual are often not compatible.—One of the misfortunes of an emergency,
particularly, that which pertains to security, is that military necessity and the guaranteed
rights of the individual are often not compatible. Our history reveals that in the crucible
of conflict, many rights are curtailed and trampled upon. Here, the right against
unreasonable search and seizure; the right against warrantless arrest; and the freedom of
speech, of expression, of the press, and of assembly under the Bill of Rights suffered the
greatest blow.
Same; Same; Same; Judicial Review; Courts are not at liberty to declare statutes invalid
although they may be abused and misabused and may afford an opportunity for abuse in
the manner of application—the validity of a statute or ordinance is to be determined
from its general purpose and its efficiency to accomplish the end desired, not from its
effects in a particular case.—Settled is the rule that courts are not at liberty to declare
statutes invalid although they may be abused and misabused and may afford an
opportunity for abuse in the manner of application. The validity of a statute or ordinance
is to be determined from its general purpose and its efficiency to accomplish the end
desired, not from its effects in a particular case. PP 1017 is merely an invocation of the
President’s calling-out power. Its general purpose is to command the AFP to suppress all
forms of lawless violence, invasion or rebellion. It had accomplished the end desired
which prompted President Arroyo to issue PP 1021. But there is nothing in PP 1017

139
EXECUTIVE DEPARTMENT
allowing the police, expressly or impliedly, to conduct illegal arrest, search or violate the
citizens’ constitutional rights. Now, may this Court adjudge a law or ordinance
unconstitutional on the ground that its implementor committed illegal acts? The answer
is no. The criterion by which the validity of the statute or ordinance is to be measured is
the essential basis for the exercise of power, and not a mere incidental result arising from
its exertion. This is logical. Just imagine the absurdity of situations when laws maybe
declared unconstitutional just because the officers implementing them have acted
arbitrarily. If this were so, judging from the blunders committed by policemen in the cases
passed upon by the Court, majority of the provisions of the Revised Penal Code would
have been declared unconstitutional a long time ago.
Same; Same; Same; General orders are “acts and commands of the President in his
capacity as Commander-in-Chief of the Armed Forces of the Philippines”—they are
internal rules issued by the executive officer to his subordinates precisely for the proper
and efficient administration of law.—President Arroyo issued G.O. No. 5 to carry into
effect the provisions of PP 1017. General orders are “acts and commands of the President
in his capacity as Commander-in-Chief of the Armed Forces of the Philippines.” They are
internal rules issued by the executive officer to his subordinates precisely for the proper
and efficient administration of law. Such rules and regulations create no relation except
between the official who issues them and the official who receives them. They are based
on and are the product of, a relationship in which power is their source, and obedience,
their object. For these reasons, one requirement for these rules to be valid is that they
must be reasonable, not arbitrary or capricious. G.O. No. 5 mandates the AFP and the
PNP to immediately carry out the “necessary and appropriate actions and measures to
suppress and prevent acts of terrorism and lawless violence.”
Same; Same; Same; Searches and Seizures; The plain import of the language of the
Constitution is that searches, seizures and arrests are normally unreasonable unless
authorized by a validly issued search warrant or warrant of arrest.—The Constitution
provides that “the right of the people to be secured in their persons, houses, papers and
effects against unreasonable search and seizure of whatever nature and for any purpose
shall be inviolable, and no search warrant or warrant of arrest shall issue except upon
probable cause to be determined personally by the judge after examination under oath or
affirmation of the complainant and the witnesses he may produce, and particularly
describing the place to be searched and the persons or things to be seized.”The plain
import of the language of the Constitution is that searches, seizures and arrests are
normally unreasonable unless authorized by a validly issued search warrant or warrant of
arrest. Thus, the fundamental protection given by this provision is that between person
and police must stand the protective authority of a magistrate clothed with power to
issue or refuse to issue search warrants or warrants of arrest.

140
EXECUTIVE DEPARTMENT
Same; Same; Same; Right of Assembly; Words and Phrases; “Assembly” means a right on
the part of the citizens to meet peaceably for consultation in respect to public affairs—it
is a necessary consequence of our republican institution and complements the right of
speech; The right of the people to peaceably assemble is not to be limited, much less
denied, except on a showing of a clear and present danger of a substantive evil that
Congress has a right to prevent.—”Assembly” means a right on the part of the citizens to
meet peaceably for consultation in respect to public affairs. It is a necessary consequence
of our republican institution and complements the right of speech. As in the case of
freedom of expression, this right is not to be limited, much less denied, except on a
showing of a clear and present danger of a substantive evil that Congress has a right to
prevent. In other words, like other rights embraced in the freedom of expression, the
right to assemble is not subject to previous restraint or censorship. It may not be
conditioned upon the prior issuance of a permit or authorization from the government
authorities except, of course, if the assembly is intended to be held in a public place, a
permit for the use of such place, and not for the assembly itself, may be validly required.
Same; Same; Same; Same; Peaceable assembly cannot be made a crime.—The ringing
truth here is that petitioner David, et al. were arrested while they were exercising their
right to peaceful assembly. They were not committing any crime, neither was there a
showing of a clear and present danger that warranted the limitation of that right. As can
be gleaned from circumstances, the charges of inciting to sedition and violation of BP 880
were mere afterthought. Even the Solicitor General, during the oral argument, failed to
justify the arresting officers’ conduct. In De Jonge v. Oregon, it was held that peaceable
assembly cannot be made a crime, thus: Peaceable assembly for lawful discussion cannot
be made a crime. The holding of meetings for peaceable political action cannot be
proscribed. Those who assist in the conduct of such meetings cannot be branded as
criminals on that score. The question, if the rights of free speech and peaceful assembly
are not to be preserved, is not as to the auspices under which the meeting was held but
as to its purpose; not as to the relations of the speakers, but whether their utterances
transcend the bounds of the freedom of speech which the Constitution protects. If the
persons assembling have committed crimes elsewhere, if they have formed or are
engaged in a conspiracy against the public peace and order, they may be prosecuted for
their conspiracy or other violations of valid laws. But it is a different matter when the
State, instead of prosecuting them for such offenses, seizes upon mere participation in a
peaceable assembly and a lawful public discussion as the basis for a criminal charge.
Same; Same; Same; Same; The wholesale cancellation of all permits to rally is a blatant
disregard of the principle that “freedom of assembly is not to be limited, much less
denied, except on a showing of a clear and present danger of a substantive evil that the
State has a right to prevent”—tolerance is the rule and limitation is the exception.—On
the basis of the above principles, the Court likewise considers the dispersal and arrest of
the members of KMU, et al. (G.R. No. 171483) unwarranted. Apparently, their dispersal
141
EXECUTIVE DEPARTMENT
was done merely on the basis of Malacañang’s directive canceling all permits previously
issued by local government units. This is arbitrary. The wholesale cancellation of all
permits to rally is a blatant disregard of the principle that “freedom of assembly is not to
be limited, much less denied, except on a showing of a clear and present danger of a
substantive evil that the State has a right to prevent.” Tolerance is the rule and limitation
is the exception. Only upon a showing that an assembly presents a clear and present
danger that the State may deny the citizens’ right to exercise it. Indeed, respondents
failed to show or convince the Court that the rallyists committed acts amounting to
lawless violence, invasion or rebellion. With the blanket revocation of permits, the
distinction between protected and unprotected assemblies was eliminated.
Same; Same; Same; Same; Under BP 880, the authority to regulate assemblies and rallies
is lodged with the local government units; When a person’s right is restricted by
government action, it behooves a democratic government to see to it that the restriction
is fair, reasonable, and according to procedure.—Under BP 880, the authority to regulate
assemblies and rallies is lodged with the local government units. They have the power to
issue permits and to revoke such permits after due notice and hearing on the
determination of the presence of clear and present danger. Here, petitioners were not
even notified and heard on the revocation of their permits. The first time they learned of
it was at the time of the dispersal. Such absence of notice is a fatal defect. When a
person’s right is restricted by government action, it behooves a democratic government
to see to it that the restriction is fair, reasonable, and according to procedure.
Same; Same; Same; Searches and Seizures; The warrantless search of the Daily Tribune’s
offices is illegal.—G.R. No. 171409, (Cacho-Olivares, et al.) presents another facet of
freedom of speech i.e., the freedom of the press. Petitioners’ narration of facts, which the
Solicitor General failed to refute, established the following: first, the Daily Tribune’s
offices were searched without warrant; second, the police operatives seized several
materials for publication; third, the search was conducted at about 1:00 o’ clock in the
morning of February 25, 2006; fourth, the search was conducted in the absence of any
official of the Daily Tribune except the security guard of the building; and fifth, policemen
stationed themselves at the vicinity of the Daily Tribune offices. x x x The search is illegal.
Rule 126 of The Revised Rules on Criminal Procedure lays down the steps in the conduct
of search and seizure. Section 4 requires that a search warrant be issued upon probable
cause in connection with one specific offence to be determined personally by the judge
after examination under oath or affirmation of the complainant and the witnesses he may
produce. Section 8 mandates that the search of a house, room, or any other premise be
made in the presence of the lawful occupant thereof or any member of his family or in
the absence of the latter, in the presence of two (2) witnesses of sufficient age and
discretion residing in the same locality. And Section 9 states that the warrant must direct
that it be served in the daytime, unless the property is on the person or in the place

142
EXECUTIVE DEPARTMENT
ordered to be searched, in which case a direction may be inserted that it be served at any
time of the day or night. All these rules were violated by the CIDG operatives.
Same; Same; Same; Same; Freedom of the Press; The search of the Daily Tribune’s offices
also violated freedom of the press; The best gauge of a free and democratic society rests
in the degree of freedom enjoyed by its media.—The search violated petitioners’ freedom
of the press. The best gauge of a free and democratic society rests in the degree of
freedom enjoyed by its media. In the Burgos v. Chief of Staff this Court held that—As
heretofore stated, the premises searched were the business and printing offices of the
“Metropolitan Mail” and the “We Forum” newspapers. As a consequence of the search
and seizure, these premises were padlocked and sealed, with the further result that the
printing and publication of said newspapers were discontinued. Such closure is in the
nature of previous restraint or censorship abhorrent to the freedom of the press
guaranteed under the fundamental law, and constitutes a virtual denial of petitioners’
freedom to express themselves in print. This state of being is patently anathematic to a
democratic framework where a free, alert and even militant press is essential for the
political enlightenment and growth of the citizenry.
Same; Same; Same; Same; Same; The search and seizure of materials for publication, the
stationing of policemen in the vicinity of the The Daily Tribune offices, and the arrogant
warning of government officials to media, are plain censorship—it is that officious
functionary of the repressive government who tells the citizen that he may speak only if
allowed to do so, and no more and no less than what he is permitted to say on pain of
punishment should he be so rash as to disobey; The Supreme Court cannot tolerate the
blatant disregard of a constitutional right even if it involves the most defiant of our
citizens—freedom to comment on public affairs is essential to the vitality of a
representative democracy.—While admittedly, the Daily Tribune was not padlocked and
sealed like the “Metropolitan Mail” and “We Forum” newspapers in the above case, yet it
cannot be denied that the CIDG operatives exceeded their enforcement duties. The
search and seizure of materials for publication, the stationing of policemen in the vicinity
of the The Daily Tribune offices, and the arrogant warning of government officials to
media, are plain censorship. It is that officious functionary of the repressive government
who tells the citizen that he may speak only if allowed to do so, and no more and no less
than what he is permitted to say on pain of punishment should he be so rash as to
disobey.Undoubtedly, the The Daily Tribune was subjected to these arbitrary intrusions
because of its anti-government sentiments. This Court cannot tolerate the blatant
disregard of a constitutional right even if it involves the most defiant of our citizens.
Freedom to comment on public affairs is essential to the vitality of a representative
democracy. It is the duty of the courts to be watchful for the constitutional rights of the
citizen, and against any stealthy encroachments thereon. The motto should always be
obsta principiis.

143
EXECUTIVE DEPARTMENT
Same; Same; Same; PP 1017 is constitutional insofar as it constitutes a call by the
President for the AFP to prevent or suppress lawless violence but PP 1017’s extraneous
provisions giving the President express or implied power (1) to issue decrees, (2) to direct
the AFP to enforce obedience to all laws even those not related to lawless violence as
well as decrees promulgated by the President, and (3) to impose standards on media or
any form of prior restraint on the press, are ultra vires and unconstitutional.—The Court
finds and so holds that PP 1017 is constitutional insofar as it constitutes a call by the
President for the AFP to prevent or suppress lawless violence. The proclamation is
sustained by Section 18, Article VII of the Constitution and the relevant jurisprudence
discussed earlier. However, PP 1017’s extraneous provisions giving the President express
or implied power (1) to issue decrees; (2) to direct the AFP to enforce obedience to all
laws even those not related to lawless violence as well as decrees promulgated by the
President; and (3) to impose standards on media or any form of prior restraint on the
press, are ultra vires and unconstitutional. The Court also rules that under Section 17,
Article XII of the Constitution, the President, in the absence of a legislation, cannot take
over privately-owned public utility and private business affected with public interest.
Same; Same; Same; Words and Phrases; The words “acts of terrorism” found in G.O. No. 5
have not been legally defined and made punishable by Congress and should thus be
deemed deleted from the said G.O.—The Court finds G.O. No. 5 valid. It is an Order issued
by the President—acting as Commander-in-Chief—addressed to subalterns in the AFP to
carry out the provisions of PP 1017. Significantly, it also provides a valid standard—that
the military and the police should take only the “necessary and appropriate actions and
measures to suppress and prevent acts of lawless violence.” But the words “acts of
terrorism” found in G.O. No. 5 have not been legally defined and made punishable by
Congress and should thus be deemed deleted from the said G.O. While “terrorism” has
been denounced generally in media, no law has been enacted to guide the military, and
eventually the courts, to determine the limits of the AFP’s authority in carrying out this
portion of G.O. No. 5.
Same; Same; Same; It is well to remember that military power is a means to an end and
substantive civil rights are ends in themselves; How to give the military the power it
needs to protect the Republic without unnecessarily trampling individual rights is one of
the eternal balancing tasks of a democratic state; Two vital principles of
constitutionalism: the maintenance of legal limits to arbitrary power, and, political
responsibility of the government to the governed.—It is well to remember that military
power is a means to an end and substantive civil rights are ends in themselves. How to
give the military the power it needs to protect the Republic without unnecessarily
trampling individual rights is one of the eternal balancing tasks of a democratic state.
During emergency, governmental action may vary in breadth and intensity from normal
times, yet they should not be arbitrary as to unduly restrain our people’s liberty. Perhaps,
the vital lesson that we must learn from the theorists who studied the various competing
144
EXECUTIVE DEPARTMENT
political philosophies is that, it is possible to grant government the authority to cope with
crises without surrendering the two vital principles of constitutionalism: the maintenance
of legal limits to arbitrary power, and political responsibility of the government to the
governed.
Randolf David vs President Gloria Macapagal-Arroyo

489 SCRA 160 – Political Law – The Executive Branch – Presidential Proclamation 1017 –
Take Care Clause – Take Over Power – Calling Out Power
Bill of Rights – Freedom of Speech – Overbreadth
In February 2006, due to the escape of some Magdalo members and the discovery of a plan
(Oplan Hackle I) to assassinate the president, then president Gloria Macapagal-Arroyo
(GMA) issued Presidential Proclamation 1017 (PP1017) and is to be implemented by
General Order No. 5 (GO 5). The said law was aimed to suppress lawlessness and the
connivance of extremists to bring down the government.
Pursuant to such PP, GMA cancelled all plans to celebrate EDSA I and at the same time
revoked all permits issued for rallies and other public organization/meeting.
Notwithstanding the cancellation of their rally permit, Kilusang Mayo Uno (KMU) head
Randolf David proceeded to rally which led to his arrest.
Later that day, the Daily Tribune, which Cacho-Olivares is the editor, was raided by the CIDG
and they seized and confiscated anti-GMA articles and write ups. Later still, another known
anti-GMA news agency (Malaya) was raided and seized. On the same day, Beltran of
Anakpawis, was also arrested. His arrest was however grounded on a warrant of arrest
issued way back in 1985 for his actions against Marcos. His supporters cannot visit him in
jail because of the current imposition of PP 1017 and GO 5.
In March, GMA issued PP 1021 which declared that the state of national emergency ceased
to exist. David and some opposition Congressmen averred that PP1017 is unconstitutional
for it has no factual basis and it cannot be validly declared by the president for such power
is reposed in Congress. Also such declaration is actually a declaration of martial law.
Olivares-Cacho also averred that the emergency contemplated in the Constitution are
those of natural calamities and that such is an overbreadth. Petitioners claim that PP 1017
is an overbreadth because it encroaches upon protected and unprotected rights. The Sol-
Gen argued that the issue has become moot and academic by reason of the lifting of PP
1017 by virtue of the declaration of PP 1021. The Sol-Gen averred that PP 1017 is within
the president’s calling out power, take care power and take over power.
ISSUE: Whether or not PP 1017 and GO 5 is constitutional.
HELD: PP 1017 and its implementing GO are partly constitutional and partly
unconstitutional.
The issue cannot be considered as moot and academic by reason of the lifting of the
questioned PP. It is still in fact operative because there are parties still affected due to the
alleged violation of the said PP. Hence, the SC can take cognition of the case at bar. The SC
ruled that PP 1017 is constitutional in part and at the same time some provisions of which
are unconstitutional. The SC ruled in the following way;
145
EXECUTIVE DEPARTMENT
Resolution by the SC on the Factual Basis of its declaration
The petitioners were not able to prove that GMA has no factual basis in issuing PP 1017
and GO 5. A reading of the Solicitor General’s Consolidated Comment and Memorandum
shows a detailed narration of the events leading to the issuance of PP 1017, with supporting
reports forming part of the records. Mentioned are the escape of the Magdalo Group, their
audacious threat of the Magdalo D-Day, the defections in the military, particularly in the
Philippine Marines, and the reproving statements from the communist leaders. There was
also the Minutes of the Intelligence Report and Security Group of the Philippine Army
showing the growing alliance between the NPA and the military. Petitioners presented
nothing to refute such events. Thus, absent any contrary allegations, the Court is convinced
that the President was justified in issuing PP 1017 calling for military aid. Indeed, judging
the seriousness of the incidents, GMA was not expected to simply fold her arms and do
nothing to prevent or suppress what she believed was lawless violence, invasion or
rebellion. However, the exercise of such power or duty must not stifle liberty.
Resolution by the SC on the Overbreadth Theory
First and foremost, the overbreadth doctrine is an analytical tool developed for testing ‘on
their faces’ statutes in free speech cases. The 7 consolidated cases at bar are not primarily
‘freedom of speech’ cases. Also, a plain reading of PP 1017 shows that it is not primarily
directed to speech or even speech-related conduct. It is actually a call upon the AFP to
prevent or suppress all forms of lawless violence. Moreover, the overbreadth doctrine is
not intended for testing the validity of a law that ‘reflects legitimate state interest in
maintaining comprehensive control over harmful, constitutionally unprotected conduct.’
Undoubtedly, lawless violence, insurrection and rebellion are considered ‘harmful’ and
‘constitutionally unprotected conduct.’ Thus, claims of facial overbreadth are entertained
in cases involving statutes which, by their terms, seek to regulate only ‘spoken words’ and
again, that ‘overbreadth claims, if entertained at all, have been curtailed when invoked
against ordinary criminal laws that are sought to be applied to protected conduct.’ Here,
the incontrovertible fact remains that PP 1017 pertains to a spectrum of conduct, not free
speech, which is manifestly subject to state regulation.
Resolution by the SC on the Calling Out Power Doctrine
On the basis of Sec 17, Art 7 of the Constitution, GMA declared PP 1017. The SC considered
the President’s ‘calling-out’ power as a discretionary power solely vested in his wisdom, it
stressed that ‘this does not prevent an examination of whether such power was exercised
within permissible constitutional limits or whether it was exercised in a manner
constituting grave abuse of discretion. The SC ruled that GMA has validly declared PP 1017
for the Constitution grants the President, as Commander-in-Chief, a ‘sequence’ of
graduated powers. From the most to the least benign, these are: the calling-out power,
the power to suspend the privilege of the writ of habeas corpus, and the power to declare
Martial Law. The only criterion for the exercise of the calling-out power is that ‘whenever
it becomes necessary,’ the President may call the armed forces ‘to prevent or suppress
lawless violence, invasion or rebellion.’ And such criterion has been met.
Resolution by the SC on the Take Care Doctrine
146
EXECUTIVE DEPARTMENT
Pursuant to the 2nd sentence of Sec 17, Art 7 of the Constitution (He shall ensure that the
laws be faithfully executed.) the president declared PP 1017. David et al averred that PP
1017 however violated Sec 1, Art 6 of the Constitution for it arrogated legislative power to
the President. Such power is vested in Congress. They assail the clause ‘to enforce
obedience to all the laws and to all decrees, orders and regulations promulgated by me
personally or upon my direction.’ The SC noted that such provision is similar to the power
that granted former President Marcos legislative powers (as provided in PP 1081). The SC
ruled that the assailed PP 1017 is unconstitutional insofar as it grants GMA the authority to
promulgate ‘decrees.’ Legislative power is peculiarly within the province of the
Legislature. Sec 1, Article 6 categorically states that ‘[t]he legislative power shall be vested
in the Congress of the Philippines which shall consist of a Senate and a House of
Representatives.’ To be sure, neither Martial Law nor a state of rebellion nor a state of
emergency can justify GMA’[s exercise of legislative power by issuing decrees. The
president can only “take care” of the carrying out of laws but cannot create or enact laws.
Resolution by the SC on the Take Over Power Doctrine
The president cannot validly order the taking over of private corporations or institutions
such as the Daily Tribune without any authority from Congress. On the other hand, the
word emergency contemplated in the constitution is not limited to natural calamities but
rather it also includes rebellion. The SC made a distinction; the president can declare the
state of national emergency but her exercise of emergency powers does not come
automatically after it for such exercise needs authority from Congress. The authority from
Congress must be based on the following:
(1) There must be a war or other emergency.
(2) The delegation must be for a limited period only.
(3) The delegation must be subject to such restrictions as the Congress may prescribe.
(4) The emergency powers must be exercised to carry out a national policy declared by
Congress.
Resolution by the SC on the Issue that PP 1017 is a Martial Law Declaration
The SC ruled that PP 1017 is not a Martial Law declaration and is not tantamount to it. It is
a valid exercise of the calling out power of the president by the president.

33. AQUINO v. MILITARY COMMISSION NO. 2

Ponente/Other Opinion: ANTONIO


Dispositive Portion: IN VIEW OF ALL THE FOREGOING, judgment is hereby rendered
dismissing the petitions for prohibition with preliminary injunction and setting aside the
te mporary restraining order issued on April 8, 1975, with costs against petitioner.
Prohibition; Withdrawal of petition for prohibition will be denied where it raises issues of
paramount public interest.—The Justices (seven against three) who voted to deny the
withdrawal are of the opinion that since all matters in issue in this case have already been
147
EXECUTIVE DEPARTMENT
submitted for resolution, and they are of paramount public interest, it is imperative that
the questions raised by petitioner on the constitutionality and legality of proceedings
against civilians in the military commissions, pursuant to pertinent General Orders,
Presidential Decrees and Letters of Instruction, should be definitely resolved.
Military Law; A military commission has jurisdiction to hear cases filed against civilians
during period of martial law.—We hold that the respondent Military Commi ssion No. 2
has been lawfully constituted and validly vested with jurisdiction to hear the cases against
civilians, including the petitioner. The Court has previously declared that the proclamation
of Martial Law (Proclamation No. 1081) on September 21, 1972, by the President of the
Philippines is valid and constitutional and that its continuance is justified by the danger
posed to the public safety . To preserve the safety of the nation in times of national peril,
the President of the Philippines necessarily possesses broad authority compatible with
the imperative requirements of the emergency . On the basis of this, he has authorized in
General Order No. 8 (September 27, 1972) the Chief of Staff, AFP, to create military
tribunals to try and decide cases “of military personnel and such other cases as may be
referred to them.” In General Order No. 12 (September 30, 1972), the military tribunals
were vested with jurisdiction “exclusive of the civil courts”, among others, over crimes
against public order, violations of the Anti-Subversion Act, violations of the laws on
firearms, and other crimes which, in the face of the emergency, are directly related to the
quelling of the rebellion and preservation of the safety and security of the Republic.
Constitutional law; Due process; Military law; During martial law the administrator should
have ample means to quell rebellion and restore civil order.—Petitioner nevertheless
insists that he being a civilian, his trial by a military commission deprives him of the right
to due process, x x x This argument ignores the reality of the rebellion and the existence
of martial law. It is, of course, essential that in a martial law situation, the martial law
administrator must have ample and sufficient means to quell the rebellion and restore
civil order. Prompt and e ffective trial and punishment of offenders have been considered
as necessary in a state of martial law, as a mere power of detention may be wholly
inadequate for the exigency , x x x Indeed, it has been said that in time of overpowering
necessity , “Public danger warrants the substitution of executive process for judicial
process.” According to Schwartz, “The immunity of civilians from military jurisdiction
must, however, give way in areas governed by martial law. When it is absolutely
imperative for public safety , legal processes can be superseded and military tribunals
authorized to exercise the jurisdiction normally vested in courts.”
Military Law; Constitutional law; Continued existence of military tribunals within
intendment of Sec. 3, par. 2 of Transitory Provisions of Constitution.—In any case, We
cannot close Our ey es to the fact that the continued existence of these military tribunals
and the exercise by them of jurisdiction over civilians during the period of martial law are
within the contemplation and intendment of Section 3, paragraph 2 of Article XVII of the

148
EXECUTIVE DEPARTMENT
Constitution. These are tribunals of special and restricted jurisdiction created under the
stress of an emergency and national security. This is the only logical way to construe said
Sec. 3, par. 2 of Art. XVII of the Constitution, in relation to Gen. Orders Nos. 8, 12 and 39,
in the context of contemporary history and the circumstances attendant to the framing of
the new Charter.
Same; Same; Responsibility for acts done during martial law must be taken by authorities
administering it.—When it has been established that martial law is in force, the
responsibility for all acts done thereunder must be taken by the authorities administering
it. It is a serious responsibility which merits the cooperation of all in the collective desire
for the re storation of civil order.
Same; Same; Due process; Due process is not a guarantee of any particular form of
tribunal in criminal cases.—Neither are We impressed with petitioner’s argument that
only thru a judicial proceeding before the regular courts can his right to due process be
preserved. The guarantee of due process is not a guarantee of any particular form of
tribunal in criminal cases. A military tribunal of competent jurisdiction, accusation in due
form, notice and opportunity to defend and trial before an impartial tribunal, adequately
meet the due process re quirement. Due process of law does not necessarily mean a
judicial proceeding in the regular courts, x x x Here, the procedure before the Military
Commission, as prescribed in Presidential Decree No. 39, assures observance of the
fundamental requisites of procedural due process, due notice, an essentially fair and
impartial trial and reasonable opportunity for the preparation of the defense.
Same; Same; Same; Prejudice on part of President against an accused before military
tribunal cannot be presumed.—It is, however, asserted that petitioner’s trial before the
Military commission will not be fair and impartial, as the President had already prejudged
petitioner’s cases and the military tribunal is a mere creation of the President, and
“subject to his control and direction.” We cannot, however, indulge in unjustified
assumption. Prejudice cannot be presumed, especially if weighed against the great
confidence and trust reposed by the people upon the President and the latter’s legal
obligation under his oath “t o do justice to every man”, x x x This assumption mu st be
made because innocence, not wrongdoing, is presumed. The presumption of innocence
includes that of good faith, fair dealing and honesty . This presumption is accorded to
every official of the land in the performance of his public duty . There is no reason why
such presumption cannot be accorded to the President of the Philippines upon whom the
people during this period has confided powers and responsibilities which are of a very
high and delicate nature.
Same; Same; Same; Administrative Order No. 355 creating a committee to investigate
charges against Aquino valid.—It was precisely because of petitioner’s complaint that he
was denied the opportunity to be heard in the preliminary investigation of his charges

149
EXECUTIVE DEPARTMENT
that the President created a Special Committee to reinvestigate the charges filed against
him in the military commission.
Same; Same; Same; Preliminary Investigation; Accused has right to cross-examine
witnesses in preliminary investigation under Administrative Order 355.—The infirmity of
this contention is apparent from the fact that the Committee “shall have all the powers
vested by law in officials authorized to conduct preliminary investigations.” We have held
as implicit in the power of the Investigating Fiscal or Judge in the discharge of his grave
responsibility of ascertaining the existence of probable cause, is his right to cross-examine
the witnesses since “cross-examination whether by the judge or by the prosecution
supplies the gap by permitting an instant contrast of falsehoods and opposing half-truths,
mixed with elements of truth, from which the examining judge or officer is better able to
form a correct sy nthesis of the real facts.”
Same; Same; Same; Same; Presidential Decree No. 39, as amended by Presidential Decree
No. 77 grants accused right to counsel.—Contrary to petitioner’s contention, Section 1(b)
of Pres. Decree No. 77 specifically grants him the right to counsel, and Pres. Decree No.
328 amended Pres. Decree No. 39, precisely to secure the substantial rights of the
accused by granting him the right to counsel during preliminary investigation.
Same; Same; Same; Same; Preliminary investigation not essential part of due process.—
The Constitution does not require the holding of preliminary investigations. The right
exists only , if and when created by statute. It is not an essential part of due process of
law. The absence thereof does not impair the validity of a criminal information or affect
the jurisdiction of the court over the case. As a creation of the statute it can, therefore, be
a modified or amended by law.
Same; Same; Same; Same; Right to cross-examine witnesses during preliminary
investigation depends upon discretion of investigating officer.—There is no curtailment of
the constitutional right of an accused person when he is not given the opportunity to
cross-examine the witnesses presented against him in the preliminary investigation
before his arrest, this being a matter that depends on the sound discretion of the Judge
or investigating officer concerned.
Same; Same; Same; Same; Reason why Pres. Decree No. 77 eliminated provision for cross-
examination is justified by necessity of martial law.—It was found necessary in Pres.
Decree No. 77 to simplify the procedure of preliminary investigation to conform to its
summary character, by eliminating the cross-examination by the contending parties of
their respective witnesses which in the past had made the proceeding the occasion for
the full and exhaustive display of parties’ evidence. The procedure prescribed in the
aforecited decrees appears justified by the necessity of disposing cases during martial
law, especially those affecting national security, at the earliest date.
Same; Same; Same; Same; Pres. Decree No. 77 is part of Constitution.—It may be relevant
to note that recently in Littong v. Castillo, this Court denied for lack of merit a petition
150
EXECUTIVE DEPARTMENT
challenging the validity of Pres. Decree No. 77 issued on December 6, 1972 on the ground
that aforesaid decree now “forms part of the law of the land.”
Criminal procedure; Discovery procedures; Taking of testimony or deposition under Pres.
Decree No 328 being similar to Sec. 7, Rule 119 of Rules of Court is proper and valid.—The
provisions of Pres. Decree No. 328, dated October 31, 1973, for the conditional
examination of prosecution witnesses before trial, is similar to the provisions of Section 7
of Rule 119 of the R evised Rules of Court, x x x We, therefore, hold that the taking of the
testimony or deposition was proper and valid.
Same; Same; In the taking of deposition only the order of the court authorizing the taking
of deposition must be served on accused; the petition to allow taking of deposition need
not be served on accused.—The thrust of Elago is that the order of the court authorizing
the taking of the deposition of the witnesses of the prosecution and fixing the date and
time thereof is the one that must be served on the accused within a reasonable time prior
to that fixed for the examination of the witnesses so that the accused may be present and
cross-examine the witnesses.
Constitutional law; Bill of Rights; Criminal procedure; Discovery procedures; Accused may
waive his right to be present in the proceeding for perpetuation of testimony even if
charged with a capital offense.—Considering the aforecited provisions of the
Constitution, that “after arraignment, trial may proceed notwithstanding the absence of
the accused provided that he has been duly notified and his failure to appear is
unjustified.”, and the absence of any law specifically requiring his presence at all stages of
his trial, there appears, therefore, no logical reason why petitioner, although he is
charged with a capital offense, should be precluded from waiving his right to be present
in the proceedings for the perpetuation of testimony, since this right, like the others
aforestated, was conferred upon him for his protection and benefit. It is also important to
note that under Section 7 of Rule 119 of the Revised Rules of Court (Deposition of witness
for the prosecution) the “Failure or refusal on the part of the defendant to attend the
examination or the taking of the deposition after notice hereinbefore provided, shall be
considered a waiver . . .” Similarly , Presidential Decree No. 328 expressly provides that “.
. . the failure or refusal to attend the examination or the taking of the deposition shall be
considered a waiver.”
Same; Same Same; Same; Exception to accused’s right to waive his presence at all stages
of a criminal trial is where his presence at all stages of a criminal trial is where his
presence is necessary for identification.—Since only 6 Justices (Fernando, Teehankee,
Barredo, Antonio, Muñoz Palma and Aquino) are of the view that petitioner may waive
his right to be present at all stages of the proceedings while 5 Justices (Castro, Makasiar,
Esguerra, Concepcion, Jr. and Martin) are in agreement that he may so waive such right,
except when he is to be identified, the result is that the respondent Commission’s Order
requiring his presence at all times during the proceedings before it should be modified, in

151
EXECUTIVE DEPARTMENT
the sense that petitioner’s presence shall be required only in the instance just indicated.
The ruling in People vs. Avanceña is thus pro tan to modified.
Military Law; Constitutional Law; There must be an official proclamation, by the President
of cessation of public emergency that gave rise to declaration of martial law.—In the
absence of any official proclamation by the President of the cessation of the public
emergency , We have no basis to conclude that the rebellion and communist su bversion
which compelled the declaration of martial law, no longer pose a danger to public safety .
Same; Same; Military tribunals bound to observe fundamental principles of law and
justice.—As in trial before civil courts, the presumption of innocence can only be
overcome by evidence bey ond reasonable doubt of the guilt of the accused. These
tribunals, in general, are “bound to observe the fundamental rules of law and principles
of justice observed and expounded by the civil judicature.” Section 11 of the Manual for
Courts-Martial specifically provides that the “rules of evidence generally recognized in the
trial of criminal cases in the courts of the Philippines shall be applied by courts-martial.”
This is applicable in the military commission. There is, therefore, no justification for
petitioner’s contention that such military tribunals are concerned primarily with the
conviction of an accused and that proceedings therein involve the complete destruction
and abolition of petitioner’s constitutional rights. This is not, however, to preclude the
President from considering the advisability of the transfer of these cases to the civil
courts, as he has previously announced.
Benigno Aquino Jr. vs Military Commission No. 2, Chief of Staff, Chief Justice et al

Martial Law – Open Court Theory – Military Courts


In September 1972, after the declaration of Martial Law, Ninoy was arrested and was
placed under custody. He was brought Fort Bonifacio. He filed for the issuance of the Writ
of Habeas Corpus which was denied by the SC. Ninoy then questioned the validity of such
denial and the declaration of martial law; at the same time he questioned the authority of
the military court [No. 2] created [pursuant to GO 2-A] to try him and his other companions.
He was being charged for illegal possession of firearms, ammunition and explosives. He was
also being charged for violation of the Anti-Subversion Act and for murder. All were filed
before the military court. Ninoy argued that the military court has no jurisdiction or civilian
courts are still operational.
ISSUE: Whether or not Ninoy can be validly charged before the military court.
HELD: The SC upheld the power of the president to create military tribunals or military
courts which are authorized to try not only military personnel but also civilians even at that
time civil courts were open and functioning. The SC basically rejected the “open court’
theory observed in the USA.

BENIGNO S. AQUINO, JR. vs. MILITARY COMMISSION

G.R. No. L-37364 ; May 9, 1975


152
EXECUTIVE DEPARTMENT
Facts: In September 1972, after the declaration of Martial Law, Ninoy was arrested and
was placed under custody. He was brought Fort Bonifacio. He filed for the issuance of the
Writ of Habeas Corpus which was denied by the SC. Aquino then questioned the validity
of such denial and the declaration of martial law; at the same time he questioned the
authority of the military court [No. 2] created [pursuant to GO 2-A] to try him and his
other companions. He was being charged for illegal possession of firearms, ammunition
and explosives. He was also being charged for violation of the Anti-Subversion Act and for
murder. All were filed before the military court. Aquino argued that the military court has
no jurisdiction or civilian courts are still operational and that being a civilian, his trial by a
military commission deprives him of his right to due process.
Issue: Whether or not Aquino was afforded due process in a military court.
Held: YES. According to Schwartz, “The immunity of civilians from military jurisdiction
must, however, give way in areas governed by martial law. When it is absolutely
imperative for public safety, legal processes can be superseded and military tribunals
authorized to exercise the jurisdiction normally vested in court.”
Neither are We impressed with petitioner’s argument that only thru a judicial proceeding
before the regular courts can his right to due process be preserved. The guarantee of due
process is not a guarantee of any particular form of tribunal in criminal cases. A military
tribunal of competent jurisdiction, accusation in due form, notice and opportunity to
defend and trial before an impartial tribunal, adequately meet the due process
requirement. Due process of law does not necessarily means a judicial proceeding in the
regular courts. 14 The guarantee of due process, viewed in its procedural aspect,
requires no particular form of procedure. It implies due notice to the individual of the
proceedings, an opportunity to defend himself and “the problem of the propriety of the
deprivations, under the circumstances presented, must be resolved in a manner
consistent with essential fairness.” It means essentially a fair and impartial trial and
reasonable opportunity for the preparation of defense.
Here, the procedure before the Military Commission, as prescribed in Presidential Decree
No. 39, assures observance of the fundamental requisites of procedural due process, due
notice, an essentially fair and impartial trial and reasonable opportunity for the
preparation of the defense Section 11 of the Manual for Courts-Martial specifically
provides that the “rules of evidence generally recognized in the trial of criminal cases in
the courts of the Philippines shall be applied by courts-martial.” This is applicable to trials
in the military commission . There is, therefore, no justification for petitioner’s contention
that such military tribunals are concerned primarily with the conviction of an accused and
that proceedings therein involve the complete destruction and abolition of petitioner’s
constitutional rights.

G.R. No. L-37364 May 9, 1975


BENIGNO S. AQUINO, JR., petitioner,
vs.
MILITARY COMMISSION 2, CHIEF OF STAFF, ARMED FORCES OF THE PHILIPPINES, and
153
EXECUTIVE DEPARTMENT
SECRETARY OF NATIONAL DEFENSE, THE CHIEF JUSTICE OF THE SUPREME COURT, and
SECRETARY OF JUSTICE, * respondents.

Facts: In September 1972, after the declaration of Martial Law, Ninoy was arrested and
was placed under custody. He was brought to Fort Bonifacio. He filed for the issuance of
the Writ of Habeas Corpus which was denied by the SC. Aquino then questioned the
validity of such denial and the declaration of martial law; at the same time he questioned
the authority of the military court [No. 2] created [pursuant to GO 2-A] to try him and his
other companions. He was being charged for illegal possession of firearms, ammunition
and explosives. He was also being charged for violation of the Anti-Subversion Act and for
murder. All were filed before the military court. Aquino argued that the military court had
no jurisdiction because civilian courts were still operational and that, being a civilian, his
trial by a military commission deprives him of his right to due process.
When the proceedings before the MilitaryCommission opened, petitioner questioned thef
airness of the trial and announced that he did not wish to participate in the proceedings
even as he discharged both his defense counsel of choice and his military defense
counsel. For the petitioner's assurance, a Special Committee was created
to reinvestigate the charges against petitioner. Petitioner filed supplemental petition
questioning the legality of the creation of the Special Committee. On March 24, 1975,
petitioner filed an "Urgent Motion for Issuance of Temporary Restraining Order Against
Military Commission No. 2"; praying that said Commission be prohibited from proceeding
with the perpetuation of testimony under its Order dated March 10, 1975, the same
being illegal, until further orders from the Supreme Court
Issue: Whether or not Aquino was afforded due process in a military court.
Held: YES. According to Schwartz, “The immunity of civilians from military jurisdiction
must, however, give way in areas governed by martial law. When it is absolutely
imperative for public safety, legal processes can be superseded and military tribunals
authorized to exercise the jurisdiction normally vested in court.”
Neither are We impressed with petitioner’s argument that only thru a judicial proceeding
before the regular courts can his right to due process be preserved. The guarantee of due
process is not a guarantee of any particular form of tribunal in criminal cases. A military
tribunal of competent jurisdiction, accusation in due form, notice and opportunity to
defend and trial before an impartial tribunal, adequately meet the due process
requirement. Due process of law does not necessarily means a judicial proceeding in the
regular courts. The guarantee of due process, viewed in its procedural aspect, requires
no particular form of procedure. It implies due notice to the individual of the
proceedings, an opportunity to defend himself and “the problem of the propriety of the
deprivations, under the circumstances presented, must be resolved in a manner
consistent with essential fairness.” It means essentially a fair and impartial trial and
reasonable opportunity for the preparation of defense.
Here, the procedure before the Military Commission, as prescribed in Presidential Decree
154
EXECUTIVE DEPARTMENT
No. 39 which is was promulgated on November 7,1972, providing for the "Rules
Governing the Creation, Composition, Jurisdiction, Procedure and Other Matters Relevant
to Military Tribunals,” assures observance of the fundamental requisites of procedural
due process, due notice, an essentially fair and impartial trial and reasonable opportunity
for the preparation of the defense Section 11 of the Manual for Courts-Martial specifically
provides that the “rules of evidence generally recognized in the trial of criminal cases in
the courts of the Philippines shall be applied by courts-martial.” This is applicable to trials
in the military commission. There is, therefore, no justification for petitioner’s contention
that such military tribunals are concerned primarily with the conviction of an accused and
that proceedings therein involve the complete destruction and abolition of petitioner’s
constitutional rights. This is not, however, to preclude the President from considering the
advisability of the transfer of these cases to the civil courts, as he has previously
announced.

34. OLAGUER v. MILITARY COMMISSION NO. 34

The issue on the jurisdiction of military commissions or tribunals to try civilians for
offenses allegedly committed before, and more particularly during a period of martial
law, as well as the other issues raised by the petitioners, have been ruled upon by a
divided Supreme Court inAquino, Jr. v. Military Commission No. 2.
This ruling has been affirmed, although not unanimously, in at least six other cases, to
wit: Gumaua v. Espino,[27] Buscayno v. Enrile,[28] Sison v. Enrile,[29] Luneta v. Special
[30] [31]
Military Commission No. 1, Ocampo v. Military Commission No. 25, and Buscayno v.
Military Commission Nos. 1, 2, 6 and 25.[32]
These rulings notwithstanding, the petitioners anchor their argument on their prayer
that the ruling in Aquino, Jr. be appraised anew and abandoned or modified
accordingly. After a thorough deliberation on the matter, We find cogent basis for re-
examining the same.
We agree with the dissenting views of then Justice, now Chief Justice
ClaudioTeehankee[35] and Madame Justice Cecilia Muñoz-Palma[36] in Aquino, Jr. in so far
as they hold that military commissions or tribunals have no jurisdiction to try civilians for
alleged offenses when the civil courts are open and functioning.
"And in Toth v. Quarles,[40] the U.S. Supreme Court further stressed that 'the assertion of
military authority over civilians cannot rest on the President's power as Commander-in-
Chief or on any theory of martial law.'
Moreover, military tribunals pertain to the Executive Department of the Government and
are simply instrumentalities of the executive power, provided by the legislature for the
President as Commander-in-Chief to aid him in properly commanding the army and navy
and enforcing discipline therein, and utilized under his orders or those of his authorized
military representatives.[41] Following the principle of separation of powers underlying the
155
EXECUTIVE DEPARTMENT
existing constitutional organization of the Government of the Philippines, the power and
the duty of interpreting the laws (as when an individual should be considered to have
violated the law) is primarily a function of the judiciary.[42] It is not, and it cannot be the
function of the Executive Department, through the military authorities. And as long as
the civil courts in the land remain open and are regularly functioning, as they do so today
and as they did during the period of martial law in the country, military tribunals cannot
try and exercise jurisdiction over civilians for offenses committed by them and which are
properly cognizable by the civil courts.[43] To have it otherwise would be a violation of the
constitutional right to due process of the civilian concerned.

Olaguer vs Military Commission

Habeas Corpus
In 1979, Olaguer and some others were detained by military personnel and they were
placed in Camp Bagong Diwa. Logauer and his group are all civilians. They were charged
with (1) unlawful possession of explosives and incendiary devices; (2) conspiracy to
assassinate President and Mrs. Marcos; (3) conspiracy to assassinate cabinet members Juan
Ponce Enrile, Francisco Tatad and Vicente Paterno; (4) conspiracy to assassinate Messrs.
Arturo Tangco, Jose Roño and Onofre Corpus; (5) arson of nine buildings; (6) attempted
murder of Messrs. Leonardo Perez, Teodoro Valencia and Generals Romeo Espino and
Fabian Ver; and (7) conspiracy and proposal to commit rebellion, and inciting to rebellion.
On August 19, 1980, the petitioners went to the SC and filed the instant Petition for
prohibition and habeas corpus.
ISSUE: Whether or not the petition for habeas corpus be granted.
HELD: The petition for habeas corpus has become moot and academic because by the time
the case reached the SC Olaguer and his companions were already released from military
confinement. “When the release of the persons in whose behalf the application for a writ
of habeas corpus was filed is effected, the Petition for the issuance of the writ becomes
moot and academic. 18 Inasmuch as the herein petitioners have been released from their
confinement in military detention centers, the instant Petitions for the issuance of a writ
of habeas corpus should be dismissed for having become moot and academic.” But the
military court created to try the case of Olaguer (and the decision it rendered) still
continues to subsist.
ISSUE2: The issue is then shifted to: Whether or not a military tribunal has the jurisdiction
to try civilians while the civil courts are open and functioning.
HELD: The SC nullified for lack of jurisdiction all decisions rendered by the military courts
or tribunals during the period of martial law in all cases involving civilian defendants. A
military commission or tribunal cannot try and exercise jurisdiction, even during the period
of martial law, over civilians for offenses allegedly committed by them as long as the civil
courts are open and functioning, and that any judgment rendered by such body relating to

156
EXECUTIVE DEPARTMENT
a civilian is null and void for lack of jurisdiction on the part of the military tribunal
concerned.

Olaguer etal. v. Military Commission, G.R. No. L-54558, May 22, 1987

FACTS: Petitioners, as civilians, have been charged the crime of subversion. Consequently,
the Chief-of-Staff of the AFP created a military tribunal, named Military Commission No.
34, to try criminal case against petitioners. Petitioners were then convicted and have
been imposed a penalty of death penalty. Thereafter, petitioners filed a petition to enjoin
the military tribunal from taking further action on their case for the tribunal should be
considered null and void. Respondents invoked that the creation of Military Commission
is constitutional as ruled upon in a previous case – Aquino v. Military Commission No.
2.- as decided upon by the Supreme Court. However, petitioners contend that such ruling
must be overturned because the ruling is now inapplicable since Martial Law has already
been lifted.

ISSUE: Whether or not the ruling in Aquino v. Military Commission be abandoned and/or
modified in so far as the case at bar is concerned?

HELD: Yes.

REASONING: First, the Court considered that since the martial law has been lifted during
the case is still pending, military tibunals, which were created for the purpose of martial
law, shall be held void already since the law itself is lifted. Second, the Court relied on the
dissenting views of some justices in Aquino v. MilComm, stating that ‘…Civilians like the
petitioner placed on tiral for civil offenses under general law are entited o trial by judicial
process, not by executive or military process…xxx..Judicial power exist only in
courts’.1Moreover, the Court emphasized that“Reverence for precedent, simply as
precedent, cannot prevail when constitutionalism and the public interest demand
otherwise. Thus, a doctrine which should be abandoned or modified should be
abandoned or modified accordingly. after all, more important than anything else is that
this Court should be right.’

GANCAYCO, J:

FACTS:

157
EXECUTIVE DEPARTMENT
June 13. 1980 -the respondent Chief of Staff of the Armed Forces of the Philippines 3
created the respondentMilitary Commission No 34 to try criminal case filed against the
petitioners.
July 30, 1980 - an amendedcharge sheet was filed for seven (7) offenses, namely:
(1) Unlawful possession of explosives and incendiarydevices;
(2) Conspiracy to assassinate President, and Mrs. Marcos;
(3) Conspiracy to assassinate cabinetmembers Juan Ponce Enrile, Francisco Tatad and
Vicente Paterno;
(4) Conspiracy to assassinate Messrs. ArturoTangco, Jose Roño and Onofre Corpus;
(5) Arson of nine buildings;
(6) Attempted murder of Messrs. LeonardoPerez, Teodoro Valencia and Generals Romeo
Espino and Fabian Ver; and
(7) Conspiracy and proposal tocommit rebellion, and inciting to rebellion.
December 4, 1984 - pending the resolution of the Petition, the respondent Military
Commission No. 34 passedsentence convicting the petitioners and imposed upon them the
penalty of death by electrocution.
The thrust of petitioner’s arguments is that military commissions have no jurisdiction to
try civiliansfor offenses alleged to have been committed during the period of martial law.
They also maintain that theproceedings before the respondent Military Commission No. 34
are in gross violation of their constitutional right todue process of law.

ISSUE:
Whether or not a military tribunal has the jurisdiction to try civilians while the civil courts
are open and functioning

HELD:
WHEREFORE,

DISMISSED - the Petitions for habeas corpus are for having become moot and academic.
GRANTED - The Petitions for certiorari and prohibition.
DECLARED UNCONSTITUTIONAL - The creation of the respondent Military Commission No.
34 to try civilians like the petitioners is hereby declared unconstitutional and all its
proceedings are deemed null and void. The temporary restraining order issued against the
respondents enjoining them from executing the Decision of the respondent Military
Commission No. 34 is hereby made permanent and the said respondents are permanently
prohibited from further pursuing Criminal Case No. MC-34-1 against the petitioners. The
sentence rendered by the respondent Military Commission No. 34 imposing the death
penalty on the petitioners is hereby vacated for being null and void, and all the items or
158
EXECUTIVE DEPARTMENT
properties taken from the petitioners in relation to the said criminal case should be
returned to them immediately.

RATIO:
(1) Military commissions or tribunals have no jurisdiction to try civilians for alleged offenses
when the civil courts are open and functioning.Due process of law demands that in all
criminal prosecutions (where the accused stands to lose either his life or his liberty), the
accused shall be entitled to, among others, a trial. Civilians like (the) petitioner placed on
trial for civil offenses under general law are entitled to trial by judicial process, not by
executive or military process.
(2) Judicial power exists only in the courts, which have "exclusive power to hear and
determine those matters which affect the life or liberty or property of a citizen.” In Toth v.
Quarles, 40 the U.S. Supreme Court furtherstressed that the assertion of military authority
over civilians cannot rest on the President's power as Commander-in-Chief or on any theory
of martial law.
(3) Following the principle of separation of powers underlyingthe existing constitutional
organization of the Government of the Philippines, the power and the duty of
interpretingthe laws as when an individual should be considered to have violated the law)
is primarily a function of the judiciary. It is not, and it cannot be the function of the
Executive Department, through the military authorities. And as long as the civil courts in
the land remain open and are regularly functioning, as they do so today and as they did
during the period of martial law in the country, military tribunals cannot try and exercise
jurisdiction over civilians for offenses committed by them and which are properly
cognizable by the civil courts.
(4) Respondent Military Commission No. 34 appears to have been rendered too hastily to
the prejudice to the petitioners, and in complete disregard of their constitutional right to
adduce evidence on their behalf.Thus, even assuming arguendo that the respondent
Military Commission No. 34 does have the jurisdiction to try the petitioners, the
Commission should be deemed ousted of its jurisdiction when, as observed by the
SolicitorGeneral, the said tribunal acted in disregard of the constitutional rights of the
accused. Indeed, it is well-settled that once a deprivation of a constitutional right is shown
to exist, the tribunal that rendered the judgment in question is deemed ousted of
jurisdiction.
(5) Proclamation No. 2045 (dated January 17, 1981) officially lifting martial law in the
Philippines and abolishing all military tribunals created pursuant to the national emergency
effectively divests the respondent Military Commission No. 34 (and all military tribunals for
that matter) of its supposed authority to try civilians, including the herein petitioners. The
said proclamation states:

159
EXECUTIVE DEPARTMENT
"The military tribunals created pursuant thereto are hereby dissolved upon final
determination of case's pending therein which may not be transferred to the civil courts
without irreparable prejudice to the state in view of the rules on double jeopardy, or other
circumstances which render prosecution of the cases difficult, if not impossible."

(6) Certainly, the rule of stare decisis is entitled to respect because stability in jurisprudence
is desirable. Nonetheless, reverence for precedent, simply as precedent, cannot prevail
when constitutionalism and the public interest demand otherwise. Thus, a doctrine which
should be abandoned or modified should be abandoned or modified accordingly. After all,
more important than anything else is that this Court should be right.

35. Guazon vs. De Villa, 181 SCRA 623, G.R. No. 80508 January 30, 1990

GUTIERREZ, JR., J.

WHEREFORE, the petition is hereby REMANDED to the Regional Trial Courts of Manila,
Malabon, and Pasay City where the petitioners may present evidence supporting their
allegations and where specific erring parties may be pinpointed and prosecuted.
Copies of this decision are likewise forwarded to the Commission on Human Rights, the
Secretary of Justice, the Secretary of National Defense, and the Commanding General
PC-INP for the drawing up and enforcement of clear guidelines to govern police actions
intended to abate riots and civil disturbances, flush out criminal elements, and subdue
terrorist activities.
In the meantime, the acts violative of human rights alleged by the petitioners as
committed during the police actions are ENJOINED until such time as permanent rules
to govern such actions are promulgated.
SO ORDERED. Guazon vs. De Villa, 181 SCRA 623, G.R. No. 80508 January 30, 1990
Constitutional Law; Police Power; Nothing in the Constitution denies the authority of the
Chief Executive to order police actions to stop unabated criminality, rising lawlessness
and alarming communist activities.—There is, of course, nothing in the Constitution which
denies the authority of the Chief Executive, invoked by the Solicitor General, to order
police actions to stop unabated criminality, rising lawlessness, and alarming communist
activities. The Constitution grants to Government the power to seek and cripple
subversive movements which would bring down constituted authority and substitute a
regime where individual liberties are suppressed as a matter of policy in the name of
security of the State. However, all police actions are governed by the limitations of the Bill

160
EXECUTIVE DEPARTMENT
of Rights. The Government cannot adopt the same reprehensible methods of
authoritarian systems both of the right and of the left, the enlargement of whose spheres
of influence it is trying hard to suppress. Our democratic institutions may still be fragile
but they are not in the least bit strengthened through violations of the constitutional
protections which are their distinguishing features.
Same; Same; Illegal Searches and Seizures; Right to be left alone in the privacy of his own
house guaranteed under Article IV of the 1973 Constitution.—“That right is guaranteed in
the following provisions of Article IV of the 1973 Constitution: “SEC. 3. The right of the
people to be secure in their persons, houses, papers and effects against unreasonable
searches and seizures of whatever nature and for any purpose shall not be violated, and
no search warrant or warrant of arrest shall issue except upon probable cause to be
determined by the judge, or such other responsible officer as may be authorized by law,
after examination under oath or affirmation of the complainant and the witnesses he may
produce, and particularly describing the place to be searched, and the persons or things
to be siezed.”
Same; Same; Same; Same; Importance of the right against unreasonable searches and
seizure explained.—“It is deference to one’s personality that lies at the core of this right,
but it could be also looked upon as a recognition of a constitutionally protected area,
primarily one’s home, but not necessarily thereto confined. (Cf. Hoffa v. United States,
385 US 293 [1966]) What is sought to be guarded is a man’s prerogative to choose who is
allowed entry to his residence. In that haven of refuge, his individuality can assert itself
not only in the choice of who shall be welcome but likewise in the kind of objects he
wants around him. There the state, however powerful, does not as such have access
except under the circumstances above noted, for in the traditional formulation, his
house, however humble, is his castle. Thus is outlawed any unwarranted intrusion by
government, which is called upon to refrain from any invasion of his dwelling and to
respect the privacies of his life. (Cf. Schmerber v. California, 384 US 757 [1966], Brennan,
J. and Boyd v. United States, 116 630 [1886]). In the same vein, Landynski in his
authoritative work (Search and Seizure and the Supreme Court [1966]), could fitly
characterize constitutional right as the embodiment of a ‘spiritual concept: the belief that
to value the privacy of home and person and to afford its constitutional protection
against the long reach of government is no less than to value human dignity, and that his
privacy must not be disturbed except in case of overriding social need, and then only
under stringent procedural safeguards.’
Same; Same; Police actions should not be characterized by methods that offend a sense
of justice.—The decision of the United States Supreme Court in Rochin v. California, (342
US 165; 96 L. Ed. 183 [1952]) emphasizes clearly that police actions should not be
characterized by methods that offend a sense of justice. The court ruled: “Applying these
general considerations to the circumstances of the present case, we are compelled to

161
EXECUTIVE DEPARTMENT
conclude that the proceedings by which this conviction was obtained do more than
offend some fastidious squeamishness or private sentimentalism about combatting crime
too energetically. This is conduct that shocks the conscience. Illegally breaking into the
privacy of the petitioner, the struggle to open his mouth and remove what was there, the
forcible extraction of his stomach’s contents—this course of proceeding by agents of
government to obtain evidence is bound to offend even hardened sensibilities. They are
methods too close to the rack and the screw to permit of constitutional differentiation.”
Same; Same; Same; It is not the police action per se which is impermissible and which
should be prohibited but the procedure used or methods which offend even hardened
sensibilities.—It is significant that it is not the police action per se which is impermissible
and which should be prohibited. Rather, it is the procedure used or in the words of the
court, methods which “offend even hardened sensibilities.”
Same; Same; Zonings intended to flush out subversives and criminal elements.—The areal
target zonings in this petition were intended to flush out subversives and criminal
elements particularly because of the blatant assassinations of public officers and police
officials by elements supposedly coddled by the communities where the “drives” were
conducted.
Same; Same; Where a violation of human rights is involved, it is the duty of the court to
stop the transgression.—Where a violation of human rights specifically guaranteed by the
Constitution is involved, it is the duty of the court to stop the transgression and state
where even the awesome power of the state may not encroach upon the rights of the
individual.
Same; Same; Same; The remedy is not to stop all police actions including the essential and
legitimate ones.—The Court believes it highly probable that some violations were actually
committed. This is so inspite of the alleged pleas of barangay officials for the thousands of
residents “to submit themselves voluntarily for character and personal verification.” We
cannot imagine police actions of the magnitude described in the petitions and admitted
by the respondents, being undertaken without some undisciplined soldiers and
policemen committing certain abuses. However, the remedy is not to stop all police
actions, including the essential and legitimate ones. We see nothing wrong in police
making their presence visibly felt in troubled areas. Police cannot respond to riots or
violent demonstrations if they do not move in sufficient numbers. A show of force is
sometimes necessary as long as the rights of people are protected and not violated.
Same; Same; Same; Same; The problem is not initially for the Supreme Court but basically
one for the executive departments and for trial courts.—The remedy is not an original
action for prohibition brought through a taxpayers’ suit. Where not one victim complains
and not one violator is properly charged, the problem is not initially for the Supreme
Court. It is basically one for the executive departments and for trial courts. Well meaning
citizens with only second hand knowledge of the events cannot keep on indiscriminately

162
EXECUTIVE DEPARTMENT
tossing problems of the executive, the military, and the police to the Supreme Court as if
we are the repository of all remedies for all evils.
Same; Same; Same; Same; Same; The problem is appropriate for the Commission on
Human Rights.—The problem is appropriate for the Commission on Human Rights. A high
level conference should bring together the heads of the Department of Justice,
Department of National Defense and the operating heads of affected agencies and
institutions to devise procedures for the prevention of abuses.
Facts: The 41 petitioners alleged that the "saturation drive" or "aerial target zoning" that
were conducted in their place (Tondo Manila) were unconstitutional. They alleged that
there is no specific target house to be search and that there is no search warrant or
warrant of arrest served. Most of the policemen are in their civilian clothes and
without nameplates oridentification cards. The residents were rudely rouse from their
sleep by banging on the walls and windows of their houses. The residents were at
the point of high-powered guns and herded like cows. Men were ordered to strip down to
their briefs for the police to examine their tattoo marks. The residents complained that
they're homes were ransacked, tossing theirbelongings and destroying their valuables.
Some of their money and valuables had disappeared after the operation. The residents
also reported incidents of maulings, spot-beatings and maltreatment. Those who were
detained also suffered mental and physical torture to extract confessions and tactical
informations. The respondents said that such accusations were all lies. Respondents
contends that the Constitution grants to government the power to seek and cripple
subversive movements for the maintenance of peace in the state. The aerial target zoning
were intended to flush out subversives and criminal elements coddled by the
communities were the said drives were conducted. They said that they have intelligently
and carefully planned months ahead for the actual operation and that local and foreign
media joined the operation to witness and record such event.

Issue: Whether or Not the saturation drive committed consisted of violation of human
rights.

Held: It is not the police action per se which should be prohibited rather it is the
procedure used or the methods which "offend even hardened sensibilities" .Based on the
facts stated by the parties, it appears to have been no impediment to securing search
warrants or warrants of arrest before any houses were searched or individuals roused
from sleep were arrested. There is no showing that the objectives sought to be attained
by the "aerial zoning" could not be achieved even as th rights of the squatters and low
income families are fully protected. However, the remedy should not be brought by a
tazpaer suit where not one victim complaints and not one violator is properly charged. In
the circumstances of this taxpayers' suit, there is no erring soldier or policeman whom the
court can order prosecuted. In the absence of clear facts no permanent relief can be
163
EXECUTIVE DEPARTMENT
given.

In the meantime where there is showing that some abuses were committed, the court
temporary restraint the alleged violations which are shocking to the senses. Petition is
remanded to the RTC of Manila.
Section 13, Rule 126 of the Rules of Court and some cases decided by the Supreme Court
provide the instances when search is lawful without search warrant:
1. In times of war within the area of military operation.
(People v. de Gracia, 233 SCRA 716, Guanzon v. de Villa, 181 SCRA 623)

CALLING OUT POWERS

36. IBP v. ZAMORA


SYNOPSIS
The President of the Philippines, Joseph Ejercito Estrada, in a verbal directive, ordered the
PNP and the Marines to conduct joint visibility patrols for the purpose of crime prevention
and suppression. In compliance with the presidential mandate, the PNP Chief, through
Police Chief Superintendent Edgar B. Aglipay, formulated Letter of Instruction 02/2000 (the
"LOI") which detailed the manner by which the joint visibility patrols, called Task Force
Tulungan, would be conducted. Task Force Tulungan was placed under the leadership of
the Police Chief of Metro Manila. Invoking his powers as Commander-in-Chief under
Section 18, Article VII of the Constitution, the President directed the AFP Chief of Staff and
PNP Chief to coordinate with each other for the proper deployment and utilization of the
Marines to assist the PNP in preventing or suppressing criminal or lawless violence. The
President also declared that the services of the Marines in the anti-crime campaign are
merely temporary in nature and for a reasonable period only, until such time when the
situation shall have improved. The Integrated Bar of the Philippines (the "IBP") filed the
instant petition to annul LOI 02/2000 and to declare the deployment of the Philippine
Marines null and void and unconstitutional, arguing that the deployment of marines in
Metro Manila is violative of the Constitution because no emergency situation obtains in
Metro Manila as would justify, even only remotely, the deployment of soldiers for law
enforcement work; hence, said deployment in derogation of Article II, Section 3 of the
Constitution.
The Supreme Court found no merit in the petition. When the President calls the armed
forces to prevent or suppress lawless violence, invasion or rebellion, he necessarily
exercises a discretionary power solely vested in his wisdom. This is clear from the intent of
the framers and from the text of the Constitution itself. The Court, thus, cannot be called
upon to overrule the President's wisdom or substitute its own. It does not, however,
prevent an examination of whether such power was exercised within permissible
164
EXECUTIVE DEPARTMENT
constitutional limits or whether it was exercised in a manner constituting grave abuse of
discretion. In view of the constitutional intent to give the President full discretionary power
to determine the necessity of calling out the armed forces, it is incumbent upon the
petitioner to show that the President's decision is totally bereft of factual basis. The
petition failed to discharge such heavy burden as there was no evidence to support the
assertion that there exists no justification for calling out the armed forces nor was grave
abuse committed because the power to call was exercised in such a manner as to violate
the constitutional provision on civilian supremacy over the military. In the performance of
the Court's duty of "purposeful hesitation" before declaring an act of another branch as
unconstitutional, only where such grave abuse of discretion is clearly shown shall the Court
interfere with the President's judgment and to doubt is to sustain. The Court also ruled that
the calling of the Marines in this case constitutes permissible use of military assets for
civilian law enforcement. The participation of the Marines in the conduct of joint visibility
patrols is appropriately circumscribed. The limited participation of the Marines is evident
in the provisions of the LOI itself, which sufficiently provides the metes and bounds of the
Marines' authority. It is noteworthy that the local police forces are the ones in charge of
the visibility patrols at all times, the real authority belonging to the PNP. Under the LOI, the
police forces are tasked to brief or orient the soldiers on police patrol procedures. It is their
responsibility to direct and manage the deployment of the Marines. It is, likewise, their
duty to provide the necessary equipment to the Marines and render logistical support to
these soldiers. It cannot be properly argued then that military authority is supreme over
civilian authority. Moreover, the deployment of the Marines to assist the PNP does not
unmake the civilian character of the police force. Neither does it amount to an "insidious
incursion" of the military in the task of law enforcement in violation of Section 5(4), Article
XVI of the Constitution.
FACTS:
Invoking his powers as Commander-in-Chief under Sec 18, Art. VII of the Constitution,
President Estrada, in verbal directive, directed the AFP Chief of Staff and PNP Chief to
coordinate with each other for the proper deployment and campaign for a temporary
period only. The IBP questioned the validity of the deployment and utilization of the
Marines to assist the PNP in law enforcement.

ISSUE:
1. WoN the President's factual determination of the necessity of calling the armed forces
is subject to judicial review.
2. WoN the calling of AFP to assist the PNP in joint visibility patrols violate the
constitutional provisions on civilian supremacy over the military.

RULING:
165
EXECUTIVE DEPARTMENT
1. The power of judicial review is set forth in Section 1, Article VIII of the Constitution, to
wit:
Section 1. The judicial power shall be vested in one Supreme Court and in such lower
courts as may be established by law.
Judicial power includes the duty of the courts of justice to settle actual controversies
involving rights which are legally demandable and enforceable, and to determine whether
or not there has been grave abuse of discretion amounting to lack or excess of jurisdiction
on the part of any branch or instrumentality of the Government.
When questions of constitutional significance are raised, the Court can exercise its power
of judicial review only if the following requisites are complied with, namely: (1) the
existence of an actual and appropriate case; (2) a personal and substantial interest of the
party raising the constitutional question; (3) the exercise of judicial review is pleaded at
the earliest opportunity; and (4) the constitutional question is the lis mota of the case.

2. The deployment of the Marines does not constitute a breach of the civilian supremacy
clause. The calling of the Marines in this case constitutes permissible use of military
assets for civilian law enforcement. The participation of the Marines in the conduct of
joint visibility patrols is appropriately circumscribed. It is their responsibility to direct and
manage the deployment of the Marines. It is, likewise, their duty to provide the necessary
equipment to the Marines and render logistical support to these soldiers. In view of the
foregoing, it cannot be properly argued that military authority is supreme over civilian
authority. Moreover, the deployment of the Marines to assist the PNP does not unmake
the civilian character of the police force. Neither does it amount to an “insidious
incursion” of the military in the task of law enforcement in violation of Section 5(4),
Article XVI of the Constitution.
Judicial Review; Requisites.—When questions of constitutional significance are raised, the
Court can exercise its power of judicial review only if the following requisites are
complied with, namely: (1) the existence of an actual and appropriate case; (2) a personal
and substantial interest of the party raising the constitutional question; (3) the exercise of
judicial review is pleaded at the earliest opportunity; and (4) the constitutional question is
the lis mota of the case.
Same; Same; Parties; “Locus Standi”; Words and Phrases; “Legal Standing” or “Locus
Standi,” “Interest,” Explained.—“Legal standing” or locus standi has been defined as a
personal and substantial interest in the case such that the party has sustained or will
sustain direct injury as a result of the governmental act that is being challenged. The term
“interest” means a material interest, an interest in issue affected by the decree, as
distinguished from mere interest in the question involved, or a mere incidental interest.
The gist of the question of standing is whether a party alleges “such personal stake in the
outcome of the controversy as to assure that concrete adverseness which sharpens the
166
EXECUTIVE DEPARTMENT
presentation of issues upon which the court depends for illumination of difficult
constitutional questions.”
Same; Same; Same; Integrated Bar of the Philippines; The mere invocation by the
Integrated Bar of the Philippines of its duty to preserve the rule of law and nothing more,
while undoubtedly true, is not sufficient to clothe it with standing in this case—this is too
general an interest which is shared by other groups and the whole citizenry.—The IBP
primarily anchors its standing on its alleged responsibility to uphold the rule of law and
the Constitution. Apart from this declaration, however, the IBP asserts no other basis in
support of its locus standi. The mere invocation by the IBP of its duty to preserve the rule
of law and nothing more, while undoubtedly true, is not sufficient to clothe it with
standing in this case. This is too general an interest which is shared by other groups and
the whole citizenry. Based on the standards above-stated; the IBP has failed to present a
specific and substantial interest in the resolution of the case.
Its fundamental purpose which, under Section 2, Rule 139-A of the Rules of Court, is to
elevate the standards of the law profession and to improve the administration of justice is
alien to, and cannot be affected by the deployment of the Marines.
Same; Same; Same; Same; The interest of the National President of the Integrated Bar of
the Philippines who signed the petition, is his alone, absent a formal board resolution
authorizing him to file the present action.—It should also be noted that the interest of the
National President of the IBP who signed the petition, is his alone, absent a formal board
resolution authorizing him to file the present action. To be sure, members of the BAR,
those in the judiciary included, have varying opinions on the issue. Moreover, the IBP,
assuming that it has duly authorized the National President to file the petition, has not
shown any specific injury which it has suffered or may suffer by virtue of the questioned
governmental act. Indeed, none of its members, whom the IBP purportedly represents,
has sustained any form of injury as a result of the operation of the joint visibility patrols.
Neither is it alleged that any of its members has been arrested or that their civil liberties
have been violated by the deployment of the Marines. What the IBP projects as injurious
is the supposed “militarization” of law enforcement which might threaten Philippine
democratic institutions and may cause more harm than good in the long run. Not only is
the presumed “injury” not personal in character, it is likewise too vague, highly
speculative and uncertain to satisfy the requirement of standing.
Same; Same; Same; Same; The Supreme Court, however, does not categorically rule that
the Integrated Bar of the Philippines has absolutely no standing to raise constitutional
issues how or in the future, but the Integrated Bar of the Philippines must, by way of
allegations and proof, satisfy the Court that it has sufficient stake to obtain judicial
resolution of the controversy.—Since petitioner has not successfully established a direct
and personal injury as a consequence of the questioned act, it does not possess the
personality to assail the validity of the deployment of the Marines. This Court, however,

167
EXECUTIVE DEPARTMENT
does not categorically rule that the IBP has absolutely no standing to raise constitutional
issues now or in the future. The IBP must, by way of allegations and proof, satisfy this
Court that it has sufficient stake to obtain judicial resolution of the controversy.
Same; Same; Same; Same; The Supreme Court has the discretion to take cognizance of a
suit which does not satisfy the requirement of legal standing when paramount interest is
involved; In this case, a reading of the petition shows that the Integrated Bar of the
Philippines has advanced constitutional issues which deserve the attention of the
Supreme Court in view of their seriousness, novelty and weight as precedents.—Having
stated the foregoing, it must be emphasized that this Court has the discretion to take
cognizance of a suit which does not satisfy the requirement of legal standing when
paramount interest is involved. In not a few cases, the Court has adopted a liberal
attitude on the locus standi of a petitioner where the petitioner is able to craft an issue of
transcendental significance to the people. Thus, when the issues raised are of paramount
importance to the public, the Court may brush aside technicalities of procedure. In this
case, a reading of the petition shows that the IBP has advanced constitutional issues
which deserve the attention of this Court in view of their seriousness, novelty and weight
as precedents. Moreover, because peace and order are under constant threat and lawless
violence occurs in increasing tempo, undoubtedly aggravated by the Mindanao
insurgency problem, the legal controversy raised in the petition almost certainly will not
go away. It will stare us in the face again. It, therefore, behooves the Court to relax the
rules on standing and to resolve the issue now, rather than later.
Same; Presidency; Commander-in-Chief Clause; “Calling Out” Power; While the Supreme
Court gives considerable weight to the parties’ formulation of the issues, the resolution of
the controversy may warrant a creative approach that goes beyond the narrow confines
of the issues raised; Even as the parties are in agreement that the power exercised by the
President is the power to call out the armed forces, the Court is of the view that the
power involved may be no more than the maintenance of peace and order and
promotion of the general welfare.—As framed by the parties, the underlying issues are
the scope of presidential powers and limits, and the extent of judicial review. But, while
this Court gives considerable weight to the parties’ formulation of the issues, the
resolution of the controversy may warrant a creative approach that goes beyond the
narrow confines of the issues raised. Thus, while the parties are in agreement that the
power exercised by the President is the power to call out the armed forces, the Court is of
the view that the power involved may be no more than the maintenance of peace and
order and promotion of the general welfare. For one, the realities on the ground do not
show that there exist a state of warfare, widespread civil unrest or anarchy. Secondly, the
full brunt of the military is not brought upon the citizenry, a point discussed in the latter
part of this decision.

168
EXECUTIVE DEPARTMENT
Same; Same; Political Questions; Separation of Powers; As a general proposition, a
controversy is justiciable if it refers to a matter which is appropriate for court review; One
class of cases wherein the Court hesitates to rule on are “political questions”; The political
question being a function of the separation of powers, the courts will not normally
interfere with the workings of another co-equal branch unless the case shows a clear
need for the courts to step in to uphold the law and the Constitution.—As a general
proposition, a controversy is justiciable if it refers to a matter which is appropriate for
court review. It pertains to issues which are inherently susceptible of being decided on
grounds recognized by law. Nevertheless, the Court does not automatically assume
jurisdiction over actual constitutional cases brought before it even in instances that are
ripe for resolution. One class of cases wherein the Court hesitates to rule on are “political
questions.” The reason is that political questions are concerned with issues dependent
upon the wisdom, not the legality, of a particular act or measure being assailed.
Moreover, the political question being a function of the separation of powers, the courts
will not normally interfere with the workings of another co-equal branch unless the case
shows a clear need for the courts to step in to uphold the law and the Constitution.
Same; Same; Same; Same; When the grant of power is qualified, conditional or subject to
limitations, the issue of whether the prescribed qualifications or conditions have been
met or the limitations respected, is justiciable—the problem being one of legality or
validity, not its wisdom.—The 1987 Constitution expands the concept of judicial review by
providing that “(T)he Judicial power shall be vested in one Supreme Court and in such
lower courts as may be established by law. Judicial power includes the duty of the courts
of justice to settle actual controversies involving rights which are legally demandable and
enforceable, and to determine whether or not there has been a grave abuse of discretion
amounting to lack or excess of jurisdiction on the part of any branch or instrumentality of
the Government.” Under this definition, the Court cannot agree with the Solicitor General
that the issue involved is a political question beyond the jurisdiction of this Court to
review. When the grant of power is qualified, conditional or subject to limitations, the
issue of whether the prescribed qualifications or conditions have been met or the
limitations respected, is justiciable—the problem being one of legality or validity, not its
wisdom. Moreover, the jurisdiction to delimit constitutional boundaries has been given to
this Court. When political questions are involved, the Constitution limits the
determination as to whether or not there has been a grave abuse of discretion amounting
to lack or excess of jurisdiction on the part of the official whose action is being
questioned.
Same; Same; Same; Words and Phrases; By grave abuse of discretion is meant simply
capricious or whimsical exercise of judgment that is patent and gross as to amount to an
evasion of positive duty or a virtual refusal to perform a duty enjoined by law, or to act at
all in contemplation of law, as where the power is exercised in an arbitrary and despotic
manner by reason of passion or hostility; A showing that plenary power is granted either
169
EXECUTIVE DEPARTMENT
department of government, may not be an obstacle to judicial inquiry, for the
improvident exercise or abuse thereof may give rise to justiciable controversy.—By grave
abuse of discretion is meant simply capricious or whimsical exercise of judgment that is
patent and gross as to amount to an evasion of positive duty or a virtual refusal to
perform a duty enjoined by law, or to act at all in contemplation of law, as where the
power is exercised in an arbitrary and despotic manner by reason of passion or hostility.
Under this definition, a court is without power to directly decide matters over which full
discretionary authority has been delegated. But while this Court has no power to
substitute its judgment for that of Congress or of the President, it may look into the
question of whether such exercise has been made in grave abuse of discretion. A showing
that plenary power is granted either department of government, may not be an obstacle
to judicial inquiry, for the improvident exercise or abuse thereof may give rise to
justiciable controversy.
Same; Same; Commander-in-Chief Clause; “Calling Out” Power; In view of the
constitutional intent to give the President full discretionary power to determine the
necessity of calling out the armed forces, it is incumbent upon the petitioner to show that
the President’s decision is totally bereft of factual basis; In the performance of the
Supreme Court’s duty of “purposeful hesitation” before declaring an act of another
branch as unconstitutional, only where such grave abuse of discretion is clearly shown
shall the Court interfere with the President’s judgment—to doubt is to sustain.—When
the President calls the armed forces to prevent or suppress lawless violence, invasion or
rebellion, he necessarily exercises a discretionary power solely vested in his wisdom. This
is clear from the intent of the framers and from the text of the Constitution itself. The
Court, thus, cannot be called upon to overrule the President’s wisdom or substitute its
own. However, this does not prevent an examination of whether such power was
exercised within permissible constitutional limits or whether it was exercised in a manner
constituting grave abuse of discretion. In view of the constitutional intent to give the
President full discretionary power to determine the necessity of calling out the armed
forces, it is incumbent upon the petitioner to show that the President’s decision is totally
bereft of factual basis. The present petition fails to discharge such heavy burden as there
is no evidence to support the assertion that there exist no justification for calling out the
armed forces. There is, likewise, no evidence to support the proposition that grave abuse
was committed because the power to call was exercised in such a manner as to violate
the constitutional provision on civilian supremacy over the military. In the performance of
this Court’s duty of “purposeful hesitation” before declaring an act of another branch as
unconstitutional, only where such grave abuse of discretion is clearly shown shall the
Court interfere with the President’s judgment. To doubt is to sustain.
Same; Same; Same; Same; Statutory Construction; Unlike in the power to suspend the
privilege of the writ of habeas corpus or the power to proclaim martial law in relation to
which the Constitution has empowered Congress to revoke such suspension or
170
EXECUTIVE DEPARTMENT
proclamation and the Supreme Court to review the sufficiency of the factual basis thereof
there is no such equivalent provision dealing with the revocation or review of the
President’s action to call out the armed forces, a distinction which places the calling out
power in a different category from the power to declare martial law and the power to
suspend the privilege of the writ of habeas corpus.—Under the foregoing provisions,
Congress may revoke such proclamation or suspension and the Court may review the
sufficiency of the factual basis thereof. However, there is no such equivalent provision
dealing with the revocation or review of the President’s action to call out the armed
forces. The distinction places the calling out power in a different category from the power
to declare martial law and the power to suspend the privilege of the writ of habeas
corpus, otherwise, the framers of the Constitution would have simply lumped together
the three powers and provided for their revocation and review without any qualification.
Expressio unius est exclusio alterius. Where the terms are expressly limited to certain
matters, it may not, by interpretation or construction, be extended to other matters. That
the intent of the Constitution is exactly what its letter says, i.e., that the power to call is
fully discretionary to the President, is extant in the deliberation of the Constitutional
Commission.
Same; Same; Same; Same; The reason for the difference in the treatment of the power to
suspend the privilege of the writ of habeas corpus, the power to declare martial law and
the power to call out the armed forces highlights the intent to grant the President the
widest leeway and broadest discretion in using the power to call out because it is
considered as the lesser and more benign power compared to the two other powers.---
The reason for the difference in the treatment of the aforementioned powers highlights
the intent to grant the President the widest leeway and broadest discretion in using the
power to call out because it is considered as the lesser and more benign power compared
to the power to suspend the privilege of the writ of habeas corpus and the power to
impose martial law, both of which involve the curtailment and suppression of certain
basic civil rights and individual freedoms, and thus necessitating safeguards by Congress
and review by this Court. Moreover, under Section 18, Article VII of the Constitution, in
the exercise of the power to suspend the privilege of the writ of habeas corpus or to
impose martial law, two conditions must concur: (1) there must be an actual invasion or
rebellion and, (2) public safety must require it. These conditions are not required in the
case of the power to call out the armed forces. The only criterion is that “whenever it
becomes necessary,” the President may call the armed forces “to prevent or suppress
lawless violence, invasion or rebellion.” The implication is that the President is given full
discretion and wide latitude in the exercise of the power to call as compared to the two
other powers.
Same; Same; Same; Same; If the petitioner fails, by way of proof to support the assertion
that the President acted without factual basis, then the Supreme Court cannot undertake
an independent investigation beyond the pleadings.—If the petitioner fails, by way of
171
EXECUTIVE DEPARTMENT
proof, to support the assertion that the President acted without factual basis, then this
Court cannot undertake an independent investigation beyond the pleadings. The factual
necessity of calling out the armed forces is not easily quantifiable and cannot be
objectively established since matters considered for satisfying the same is a combination
of several factors which are not always accessible to the courts. Besides the absence of
textual standards that the court may use to judge necessity, information necessary to
arrive at such judgment might also prove unmanageable for the courts. Certain pertinent
information might be difficult to verify, or wholly unavailable to the courts. In many
instances, the evidence upon which the President might decide that there is a need to call
out the armed forces may be of a nature not constituting technical proof.
Same; Same; Same; Same; Judicial Notice; The Court takes judicial notice of the recent
bombings perpetrated by lawless elements in shopping malls, public utilities, and other
public places.—The President has already determined the necessity and factual basis for
calling the armed forces. In his Memorandum, he categorically asserted that, “[V]iolent
crimes like bank/store robberies, holdups, kidnappings and carnappings continue to occur
in Metro Manila . . .” We do not doubt the veracity of the President’s assessment of the
situation, especially in the light of present developments. The Court takes judicial notice
of the recent bombings perpetrated by lawless elements in the shopping malls, public
utilities, and other public places. These are among the areas of deployment described in
the LOI 2000. Considering all these facts, we hold that the President has sufficient factual
basis to call for military aid in law enforcement and in the exercise of this constitutional
power.
Same; Same; Same; Same; Civilian Supremacy Clause; The deployment of the Marines in
the metropolis for civilian law enforcement does not constitute a breach of the civilian
supremacy clause.—The deployment of the Marines does not constitute a breach of the
civilian supremacy clause. The calling of the Marines in this case constitutes permissible
use of military assets for civilian law enforcement. The participation of the Marines in the
conduct of joint visibility patrols is appropriately circumscribed. The limited participation
of the Marines is evident in the provisions of the LOI itself, which sufficiently provides the
metes and bounds of the Marines’ authority. It is noteworthy that the local police forces
are the ones in charge of the visibility patrols at all times, the real authority belonging to
the PNP. In fact, the Metro Manila Police Chief is the overall leader of the PNP-Philippine
Marines joint visibility patrols. Under the LOI, the police forces are tasked to brief or
orient the soldiers on police patrol procedures. It is their responsibility to direct and
manage the deployment of the Marines. It is, likewise, their duty to provide the necessary
equipment to the Marines and render logistical support to these soldiers. In view of the
foregoing, it cannot be properly argued that military authority is supreme over civilian
authority. Moreover, the deployment of the Marines to assist the PNP does not unmake
the civilian character of the police force. Neither does it amount to an “insidious

172
EXECUTIVE DEPARTMENT
incursion” of the military in the task of law enforcement in violation of Section 5(4),
Article XVI of the Constitution.
Same; Same; Same; Same; Same; Philippine National Police (PNP); Where none of the
Marines was incorporated or enlisted as members of the Philippine National Police, there
can be no appointment to a civilian position to speak of—the deployment of the Marines
in the joint visibility patrols does not destroy the civilian character of the Philippine
National Police.—In this regard, it is not correct to say that General Angelo Reyes, Chief of
Staff of the AFP, by his alleged involvement in civilian law enforcement, has been virtually
appointed to a civilian post in derogation of the aforecited provision. The real authority in
these operations, as stated in the LOI, is lodged with the head of a civilian institution, the
PNP, and not with the military. Such being the case, it does not matter whether the APP
Chief actually participates in the Task Force Tulungan since he does not exercise any
authority or control over the same. Since none of the Marines was incorporated or
enlisted as members of the PNP, there can be no appointment to a civilian position to
speak of. Hence, the deployment of the Marines in the joint visibility patrols does not
destroy the civilian character of the PNP.
Same; Same; Same; Same; Same; Words and Phrases; “Regulatory Power” “Proscriptive
Power,” and “Compulsory Power,” Distinguished.—A power regulatory in nature is one
which controls or directs. It is proscriptive if it prohibits or condemns and compulsory if it
exerts some coercive force. See US v. Yunis, 681 F.Supp 891 (D.D.C., 1988). See also
FOURTH AMENDMENT AND POSSE COMITATUS ACT RESTRICTIONS ON MILITARY
INVOLVEMENT IN CIVIL LAW ENFORCEMENT, 54 George Washington Law Review, pp.
404-433 (1986), which discusses the four divergent standards for assessing acceptable
involvement of military personnel in civil law enforcement. See likewise HONORED IN THE
BREECH: PRESIDENTIAL AUTHORITY TO EXECUTE THE LAWS WITH MILITARY FORCE, 83
Yale Law Journal, pp. 130-152, 1973.
Same; Same; Same; Same; Same; Even if the Court were to apply the rigid standards to
determine whether there is permissible use of the military in civilian law enforcement,
the conclusion is inevitable that no violation of the civilian supremacy clause in the
Constitution is committed.—Even if the Court were to apply the above rigid standards to
the present case to determine whether there is permissible use of the military in civilian
law enforcement, the conclusion is inevitable that no violation of the civilian supremacy
clause in the Constitution is committed. On this point, the Court agrees with the
observation of the Solicitor General: 3. The designation of tasks in Annex A does not
constitute the exercise of regulatory, proscriptive, or compulsory military power. First,
the soldiers do not control or direct the operation. This is evident from Nos. 6, 8(k) and
9(a) of Annex A. These soldiers, second, also have no power to prohibit or condemn. In
No. 9(d) of Annex A, all arrested persons are brought to the nearest police stations for
proper disposition. And last, these soldiers apply no coercive force. The materials or

173
EXECUTIVE DEPARTMENT
equipment issued to them, as shown in No. 8(c) of Annex A, are all low impact and
defensive in character. The conclusion is that there being no exercise of regulatory,
proscriptive or compulsory military power, the deployment of a handful of Philippine
Marines constitutes no impermissible use of military power for civilian law enforcement.
Same; Same; Same; Same; Unless the petitioner can show that in the deployment of the
Marines, the President has violated the fundamental law, exceeded his authority or
jeopardized the civil liberties of the people, the Supreme Court is not inclined to overrule
the President’s determination of the factual basis for the calling of the Marines to prevent
or suppress lawless violence.—It appears that the present petition is anchored on fear
that once the armed forces are deployed, the military will gain ascendancy, and thus
place in peril our cherished liberties. Such apprehensions, however, are unfounded. The
power to call the armed forces is just that—calling out the armed forces. Unless,
petitioner IBP can show, which it has not, that in the deployment of the Marines, the
President has violated the fundamental law, exceeded his authority or jeopardized the
civil liberties of the people, this Court is not inclined to overrule the President’s
determination of the factual basis for the calling of the Marines to prevent or suppress
lawless violence.
Freedom; Civil Liberties; Freedom and democracy will be in full bloom only when people
feel secure in their homes and in the streets, not when the shadows of violence and
anarchy constantly lurk in their midst.—Since the institution of the joint visibility patrol in
January, 2000, not a single citizen has complained that his political or civil rights have
been violated as a result of the deployment of the Marines. It was precisely to safeguard
peace, tranquility and the civil liberties of the people that the joint visibility patrol was
conceived. Freedom and democracy will be in full bloom only when people feel secure in
their homes and in the streets, not when the shadows of violence and anarchy constantly
lurk in their midst.

37. KULAYAN v. TAN


Civil Procedure; Courts; Hierarchy of Courts; The doctrine of hierarchy of courts provides
that where the issuance of an extraordinary writ is also within the competence of the
Court of Appeals (CA) or the Regional Trial Court (RTC), it is in either of these courts and
not in the Supreme Court, that the specific action for the issuance of such writ must be
sought unless special and important laws are clearly and specifically set forth in the
petition.—We first dispose of respondents’ invocation of the doctrine of hierarchy of
courts which allegedly prevents judicial review by this Court in the present case, citing for
this specific purpose, Montes v. Court of Appeals, 489 SCRA 382 (2006), and Purok
Bagong Silang Association, Inc. v. Yuipco, 384 SCRA 152 (2002), Simply put, the doctrine
provides that where the issuance of an extraordinary writ is also within the competence
of the CA or the RTC, it is in either of these courts and not in the Supreme Court, that the

174
EXECUTIVE DEPARTMENT
specific action for the issuance of such writ must be sought unless special and important
laws are clearly and specifically set forth in the petition. The reason for this is that this
Court is a court of last resort and must so remain if it is to perform the functions assigned
to it by the Constitution and immemorial tradition. It cannot be burdened with deciding
cases in the first instance. x x x The instant case stems from a petition for certiorari and
prohibition, over which the Supreme Court possesses original jurisdiction. More crucially,
this case involves acts of a public official which pertain to restrictive custody, and is thus
impressed with transcendental public importance that would warrant the relaxation of
the general rule. The Court would be remiss in its constitutional duties were it to dismiss
the present petition solely due to claims of judicial hierarchy.
Presidency; There is one repository of executive powers, and that is the President of the
Republic—this means that when Section 1, Article VII of the Constitution speaks of
executive power, it is granted to the President and no one else.—As early as Villena v.
Secretary of Interior, 67 Phil. 451 (1939), it has already been established that there is one
repository of executive powers, and that is the President of the Republic. This means that
when Section 1, Article VII of the Constitution speaks of executive power, it is granted to
the President and no one else. As emphasized by Justice Jose P. Laurel, in his ponencia in
Villena: With reference to the Executive Department of the government, there is one
purpose which is crystal-clear and is readily visible without the projection of judicial
searchlight, and that is the establishment of a single, not plural, Executive. The first
section of Article VII of the Constitution, dealing with the Executive Department, begins
with the enunciation of the principle that “The executive power shall be vested in a
President of the Philippines.” This means that the President of the Philippines is the
Executive of the Government of the Philippines, and no other. Corollarily, it is only the
President, as Executive, who is authorized to exercise emergency powers as provided
under Section 23, Article VI, of the Constitution, as well as what became known as the
calling-out powers under Section 7, Article VII thereof.
Same; Calling-Out Powers; By constitutional fiat, the calling-out powers, which is of lesser
gravity than the power to declare martial law, is bestowed upon the President alone.—
The power to declare a state of martial law is subject to the Supreme Court’s authority to
review the factual basis thereof. By constitutional fiat, the calling-out powers, which is of
lesser gravity than the power to declare martial law, is bestowed upon the President
alone. As noted in Villena, “(t)here are certain constitutional powers and prerogatives of
the Chief Executive of the Nation which must be exercised by him in person and no
amount of approval or ratification will validate the exercise of any of those powers by any
other person. Such, for instance, is his power to suspend the writ of habeas corpus and
proclaim martial law x x x.
Same; While the President is still a civilian, Article II, Section 3 of the Constitution
mandates that civilian authority is, at all times, supreme over the military, making the

175
EXECUTIVE DEPARTMENT
civilian president the nation’s supreme military leader; The Constitution does not require
that the President must be possessed of military training and talents, but as Commander-
in-Chief, he has the power to direct military operations and to determine military
strategy.—Indeed, while the President is still a civilian, Article II, Section 3 of the
Constitution mandates that civilian authority is, at all times, supreme over the military,
making the civilian president the nation’s supreme military leader. The net effect of
Article II, Section 3, when read with Article VII, Section 18, is that a civilian President is the
ceremonial, legal and administrative head of the armed forces. The Constitution does not
require that the President must be possessed of military training and talents, but as
Commander-in-Chief, he has the power to direct military operations and to determine
military strategy. Normally, he would be expected to delegate the actual command of the
armed forces to military experts; but the ultimate power is his. As Commander-in-Chief,
he is authorized to direct the movements of the naval and military forces placed by law at
his command, and to employ them in the manner he may deem most effectual.
Same; Philippine National Police; Regarding the country’s police force, Section 6, Article
XVI of the Constitution states that: the State shall establish and maintain one police force,
which shall be national in scope and civilian in character, to be administered and
controlled by a national police commission; A local chief executive, such as the provincial
governor, exercises operational supervision over the police, and may exercise control only
in day-to-day operations.—In addition to being the commander-in-chief of the armed
forces, the President also acts as the leader of the country’s police forces, under the
mandate of Section 17, Article VII of the Constitution, which provides that, “The President
shall have control of all the executive departments, bureaus, and offices. He shall ensure
that the laws be faithfully executed.” During the deliberations of the Constitutional
Commission on the framing of this provision, Fr. Bernas defended the retention of the
word “control,” employing the same rationale of singularity of the office of the president,
as the only Executive under the presidential form of government. Regarding the country’s
police force, Section 6, Article XVI of the Constitution states that: “The State shall
establish and maintain one police force, which shall be national in scope and civilian in
character, to be administered and controlled by a national police commission. The
authority of local executives over the police units in their jurisdiction shall be provided by
law.” A local chief executive, such as the provincial governor, exercises operational
supervision over the police, and may exercise control only in day-to-day operations
Same; Calling-Out Powers; Respondent provincial governor is not endowed with the
power to call upon the armed forces at his own bidding; the calling-out powers
contemplated under the Constitution is exclusive to the President.—Respondent
provincial governor is not endowed with the power to call upon the armed forces at his
own bidding. In issuing the assailed proclamation, Governor Tan exceeded his authority
when he declared a state of emergency and called upon the Armed Forces, the police,
and his own Civilian Emergency Force. The calling-out powers contemplated under the
176
EXECUTIVE DEPARTMENT
Constitution is exclusive to the President. An exercise by another official, even if he is the
local chief executive, is ultra vires, and may not be justified by the invocation of Section
465 of the Local Government Code.
Same; A kidnapping incident cannot be considered as a calamity or a disaster.—
Respondents cannot rely on paragraph 1, subparagraph (vii) of Article 465 above, as the
said provision expressly refers to calamities and disasters, whether man-made or natural.
The governor, as local chief executive of the province, is certainly empowered to enact
and implement emergency measures during these occurrences. But the kidnapping
incident in the case at bar cannot be considered as a calamity or a disaster. Respondents
cannot find any legal mooring under this provision to justify their actions.
Local Government Units; The intent behind the powers granted to local government units
is fiscal, economic, and administrative in nature.—The Local Government Code does not
involve the diminution of central powers inherently vested in the National Government,
especially not the prerogatives solely granted by the Constitution to the President in
matters of security and defense. The intent behind the powers granted to local
government units is fiscal, economic, and administrative in nature. The Code is concerned
only with powers that would make the delivery of basic services more effective to the
constituents, and should not be unduly stretched to confer calling-out powers on local
executives.
Philippine National Police; Private Armies; Pursuant to the national policy to establish one
police force, the organization of private citizen armies is proscribed.—Pursuant to the
national policy to establish one police force, the organization of private citizen armies is
proscribed. Section 24 of Article XVIII of the Constitution mandates that: Private armies
and other armed groups not recognized by duly constituted authority shall be dismantled.
All paramilitary forces including Civilian Home Defense Forces (CHDF) not consistent with
the citizen armed force established in this Constitution, shall be dissolved or, where
appropriate, converted into the regular force. Additionally, Section 21 of Article XI states
that, “The preservation of peace and order within the regions shall be the responsibility
of the local police agencies which shall be organized, maintained, supervised, and utilized
in accordance with applicable laws. The defense and security of the regions shall be the
responsibility of the National Government.”
The calling-out powers contemplated under the Constitution is exclusive to the President.
An exercise by another offcial, even if he is the local chief executive, is ultra vires, and
may not be justifed by the invocation of Section 465 of the Local Government Code.

Three members from the International Committee of the Red Cross (ICRC) were
kidnapped in the vicinity of the Provincial Capitol in Patikul, Sulu. Andres Notter, Eugenio
Vagni, and Marie Jean Lacaba, were purportedly inspecting a water sanitation project
for the Sulu Provincial Jail when they were seized by three armed men who were
177
EXECUTIVE DEPARTMENT
later confrmed to be members of the Abu Sayyaf Group (ASG). A Local Crisis Committee,
later renamed Sulu Crisis Management Committee (Committee) was then formed to
investigate the kidnapping incident. The Committee convened under the leadership of
respondent Abdusakur Mahail Tan, the Provincial Governor of Sulu.
Governor Tan issued Proclamation No. 1, Series of 2009, declaring a state of emergency
in the province of Sulu. The Proclamation cited the kidnapping incident as a ground
for the said declaration, describing it as a terrorist act pursuant to the Human Security Act
(R.A. 9372). It also invoked Section 465 of the Local Government Code of 1991 (R.A.
7160), which bestows on the Provincial Governor the power to carry out emergency
measures during man-made and natural disasters and calamities, and to call upon the
appropriate national law enforcement agencies to suppress disorder and lawless violence.
In the Proclamation, Tan called upon the PNP and the Civilian Emergency Force (CEF)
to set up checkpoints and chokepoints, conduct general search and seizures
including arrests, and other actions necessary to ensure public safety.
Petitioners, Jamar Kulayan, et al. claimed that Proclamation No. 1-09 was issued ultra
vires, and thus null and void, for violating Sections 1 and 18, Article VII of the
Constitution, which grants the President sole authority to exercise emergency
powers and calling-out powers as the chief executive of the Republic and commander-in-
chief of the armed forces.
ISSUE:
Whether or not a governor can exercise the calling-out powers of a President
HELD:
It has already been established that there is one repository of executive powers, and that
is the President of the Republic. This means that when Section 1, Article VII of the
Constitution speaks of executive power, it is granted to the President and no one else.
Corollarily, it is only the President, as Executive, who is authorized to exercise emergency
powers as provided under Section 23, Article VI, of the Constitution, as well as what
became known as the calling-out powers under Section 7, Article VII thereof.
While the President is still a civilian, Article II, Section 3 of the Constitution
mandates that civilian authority is, at all times, supreme over the military, making
the civilian president the nation’s supreme military leader. The net effect of Article II,
Section 3, when read with Article VII, Section 18, is that a civilian President is the
ceremonial, legal and administrative head of the armed forces. The Constitution does not
require that the President must be possessed of military training and talents, but as
Commander-in-Chief, he has the power to direct military operations and to determine
military strategy. Normally, he would be expected to delegate the actual command of
the armed forces to military experts; but the ultimate power is his.

178
EXECUTIVE DEPARTMENT
Given the foregoing, Governor Tan is not endowed with the power to call upon the armed
forces at his own bidding. In issuing the assailed proclamation, Governor Tan exceeded
his authority when he declared a state of emergency and called upon the Armed Forces,
the police, and his own Civilian Emergency Force. The calling-out powers contemplated
under the Constitution is exclusive to the President. An exercise by another offcial,
even if he is the local chief executive, is ultra vires, and may not be justifed by the
invocation of Section 465 of the Local Government Code.

DECLARATION OF STATE OF REBELLION


38. LACSON v. PEREZ
Constitution Law; Arrest; The instant petitions have been rendered moot and academic.—
All the foregoing petitions assail the declaration of a state of rebellion by President Gloria
Macapagal-Arroyo and the warrantless arrests allegedly effected by virtue thereof, as
having no basis both in fact and in law. Significantly, on May 6, 2001, President
MacapagalArroyo ordered the lifting of the declaration of a “state of rebellion” in Metro
Manila. Accordingly, the instant petitions have been rendered moot and academic.
Same; Same; The warrantless arrest feared by petitioners is not based on the declaration
of a “state of rebellion.”—In quelling or suppressing the rebellion, the authorities may
only resort to warrantless arrests of persons suspected of rebellion, as provided under
Section 5, Rule 113 of the Rules of Court, if the circumstances so warrant. The warrantless
arrest feared by petitioners is, thus, not based on the declaration of a “state of rebellion.”
Same; Same; Resort to the extraordinary remedies of mandamus and prohibition not
justified since an individual subjected to warrantless arrest is not without adequate
remedies in the ordinary course of law.—Moreover, petitioners’ contention in G.R. No.
147780 (Lacson Petition), 147781 (Defensor-Santiago Petition), and 147799 (Lumbao
Petition) that they are under imminent danger of being arrested without warrant do not
justify their resort to the extraordinary remedies of mandamus and prohibition, since an
individual subjected to warrantless arrest is not without adequate remedies in the
ordinary course of law. Such an individual may ask for a preliminary investigation under
Rule 112 of the Rules of Court, where he may adduce evidence in his defense, or he may
submit himself to inquest proceedings to determine whether or not he should remain
under custody and correspondingly be charged in court. Further, a person subject of a
warrantless arrest must be delivered to the proper judicial authorities within the periods
provided in Article 125 of the Revised Penal Code, otherwise the arresting officer could be
held liable for delay in the delivery of detained persons. Should the detention be without
legal ground, the person arrested can charge the arresting officer with arbitrary
detention. All this is without prejudice to his filing an action for damages against the
arresting officer under Article 32 of the Civil Code. Verily, petitioners have a surfeit of
other remedies which they can avail themselves of, thereby making the prayer for

179
EXECUTIVE DEPARTMENT
prohibition and mandamus improper at this time (Sections 2 and 3, Rule 65, Rules of
Court).
Same; Same; Court cannot enjoin criminal prosecution conducted in accordance with the
Rules of Court for by that time any arrest would have been in pursuance of a duly issued
warrant.—In connection with their alleged impending warrantless arrest, petitioners
Lacson, Aquino, and Mancao pray that the “appropriate court before whom the
informations against petitioners are filed be directed to desist from arraigning and
proceeding with the trial of the case, until the instant petition is finally resolved.” This
relief is clearly premature considering that as of this date, no complaints or charges have
been filed against any of the petitioners for any crime. And in the event that the same are
later filed, this Court cannot enjoin criminal prosecution conducted in accordance with
the Rules of Court, for by that time any arrest would have been in pursuance of a duly
issued warrant.
Same; Same; Mandamus will not issue unless the right to relief is clear at the time of the
award.—The petition herein is denominated by petitioner Defensor-Santiago as one for
mandamus. It is basic in matters relating to petitions for mandamus that the legal right of
the petitioner to the performance of a particular act which is sought to be compelled
must be clear and complete. Mandamus will not issue unless the right to relief is clear at
the time of the award (Palileo v. Ruiz Castro, 85 Phil. 272). Up to the present time,
petitioner Defensor-Santiago has not shown that she is in imminent danger of being
arrested without a warrant. In point of fact, the authorities have categorically stated that
petitioner will not be arrested without a warrant.
Remedial Law; Action; Party; Every action must be brought in the name of the party
whose legal right has been invaded or infringed, or whose legal right is under imminent
threat of invasion or infringement.—Petitioner Laban ng Demokratikong Pilipino is not a
real party-in-interest. The rule requires that a party must show a personal stake in the
outcome of the case or an injury to himself that can be redressed by a favorable decision
so as to warrant an invocation of the court’s jurisdiction and to justify the exercise of the
court’s remedial powers in his behalf (KMU Labor Center v. Garcia, Jr., 239 SCRA 386
[1994]). Here, petitioner has not demonstrated any injury to itself which would justify
resort to the Court. Petitioner is a juridical person not subject to arrest. Thus, it cannot
claim to be threatened by a warrantless arrest. Nor is it alleged that its leaders, members,
and supporters are being threatened with warrantless arrest and detention for the crime
of rebellion. Every action must be brought in the name of the party whose legal right has
been invaded or infringed, or whose legal right is under imminent threat of invasion or
infringement.
Facts: President Macapagal-Arroyo declared a State of Rebellion (Proclamation No. 38) on
May 1, 2001 as well as General Order No. 1 ordering the AFP and the PNP to suppress the
rebellion in the NCR. Warrantless arrests of several alleged leaders and promoters of the

180
EXECUTIVE DEPARTMENT
“rebellion” were thereafter effected. Petitioner filed for prohibition, injunction, mandamus
and habeas corpus with an application for the issuance of temporary restraining order
and/or writ of preliminary injunction. Petitioners assail the declaration of Proc. No. 38 and
the warrantless arrests allegedly effected by virtue thereof. Petitioners furthermore pray
that the appropriate court, wherein the information against them were filed, would desist
arraignment and trial until this instant petition is resolved. They also contend that they are
allegedly faced with impending warrantless arrests and unlawful restraint being that hold
departure orders were issued against them.

Issue: Whether or Not Proclamation No. 38 is valid, along with the warrantless arrests and
hold departure orders allegedly effected by the same.

Held: President Macapagal-Arroyo ordered the lifting of Proc. No. 38 on May 6, 2006,
accordingly the instant petition has been rendered moot and academic. Respondents have
declared that the Justice Department and the police authorities intend to obtain regular
warrants of arrests from the courts for all acts committed prior to and until May 1, 2001.
Under Section 5, Rule 113 of the Rules of Court, authorities may only resort to warrantless
arrests of persons suspected of rebellion in suppressing the rebellion if the circumstances
so warrant, thus the warrantless arrests are not based on Proc. No. 38. Petitioner’s prayer
for mandamus and prohibition is improper at this time because an individual warrantlessly
arrested has adequate remedies in law: Rule 112 of the Rules of Court, providing for
preliminary investigation, Article 125 of the Revised Penal Code, providing for the period in
which a warrantlessly arrested person must be delivered to the proper judicial authorities,
otherwise the officer responsible for such may be penalized for the delay of the same. If
the detention should have no legal ground, the arresting officer can be charged with
arbitrary detention, not prejudicial to claim of damages under Article 32 of the Civil Code.
Petitioners were neither assailing the validity of the subject hold departure orders, nor
were they expressing any intention to leave the country in the near future. To declare the
hold departure orders null and void ab initio must be made in the proper proceedings
initiated for that purpose. Petitioners’ prayer for relief regarding their alleged impending
warrantless arrests is premature being that no complaints have been filed against them for
any crime, furthermore, the writ of habeas corpus is uncalled for since its purpose is to
relieve unlawful restraint which Petitioners are not subjected to.

Petition is dismissed. Respondents, consistent and congruent with their undertaking earlier
adverted to, together with their agents, representatives, and all persons acting in their
behalf, are hereby enjoined from arresting Petitioners without the required judicial
warrants for all acts committed in relation to or in connection with the May 1, 2001 siege
of Malacañang.
181
EXECUTIVE DEPARTMENT
FACTS:

On May 1, 2001, President Macapagal-Arroyo, faced by an “angry and violent mob armed
with explosives, firearms, bladed weapons, clubs, stones and other deadly weapons”
assaulting and attempting to break into Malacañang, issued Proclamation No. 38
declaring that there was a state of rebellion in the National Capital Region. She likewise
issued General Order No. 1 directing the Armed Forces of the Philippines and the
Philippine National Police to suppress the rebellion in the National Capital Region.
Warrantless arrests of several alleged leaders and promoters of the “rebellion” were
thereafter effected.

Aggrieved by the warrantless arrests, and the declaration of a “state of rebellion,” which
allegedly gave a semblance of legality to the arrests, the following four related petitions
were filed before the Court. Prior to resolution, the “state of rebellion” was lifted in
Metro Manila.

ISSUE:

o Whether or not the declaration of a state of rebellion is constitutional

RULING:

As to warrantless arrests

As to petitioner’s claim that the proclamation of a “state of rebellion” is being used by the
authorities to justify warrantless arrests, the Secretary of Justice denies that it has issued
a particular order to arrest specific persons in connection with the “rebellion.” xxx

With this declaration, petitioners’ apprehensions as to warrantless arrests should be laid


to rest.

In quelling or suppressing the rebellion, the authorities may only resort to warrantless
arrests of persons suspected of rebellion, as provided under Section 5, Rule 113 of the
Rules of Court, if the circumstances so warrant. The warrantless arrest feared by
petitioners is, thus, not based on the declaration of a “state of rebellion.”

Was there violation of doctrine of separation of powers?

182
EXECUTIVE DEPARTMENT
Petitioner Lumbao, leader of the People’s Movement against Poverty (PMAP), for his part,
argues that the declaration of a “state of rebellion” is violative of the doctrine of
separation of powers, being an encroachment on the domain of the judiciary which has
the constitutional prerogative to “determine or interpret” what took place on May 1,
2001, and that the declaration of a state of rebellion cannot be an exception to the
general rule on the allocation of the governmental powers.

We disagree. To be sure, section 18, Article VII of the Constitution expressly provides that
“[t]he President shall be the Commander-in-Chief of all armed forces of the Philippines
and whenever it becomes necessary, he may call out such armed forces to prevent or
suppress lawless violence, invasion or rebellion…” thus, we held in Integrated Bar of the
Philippines v. Hon. Zamora, (G.R. No. 141284, August 15, 2000):

xxx The factual necessity of calling out the armed forces is not easily quantifiable and
cannot be objectively established since matters considered for satisfying the same is a
combination of several factors which are not always accessible to the courts. Besides the
absence of testual standards that the court may use to judge necessity, information
necessary to arrive at such judgment might also prove unmanageable for the courts.
Certain pertinent information necessary to arrive at such judgment might also prove
unmanageable for the courts. Certain pertinent information might be difficult to verify, or
wholly unavailable to the courts. In many instances, the evidence upon which the
President might decide that there is a need to call out the armed forces may be of a
nature not constituting technical proof.

On the other hand, the President as Commander-in-Chief has a vast intelligence network
to gather information, some of which may be classified as highly confidential or affecting
the security of the state. In the exercise of the power to call, on-the-spot decisions may
be imperatively necessary in emergency situations to avert great loss of human lives and
mass destruction of property. xxx

The Court, in a proper case, may look into the sufficiency of the factual basis of the
exercise of this power. However, this is no longer feasible at this time, Proclamation No.
38 having been lifted.
FACTS:
On May 1, 2001, President Macapagal-Arroyo, faced by an angry mob assaulting
and attempting to break into Malacañang, issued Proclamation No. 38 declaring that
there was a state of rebellion in the National Capital Region. She likewise issued General
Order No. 1 directing the Armed Forces of the Philippines and the Philippine National

183
EXECUTIVE DEPARTMENT
Police to suppress the rebellion in the National Capital Region. Warrantless arrests of
several alleged leaders and promoters of the “rebellion” were thereafter effected.

Aggrieved by the warrantless arrests, and the declaration of a “state of rebellion,”


which allegedly gave a semblance of legality to the arrests, four related petitions were
filed before the Court assailing the declaration of a state of rebellion by the President and
the warrantless arrests allegedly effected by virtue thereof, as having no basis both in fact
and in law.

1. On May 6, 2001, the President ordered the lifting of the declaration of a “state of
rebellion” in Metro Manila. Accordingly, the instant petitions have been rendered moot
and academic.

2. As to petitioners’ claim that the proclamation of a “state of rebellion” is being used by


the authorities to justify warrantless arrests, there are actually general instructions to law
enforcement officers and military agencies to implement Proclamation No. 38 and obtain
regular warrants of arrests from the courts. This means that preliminary investigations
will be conducted.

3. Moreover, petitioners’ contention that they are under imminent danger of being
arrested without warrant do not justify their resort to the extraordinary remedies of
mandamus and prohibition, since an individual subjected to warrantless arrest is not
without adequate remedies in the ordinary course of law.

4. Petitioners cannot ask the Court to direct the courts before whom the informations
against the petitioners are filed to desist from arraigning and proceeding with the trial of
the case. Such relief is clearly premature considering that as of this date, no complaints or
charges have been filed against any of the petitioners for any crime.

5. Hold departure orders issued against petitioners cannot be declared null and void since
petitioners are not directly assailing the validity of the subject hold departure orders in
their petition.

6. Petitioner Defensor-Santiago has not shown that she is in imminent danger of being
arrested without a warrant. Hence, her petition of mandamus cannot be issued since such
right to relief must be clear at the time of the award.

184
EXECUTIVE DEPARTMENT
7. Petitioner Lumbao, leader of the People’s Movement against Poverty (PMAP), argues
that the declaration of a “state of rebellion” is violative of the doctrine of separation of
powers, being an encroachment on the domain of the judiciary to interpret what took
place on May 1. The Court disagreed since the President as the Commander-in-Chief of all
armed forces of the Philippines, may call out such armed forces to prevent or suppress
lawless violence.

8. As for petitioner Laban ng Demokratikong Pilipino (LDP), it is not a real party-in-interest.


LDP has not demonstrated any injury to itself which would justify resort to the Court.
Petitioner is a juridical person not subject to arrest. Thus, it cannot claim to be threatened
by a warrantless arrest. Nor is it alleged that its leaders, members and supporters are
being threatened with warrantless arrest and detention for the crime of rebellion.

Even if instant petition may be considered as an action for declaratory relief, the Supreme
Court does not have jurisdiction in the first instance over such a petition.

PETITIONS DISMISSED (However, petitioners cannot be arrested without the required


judicial warrant for all acts committed in relation to or in connection with the May 1,
2001 siege)

DECLARATION OF MARTIAL LAW


39. FORTUN v. ARROYO
Constitutional Law; Executive Department; Congress; Martial Law; Writ of Habeas Corpus;
It is evident that under the 1986 Constitution the President and the Congress act in
tandem in exercising the power to proclaim martial law or suspend the privilege of the
writ of habeas corpus. They exercise the power, not only sequentially, but in a sense
jointly.—The pertinent provisions of Section 18, Article VII of the 1987 Constitution state:
Sec. 18. The President shall be the Commander‑in‑Chief of all armed forces of the
Philippines and whenever it becomes necessary, he may call out such armed forces to
prevent or suppress lawless violence, invasion or rebellion. In case of invasion or
rebellion, when the public safety requires it, he may, for a period not exceeding sixty
days, suspend the privilege of the writ of habeas corpus or place the Philippines or any
part thereof under martial law. Within forty‑eight hours from the proclamation of martial
law or the suspension of the privilege of writ of habeas corpus, the President shall submit
a report in person or in writing to the Congress. The Congress, voting jointly, by a vote of
at least a majority of all its Members in regular or special session, may revoke such
proclamation or suspension, which revocation shall not be set aside by the President.
185
EXECUTIVE DEPARTMENT
Upon the initiative of the President, the Congress may, in the same manner, extend such
proclamation or suspension for a period to be determined by the Congress, if the invasion
or rebellion shall persist and public safety requires it. The Congress, if not in session, shall,
within twenty‑four hours following such proclamation or suspension, convene in
accordance with its rules without any need of a call. x x x x Although the above vests in
the President the power to proclaim martial law or suspend the privilege of the writ of
habeas corpus, he shares such power with the Congress. Thus: 1. The President’s
proclamation or suspension is temporary, good for only 60 days; 2. He must, within 48
hours of the proclamation or suspension, report his action in person or in writing to
Congress; 3. Both houses of Congress, if not in session must jointly convene within 24
hours of the proclamation or suspension for the purpose of reviewing its validity; and 4.
The Congress, voting jointly, may revoke or affirm the President’s proclamation or
suspension, allow their limited effectivity to lapse, or extend the same if Congress deems
warranted. It is evident that under the 1987 Constitution the President and the Congress
act in tandem in exercising the power to proclaim martial law or suspend the privilege of
the writ of habeas corpus. They exercise the power, not only sequentially, but in a sense
jointly since, after the President has initiated the proclamation or the suspension, only
the Congress can maintain the same based on its own evaluation of the situation on the
ground, a power that the President does not have.
Same; Courts; Judicial Review; The Court’s duty is to steer clear of declaring
unconstitutional the acts of the Executive or the Legislative department, given the
assumption that it carefully studied those acts and found them consistent with the
fundamental law before taking them.—The Court does not resolve purely academic
questions to satisfy scholarly interest, however intellectually challenging these are. This is
especially true, said the Court in Philippine Association of Colleges and Universities v.
Secretary of Education, where the issues “reach constitutional dimensions, for then there
comes into play regard for the court’s duty to avoid decision of constitutional issues
unless avoidance becomes evasion.” The Court’s duty is to steer clear of declaring
unconstitutional the acts of the Executive or the Legislative department, given the
assumption that it carefully studied those acts and found them consistent with the
fundamental law before taking them. “To doubt is to sustain.”
Same; Same; Executive Department; Congress; Martial Law; Writ of Habeas Corpus; If the
Congress procrastinates or altogether fails to fulfill its duty respecting the proclamation or
suspension within the short time expected of it, then the Court can step in, hear the
petitions challenging the President’s action, and ascertain if it has a factual basis.—If the
Congress procrastinates or altogether fails to fulfill its duty respecting the proclamation or
suspension within the short time expected of it, then the Court can step in, hear the
petitions challenging the President’s action, and ascertain if it has a factual basis. If the
Court finds none, then it can annul the proclamation or the suspension. But what if the 30
days given it by the Constitution proves inadequate? Justice Carpio himself offers the
186
EXECUTIVE DEPARTMENT
answer in his dissent: that 30-day period does not operate to divest this Court of its
jurisdiction over the case. The settled rule is that jurisdiction once acquired is not lost
until the case has been terminated.

Fortun v. Macapagal-Arroyo

Petition: Consolidated petitions for the writs of certiorari and prohibition Challenging the
Constitutionality of PD No. 1959
Petitioner: Philip Sigfrid A. Fortun et al
Respondent: Gloria Macapagal-Arroyo
Ponente: J. Abad (As in the father of Ma’am Abad-Gamo, caution when reciting nalang
siguro )

DOCTRINE: Power of Supreme Court to inquire into the factual basis for the declaration of
martial law and the suspension of the privilege of the writ of habeas corpus by the
President

FACTS:

November 23, 2009 heavily armed men, believed led by the ruling Ampatuan family,
gunned down and buried under shoveled dirt 57 innocent civilians on a highway in
Maguindanao.

Following these incidents of which the facts of such we all know very well, President
Arroyo issued the following Presidential Decrees, herein presented sequentially:

1. Proclamation 1946 - declaring a state of emergency in Maguindanao, Sultan


Kudarat, and Cotabato City to prevent and suppress similar lawless violence in
Central Mindanao; issued November 4, 2009.

2. Presidential Proclamation 1959 (the PD mainly assailed in this case) - declaring


martial law and suspending the privilege of the writ of habeas corpus in that
province except for identified areas of the Moro Islamic Liberation Front; issued
December 4, 2009.

Two days later or on December 6, 2009 President Arroyo submitted her report to
Congress in accordance with Section 18, Article VII of the 1987. Constitution which
required her, within 48 hours from the proclamation of martial law or the suspension of
187
EXECUTIVE DEPARTMENT
the privilege of the writ of habeas corpus, to submit to that body a report in person or in
writing of her action.

In her report, PGMA said that her actions were based on her finding lawless men have
taken up arms in Maguindanao and risen against the government. The President
described the scope of the uprising, the nature, quantity, and quality of the rebels’
weaponry, the movement of their heavily armed units in strategic positions, the closure
of the Maguindanao Provincial Capitol, Ampatuan Municipal Hall, Datu Unsay Municipal
Hall, and 14 other municipal halls, and the use of armored vehicles, tanks, and patrol cars
with unauthorized “PNP/Police” markings.

On December 9, 2009 Congress, in joint session, convened pursuant to Section 18,


Article VII of the 1987 Constitution to review the validity of the President’s action.

But, two days later or on December 12, 2009 before Congress could act, the President
issued Presidential Proclamation 1963.

3. Presidential Proclamation 1963 - lifting martial law and restoring the privilege
of the writ of habeas corpus in Maguindanao; issued December 12, 2009.

ISSUES:
WoN there is a need for the Court to review the sufficiency of the factual basis of the
proclamation of martial law or the suspension of the privilege of the writ of habeas
corpus in this case, considering the same were lifted within a few days of being issued and
thus Congress was not able to affirm/maintain the same based on its own evaluation.

PROVISION:

Article 7, Section 18 of the 1987 Constitution.

The President shall be the Commander-in-Chief of all armed forces of the Philippines and
whenever it becomes necessary, he may call out such armed forces to prevent or
suppress lawless violence, invasion or rebellion. In case of invasion or rebellion, when the
public safety requires it, he may, for a period not exceeding sixty days, suspend the
privilege of the writ of habeas corpus or place the Philippines or any part thereof under
martial law. Within forty-eight hours from the proclamation of martial law or the
suspension of the privilege of the writ of habeas corpus, the President shall submit a
report in person or in writing to the Congress. The Congress, voting jointly, by a vote of
at least a majority of all its Members in regular or special session, may revoke such
188
EXECUTIVE DEPARTMENT
proclamation or suspension, which revocation shall not be set aside by the President.
Upon the initiative of the President, the Congress may, in the same manner, extend such
proclamation or suspension for a period to be determined by the Congress, if the invasion
or rebellion shall persist and public safety requires it.

The Congress, if not in session, shall, within twenty-four hours following such
proclamation or suspension, convene in accordance with its rules without need of a call.

The Supreme Court may review, in an appropriate proceeding filed by any citizen, the
sufficiency of the factual basis of the proclamation of martial law or the suspension of
the privilege of the writ of habeas corpus or the extension thereof, and must
promulgate its decision thereon within thirty days from its filing.

A state of martial law does not suspend the operation of the Constitution, nor supplant
the functioning of the civil courts or legislative assemblies, nor authorize the conferment
of jurisdiction on military courts and agencies over civilians where civil courts are able to
function, nor automatically suspend the privilege of the writ of habeas corpus.

The suspension of the privilege of the writ of habeas corpus shall apply only to persons
judicially charged for rebellion or offenses inherent in, or directly connected with,
invasion.

During the suspension of the privilege of the writ of habeas corpus, any person thus
arrested or detained shall be judicially charged within three days, otherwise he shall be
released.

RULING + RATIO:

No. It is evident that under the 1987 Constitution the President and the Congress act in
tandem in exercising the power to proclaim martial law or suspend the privilege of the
writ of habeas corpus.

They exercise the power, not only sequentially, but in a sense jointly since, after the
President has initiated the proclamation or the suspension, only the Congress can
maintain the same based on its own evaluation of the situation on the ground, a power
that the President does not have.

Consequently, although the Constitution reserves to the Supreme Court the power to
review the sufficiency of the factual basis of the proclamation or suspension in a proper

189
EXECUTIVE DEPARTMENT
suit, it is implicit that the Court must allow Congress to exercise its own review powers,
which is automatic rather than initiated.

Only when Congress defaults in its express duty to defend the Constitution through
such review should the Supreme Court step in as its final rampart. The Court then can
hear the petitions challenging the President’s action, and ascertain if it has a factual basis.
If the Court finds none, then it can annul the proclamation or the suspension.

The problem in this case is that the President aborted the proclamation of martial law and
the suspension of the privilege of the writ of habeas corpus in Maguindanao in just eight
days. In a real sense, the proclamation and the suspension never took off. The Congress
itself adjourned without touching the matter, it having become moot and academic. The
Court does not resolve purely academic questions to satisfy scholarly interest, however
intellectually challenging these are.

Additional Note:
Should the 30 days given by the Constitution prove to be inadequate, it is important to
note that the 30-day period does not operate to divest this Court of its jurisdiction over
the case. The settled rule is that jurisdiction once acquired is not lost until the case has
been terminated.

DISPOSITION: Petitions dismissed for being moot and academic.

Carpio Dissenting:
The President has the sole power to declare martial law or suspend the writ. This power
of the President is subject to review separately by Congress and the Supreme Court.
Justice Mendoza stresses, “Thus, Congress and this Court have separate spheres of
competence. They do not act ‘jointly and sequentially’ but independently of each other.
Father Bernas points out, “Since the powers of Congress and the Court are independent
of each other, there is nothing to prevent the Congress and the Court from
simultaneously exercising their separate powers.”

SUSPENSION OF THE PRIVILEGE OF THE WRIT OF HABEAS CORPUS


40. BARCELON v. BAKER
1.WRIT OF HABEAS CORPUS, SUSPENSION OF THE PRIVILEGE OF THE SAME BY THE
EXECUTIVE AM) LEGlSLATIVE DEPARTMENTS OF THE GOVERNMENT.—The privilege of the
writ of habeas corpus may be suspended in the Philippine Islands in the case of rebellion,
190
EXECUTIVE DEPARTMENT
insurrection, and invasion, when the public safety requires it, by the President of the
United States, or by the Governor-General of the Philippine Islands with the approval of
the Philippine Commission.
2.ID.; ID.—A paragraph of section 5 of the act of Congress of July 1, 1002, confers upon
the President, or the Governor-General and the Philippine Commission discretionary
power to determine (a) whether or not there exists the status of rebellion, insurrection,
or invasion; (b) whether the public safety requires the suspension of the privilege of the
writ of habeas corpus.
3.ID.; ID.—When the President, or the Governor-General with the approval of the
Philippine Commission, declares that a state of rebeltion, insurrection, or invasion exists,
this declaration or conclusion is conclusive against the judicial department of the
Government.
4.ID.; ID.—Whenever a statute gives discretionary power to any person, to be exercised
by him upon his own opinion of certain facts, such statute constitutes him the sole and
exclusive judge of the existence of those facts.
5.ID.; ID.—After the President, or the Governor-General with the approval of the
Philippine Commission, declares that a state of rebellion, insurrection, or invasion exists,
that condition will be considered by the judicial department of the Government as
continuing until the President or the Governor-General shall declare it to be at an end.

This case was an application for a writ of habeas corpus which it alleged that Barcelon is
detained and restrained of his liberty at the town of Batangas, in the Province of
Batangas, and that the detention and restraint of the said applicant is is wholly without
legal authority and not under or by virtue of any process issued by any court.
Respondents admit that they are detaining the body of the said Felix Barcelon, but deny
the right of the court to inquire into the reasons therefor by virtue of the resolution
issued by the Philippine Commission and the executive order of the Governor-General
suspending the privilege of the writ of habeas corpus in the Provinces of Cavite and
Batangas. The Philippine Bill section 5 provides that the Governor-General is hereby
authorized to suspend writ of habeas corpus in the said provinces because of the fact that
certain organized bands of ladrones in said provinces were in open insurrection against
the constituted authorities; and the said bands, or parts of them, and some of their
leaders, were still in open resistance to the constituted authorities.

ISSUE: Whether or not the judicial department of the Government may investigate the
facts upon which the legislative and executive branches of the Government acted in
providing for the suspension of the privilege of the writ of habeas corpus in the province
of Cavite and Batangas

191
EXECUTIVE DEPARTMENT
HELD: NO. It is the duty of the legislative branch of the Government to make such laws
and regulations as will effectually conserve peace and good order and protect the lives
and property of the citizens of the State. It is the duty of the Governor-General to take
such steps as he deems wise and necessary for the purpose of enforcing such laws. If the
judicial department of the Government, or any officer in the Government, has a right to
contest the orders of the President or of the Governor-General under the conditions
above supposed, before complying with such orders, then the hands of the President or
the Governor-General may be tied until the very object of the rebels or insurrectos or
invaders has been accomplished.IN THIS CASE, Congress had authority to provide that the
President, or the Governor-General, with the approval of the Philippine Commission,
might suspend the privilege of the writ of habeas corpus in cases of rebellion,
insurrection, or invasion, when the public safety might require it. The conclusion set forth
in the said resolution and the said executive order, as to the fact that there existed in the
Provinces of Cavite and Batangas open insurrection against the constituted authorities,
was a conclusion entirely within the discretion of the legislative and executive branches of
the Government, after an investigation of the facts. That one branch of the United States
Government in the Philippine Islands has no right to interfere or inquire into, for the
purpose of nullifying the same, the discretionary acts of another independent department
of the Government. The doctrine that whenever the Constitution or a statute gives a
discretionary power to any person, to be exercised by him upon his own opinion of
certain facts, such person is to be considered the sole and exclusive judge of the existence
of those facts has been recognized in this case. The authority to suspend the privilege of
writ of habeas corpus is exclusively vested in the legislative and executive branches of the
government and their decision is final and conclusive upon the Judicial Department and
upon all persons. Therefore, the application for the writ of habeas corpus is denied.

E. SUSPENSION OF THE PRIVILEGE OF THE WRIT OF HABEAS CORPUS


BARCELON VS BAKER OF THE PC - Suspension of the Privilege of the Writ Habeas Corpus as
a Political Question being a Prerogative by the President
In the early 1900’s in Batangas, Barcelon was detained by orders of Baker. Barcelon’s
lawyers petitioned before the court for a writ of habeas corpus demanding Barcelon and
Thompson, one of his men, to explain why Barcelon was detained. They alleged that there
is no legal authority behind Barcelon’s arrest and it was w/o due process. The Atty-Gen
averred that Baker et al acted only pursuant to the Gov-Gen’s resolution in 1905 which
suspended the privilege of the writ of habeas corpus in Cavite and Batangas (Sec 5 of The
Philippine Bill). Barcelon argued that there is no rebellion or invasion or insurrection during
his arrest hence he should be set free.
ISSUE: Whether or not Barcelon was arrested w/ due process.

192
EXECUTIVE DEPARTMENT
HELD: The SC held that the issue is a political question. Only the president can determine
the existence of the grounds specified in the Constitution for the suspension o the privilege
o the writ of habeas corpus. This power is discretionary and therefore not justiciable. The
president has superior competence to assess the peace and order condition of the country.
Hence, the determination held by the president (GG) of the Philippines of the existence of
any of the grounds prescribed by the Constitution for the suspension of the privilege of the
writ of habeas corpus should be conclusive upon the courts. The justification was that the
president (GG), with all the intelligence sources available to him as commander-in-chief,
was in a better position than the SC to ascertain the real state of peace and order in the
country.
MONTENEGRO V CASTANEDA - Suspension of the Privilege of the Writ Habeas Corpus as a
Political Question being a Prerogative by the President
About five o’clock in the morning of October 18, 1950, Maximino Montenegro was arrested
with others at the Samanillo Bldg. Manila, by agents of the Military Intelligence Service of
the Armed Forces of the Philippines, for complicity with a communistic organization in the
commission of acts of rebellion, insurrection or sedition. So far as the record discloses, he is
still under arrest in the custody by respondents. On October 22, 1950, The President issued
Proclamation No. 210 suspending the privilege of the writ of habeas corpus. On October 21,
1950, Maximino’s father, the petitioner, submitted this application for a writ of habeas
corpus seeking the release of his son.
Opposing the writ, respondents admitted having the body of Maximino, but questioned
judicial authority to go further in the matter, invoking the above-mentioned proclamation.
Petitioner replied that such proclamation was void, and that, anyway, it did not apply to his
son, who had been arrested before its promulgation. Heeding the suspension order, the
court of first instance denied the release prayed for. Hence this appeal, founded mainly on
the petitioner’s propositions:.
(a) The proclamation is unconstitutional “because it partakes of bill of attainder, or an ex
post facto law; and unlawfully includes sedition which under the Constitution is not a
ground for suspension”;
(b) Supposing the proclamation is valid, no prima facie.
(c) “There is no state of invasion, insurrection or rebellion, or imminent danger thereof,”
the only situations permitting discontinuance of the writ of habeas corpus; showing was
made that the petitioner’s son was included within the terms thereof.
Montenegro then filed a petition for the writ of habeas corpus demanding the detainers to
bring his son’s body and explain his detention. Castaňeda et al argued that the court has no

193
EXECUTIVE DEPARTMENT
judicial authority over the matter invoking the PP and the previous ruling in Barcelon vs
Baker.
ISSUE: Whether or not Montenegro’s petition should be granted.
HELD: As ruled by the SC in the Barcelon case, Montenegro’s petition is likewise denied.
It is first argued that the proclamation is invalid because it “partakes” of a bill of attainder
or an ex post facto law, and violates the constitutional percept that no bill of attainder or ex
post facto law shall be passed. The argument is devoid of merit. The prohibition applies
only to statutes. U.S. vs. Gen. El., 80 Fed. Supp. 989; De Pass vs. Bidwell, 24 Fed., 615.2 A bill
of attainder is a legislative act which inflicts punishment without judicial trial. (16 C.J.S. p.
902; U.S. vs. Lovett (1946) 328 U.S. 303). Anyway, if, as we find, the stay of the writ was
ordered in accordance with the powers expressly vested in the President by the
Constitution, such order must be deemed an exception to the general prohibition against ex
post facto laws and bills of attainder — supposing there is a conflict between the
prohibition and the suspension.
And we agree with the Solicitor General that in the light of the views of the United States
Supreme Court thru, Marshall, Taney and Story quoted with approval in Barcelon vs.
Baker (5 Phil., 87, pp. 98 and 100) the authority to decide whenever the exigency has arisen
requiring the suspension belongs to the President and “his decision is final and conclusive”
upon the courts and upon all other persons.
But even supposing the President’s appraisal of the situation is merely prima facie, we see
that petitioner in this litigation has failed to overcome the presumption of correctness
which the judiciary accords to acts of the Executive and Legislative Departments of our
Government.
The constitutional authority of the President to suspend in case of imminent danger of
invasion, insurrection or rebellion under Article 7 may not correctly be placed in doubt.
LANSANG V GARCIA
Abandonment of the Doctrine Held in the Barcelon Case & the Montenegro Case
Due to the throwing of two hand grenades in a Liberal Party caucus in 1971 causing the
death of 8 people, Marcos issued PP 889 which suspended the privilege of the writ of
habeas corpus. Marcos urged that there is a need to curtail the growth of Maoist groups.
Subsequently, Lansang et al were invited by the PC headed by Garcia for interrogation and
investigation. Lansang et al questioned the validity of the suspension of the writ averring
that the suspension does not meet the constitutional requisites.
ISSUE: Whether or not the suspension is constitutional.

194
EXECUTIVE DEPARTMENT
HELD: The doctrine established in Barcelon and Montenegro was subsequently abandoned
in this case where the SC declared that it had the power to inquire into the factual basis of
the suspension of the privilege of the writ of habeas corpus by Marcos in Aug 1971 and to
annul the same if no legal ground could be established. Accordingly, hearings were
conducted to receive evidence on this matter, including two closed-door sessions in which
relevant classified information was divulged by the government to the members of the SC
and 3 selected lawyers of the petitioners. In the end, after satisfying itself that there was
actually a massive and systematic Communist-oriented campaign to overthrow the
government by force, as claimed by Marcos, the SC unanimously decided to uphold the
suspension of the privilege of the Writ of Habeas Corpus.
LANSANG VS. COURT OF APPEALS. 326 SCRA 259 25 FEBRUARY 2000
QUISIMBING, J.
FACTS:
Private respondents General Assembly of the Blind (GABI) were allegedly awarded a
“verbal contract of lease” in 1970 by the National Parks Development Committee (NPDC), a
government initiated civic body engaged in the development of national perks including
Rizal Park. No document or instrument appears on record to show the grantor of the verbal
license to private respondents to occupy a portion of the government park. They were
given office and library space as well as kiosks for selling food and drinks along TM Kalaw.
40 % of the profits derived from the kiosks were to remit to NPDC again without anything
shown on the record.
With the change of the Government after the EDSA Revolution, a new chairman of
the NPDC, Amado J. Lansang (herein petitioner), sought to clean up Rizal Park. Petitioner
terminated the so-called verbal agreement with GABI and demanded that the latter vacate
the premises and the kiosks it ran privately within the park. Notice was given March 5, 1988
and respondents were given until March 8 to vacate. The notice was signed by Jose Iglesias,
GABI president, allegedly to indicate his conformity to its contents. However, Iglesias, who
was totally blind, claims that he was deceived into signing the notice. GABI filed an action
for damages and injunction in the Regional Trial Court against petitioner. The trial court
issued a TRO and expired on March 28, 1988. The following day GABI was finally evicted by
NPDC.
RTC’s ruling: The case was dismissed ruling that the complaint was against the state which
could not be sued without its consent.
Court of Appeal’s ruling: Reversed the decision. The mere allegation that the government
official is being sued in his official capacity is not enough to protect such official from
liability for acts done without or in excess of his authority.
ISSUES:

195
EXECUTIVE DEPARTMENT
W/N respondent court erred in not holding that private respondents’ complaint against
petitioner, as chairman of NPDC, and his co-defendants in civil case no. 88-43887, is in
effect a suit against the state which cannot be sued without its consent.
W/N respondent court erred in not holding that petitioner’s act of terminating respondent
GABI’s concession is valid and done in the lawful performance of official duty.
HELD:
The rule does not apply where the public official is charged in his official capacity for acts
that are unlawful and injurious to the rights of others. Public officials are not exempt, in
their personal capacity, from liability arising from acts committed in bad faith. There is no
question in the capacity of the petitioner as NPDC chairman and his authority to terminate
the agreement. The question now is whether or not the petitioner abused his authority in
ordering the ejectment of the private respondents.
However, no evidence of such abuse of authority is on record. Rizal part is beyond the
commerce of man, thus could not be subject to lease of contract. Private respondents
cannot and does nit claim a vested right to continue to occupy Rizal Park.
However, the petitioners cannot be awarded with moral and exemplary damages as well as
attorney’s fees. There is no evidence on record to support Iglesia’s claim that he suffered
moral injury as a result of GABI’s ejectment from Rizal Park. ]
WHEREFORE, the instant petition is GRANTED and the decision of the Court of Appeals is
set aside.
Aquino v. Enrile 59 SCRA 183
FACTS:
The cases are all petitions for habeas corpus, the petitioners having been arrested and
detained by the military by virtue of Proclamation 1081. The petitioners were arrested and
held pursuant to General Order No.2 of the President "for being participants or for having
given aid and comfort in the conspiracy to seize political and state power in the country and
to take over the Government by force..." General Order No. 2 was issued by the President
in the exercise of the power he assumed by virtue of Proclamation 1081 placing the entire
country under martial law.

ISSUES:
1) Is the existence of conditions claimed to justify the exercise of the power to declare
martial law subject to judicial inquiry?; and
2) Is the detention of the petitioners legal in accordance to the declaration of martial law?

HELD:
5 Justices held that the issue is a political question, hence, not subject to judicial inquiry,
196
EXECUTIVE DEPARTMENT
while 4 Justices held that the issue is a justiciable one. However, any inquiry by this Court in
the present cases into the constitutional sufficiency of the factual bases for the
proclamation of martial law has become moot and academic. Implicit in the state of martial
law is the suspension of the privilege of writ of habeas corpus with respect to persons
arrested or detained for acts related to the basic objective of the proclamation, which is to
suppress invasion, insurrection or rebellion, or to safeguard public safety against imminent
danger thereof. The preservation of society and national survival takes precedence. The
proclamation of martial law automatically suspends the privilege of the writ as to the
persons referred to in this case.
Aquino v. Enrile, 59 SCRA 183, September 17, 1974 (En Banc (all Justices wrote their
opinion)
Petitioners are: Ninoy, Mitra, F. Rordrigo, N. Rama, J. Roces, Locsin, Fadul, Galang, Go Eng
Guan, Maximo Soliven, Constantino, Luis Mauricio, Jose Diokno and wife, Carmen, Voltaire
Garcia (case were withdrawn bec. Petitioner died), Yuyitung, Tan Chin Hian, Doronila,
Mercado, Abaya, Granada, Beltran, Bren Guiao, Cusipag, Ordonez, Almario, Baun, Guiao
and T. Guiao (also died) and Rondon.
Respondents are: Enrile as Sec. Nat’l Defense, Espino as Chief of Staff AFP, Ramos as Chief,
Phil. Constabulary
FACTS: According to Chief Justice Makalintal:
These nine cases are all about the petitions for habeas corpus, the petitioners having been
arrested and detained unlawfully by the military by virtue of Proclamation no. 1081 dated
September 21, 1972 through the President exercising his powers he assumed by virtue of
Martial Law.
The petitioners were arrested pursuant to Gen. Order no 2 “for being participants or for
having giving aid and comfort in the conspiracy to seize political and state power in the
country and to take over the Government by force…” (September 22, 1972).
The provision of the 1935 constitution reads “the President shall be commander-in-chief of
all armed forces in the Philippines and, whenever it becomes necessary, he may call out such
armed forces to prevent or suppress lawless violence, invasion, insurrection or rebellion. In
case of invasion, insurrection, or rebellion, or imminent danger thereof, when the public
safety requires it, he may suspend the privilege of writ of habeas corpus or place the
Philippines or any part thereof under martial law.” Art VII Section 10(2)
Accdg to Castro, J.:
On Sept 21 1972, the country was placed under Martial Law. From Sept 22 to 30,
petitioners were arrested by the military authorities and detained, some at Fort Bonifacio,
Rizal, Camp Aguinaldo and Camp Crame. They aver that the arrest and detention were
197
EXECUTIVE DEPARTMENT
illegal having been effected without valid order of a court of justice. Writ of habeas corpus
were directed by the Court directing respondents to produce the bodies of the petitioners
in Court. Respondents, through the Solicitor General, answered that such arrests were
legally ordered by the President pursuant to Proclamation of Martial Law as “participant or
as having giving aid and comfort in the conspiracy to seize political and state power and to
take the government by force.” Hearings were held on 26 and 29 September and October 6.
Meanwhile, some of the petitioners, with leave of Court, withdrew their petitions, others
were released from custody under certain restrictive conditions. Voltaire died after his
release, the action was deemed abated.
Only Diokno AND Benigno Aquino was still in military custody (September 9, 1972—the
date of the supposed promulgation of the nine cases.) On September 11 1972, the
petitioner Diokno was released. Eleven members voted to dismiss Diokno’s petition as
being “moot and academic” except Castro, who find Diokno’s derogatory imputations grave
and highly insulting.
On August 23 1973, petitioner Ninoy filed an action for certiorari and prohibition with this
Court, alleging that on 11 August 1973 charges of murder, subversion and illegal possession
of firearm were filed against him, that his trial held on August 27, 29, 31 was illegal because
the proclamation of Martial law was unconstitutional and that he could not expect a fair
trial because the President could reverse any judgment of acquittal by the military court
and sentence him to death. “Benigno S. Aquino, Jr. vs. Military Commission No. 2”
On the other hand, December 28 1973, Jose Diokno filed a motion to withdraw his petition
filed in his behalf, imputing the (1) delay in the disposition of the case, (2) that the decision
of the Court in the Ratification cases contrary to the Court’s ruling that the 1973
Constitution was not validly ratified and (3) the action of the members of the Court taking
an oath to the new Constitution and which becomes a different court in which he filed his
petition.
Diokno asserts that “a conscience that allows man to rot behind bars for more than one
year and three months without trial—of course, without any charges at all—is a conscience
that has become stunted, if not stultified..” and “… I can not continue to entrust my case to
them; and I have become thoroughly convinced that our quest for justice in my case is
futile.”
Issue(s):
Whether or not this court may inquire into the validity of Proclamation no 1081. Is the
existence of conditions claimed to justify the exercise of power to declare martial law
subject to judicial inquiry? Is the question political or justiciable in character?
Ruling:

198
EXECUTIVE DEPARTMENT
YES. Five justices held that the question is political and should not be determined by court.
(Makasiar, Antonio, Esguerra, Fernandez and Aquino) Fernandez adds that as a member of
the 1973 Convention he believes that “the as a member of the Convention, they have put
an imprimatur on the proposition of the validity of a martial law proclamation…” Barredo
believes that political question are not per se beyond the court’s jurisdiction, judicial power
vested in it by the Constitution being all-embracing and plenary but as a matter of policy
should abstain from interfering with the Executive’s Proclamation. Esguerra finds that the
declaration of martial law is final and conclusive upon the courts. Antonio finds that there is
no dispute as to the existence of a state of rebellion and on that premise emphasizes the
factor of necessity for the exercise of the president under the 1935 Constitution to declare
martial law.
Four on the side of justiciability: Castro, Fernando, Teehanke and Munoz Palma. The
constitutional sufficiency may be inquired into by court and would thus apply the principle
laid down by Lansang although the case refers to the power of President to suspend habeas
corpus. The recognition of justiciability in Lansang is there distinguished from the power of
judicial review and is limited to ascertaining whether the President has gone beyond the
constitutional limits of his jurisdiction, not to exercise the power vested in him or to
determine the wisdom of the act. The Test is whether in suspending the writ of habeas
corpus, the president he did or did not acted arbitrarily (bias, capricious). Applying the test,
the Justices find no arbitrariness in the President’s proclamation of martial law pursuant to
the 1935 Constitution. The bases for the suspension of the privilege of writ of habeas
corpus, with regards to the existence of a state rebellion in the country, had not
disappeared but had even worsened.
The question of the validity of the Proclamation no 1081 has been foreclosed by the
transitory provision of the 1973 Constitution (Art XVII. Sec 3 (2)) that “all proclamations,
orders, decrees, instructions, and acts promulgated, issued or done by the incumbent
President shall be part of the law of the land and shall remain valid, legal, binding and
effective even after … the ratification of this Constitution.”
The political or justiciable question controversy has become moot and purposeless as a
consequence of the referendum of July 27-28, 1973. The question which was
overwhelmingly voted upon by a majority of voters, even between 15 and 18 years of age
in affirmative: “Under the 1973 Constitution, the President, if he so desires, can continue in
office beyond 1973 and finish the reforms he initiated under martial law?”
Whether or not the petitions for writ of habeas corpus should be suspended contending
that the proclamation of Martial Law was unconstitutional.
YES. The petitions should be dismissed with respect to petitioners who have been released
from detention but have not withdrawn their petitions because they are still subject to
certain restrictions. Implicit in the state of martial law is the suspension of the privilege of
199
EXECUTIVE DEPARTMENT
writ of habeas corpus with respect to persons arrested or detained for acts related to the
basic objective of the proclamation: to suppress invasion, insurrection, rebellion or to
safeguard public safety against imminent danger thereof.
RULING SUMMARIZED (Castro):
That the proclamation of Martial Law in September 1972 by the President was within the
1932 Constitution
That because the Communist rebellion had not been abated and instead the subversion had
proliferated throughout the country, the imposition of martial law was an “imperative of
national survival.”
that the arrest and detention of the persons who were “participants or gave aid and
comfort in the conspiracy to seize political and state power in the country and to take over
the Government by force” were not unconstitutional nor arbitrary
that subsumed in the declaration of martial law is the suspension of the writ of habeas
corpus
that the fact that the regular courts are open cannot be accepted as evidence that rebellion
and insurrection no longer imperil the safety of the state
that actual armed combat has been and still raging in parts of Mindanao, Bicol and Cagayan
that the hosts of doubts with respect to the validity of the ratification and effectivity of the
1973 Constitution has been dispelled by the national referendum of July 1973
that the issue of the validity and constitutionality of the arrest and detention of all the
petitioners and of the restrictions imposed upon those who were freed, is now foreclosed
by the transitory provision of 1973 CONSTITUTION (Article XVII Sec 3 (2)) which validates all
acts made by the President.
**Habeas Corpus- the purpose of the writ is to inquire into the cause or reason why a
person is being restrained of his liberty against his will and if there is no legal and/or valid
justification shown for such restraint the writ will forthwith issue to restore to that person
his liberty or freedom.
***The complete provision of Proclamaccion numero 1081, page 343-359…
GARCIA-PADILLA V ENRILE
Reversal of the Lansang Doctrine & Reinstatement of the Montenegro Doctrine
In July 1982, Sabino Padilla, together w/ 8 others who were having a conference in a house
in Bayombong, NV, were arrested by members of the PC. The raid of the house was
authorized by a search warrant issued by Judge Sayo. Josefina, mother of Sabino, opposed
the arrest averring that no warrant of arrest was issued but rather it was just a warrant of
200
EXECUTIVE DEPARTMENT
arrest hence the arrest of her son and the others was w/o just cause. Sabino and
companions together with 4 others were later transferred to a facility only the PCs know.
Josefina petitioned the court for the issuance of the writ of habeas corpus.
ISSUE: Whether or not the arrests done against Sabino et al is valid.
HELD: In a complete about face, the SC decision in the Lansang Case was reversed and the
ruling in the Barcelon Case & the Montenegro Case was again reinstated. The questioned
power of the president to suspend the privilege of the WoHC was once again held as
discretionary in the president. The SC again reiterated that the suspension of the writ was a
political question to be resolved solely by the president. It was also noted that the
suspension of the privilege of the writ of habeas corpus must, indeed, carry with it the
suspension of the right to bail, if the government’s campaign to suppress the rebellion is to
be enhanced and rendered effective. If the right to bail may be demanded during the
continuance of the rebellion, and those arrested, captured and detained in the course
thereof will be released, they would, without the least doubt, rejoin their comrades in the
field thereby jeopardizing the success of government efforts to bring to an end the
invasion, rebellion or insurrection.
NOTE: This ruling was abrogated by Sec 18, Art 7 of the 1987 Constitution which expressly
constitutionalized the Lansang Doctrine. Note as well that under Art 3 (Sec 13) of the
Constitution it is stated that “the right to bail shall not be impaired even if the privilege of
the writ of habeas corpus is suspended.”
6. PARDONING POWER
PEOPLE V SALLE
Facts:
On November 1991, Francisco Salle, Jr. and Ricky Mengote were convicted of the
compound crime of murder and destructive arson before the RTC of Quezon City. Salle and
Mengote filed their Notice of Appeal which was accepted by the Supreme Court on March
24, 1993.
In 1994, Salle filed an Urgent Motion to Withdraw Appeal. The Court required Salle's
counsel, Atty. Ida May La'o of the Free Legal Assistance Group (FLAG) to verify the
voluntariness of the motion.
Atty. La'o manifested that Salle signed the motion without the assistance of counsel on his
misimpression that the motion was necessary for his early release from the New Bilibid
Prison following the grant of a conditional pardon by the President on December 9, 1993.
She also stated that Mengote was also granted conditional pardon and that he immediately
left for his province without consulting her. She prayed that the Court grant Salle's motion
to withdraw hisappeal.
201
EXECUTIVE DEPARTMENT
On March 23, 1994, the Court granted Salle's motion.
After taking into consideration Section 19, Article VII of the Constitution which provides
that the President may, except in cases of impeachment or as otherwise provided in the
Constitution, grant pardon after conviction by final judgment, the Court required (1) the
Solicitor General and the counsel for accused-appellants to submit their memoranda on the
issue of the enforceability of the conditional pardon and (2) the Presidential Committee for
the Grant of Bail, Release or Pardon to inform the Court why it recommended to the
President the grant of the conditional pardon despite the pendency of the appeal.
In its Memorandum, the Office of the Solicitor General maintains that
theconditional pardon granted to appellant Mengote is unenforceable because the
judgment of conviction is not yet final in view of the pendency in this Court of his appeal.
On the other hand, the FLAG, through Atty. La'o, submits that the conditionalpardon
extended to Mengote is valid and enforceable. Citing Monsanto vs. Factoran, Jr., it argues
that although Mengote did not file a motion to withdraw the appeal, he was deemed to
have abandoned the appeal by his acceptance of the conditional pardon which resulted in
the finality of his conviction.
Issue: Whether or not a pardon granted to an accused during the pendency of
hisappeal from a judgment of conviction by the trial court is enforceable.
Held: Section 19, Article VII thereof reads as follows:
“Except in cases of impeachment, or as otherwise provided in this Constitution, the
President may grant reprieves, commutations, and pardons, and remit fines and
forfeitures, after conviction by final judgment.
He shall also have the power to grant amnesty with the concurrence of a majority of all the
Members of the Congress.”
Where the pardoning power is subject to the limitation of conviction, it may be exercised at
any time after conviction even if the judgment is on appeal. It is, of course, entirely
different where the requirement is " final conviction, " as was mandated in the original
provision of Section 14, Article IX of the 1973 Constitution, or "conviction by final
judgment," as presently prescribed in Section 19, Article VII of the 1987 Constitution. In
such a case, no pardon may be extended before a judgment of conviction becomes final.
A judgment of conviction becomes final (a) when no appeal is seasonably perfected, (b)
when the accused commences to serve the sentence, (c) when the right to appeal is
expressly waived in writing, except where the death penalty was imposed by the trial court,
and (d) when the accused applies for probation, thereby waiving his right to appeal. Where
the judgment of conviction is still pending appeal and has not yet therefore attained
finality, as in the instant case, executive clemency may not yet be granted to the appellant.
202
EXECUTIVE DEPARTMENT
The "conviction by final judgment" limitation under Section 19, Article VII of the present
Constitution prohibits the grant of pardon, whether full or conditional, to an accused during
the pendency of his appeal from his conviction by the trial court. Any application therefor, if
one is made, should not be acted upon or the process toward its grant should not be begun
unless the appeal is withdrawn. Accordingly, the agencies or instrumentalities of the
Government concerned must require proof from the accused that he has not appealed
from his conviction or that he has withdrawn his appeal. Such proof may be in the formof a
certification issued by the trial court or the appellate court, as the case may be.
The acceptance of the pardon shall not operate as an abandonment or waiver of
the appeal, and the release of an accused by virtue of a pardon, commutation of sentence,
or parole before the withdrawal of an appeal shall render those responsible therefor
administratively liable. Accordingly, those in custody of the accused must not solely rely on
the pardon as a basis for the release of the accused from confinement.
WHEREFORE, counsel for accused-appellant Ricky Mengote y Cuntado is hereby given thirty
(30) days from notice hereof within which to secure from the latter the withdrawal of
his appeal and to submit it to this Court. The conditionalpardon granted the
said appellant shall be deemed to take effect only upon the grant of such withdrawal. In
case of non-compliance with this Resolution, the Director of the Bureau of Corrections must
exert every possible effort to take back into his custody the said appellant, for which
purpose he may seek the assistance of the Philippine National Police or the National Bureau
of Investigation. (People vs. Francisco Salle, Jr. and Ricky Mengote, G.R. No. 103567,
December 4, 1995)
SALVACION A. MONSANTO vs FACTORAN, February 9, 1989 (G.R. No. 78239).
FACTS:
• In a decision by the Sandiganbayan convicted petitioner Salvacion A. Monsanto was
accused of the crime of estafa thru falsification of public documents and sentenced them to
imprisonment and to indemnify the government in the sum of P4,892.50 representing the
balance of the amount defrauded and to pay the costs proportionately.
• She was given an absolute pardon by President Marcos which she accepted.
• Petitioner requested that she be restored to her former post as assistant city treasurer
since the same was still vacant, she also asked for the backpay for the entire period of her
suspension.
• Finance Ministry ruled that petitioner may be reinstated to her position without the
necessity of a new appointment
• The Office of the President said that that acquittal, not absolute pardon, of a former
public officer is the only ground for reinstatement to his former position and entitlement to
payment of his salaries, benefits and emoluments due to him during the period of his
suspension pendente lite.
• In fact, in such a situation, the former public official must secure a reappointment before
203
EXECUTIVE DEPARTMENT
he can reassume his former position. And a pardon shall in no case exempt the culprit from
payment of the civil indemnity imposed upon him by the sentence.
• Petitioner argued that general rules on pardon cannot apply to her case by reason of the
fact that she was extended executive clemency while her conviction was still pending
appeal in this Court. There having been no final judgment of conviction, her employment
therefore as assistant city treasurer could not be said to have been terminated or forfeited.
• The court viewed that is not material when the pardon was bestowed, whether before or
after conviction, for the result would still be the same
ISSUE:
(1) Effects of a full and absolute pardon
(2) WON a public officer, who has been granted an absolute pardon by the Chief Executive,
is entitled to reinstatement to her former position without need of a new appointment.
HELD:
(1) A pardon reaches both the punishment prescribed for the offense and the guilt of the
offender; and when the pardon is full, it releases the punishment and blots out of existence
the guilt, so that in the eye of the law the offender is as innocent as if he had never
committed the offense. If granted before conviction, it prevents any of the penalties and
disabilities, consequent upon conviction, from attaching; if granted after conviction, it
removes the penalties and disabilities and restores him to all his civil rights; it makes him, as
it were, a new man, and gives him a new credit and capacity. But unless expressly grounded
on the person’s innocence (which is rare), it cannot bring back lost reputation for honesty,
integrity and fair dealing.
A pardon looks to the future. It is not retrospective. It makes no amends for the past. It
affords no relief for what has been suffered by the offender. It does not impose upon the
government any obligation to make reparation for what has been suffered.
(2) No. To insist on automatic reinstatement because of a mistaken notion that the pardon
virtually acquitted one from the offense of estafa would be grossly untenable. A pardon,
albeit full and plenary, cannot preclude the appointing power from refusing appointment to
anyone deemed to be of bad character, a poor moral risk, or who is unsuitable by reason of
the pardoned conviction.
The absolute disqualification or ineligibility from public office forms part of the punishment
prescribed by the Revised Penal Code for estafa thru falsification of public documents.
The pardon granted to petitioner has resulted in removing her disqualification from holding
public employment but it cannot go beyond that. To regain her former post as assistant city
treasurer, she must re-apply and undergo the usual procedure required for a new
appointment.
GARCIA VS COA CHAIRMAN
204
EXECUTIVE DEPARTMENT
FACTS:
Petitioner was a supervising lineman in the Region IV Station of the Bureau of
Telecommunications in Lucena City. A criminal case of qualified theft was filed against him.
The president grated him an executive clemency. The petitioner filed a claim for back
payment of salaries. The petitioner was later recalled to the service on 12 March 1984 but
the records do not show whether petitioner’s reinstatement was to the same position of
Supervising Lineman.

ISSUE: Whether Garcia is entitled to the payment of back wages after having been
reinstated pursuant to the grant of executive clemency.
HELD: The pardoned offender regains his eligibility for appointment to public office which
was forfeited by reason of the conviction of the offense. But since pardon does not
generally result in automatic reinstatement because the offender has to apply for
reappointment, he is not entitled to back wages.
If the pardon is based on the innocence of the individual, it affirms this innocence and
makes him a new man and as innocent; as if he had not been found guilty of the offense
charged. 7 When a person is given pardon because he did not truly commit the offense, the
pardon relieves the party from all punitive consequences of his criminal act, thereby
restoring to him his clean name, good reputation and unstained character prior to the
finding of guilt.
In the case at bar, the acquittal of petitioner by the trial court was founded not on lack of
proof beyond reasonable doubt but on the fact that petitioner did not commit the offense
imputed to him. Aside from finding him innocent of the charge, the trial court commended
petitioner for his concern and dedication as a public servant. Verily, petitioner’s innocence
is the primary reason behind the grant of executive clemency to him, bolstered by the
favorable recommendations for his reinstatement. This signifies that petitioner need no
longer apply to be reinstated to his former employment; he is restored to his office ipso
facto upon the issuance of the clemency.
Petitioner’s automatic reinstatement to the government service entitles him to back wages.
This is meant to afford relief to petitioner who is innocent from the start and to make
reparation for what he has suffered as a result of his unjust dismissal from the service. The
right to back wages is afforded to those with have been illegally dismissed and were thus
ordered reinstated or to those otherwise acquitted of the charges against them.
Therefore, the court ordered the full back wages from April 1 1975 (date when he was
illegally dismissed) to March 12 1984 (reinstated) to the petitioner.
2. PARDON VS PAROLE
TORRES v. GONZALES

205
EXECUTIVE DEPARTMENT
July 23, 1987 (G.R. No. 76872)
FACTS:
1978, Torres was convicted of estafa. In 1979, he was pardoned by the president w/ the
condition that he shall not violate any penal laws again. Should this condition be violated,
he will be proceeded against in the manner prescribed by law. Petitioner accepted the
conditional pardon and was consequently released from confinement. In 1982, Torres was
charged with multiple crimes of estafa. In 1986, Gonzales petitioned for the cancellation of
Torres’ pardon. Hence, the president cancelled the pardon. Torres appealed the issue
before the SC averring that the Exec Dep’t erred in convicting him for violating the
conditions of his pardon because the estafa charges against him were not yet final and
executory as they were still on appeal.

ISSUE: whether or not conviction of a crime by final judgment of a court is necessary before
the petitioner can be validly rearrested and recommitted for violation of the terms of his
conditional pardon and accordingly to serve the balance of his original sentence.
HELD: In proceeding against a convict who has been conditionally pardoned and who is
alleged to have breached the conditions of his pardon, the Executive Department has two
options: (1) Section 64 (i) of the Revised Administrative Code, a purely executive act, not
subject to judicial scrutiny, or (2) Article 159 of the Revised Penal Code, a judicial act
consisting of trial for and conviction of violation of a conditional pardon.
Where the President opts to proceed under Section 64 (i) of the Revised Administrative
Code, no judicial pronouncement of guilt of a subsequent crime is necessary, much less
conviction therefor by final judgment of a court, in order that a convict may be
recommended for the violation of his conditional pardon.
Under art. 159 of the RPC, parolee or convict who is regarded as having violated the
provisions thereof must be charged, prosecuted and convicted by final judgment before he
can be made to suffer the penalty prescribed.
In the case at bar, President has chosen to proceed against the petitioner under Section 64
(i) of the Revised Administrative Code. That choice is an exercise of the President’s
executive prerogative and is not subject to judicial scrutiny.
*Who determines if violated? The PRESIDENT. When the person was conditionally
pardoned it was a generous exercise by the Chief Executive of his constitutional
prerogative. The acceptance thereof by the convict or prisoner carrie[d] with it the
authority or power of the Executive to determine whether a condition or conditions of the
pardon has or have been violated. To no other department of the Government [has] such
power been intrusted.
TORRES VS GONZALES
206
EXECUTIVE DEPARTMENT
152 SCRA 272 – Political Law – Constitutional Law – Pardon – Not Subject to Judicial
Review/Scrutiny
In 1978, Wilfredo Torres was convicted of estafa. In 1979, he was pardoned by the
president with the condition that he shall not violate any penal laws again. In 1982, Torres
was charged with multiple crimes of estafa. In 1986, then Chairman of the Board of Paroles
Neptali Gonzales petitioned for the cancellation of Torres’ pardon. Hence, the president
cancelled the pardon. Torres appealed the issue before the Supreme Court averring that
the Executive Department erred in convicting him for violating the conditions of his pardon
because the estafa charges against him were not yet final and executory as they were still
on appeal.
ISSUE: Whether or not conviction of a crime by final judgment of a court is necessary
before Torres can be validly rearrested and recommitted for violation of the terms of his
conditional pardon and accordingly to serve the balance of his original sentence.
HELD: The SC affirmed the following:
1. The grant of pardon and the determination of the terms and conditions of a conditional
pardon are purely executive acts which are not subject to judicial scrutiny.
2. The determination of the occurrence of a breach of a condition of a pardon, and the
proper consequences of such breach, may be either a purely executive act, not subject to
judicial scrutiny under Section 64 (i) of the Revised Administrative Code; or it may be a
judicial act consisting of trial for and conviction of violation of a conditional pardon under
Article 159 of the Revised Penal Code. Where the President opts to proceed under Section
64 (i) of the Revised Administrative Code, no judicial pronouncement of guilt of a
subsequent crime is necessary, much less conviction therefor by final judgment of a court,
in order that a convict may be recommended for the violation of his conditional pardon.
3. Because due process is not semper et ubique judicial process, and because the
conditionally pardoned convict had already been accorded judicial due process in his trial
and conviction for the offense for which he was conditionally pardoned, Section 64 (i) of
the Revised Administrative Code is not afflicted with a constitutional vice.
In proceeding against a convict who has been conditionally pardoned and who is alleged to
have breached the conditions of his pardon, the Executive Department has two options: (i)
to proceed against him under Section 64 (i) of the Revised Administrative Code; or (ii) to
proceed against him under Article 159 of the RPC which imposes the penalty of prision
correccional, minimum period, upon a convict who “having been granted conditional
pardon by the Chief Executive, shall violate any of the conditions of such pardon.” Here, the
President has chosen to proceed against the petitioner under Section 64 (i) of the Revised
Administrative Code. That choice is an exercise of the President’s executive prerogative and
is not subject to judicial scrutiny.
207
EXECUTIVE DEPARTMENT
3. Pardon in Administrative Cases

LLAMAS VS ORBOS and OCAMPO


“Executive Clemency to Administrative Cases?”

Court has jurisdiction in this case since the issue is on the validity of the discretionary powers of the President which
may be subjected to judicial review.

President has power to grant executive clemency to administrative cases.

President not only has power to reverse a ruling of a subordinate against an erring public official but also has the power
to grant executive clemency to the same.

President can grant executive clemency to administrative case involving the executive department only!

Ponente: Justice Paras, 1991

FACTS:
Petitioner Llamas is the incumbent Vice-Governor; Respondent Ocampo is the incumbent Governor; Orbos is the
President’s Executive Secretary.

The Secretary of the Department of Local Government (DLG) rendered a decision holding Ocampo guilty of violating
the Anti-Graft and Corrupt Practices and penalizing the latter to 90 days suspension. Ocampo’s appeals to DLG and
Office of the President were denied. The denial on the latter, pursuant to the law, rendered the immediate execution
of the decision; thus, petitioner took his oath as acting governor. However, Ocampo issued an administrative order
showing his intention to hold office as his residence in the belief that the pendency of his motion for reconsideration
precludes the finality of the DLG decision.

Without ruling on Ocampo’s motion, Orbos issued a resolution granting executive clemency to the former; in effect,
reducing the the penalty imposed to him by period already served which is more than 60 days. By virtue of this
resolution, Ocampo reassumed the governorship of the province allegedly without any notification made to petitioner;
hence this instant petition of Llamas for review.

ISSUE/S:
Petitioner contends that the phrase “after conviction by final judgment” applies solely to criminal cases, and no other
law allows the grant of executive clemency or pardon to anyone who has been “convicted in an administrative case”.

W/N the court has jurisdiction in this case.

W/N the President has the power to grant executive clemency in administrative cases.

RULING:
The court denied this petition and held that the President did not act arbitrarily or with abuse on granting executive
clemency to Ocampo.

Court has jurisdiction in this case since the issue is on the validity of the discretionary powers of the President which
may be subjected to judicial review.
While courts cannot inquire into the manner in which the President’s discretionary powers are exercised or into the
wisdom for its exercise, it is also a settled rule that when the issue involved concerns the validity of such discretionary
powers or whether said powers are within the limits presented by the Constitution, the Court will not decline to
exercise the power of judicial review.
208
EXECUTIVE DEPARTMENT
President has power to grant executive clemency to administrative cases.
Since the President can grant reprieves, commutations and pardons and remit fines and forfeitures in criminal cases
with much more reason can s/he grant executive clemency in administrative cases.

President not only has power to reverse a ruling of a subordinate against an erring public official but also has the
power to grant executive clemency to the same.
Under the Doctrine of Qualified Political Agency, the different executive department are mere adjuncts of the
President. Their acts are presumptively the acts of the President until reprobated by him/her. The President exercises
supervision and control over all executive department and may substitute her decision for that of his/her subordinate.

President can grant executive clemency to administrative case involving the executive department only!

5. Effect
SABELLO VS. DECS

Ponente: Gancayco, 1989

FACTS:

Petitioner, was the Elementary School Principal of Talisay and also the Assistant Principal of the Talisay Barangay High
School of the Division of Gingoog City. The barangay high school was in deficit at that time due to the fact that the
students could hardly pay for their monthly tuition fees.

For this matter, petitioner was authorized to withdraw the aid given by the President to cover up the salaries of the
high school teachers, with the honest thought in mind that the barrio high school was a barrio project and as such
therefore, was entitled to its share of the RICD fund in question.

The only part that the herein petitioner played was his being authorized by the said barrio council to withdraw the
above amount and which was subsequently deposited in the City Treasurer's Office in the name of the Talisay Barrio
209
EXECUTIVE DEPARTMENT
High School. That was a grave error on the part of the herein petitioner as it involves the very intricacies in the
disbursement of government funds and of its technicalities. Thus, the herein petitioner was charged of the violation
of Republic Act 3019, and convicted to suffer a sentence of one year and disqualification to hold public office.

The petitioner appealed which the Court of Appeals modified the decision by eliminating the subsidiary imprisonment
in case of insolvency in the payment of one-half of the amount being involved. The herein petitioner, being financially
battered, could no longer hire a lawyer to proceed to the highest court of the land.

Finally, the herein petitioner was granted an ABSOLUTE PARDON by the President of the Republic of the Philippines,
restoring him to 'full civil and political rights.' With this instrument on hand, the herein petitioner applied for
reinstatement to the government service, only to be reinstated to the wrong position of a mere classroom teacher
and not to his former position as Elementary School Principal.

ISSUE:
WON petitioner merits reappointment to the position he held prior to his conviction.

HELD:
YES. As a general rule, the question of whether or not petitioner should be reappointed to his former position is a
matter of discretion of the appointing authority, but under the circumstances of this case, if the petitioner had been
unfairly deprived of what is rightfully his, the discretion is qualified by the requirements of giving justice to the
petitioner. It is no longer a matter of discretion on the part of the appointing power, but discretion tempered with
fairness and justice.

Taking into consideration that this petition is filed by a non-lawyer, who claims that poverty denies him the services
of a lawyer, We also set aside the requirement of exhaustion of administrative remedies and resolved to go direct to
the merits of the petition.

In Monsanto vs. Factoran, Jr., 3 this Court held that the absolute disqualification from office or ineligibility from public
office forms part of the punishment prescribed under the penal code and that pardon frees the individual from all the
penalties and legal disabilities and restores him to all his civil rights. Although such pardon restores his eligibility to a
public office it does not entitle him to automatic reinstatement. He should apply for reappointment to said office.

In the present case after his absolute pardon, petitioner was reinstated to the service as a classroom teacher by the
Department of Education, Culture and Sports.

As there are no circumstances that would warrant the diminution in his rank, justice and equity dictate that he be
returned to his former position of Elementary School Principal I and not to that of a mere classroom teacher.

210
EXECUTIVE DEPARTMENT
7. Diplomatic Power
DAMES AND MOORE VS. REGAN (US CASE)

Brief Fact Summary. The Supreme Court of the United States held that the President may nullify attachments and
order the transfer of frozen Iranian assets pursuant to Section 1702(a)(1) of the International Emergency Economic
Powers Act (“IEEPA”). Based on the Court’s inferences from legislation passed by Congress (IEEPA and the Hostage
Act) regarding the President’s authority to deal with international crises and from the history of congressional
acquiescence in executive claims settlement, the President may also suspend claims pursuant to the Executive Order.

Synopsis of Rule of Law. Where Congress has a history of acquiescence, as with claims settlement, it thereby implicitly
approves of the President’s actions regarding that specific subject matter about which Congress was silent.

Facts.

In response to the seizure of American personnel as hostages at the American Embassy in Iran, the President issued
various Executive Orders and regulations by which the President nullified attachments and liens on Iranian assets in
the United States, directed that theses assets be transferred to Iran, and suspended claims against Iran that may be
presented to an International Claims Tribunal. On December 19, 1979, Petitioner, Dames & Moore, filed suit in the
United Sates District Court against Defendants, the government of Iran, the Atomic Energy Organization of Iran, and
many Iranian banks, alleging that its subsidiary was a party to a contract with the Atomic Energy Organization and that
the subsidiary’s interest had been assigned to Petitioner. Petitioner alleged it was owed over 3 million dollars. The
District Court issued orders of attachment directed against the Defendants’ property and the property of certain
Iranian banks. In a January 20, 1981 Executive Agreement, the

President agreed to nullify attachments and ordered the transfer of frozen Iranian assets. On February 24, 1981, the
President ratified an earlier Order wherein he “suspended” all “claims which may be presented to the Tribunal” and
provided that such claims “shall have no legal effect in any action now pending in U.S. courts.”

Issue/s

Whether the President’s acts of “nullifying” the attachments and ordering the “transfer” of all frozen assets are
specifically authorized by Congress.

Whether the President has authority to suspend claims pending in American courts.

211
EXECUTIVE DEPARTMENT
Held:

Yes. Because the President’s actions in nullifying the attachments and ordering the transfer of assets were taken
pursuant to congressional authorization (Section 1702 (a)(1) of IEEPA), it is “supported by the strongest of
presumptions and widest latitude of judicial interpretation and the burden of persuasion rests heavily on any who
might attack it.”

Yes. Based on the legislation (IEEPA and the Hostage Act) which Congress has enacted in the area of the President’s
authority to deal with international crises, and from the history of congressional acquiescence in executive claims
settlement, the President was authorized to suspend claims pursuant to the Executive Order

Discussion. The majority resorts to drawing inferences from Congress’ legislation to conclude that the President has
authority to suspend claims in American Courts.

212
EXECUTIVE DEPARTMENT
PIMENTEL, JR. VS. AGUIRRE and BONCODIN

In administrative law, Supervision means the overseeing or authority of an officer to see that subordinate officers
perform their duties. If the latter fail to fulfil them, the former may take actions as prescribed by law to make them
perform their duties. On the other hand, Control means the authority of an officer to alter/modify/nullify/set aside
what a subordinate has done in the performance of his duties and to substitute the judgment of the former for that of
the latter. The former is the power of mere oversight over an inferior body; it does not include any restraining authority
over such body present in the latter.

Cabinet members and other executive officials are merely alter egos. As such, they are subject to the power of control
of the President; thus, their actions and decisions may be changed, suspended, or reversed by the latter. In contrast,
LGUs are elected by the people. Their sovereign powers emanate from the electorate, to whom they are directly
accountable. They are subject to the President’s supervision only, not control, so long as their acts are exercised within
the sphere of their legitimate powers. Thus, the President may not withhold or alter any authority or power given them
by the Constitution and the law.

Ponente: Justice Panganiban, 2000

FACTS:
The President issued AO 372: Section 1 directs Local Government Units (LGUs) to reduce their expenditure by 25
percent; and Section 4 withholds 10 percent of their internal revenue allotments; hence, petitioner Pimentel filed this
petition for certiorari and prohibition seeking to annul Sections 1 and 4 of the order.

Petitioner contends that the President, in issuing AO 372, was in effect exercising the power of control over LGUs
contrary to the power of general supervision granted by the Constitution; and that the directive to withhold 10 percent
of their IRA is also a contravention to the Constitution.

On the other hand, the Solicitor General, in behalf of the respondents, contends that AO 372 was issued to alleviate
the "economic difficulties brought about by the peso devaluation" and constituted merely an exercise of the
President's power of supervision over LGUs; likewise, withholding of 10 percent of the LGUs’ IRA does not violate the
statutory prohibition on the imposition of any lien or holdback on their revenue shares, because such withholding is
"temporary in nature”.

ISSUE/S:
W/N (a) Section 1 of AO 372, insofar as it "directs" LGUs to reduce their expenditures by 25 percent; and (b) Section 4
of the same issuance, which withholds 10 percent of their internal revenue allotments, are valid exercises of the
President's power of general supervision over local governments.

RULING:
The court granted this petition and ordered the permanent prohibition of AO 372 insofar as the local government units
are concerned.

Supervision vs. Control.


In administrative law, Supervision means the overseeing or authority of an officer to see that subordinate officers
perform their duties. If the latter fail to fulfil them, the former may take actions as prescribed by law to make them
perform their duties. On the other hand, Control means the authority of an officer to alter/modify/nullify/set aside
what a subordinate has done in the performance of his duties and to substitute the judgment of the former for that
of the latter. The former is the power of mere oversight over an inferior body; it does not include any restraining
authority over such body present in the latter.

Cabinet Members = Supervision; LGUs = Control.


Cabinet members and other executive officials are merely alter egos. As such, they are subject to the power of control
of the President; thus, their actions and decisions may be changed, suspended, or reversed by the latter. In contrast,
LGUs are elected by the people. Their sovereign powers emanate from the electorate, to whom they are directly
accountable. They are subject to the President’s supervision only, not control, so long as their acts are exercised within
213
EXECUTIVE DEPARTMENT
the sphere of their legitimate powers. Thus, the President may not withhold or alter any authority or power given
them by the Constitution and the law.

AO 372 is merely advisory in character but no legal sanctions may be imposed upon LGUs and their officials who do
not follow such advice.
The provision of AO 372 is merely an advisory to prevail upon local executives to recognize the need for fiscal restraint
in a period of economic difficulty. It is understood, however, that no legal sanction may be imposed upon LGUs and
their officials who do not follow such advice.

The withholding of 10% of the LGUs’ IRA pending assessment and evaluation although temporary is equivalent to a
holdback; thus, contravenes the Constitution.
A basic feature of fiscal autonomy is the automatic release of the shares of LGUs in the national internal revenue as
mandated by the Constitution. The Local Government Code specifies further that the release shall be made directly to
the LGU within five days after every quarter of the year and shall not subject to any lien or holdback that may be
imposed by the national government for whatever purpose.

a. Treaties vs. Executive Agreements

USAFFE Veterans Association, Inc. v. Treasurer of the Philippines, G.R. No. L-10500, [June 30, 1959]

Ponente: Bengzon, J.

1. UNITED STATES GRANTS; GRANT FOR THE OPERATION AND MAINTENANCE OF THE PHILIPPINE ARMY; UNEXPENDED
SUMS REFUNDABLE. — The United States Congressional Act of December 17, 1941 (Public Law No. 353), appropriating
$269-million, expressly provided that the amount "shall be available for payment to the Government of the
Commonwealth of the Philippines upon its written request either in advance of or in reimbursement for all or any part
of the estimated or actual costs" of operation, mobilization oand maintenance of the Philippine Army. Held: Ownership
of the money did not vest in the Philippine Government upon delivery thereof. In any systems of accounting, advances
of funds for expenditures contemplate disbursements to be reported, and credited if apporoved, against such
advances the unexpended sums to be returned later.

2. INTERNATIONAL LAW; EXECUTIVE AGREEMENTS; BINDING EVEN WITHOUT CONCURRENCE OF SENATE.--The Court
apparently holds that Executive agreements may be entered into with other states, and are effected even without the
concurrence of the Senate.
214
EXECUTIVE DEPARTMENT
3. ID.; ID.; ID.; NATURE OF EXECUTIVE AGREEMENTS. — Executive Agreements fall into two classes: (1) agreements
entered into may be termed as presidential agreements, and (2) agreements entered into purxuance of acts of
Congress, which have been designated as Congressional-Executive Agreements.

4. ID.; ID.; ID.; ID.; ROMULO-SNYDER AGREEMENT. — Romulo Snyder Agreement (1950) whereby the Philippine
Government undertook to return to the United States Government in ten annual installments, a total of about 35
million dollars advanced by the United States to, but unexpended by the National Defense Forces of the Philippines,
may fall under any of these two classes of executive agreements.

5. ID.; ID.; ID.; ID.; ID.; RATIFICATION OF THE AGREEMENT. — The acts of Congress appropriating funds for the yearly
installments necessary to comply with the Romulo-Snyder Agreement, constitute a ratification thereof.

FACTS:
• Romulo-Snyder Agreement (1950): RP Govt undertook to return to the US Govt in 10 annual installments, a total of
about $35M advanced by the US to, but unexpended by, the Nat’l Defense Forces of the RP.
• Oct 1954: The Usaffe Veterans Associations Inc prayed in its complaint before the CFI that said Agreement be
annulled; that payments thereunder be declared illegal; & that defendants as officers of RP be restrained from
disbursing any funds in the Nat’l Treasury in pursuance of said Agreement.
• Usaffe Veterans further asked that the moneys available, instead of being remitted to the US, should be turned over
to the Finance Service of the AFP for the payment of all pending claims of the veterans represented by plaintiff.
• 3 PROPOSITIONS OF PLAINTIFF IN COMPLAINT: 1) that the funds to be “returned” under the Agreement were funds
appropriated by the US Congress for the RP Army, actually delivered to the RP Govt & actually owned by the said
Government; 2) that U.S Secretary Snyder of the Treasury, had no authority to retake such funds from the RP Govt; 3)
The RP Foreign Secretary Carlos P. Romulo had no authority to return or promise to return the aforesaid sums of
money through the Agreement.
• The court eventually upheld the validity of the Agreement. Plaintiff appealed.
• July 26, 1941: Foreseeing the War in the Pacific, Pres Roosevelt, called into the service of the US Armed Forces, for
the duration of the emergency, all organized mil forces of the Commonwealth. (His order was published here by Proc
No 740 of Pres Quezon on Aug 10, 1941)
• October 1941: By 2 special orders, MacArthur, Commanding Gen of USAFFE, placed under his command all the RP
Army units including Phil Constabulary.
• Thus, US Congress provided in its Appropriation Act of Dec 17, 1941 (Public Law No. 353): “For all expenses necessary
for the mobilization, operation & maintenance of the RP Army, including expenses connected w/ calling into the
service RP mil forces…$269,000.00; to remain available ‘til June 30, 1943, w/c shall be available for payment to the
Commonwealth upon its written request, either in advance of or in reimbursement for all or any part of the estimated
or actual costs, as authorized by the USAFFE Commanding Gen, of the necessary expenses for the purposes aforesaid.”
• Pursuant to the power reserved to him under Public Law 353, Roosevelt issued EO 9011: “2(a) Necessary
expenditures from funds in the Phil Treasury for the purposes authorized by Public Law 353, will be made by disbursing
officers of the RP Army on the approval of authority of the Commandign General, USAFFE, & such purposes as he may
deem proper…”
• P570,863,000.00 was transferred directly to the AFP by means of vouchers w/c stated “Advance of Funds under
Public Law 353 & EO 9011” This was used mostly to discharge in RP the monetary obligations assumed by the US as a
result of the induction of the AFP into the US Army, & its operations beginning in 1941.
• There remained unexpended & uncommitted $35M in the possession of the AFP as of Dec 31, 1949. Bec the RP Govt
then badly needed funds, Pres Quirino, through CB Gov Cuaderno, proposed to US officials the retention of the $35M
as a loan, & for its repayment in 10 annual installments. This was the Romulo-Snyder Agreement, signed in Washington
on Nov 6, 1950 by RP Foreign Affairs Sec Romulo, & US Sec of Treasury, John Snyder.
• PRESENT ACTION: Usaffe’s arguments – 1) the money delivered by the US to the AFP were straight payments for mil
services; ownership thus vested in RP Govt upon delivery, & consequently, there was nothing to return, nothing to
consider as a loan; 2) the Agreement was void bec there was no loan to be repaid & bec it was not binding on the RP
Govt for lack of authority of the officers who concluded it.

ISSUES
• Basic issue: Validity of the Romulo-Snyder Agreement – Court can’t pass judgment
215
EXECUTIVE DEPARTMENT
1. WON there is obligation to repay - YES
2. WON the officers who promised to repay had authority to bind this Govt – YES

RATIO:

1. YES
• Note that the $269M appropriated in Public Law 353 (see 8th bullet) expressly said that the money is to be handed
to the RP Govt either in advance of or in reimbursement thereof.
• In any system of accounting, advances of funds for expenditures contemplate disbursements to be reported, &
credited if approved, against such advances, the unexpended sums to be returned later. Congressional law itself
required accounting “in the manner prescribed by US Pres - & said Pres in EO 9011, outlined the procedure whereby
advanced funds shall be accounted for.
• It also requires as a condition sine qua non that all expenditures shall first be approved by the USAFFE Commanding
Gen.
• These ideas of “funds advanced” to meet expenditures of the Phil Army as may be approved by the USAFFE Comm-
Gen, in connection w/ the accounting requirement, evidently contradict appellant’s thesis that the moneys
represented straight payments to RP Govt for its armed services, & passed into the absolute control of such Govt
• Instead of returning such amount into one lump sum, our Exec Dept arranged for its repayment in 10 annual
installments. Prima facie such arrangement should raise no valid objection, given the obligation to return.

2. YES (They have authority to bind Govt even w/o Senate concurrence)
• There is no doubt Pres Quirino approved the negotiations. And he had the power to contract budgetary loans under
RA 213, amending RA 16.
• The most impt argument, however, rests on the lack of ratification of the Agreement by RP Senate to make it binding
on the Govt.
• The ff explanation of the defendant was considered persuasive by the Court…
The agreement is not a ‘treaty’ as that term is used in CONSTI. However, a treaty is not the only form that an int’l
agreement may assume. For the grant of treatymaking power to the Executive & the Senate does not exhaust the
power of the govt over int’l relations.
Executive agreements may be entered into w/ other states & are effective even w/o concurrence of Senate.
In int’l law, there’s no difference bet’n treaties & executive agreements in their binding effect upon states concerned
as long as the negotiating functionaries have remained w/n their powers.
The distinction bet’n executive agreements & treaties is purely a const’l one & has not int’l legal significance.
Altman v. US: An int’l compact negotiated bet’n the reps of 2 sovereign nations & made in the name or behalf of the
contracting parties & dealing w/ impt commercial rel’ns bet’n the 2 countries, is a treaty both internationally although
as an executive agreement it is not technically a treaty requiring the advice & consent of the Senate
2 classes of Executive Agreements: 1) agreements made purely as executive acts affecting external relations &
independent of or w/o legislative authorization, w/c may be termed as pres’l agreements; 2) agreements entered into
in pursuance of acts of Cong, w/c have been designated as Congressional-Executive Agreements
The Romulo-Snyder Agreement may fall under any of these 2 classes for on Sept 18, 1946, RP Congress authorized
the RP Pres to obtain such loans or incur such indebtedness w/ the US.
Even granting there’s no leg authorization, the Agreement was legally & validly entered into to conform to the 2nd
category, namely, as agreements entered into purely as executive acts w/o leg authorization’, w/c usu includes money
agreements.
• The Agreement was not submitted to the US Senate either. But the Phil Senate’s Resolution No. 15 practically admits
the validity & binding force of such Agreement.
• Further, the acts of Congress appropriating funds for the yearly installments necessary to comply w/ such Agreement
constitute a ratification thereof, W/C PLACES THE QUESTION OF VALIDITY OUT OF THE COURT’S REACH, NO CONST’L
PRINCIPLE HAVING BEEN INVOKED TO RESTRICT CONGRESS’ PLENARY POEWR TO APPROPRIATE FUNDS – LOAN OR
NO LOAN.

HELD: Plaintiff failed to make a clear case for the relief demanded. Petition DENIED.

216
EXECUTIVE DEPARTMENT
Bayan v. Zamora, G.R. No. 138570, 138572, 138587, 138680, 138698, [October 10, 2000]

PONENTE: BUENA, J.

SYNOPSIS

The instant petitions for certiorari and prohibition assailed the agreement forged between the RP and the USA — THE
VISITING FORCES AGREEMENT, which formalized, among others, the use of installations in the Philippine territory by
the US military personnel to strengthen their defense and security relationship. On October 5, 1998, President Joseph
E. Estrada ratified the VFA, and then transmitted to the Senate his letter of ratification and the VFA for concurrence
pursuant to Section 21, Art. VII of the 1987 Constitution. The Senate subsequently approved the VFA by a 2/3 vote of
its members.

From these consolidated petitions, petitioners — as legislators, non-governmental organizations, citizens and
taxpayers — assailed the constitutionality of the VFA and imputed to respondents grave abuse of discretion in ratifying
the agreement.

In dismissing the petition, the Supreme Court held: that at the outset, petitioners have no locus standi to bring the suit
because they have not shown any interest in the case nor have they substantiated that they have sustained or will
sustain direct injury as a result of the operation of the VFA; that as taxpayers, they have not established that the VFA
involves the illegal disbursement of public funds raised by taxation; that whether the President referred the VFA to
the Senate and the latter extended its concurrence under Section 21 , Article VII, or Section 25, Article XVIII, is
immaterial, for in either case, the fundamental law is crystalline that the concurrence of the Senate is mandatory; that
with regard to the ratification by the President of the VFA and the exercise by the Senate of its constitutional power
to concur with the VFA, the Court, absent clear showing of grave abuse of discretion on the part of respondents, is
without power to meddle with such affairs purely executive and legislative in character and nature; and that with the
ratification of the VFA, which is equivalent to final acceptance and with the exchange of notes between the Philippines
and the USA, it now becomes obligatory, under the principles of international law, to be bound by the terms of the
agreement.

SYLLABUS

1. REMEDIAL LAW; ACTIONS; REQUIREMENTS TO ALLOW A SUIT CHALLENGING THE CONSTITUTIONALITY OF A LAW,
ACT OR STATUTE; PETITIONERS HAVE NO LEGAL STANDING TO ASSAIL THE LEGALITY OF THE VFA IN CASE AT BAR. —
A party bringing a suit challenging the constitutionality of a law, act, or statute must show "not only that the law is
invalid, but also that he has sustained or is in immediate, or imminent danger of sustaining some direct injury as a
result of its enforcement, and not merely that he suffers thereby in some indefinite way." He must show that he has
been, or is about to be, denied some right or privilege to which he is lawfully entitled, or that he is about to be
subjected to some burdens or penalties by reason of the statute complained of. In the case before us, petitioners
failed to show, to the satisfaction of this Court, that they have sustained, or are in danger of sustaining any direct injury
as a result of the enforcement of the VFA. As taxpayers, petitioners have not established that the VFA involves the
exercise by Congress of its taxing or spending powers. On this point, it bears stressing that a taxpayer's suit refers to a
case where the act complained of directly involves the illegal disbursement of public funds derived from taxation. . .
Clearly, inasmuch as no public funds raised by taxation are involved in this case, and in the absence of any allegation
by petitioners that public funds are being misspent or illegally expended, petitioners, as taxpayers, have no legal
standing to assail the legality of the VFA. Similarly, Representatives Wigberto Tañada, Agapito Aquino and Joker
Arroyo, as petitioners-legislators, do not possess the requisite locus standi to maintain the present suit. . . [T]he
allegations of impairment of legislative power, such as the delegation of the power of Congress to grant tax
exemptions, are more apparent than real. While it may be true that petitioners pointed to provisions of the VFA which
allegedly impair their legislative powers, petitioners failed however to sufficiently show that they have in fact suffered
direct injury.

217
EXECUTIVE DEPARTMENT
2. POLITICAL LAW; INTERNATIONAL LAW; PACTA SUNT SERVANDA; EFFECT OF RP'S RATIFICATION OF THE VFA AND
USA'S ACKNOWLEDGMENT OF THE VFA AS A TREATY; CASE AT BAR. — The records reveal that the United States
Government, through Ambassador Thomas C. Hubbard, has stated that the United States government has fully
committed to living up to the terms of the VFA. For as long as the United States of America accepts or acknowledges
the VFA as a treaty, and binds itself further to comply with its obligations under the treaty, there is indeed marked
compliance with the mandate of the Constitution. Worth stressing too, is that the ratification, by the President, of the
VFA and the concurrence of the Senate should be taken as a clear an unequivocal expression of our nation's consent
to be bound by said treaty, with the concomitant duty to uphold the obligations and responsibilities embodied
thereunder. With the ratification of the VFA, which is equivalent to final acceptance, and with the exchange of notes
between the Philippines and the United States of America, it now becomes obligatory and incumbent on our part,
under the principles of international law, to be bound by the terms of the agreement. Thus, no less than Section 2,
Article II of the Constitution, declares that the Philippines adopts the generally accepted principles of international law
as part of the law of the land and adheres to the policy of peace, equality, justice, freedom, cooperation and amity
with all nations. As a member of the family of nations, the Philippines agrees to be bound by generally accepted rules
for the conduct of its international relations. While the international obligation devolves upon the state and not upon
any particular branch, institution, or individual member of its government, the Philippines is nonetheless responsible
for violations committed by any branch or subdivision of its government or any official thereof. As an integral part of
the community of nations, we are responsible to assure that our government, Constitution and laws will carry out our
international obligation. . . Article 26 of the convention provides that "Every treaty in force is binding upon the parties
to it and must be performed by them in good faith." This is known as the principle of pacta sunt servanda which
preserves the sanctity of treaties and have been one of the most fundamental principles of positive international law,
supported by the jurisprudence of international tribunals.

3. CONSTITUTIONAL LAW; EXECUTIVE DEPARTMENT; POWER TO ENTER INTO TREATIES AND INTERNATIONAL
AGREEMENTS IS VESTED IN THE PRESIDENT; CASE AT BAR. — As regards the power to enter into treaties or
international agreements, the Constitution vests the same in the President, subject only to the concurrence of at least
two-thirds vote of all the members of the Senate. In this light, the negotiation of the VFA and the subsequent
ratification of the agreement are exclusive acts which pertain solely to the President, in the lawful exercise of his vast
executive and diplomatic powers granted him no less than by the fundamental law itself. Into the field of negotiation
the Senate cannot intrude, and Congress itself is powerless to invade it. Consequently, the acts or judgment calls of
the President involving the VFA — specifically the acts of ratification and entering into a treaty and those necessary or
incidental to the exercise of such principal acts — squarely fall within the sphere of his constitutional powers and thus,
may not be validly struck down, much less calibrated by this Court, in the absence of clear showing of grave abuse of
power or discretion.

4. REMEDIAL LAW; CERTIORARI; GRAVE ABUSE OF DISCRETION; ACT OF THE PRESIDENT IN SUBMITTING THE VFA TO
THE SENATE FOR CONCURRENCE UNDER SECTION 21 OF ARTICLE VII, INSTEAD OF SECTION 25 OF ARTICLE XVIII OF THE
CONSTITUTION, NOT A CASE OF. — It is the Court's considered view that the President, in ratifying the VFA and in
submitting the same to the Senate for concurrence, acted within the confines and limits of the powers vested in him
by the Constitution. It is of no moment that the President, in the exercise of his wide latitude of discretion and in the
honest belief that the VFA falls within the ambit of Section 21, Article VII of the Constitution, referred the VFA to the
Senate for concurrence under the aforementioned provision. Certainly, no abuse of discretion, much less a grave,
patent and whimsical abuse of judgment, may be imputed to the President in his act of ratifying the VFA and referring
the same to the Senate for the purpose of complying with the concurrence requirement embodied in the fundamental
law. In doing so, the President merely performed a constitutional task and exercised a prerogative that chiefly pertains
to the functions of his office. Even if he erred in submitting the VFA to the Senate for concurrence under the provisions
of Section 21 of Article VII, instead of Section 25 of Article XVIII of the Constitution, still, the President may not be
faulted or scarred, much less be adjudged guilty of committing an abuse of discretion in some patent, gross, and
capricious manner.

5. CONSTITUTIONAL LAW; LEGISLATIVE DEPARTMENT; TREATY-CONCURRING POWER OF THE SENATE PERTAINS TO


THE WISDOM OF AN ACT WHICH IS BEYOND THE PROVINCE OF THE COURTS TO INQUIRE. — As to the power to concur
with treaties, the constitution lodges the same with the Senate alone. Thus, once the Senate performs that power, or
218
EXECUTIVE DEPARTMENT
exercises its prerogative within the boundaries prescribed by the Constitution, the concurrence manner, be viewed to
constitute an abuse of power, much less grave abuse thereof. Corollarily, the Senate, in the exercise of its discretion
and acting within the limits of such power, may not be similarly faulted for having simply performed a task conferred
and sanctioned by no less than the fundamental law. For the role of the Senate in relation to treaties is essentially
legislative in character; the Senate, as an independent body possessed of its own erudite mind, has the prerogative to
either accept or reject the proposed agreement, and whatever action it takes in the exercise of its wide latitude of
discretion, pertains to the wisdom rather than the legality of the act. In this sense, the Senate partakes a principal, yet
delicate, role in keeping the principles of separation of powers and of checks and balances alive and vigilantly ensures
that these cherished rudiments remain true to their form in a democratic government such as ours. The Constitution
thus animates, through this treaty-concurring power of the Senate, a healthy system of checks and balances
indispensable toward our nation's pursuit of political maturity and growth. True enough, rudimentary is the principle
that matters pertaining to the wisdom of a legislative act are beyond the ambit and province of the courts to inquire.

Facts:

The Philippines and the United States entered into a Mutual Defense Treaty on August 30, 1951, To further
strengthen their defense and security relationship. Under the treaty, the parties agreed to respond to any external
armed attack on their territory, armed forces, public vessels, and aircraft.

On September 16, 1991, the Philippine Senate rejected the proposed RP-US Treaty of Friendship, Cooperation
and Security which, in effect, would have extended the presence of US military bases in the Philippines.

On July 18, 1997 RP and US exchanged notes and discussed, among other things, the possible elements of the
Visiting Forces Agreement (VFA).This resulted to a series of conferences and negotiations which culminated on January
12 and 13, 1998. Thereafter, President Fidel Ramos approved the VFA, which was respectively signed by Secretary
Siazon and United States Ambassador Thomas Hubbard.

On October 5, 1998, President Joseph E. Estrada, through respondent Secretary of Foreign Affairs, ratified the
VFA. On October 6, 1998, the President, acting through respondent Executive Secretary Ronaldo Zamora, officially
transmitted to the Senate of the Philippines,the Instrument of Ratification, the letter of the President and the VFA, for
concurrence pursuant to Section 21, Article VII of the 1987 Constitution.

Petitions for certiorari and prohibition, petitioners – as legislators, non-governmental organizations, citizens and
taxpayers – assail the constitutionality of the VFA and impute to herein respondents grave abuse of discretion in
ratifying the agreement.

Petitioner contends, under they provision cited, the “foreign military bases, troops, or facilities” may be allowed
in the Philippines unless the following conditions are sufficiently met: a) it must be a treaty,b) it must be duly concurred
in by the senate, ratified by a majority of the votes cast in a national referendum held for that purpose if so required
by congress, and c) recognized as such by the other contracting state.

Respondents, on the other hand, argue that Section 21 Article VII is applicable so that, what is requires for such
treaty to be valid and effective is the concurrence in by at least two-thirds of all the members of the senate.

Issue:

Is the VFA governed by the provisions of Section 21, Art VII or of Section 25, Article XVIII of the Constitution?

Ruling:

Section 25, Article XVIII, which specifically deals with treaties involving foreign military bases, troops or facilities
should apply in the instant case.

219
EXECUTIVE DEPARTMENT
The 1987 Philippine Constitution contains two provisions requiring the concurrence of the Senate on treaties or
international agreements. Sec. 21 Art. VII, which respondent invokes, reads: “No treaty or international agreement
shall be valid and effective unless concurred in by at least 2/3 of all the Members of the Senate. Sec. 25 Art. XVIII
provides : “After the expiration in 1991 of the Agreement between the RP and the US concerning Military Bases,
foreign military bases, troops or facilities shall not be allowed in the Philippines except under a treaty duly concurred
in and when the Congress so requires, ratified by a majority of votes cast by the people in a national referendum held
for that purpose, and recognized as a treaty by the Senate by the other contracting state”.

The first cited provision applies to any form of treaties and international agreements in general with a wide
variety of subject matter. All treaties and international agreements entered into by the Philippines, regardless of
subject matter, coverage or particular designation requires the concurrence of the Senate to be valid and effective.

In contrast, the second cited provision applies to treaties which involve presence of foreign military bases, troops
and facilities in the Philippines. Both constitutional provisions share some common ground. The fact that the President
referred the VFA to the Senate under Sec. 21 Art. VII, and that Senate extended its concurrence under the same
provision is immaterial.

220
EXECUTIVE DEPARTMENT
9. POWER TO IMPOUND

Philippine Constitution Association v. Enriquez, G.R. No. 113105, 113174, 113766, 113888, [August 19, 1994] [Article
VI Section 25 - Appropriations]

Impoundment refers to a refusal by the President, for whatever reason, to spend funds made available by Congress. It
is the failure to spend or obligate budget authority of any type.

PONENTE: QUIASON, J

FACTS:
Petitioners assailed the validity of RA 7663 or General Appropriations Act of 1994.
GAA contains a special provision that allows any members of the Congress the REalignment of Allocation for
Operational Expenses, provided that the total of said allocation is not exceeded.
Philconsa claims that only the Senate President and the Speaker of the House of Representatives are the ones
authorized under the Constitution to realign savings, not the individual members of Congress themselves.

President signed the law, but Vetoes certain provisions of the law and imposed certain provisional conditions: that the
AFP Chief of Staff is authorized to use savings to augment the pension funds under the Retirement and Separation
Benefits of the AFP.

ISSUE:
Whether or not RA 7663 is violative of Article VI, Section 25 (5) of 1987 Constitution.

RULING:
Yes. Only the Senate President and the Speaker of the House are allowed to approve the realignment.
Furthermore, two conditions must be met: 1) the funds to be realigned are actually savings, and 2) the transfer is for
the purpose of augmenting the items of expenditures to which said transfer to be made.

As to the certain condition given to the AFP Chief of Staff, it is violative of of Sections 25(5) and 29(1) of the Article VI
of the Constitution. The list of those who may be authorized to transfer funds is exclusive. the AFP Chief of Staff may
not be given authority.

12. RESIDUAL POWER

Marcos v. Manglapus, G.R. No. 88211, [September 15, 1989]


PONENTE: CORTES, J.

221
EXECUTIVE DEPARTMENT
1. CONSTITUTIONAL LAW; BILL OF RIGHTS; RIGHT TO RETURN TO ONE'S COUNTRY, NOT AMONG THE RIGHTS
GUARANTEED. — The right to return to one's country is not among the rights specifically guaranteed in the Bill of
Rights, which treats only of the liberty of abode and the right to travel.

2. ID.; ID.; RIGHT TO RETURN CONSIDERED AS A GENERALLY ACCEPTED PRINCIPLE OF INTERNATIONAL LAW. — It is the
court's well-considered view that the right to return may be considered, as a generally accepted principle of
international law and under our Constitution,is part of the law of the land [Art. II Sec. 2 of the Constitution.]

3. ID.; ID.; RIGHT TO RETURN, DISTINCT AND SEPARATE FROM THE RIGHT TO TRAVEL. — It is distinct and separate
from the right to travel and enjoys a different protection under the International Covenant of Civil and Political Rights,
i.e., against being "arbitrarily deprived" thereof [Art. 12 (4).]

4. ID.; ALLOCATION OF POWER IN THE THREE BRANCHES OF GOVERNMENT A GRANT OF ALL THE POWERS INHERENT
THERETO. — As the Supreme Court in Ocampo v. Cabangis [15 Phil. 626 (1910)] pointed out "a grant of the legislative
power means a grant of all legislative power; and a grant of the judicial power means a grant of all the judicial power
which may be exercised under the government." [At 631-632.] If this can be said of the legislative power which is
exercised by two chambers with a combined membership of more than two hundred members and of the judicial
power which is vested in a hierarchy of courts, it can equally be said of the executive power which is vested in one
official — the President.

5. ID.; PRESIDENT'S POWER UNDER THE 1987 CONSTITUTION; EXTENT AND LIMITATION. — Consideration of tradition
and the development of presidential power under the different constitutions are essential for a complete
understanding of the extent of and limitations to the President's powers under the 1987 Constitution. Although the
1987 Constitution imposes limitations on the exercise of specific powers of the President, it maintains intact what is
traditionally considered as within the scope of "executive power." Corollarily, the powers of the President cannot be
said to be limited only to the specific powers enumerated in the Constitution. In other words, executive power is more
than the sum of specific powers so enumerated.

6. ID.; PRESIDENT'S RESIDUAL POWER TO PROTECT THE GENERAL WELFARE OF THE PEOPLE; THE POWERS INVOLVED.
— The power involved is the President's residual power to protect the general welfare of the people. It is founded on
the duty of the President, as steward of the people. To paraphrase Theodore Roosevelt, it is not only the power of the
President but also his duty to do anything not forbidden by the Constitution or the laws that the needs of the nation
demand. The President is not only clothed with extraordinary powers in times of emergency, but is also tasked with
attending to the day-to-day problems of maintaining peace and order and ensuring domestic tranquillity in times when
no foreign foe appears on the horizon. Wide discretion, within the bounds of law, in fulfilling presidential duties in
times of peace is not in any way diminished by the relative want of an emergency specified in the commander-in-chief
provision.

7. ID.; LIBERTY OF ABODE AND RIGHT TO TRAVEL; REQUEST TO BE ALLOWED TO RETURN TO THE PHILIPPINES; TO BE
TREATED AS ADDRESSED TO THE RESIDUAL UNSTATED POWERS OF THE PRESIDENT. — The request or demand of the
Marcoses to be allowed to return to the Philippines cannot be considered in the light solely of the constitutional
provisions guaranteeing liberty of abode and the right to travel, subject to certain exceptions, or of case law which
clearly never contemplated situations even remotely similar to the present one. It must be treated as a matter that is
appropriately addressed to those residual unstated powers of the President which are implicit in and correlative to
the paramount duty residing in that office to safeguard and protect general welfare. In that context, such request or
demand should submit to the exercise of a broader discretion on the part of the President to determine whether it
must be granted or denied.

8. ID.; JUDICIAL REVIEW; POWER TO DETERMINE GRAVE ABUSE OF DISCRETION OR EXCESS OF JURISDICTION ON ANY
BRANCH OR INSTRUMENTALITY OF THE GOVERNMENT. — The present Constitution limits resort to the political
question doctrine and broadens the scope of judicial inquiry into areas which the Court, under previous constitutions,
would have normally left to the political departments to decide. The deliberations of the Constitutional Commission
cited by petitioners show that the framers intended to widen the scope of judicial review but they did not intend
courts of justice to settle all actual controversies before them. When political questions are involved, the Constitution

222
EXECUTIVE DEPARTMENT
limits the determination to whether or not there has been a grave abuse of discretion amounting to lack or excess of
jurisdiction on the part of the official whose action is being questioned.

9. ID.; LIBERTY OF ABODE AND RIGHT TO TRAVEL; DENIAL OF REQUEST TO BE ALLOWED TO RETURN TO THE
PHILIPPINES, NOT A GRAVE ABUSE OF DISCRETION. — We find that from the pleadings filed by the parties, from their
oral arguments, and the facts revealed during the briefing in chambers by the Chief of Staff of the Armed Forces of the
Philippines and the National Security Adviser, wherein petitioners and respondents were represented, there exist
factual bases for the President's decision. The documented history of the efforts of the Marcoses and their followers
to destabilize the country, as earlier narrated in this ponencia bolsters the conclusion that the return of the Marcoses
at this time would only exacerbate and intensify the violence directed against the State and instigate more chaos. With
these before her, the President cannot be said to have acted arbitrarily and capriciously and whimsically in determining
that the return of the Marcoses poses a serious threat to the national interest and welfare and in prohibiting their
return.

Facts:

Former President Ferdinand E. Marcos was deposed from the presidency via the non-violent “people power”
revolution and was forced into exile. Marcos, in his deathbed, has signified his wish to return to the Philippines to die.
But President Corazon Aquino, considering the dire consequences to the nation of his return at a time when the
stability of government is threatened from various directions and the economy is just beginning to rise and move
forward, has stood firmly on the decision to bar the return of Marcos and his family.

Aquino barred Marcos from returning due to possible threats & following supervening events:

1. failed Manila Hotel coup in 1986 led by Marcos leaders


2. channel 7 taken over by rebels & loyalists
3. plan of Marcoses to return w/ mercenaries aboard a chartered plane of a Lebanese arms dealer. This is to
prove that they can stir trouble from afar
4. Honasan’s failed coup
5. Communist insurgency movements
6. secessionist movements in Mindanao
7. devastated economy because of

1. accumulated foreign debt


2. plunder of nation by Marcos & cronies

Marcos filed for a petition of mandamus and prohibition to order the respondents to issue them their travel
documents and prevent the implementation of President Aquino’s decision to bar Marcos from returning in the
Philippines. Petitioner questions Aquino’s power to bar his return in the country. He also questioned the claim of the
President that the decision was made in the interest of national security, public safety and health. Petitioner also
claimed that the President acted outside her jurisdiction.

According to the Marcoses, such act deprives them of their right to life, liberty, property without due process and
equal protection of the laws. They also said that it deprives them of their right to travel which according to Section 6,
Article 3 of the constitution, may only be impaired by a court order.

Issue:

1. Whether or not, in the exercise of the powers granted by the Constitution, the President may prohibit the
Marcoses from returning to the Philippines.

223
EXECUTIVE DEPARTMENT
2. Whether or not the President acted arbitrarily or with grave abuse of discretion amounting to lack or excess
of jurisdiction when she determined that the return of the Marcoses to the Philippines poses a serious threat
to national interest and welfare and decided to bar their return.

Decision:

No to both issues. Petition dismissed.

Ratio:

Separation of power dictates that each department has exclusive powers. According to Section 1, Article VII of the
1987 Philippine Constitution, “the executive power shall be vested in the President of the Philippines.” However, it
does not define what is meant by “executive power” although in the same article it touches on exercise of certain
powers by the President, i.e., the power of control over all executive departments, bureaus and offices, the power to
execute the laws, the appointing power to grant reprieves, commutations and pardons… (art VII secfs. 14-23).
Although the constitution outlines tasks of the president, this list is not defined & exclusive. She has residual &
discretionary powers not stated in the Constitution which include the power to protect the general welfare of the
people. She is obliged to protect the people, promote their welfare & advance national interest. (Art. II, Sec. 4-5 of the
Constitution). Residual powers, according to Theodore Roosevelt, dictate that the President can do anything which is
not forbidden in the Constitution (Corwin, supra at 153), inevitable to vest discretionary powers on the President
(Hyman, American President) and that the president has to maintain peace during times of emergency but also on the
day-to-day operation of the State.

The rights Marcoses are invoking are not absolute. They’re flexible depending on the circumstances. The request of
the Marcoses to be allowed to return to the Philippines cannot be considered in the light solely of the constitutional
provisions guaranteeing liberty of abode and the right to travel, subject to certain exceptions, or of case law which
clearly never contemplated situations even remotely similar to the present one. It must be treated as a matter that is
appropriately addressed to those residual unstated powers of the President which are implicit in and correlative to
the paramount duty residing in that office to safeguard and protect general welfare. In that context, such request or
demand should submit to the exercise of a broader discretion on the part of the President to determine whether it
must be granted or denied.

For issue number 2, the question for the court to determine is whether or not there exist factual basis for the President
to conclude that it was in the national interest to bar the return of the Marcoses in the Philippines. It is proven that
there are factual bases in her decision. The supervening events that happened before her decision are factual. The
President must take preemptive measures for the self-preservation of the country & protection of the people. She has
to uphold the Constitution.

224
EXECUTIVE DEPARTMENT

S-ar putea să vă placă și